Você está na página 1de 453

- .

.)

,) .)

.)

.)

. .... -,.-

-.,

H)

\"

,)

.)))

H. A.

DJ1

<1><1>

MAPOH)

E PEHUJ1AJ1bHE

11I1HTEfPAJlbHOE
YlC4HCJ1EHI1E
B npl1MEPAX VI
<1>YHKlJ.HH

O,lJ,HOYi

3ALlA4AX)
nEPEMEH

HOV1)

.)
113,lJ,

ATEJ1 beTB0 HAY K A)


\302\253

MOCKBA)))

I. A.

MARON)

Problems
in Calculus
of One Variable)
(With elementsof theory))
TRANSLATED)
FROM

THE

RUSSIAN

by

LEONIDLEVANT)

MIR

PUBLISHERS

MOSCOW)))

UDC 517=20)

First published 1973


Revised from the 1970Russian edition)

Ha
M)

0223-200
041(0I )-73)

aHcAuuCKOM

Jl3blKe)))

Contents)

the

Frolll

Chapter

I.)

I ntroduct

Au thor

..........

.)

.....

ion to Mathematical

Analysis
Numbers. The Absolute Value of a Real Number
Function. Domain of Definition
Investigation of Functions
Inverse Functions
of Functions
Graphical Representation
Number Sequences.
Limit of a Sequence
Evaluation of Limits of Sequences
Testing Sequencesfor Convergence
.
The Limit of a Function
Calculation of Limits of Functions.
Functions. Their DefiniInfinitesimal
and Infinite
.
tion and Comparison
Infinitesimals. Application
to Finding
Equivalent
Lim i ts

.. ..

1.1.
Real

...
..
. ..
.....
.
...
.. .........
.
1.11.
.........
1.12.
.................
....
....
1.13.
One-SidedLimits . . . . . . . . .
1.14.
Continuity of a Function. Points of Discontinuity
and Their Classification . . . . . . . . . . . . . . . .
1.15.
Arithmetical Operations on Continuous Functions.
Continuity of a Composite Function . . . . . . . . . .
1.16.
The Properties of a Function Continuous on a Closed Interval. Continuity of an Inverse Function .
..
1.17.Additional Problems. . . .
I.
ion of Punct ions . . . . . . .
.
2.1.
of the Derivative .
2.2.
of Explicit Functions .
\037

\037

\037

\037

\037

\037

\037

\037

\037

\037

1.2.
1.3.
1.4.
1.5.
1.6.
1.7.
1.8.
1.9.
1.10.

.)

....
..........

.)

11
11
15
22

28
30
41
48
50

.. ....

\037

\037

\037

55

60)

68)

71

75)

\037

77)

\037

84)

\037

.)

\037

Cha pter)

Different

\037

\037

\037

iat

Definition

Differentiation

98
98

.............

2.3.SuccessiveDifferentiation

Lei b n i z For m u 1a

87

91

.)

of

Explicit

100)

Functions.

107)))

6)

Contents)

2.4.

....

of Inverse, Implicit
Differentiation
and Parametri9
.
cally Represented Functions
9 2.5.Applications of the Derivative.
of a Function. Application to AppThe Differential
\037

. . . . . . . . . . . . . III
. . . . . . . . . 115)

2.6.

roximate Computations
2.7.Additional Problems

\037

Chapter

III.Application

of

Calculus

Differential

122
126)

.)

to

of

...
. . . . . . . . . . . . . . . Investigation

Functions

....

.)

3.1.BasicTheorems on

Functions. .

Differentiable

'.

3.2.Evaluation of Indeterminate Forms. L'Hospital'sRule


3.3.Taylor'sFormula. Application to Approximate Calcu at ions . . . . . . . . . . . . . . . . . . . . . . .
3.4.Application of Taylor's Formula to Evaluation of
\037

13]
131
138)

\037

..................
a Function for Monotonicity . . . . . . .
ma of a Function . . . . . . . .
Maxima and
9 3.6.
3.7.Finding the Greatest and the Least Values of a
ct on . . . . . . . . . . . . . . . . . . . . . . .
9 3.8.
Geometry and Physics . . . .
Solving Problems
3.9.Convexity and Concavity of a Curve. Points of
In
ion . . . . . . . . . . . . . . . . . . . . . . .
9 3.10.
Asymptotes
3.11.
General Plan for Investigating Functions and Sket.
ch ng Gra p hs . . . . . . . . . . . . . . . . . . . . .
3.12.
Solution of Algebraic and Transcen.
Approximate
....
dental Equations
3.13.Additional Problems.. . . . . . . . . . . .
ite ntegrals. Basic Methods of ntegrat ion . .
Cha p ter IV.
4.1.Direct Integration and the Method of Expansion
4.2.Integration
Substitution
.....
4.3. Integration
Parts .
.....
4.4. Reduction Formulas . . . . . . . . . . . .
Functions)
...
Chapter V. Basic Classesof Integrable
5.1.Integration of Rational Functions.

143)

\037

Li m its

\037

3.5.Testing

Mini

\037

Fun

&

flect

.....

\037

\037

\037

by

\037

by

\037

\037

\037

\037

\037

\037

5.2.Integration of Certain
5.3.Euler's Substitutions

Irrational

Expressions

5.4.Other Methods of Integrating Irrational Expressions


5.5.Integration of a Binomial Differential . . . .
5.6.Integration of Trigonometric and Hyperbolic Functions
5.7.Integration

162)

166
170)

174)

183
190)

190
195
199
202
211)

214)

214
2]9

222
224
228

..

of Certain Irrational Function\037 with the


AJd of TrigonornetfJc or Hyperbolic
SubstitutIons.

\037

159

..........
..
.)

\037

152)

..
..
.)

\037

\037

148

...........

in

I ndefin

147

230
237)))

Contents)

.
....
tegra Is) . . . . . . .
VI. The
....
Integral .
6.1.Statement of the Problem. The Lower and Upper
.......
Integral Sums .
\037

\037

Chapter

7)

5.8.Integration
5.9.Methods
Definite

\037

of Other Transcendental Functions


of Integration (List of Basic Forms of In-

..
.....
.............

6.2.Evaluating
niz Formula
6.3.Estimating

\037

240)

Definite

Integrals

by

242

247)

247)

Newton-Leib-

the

256)
an Integral. The Definite Integral as a
Function of Its Litnits
262
6.4.Changing the Variable in a Definite Integral
275)
Simplification of Integrals Based on the Properties
of Sym met ry of In tegrands
.
288
Formulas
294
Integration
by Parts. Reduction
301
Definite
Integrals
Approximating
Additional Problems
307

\037

\037

\037

\037

\037

\037

Chapter VI

6.5.

\037

of the

Definite

310)

Integrals.............
......
...

7.1.Computing

finite
\037

..

6.6.
6.7.
6.8.

I. Applications
\037

.................
...
...... ........
....
...
..
............
Integral . . . . . . .

the Limits of Sums with

7.2.Finding Average
7.3.Computing Areas

Values of
in

the Aid of

De-

Rectangular

310
312

Function

Coordinates

317)

7.4.Computing Areas with Parametrically Represented


. . .
Bounda ries .
The Area of a Curvilinear Sectorin Polar Coordinates
.
Computing the Volume of a Solid
The Arc Length of a Plane Curve in Rectangular

................ . .
. ......
Coord tes . . . . . . . . . . . . . . . . . . . . . .
7.8.The Arc Length of a Curve Represented Parametrica
......
....... .........
7.9.The Arc Length of a Curve Polar Coordinates
7.10.Area of Surface of Revolution . . . . . . . . .

\037

\037

\037

\037

7.5.
7.6.
7.7.

ina

327

331
336)
345)

\037

11 y

in

\037

\037

\037

7.11.
Geometrical
7.12.
Computing

Applications

cf

Pressure, Work

the Definite
Integral
and Other Physical

360)

..........
Inertia.
.
7.14.
Problems.. . . . . .
Chapter VIII.Improper Integrals.. . . . . . . . . . . . . .
8.1.Improper Integrals
Limits . . . . . .
8.2.Improper Integrals of Unbounded Functions .
Geometric and Physical Applications
of
9 8.3.
........
. . . . .Improper
....
Integrals
8.4. Additional Problems .
..
...
.
Answers and Hints . . . . . . . . . . .
\037

Quantities
by the Definite
Integrals
Computing Static Moments and Moments of
Determining Coordinates of the Centre of Gravity
Additional
\037

7.13.

.)

\037

\037

\037

\037

with

Infinite

..

.)

348
351
354

.)

367)

372

383
387
387
397)

409
415
418)))

In fond

memory

at my parents)

Fromthe Author)
This textbook on mathematical analysis is based on many years'
of lecturing at a higher technicalcollege.Its aim is to
experience
train the students in active approach to mathematical exercises,
as is done at a seminar.
1\\\\uch attention is given to problems improving the theoretical
are supplebackground.Therefore standard computational exercises
mented by examples
and problems explainingthe theory, promoits deeper underst\037nding and stimulating precisemathemating
tical thinking. Some counter-examples
explainingthe need for certain conditions in the formulation of basic theorems are also in-

cluded.
The book is designed along the following lines. Each section
opens with a concisetheoreticalintroduction containing the principal definitions, theorems and formulas. Then follows a detailed
solution of one or more typical problems.
Finally, problems without solution are given, which are similar to those solved but
Someof them are provided with hints.
contain certain peculiarities.
Each chapter (except Chap. IV and V) closeswith a separate
\037ection
of supplementaryproblemsand questions aimed at reviewing
and extendingthe material of the chapter.These sectionsshould
prove of interest to the inquiring student, and possibly also to
lecturers in selectingmaterial for classwork or seminars.
The full solutions developed in the text pursue two aims:(1)
to provide lecturerswith a time-saver, since they can refer the
students to the textbook for most of the standard exercisesof a
computational character and concentratemainly on the solution
of more sophisticatedproblems, thus gaining time for more rewar-)))

10)

From the

Author)

ding work; and (2) to meet the needs of those who are working
on their own or following correspondence
courses,providing a substitute for the oral explanationsgiven to full-time students.
The student will find the book most useful if he uses it actively, that is to say, if he studies the relevant theoreticalmaterial
carefully before going on to the worked-out solutions, and finally
reinforces the newly-acquiredknowledgeby solving the problems
given for independent work. The best results will be obtained
the student, having mastered the theoreticalpart, immedia..
\\vhen

tely attacks the unsolved problemswithout


solutionsunless in difficulty.)

referring to the text

Isaac Maron)))

Chapter
1)

INTRODUCTION
TO MATHEMATICAL

\037

ANAL YSIS)

1.1.Real Numbers.

The Absolute Value of a Real Number

Any

decimal fraction, terminating or nonterminating,

real number.
Periodicdecimal fractions

is ca1Ieel

are called rational numbers. Every


rational number may be written in the form of a ratio,P- , of two
q
integers p and q, and vice versa.
Nonperiodicdecimal fractions are calledirrational numbers.
If
X is a certain set of real numbers, then the notation x E )(
means that the number x belongs to X, and the notation x
means that the number x does not belong to X.
A set of real numbers x satisfying the inequalit ies a < x < b,
where a and b are fixed numbers, is calledan open interval (a, b).
A set of real numbers x satisfying the inequalitiesa
x b is
calleda closedinterval [a, b].A set of real numbers x, satisfying
the inequalities a x < b or a < x b, is calleda half-open interval [a, b) or (a, b]. Open, closed,and half-open intervals are
covered by a single term interval.
Any real number may be depicted as a certain point on the
coordinate axiswhich is calleda proper point. We may also introduce two more, so-called
improper points, +00 and
infinitely
removed from the origin of coordinatesin the positive and negative dirtctions,respectively. By definition, the inequalities
<
< x < + 00 hold true for any real number x.
The interval
a + 8) is calledthe 8-neighbourhoodof the
\037:\\(

\037

\037

\037

\037

\037

-00

-00

number

a.

(a-e,

>
<

The set of real numbers x M is calledthe M-neighbourhood


of the improper point
The set of
numbers x M is calledthe M -neighbourhood
of t
improper point
The absolute value of a number x (denoted x D is a nurnber

+ 00.

.e

that

r\037al

-00.

satisfies the conditions


I

-x

x ==
x ==x
I

if

if

< 0;
\037

O.)))

Ch.

12)

i. introduction

to Mathematical

The properties of absolute values are:


(1) the inequality x a means that
(2) the inequality x a means that x
x l-t- y I;
(3) x + y
\\

(4)

\037

\037

\037

Analysis)

-a
\037

\037

or x

\037

a;
\037

-a;

'x+ yl\037llxl-lyl/;

(5) Ixyl=lxlly/;
(6) 1= \\:: (y 0).
I

=1=

1.1.1.
Prove that

the number

... \037
.. ...

O.1010010001
1000.a I
n)

is i rra t ional.

Solution. To prove this, it is necessary to ascertain that the


given decimal fraction is not a periodic one. Indeed,there are n
zeros between the nth and (n + 1)th unities, which cannot occur
in a periodic
fraction.

t.t .2. Prove that any number, with zeros standing in all deciplacesnumbered Ion and only in these places,is irrational.
t.t .3. Prove that the sum of, or the difference between, a ra-

mal

tional number

number.

and an irrational number

irrational

is an

\037

= V is a
Suppose a
a rational number, since
it is the difference between two rational numbers, which contradicts the condition.Hence,the supposition is wrong and the number
is irrational.
ex
Solution. Consider the sum of a and
rational number, then = v-a is also

\037.

\037

\037

\037

.4.

t
and the quotient a/f) of a
Prove that the product
is an irrational
rational number a 0 and an irrational number

t,

a\037

=1==

\037

number.
I

.t .5.(a) Fin d

all rat iona 1 val u es 0f x at

number.
Solution. (a) Suppose x and

\\v hi

ch

is a rational

bers.Then the difference y-x


us now expressx through q

+ +
y

V'

\037

V- x

==

+x +3

-=
x

q,

3 == q + x,
x2 + x+ 3 == q2 -r--2qx -1-

x2

q\"!._\037

\037

+ x +3

V x2 -t- x 3 are rational llum== q is also a rat ional number. Let


y

x == V x2

X ==

By a direct

2q

x\037,

check it is easy to ascertain that

q =1=

1/2

,)))

\037

if

/.1.Real

Numbers.

The

Absolute Value

01 Real

Number

13)

+x+3

Prove the reverse, namely, y==Vx2


is a rational number
=
x
where q is any rational number not equal to 1/2,

'

i\0372:

Indeed,

/(q2-3)2
+ q2-3
(1-2q)2
1-2q+3
q4-2q:1+7q\037-6q+9
= -./
= -./(q2-q+3)2
=q2-q +3
V
V
(I 2q)2
(l 2q)2
== V x2-t-x+3==

\037

===

II

2q

(q

=1=

J..
2

)
to
equal 1/2,

\302\267

The latter expressionis rational at any rational q not


(b) Prove that V2 is an irrational number.
Prove that the sum V 3 + V2 is an irrational number.
Solution. Assume the contrary, Le. that the number V 3 +V 2
is rational. Then the nunlber

1.1.6.

V3

-V2 = Y3

\037

is

V2

also rational, since it is the quotient of two rational

Whence the number

V2 =

\037

num1jcrs.

[(V3+V2 )-(V3 -V2\])

contradicts the irrational nature of the number


Problem
Hence,the supposition is wrong, and the
(see
1.1.5).

is rational, which
V2\"

number

V3 +

11'\"

2 is irrational.

.1.7.

t
Prove that for every positive rational number r satisfying
the condition r 2 < 2 one can always find a larger rational number
r + h (h > 0) for which (r + h)2 < 2.
Solution. We may assume h < Then h 2 < hand (r + h)2 <
< r 2 + 2rh -t- h. That is why it is sufficient to put r 2 + 2rh + h = 2,

1.

i.e.h==(2-r2)j(2r+l).
1.1.8.
Prove that for every

positive rational number s satisfying


the condition S2 > 2 one can always find a smaller rational number
2.
(k > 0) for which (S-k)2>
t.t Solve the follo\\ving inequalities:
3

s-k

.9.
(a)12x-1<1;
(b)

(X-2)2;?4;
8

2
2x
(c) x +
2

\037

0;

-7x-i-121>
x2-7x+12.
Solution. (a) The inequality 12x-31
<1
inequalities)
-1< 2x 3 < 1,
(d) x
I

whence)

2 < 2x < 4 and 1 < x

<

2.)))

IS

eqivalent to the

I. Introduction

Ch.

14)

to Mathematical

Analysis)

(d) The given inequality is valid for those values of x at

x2-7x+12< 0, whence 3 < x < 4.

1.1.10.
Find

solut ions:
(a)

\",'hich

whether the following equations have any

out

Ixl==x+5;(b) Ixl==x-5?

Solution. (a) At x 0 we have x ==x + 5. Hence,there are no


X + 5, whence x = -5/2.
solutions. At x < 0 we have -x.=::
This
value satisfies the initial equation.
Hence,there are no solutions.
(b) At x\037O we have
\037

At

<0

we have

-x==x=x-5.
x-5,whence x

=-=

5/2, which contradicts

our supposition (x < 0). Thus, the equation has no solution.


Determinethe values of x satisfying the following equa-

1.1.11.
:

lit i es

x-

x+
(b) x2
I

= xx+ ;
5x + 6 ==

(a)

1.1.12.
:

--+

5x 6).
val ues of x satisfying the

(x

Determinethe
following equa lit i es
2
2
(a) (x + 4x+9) + (2x-3)== x + 4x+91+ 2x 3 /;
2
4
4
x 2 + 21.
(x + 2) 1== x
(b) (x
Solution. (a) The equality a + b == a + b is valid if and
only if both summands have the same sign. Since
x 2 + 4x+9 == (x+2)2+5
>0

-4)-

-41-1
I

at any values of x, the equality is satisfied at those values of x


at which
0, at x 3/2.
holds true if and only if a
The
equality
(b)
b I.
and b have the same sign and a
In our case the equality will hold true for the val ues of x at

2x-3

\037

i.e.
la-bl==lal-lbl
\037

which)

x4

Whence)

\037

\037

x2 + 2.)

'

x2-2\0371;
Ixl\037V3.

Solve the inequalities:


1.1.13.
13x-51-12x-t-31>0;
(b) x 2 5x > x2 l-j5xI.
Find the roots of the following equations.
1.1.14.
(a) jsinxl-==sinx+l;
(b) x2-2Ixl-3==0.
(a)

Solution. (a) This equation \\vill hold true

of x at which

sin x < 0, that is

\\vhy

\\ve

only for

those values

may re\\vrite

it

in

the)))

1.2.Function.

\037

following

Domain of Definition)

15)

way:)

1,or sinx=_1/2;)
-sinx==sinx+

...).

n /6 (k=O, + + 2,
whence
(b) This equation can be solved in a regularway by considering
the cases x\037O and x\037O. We may also solve this equation reit in the form)
writing
== O.
x2 2

1,

x=nk-(-I)k
I

- xl-3
1

Substituting y for I x \\, we obtain


whence
not

fit

Y1

==

3, =
Y2

in. Hence)

Le.X1==

y2-2y-3= 0,

-1.Since

y == I x

-3,

X 2 ===

= x
I

\037

0, the

val ue

Y2

-1does

== 3,)

3.)

* 1.2.Function.Domainof Definition
The independentvariable x is defined by a set X of its values.
If to each val ue of the independent variable x E X there corresponds one definite value of another variable y, then lJ is called
the function of x with a domain of definition (or domain) X or,
in functional notation, y ==Y (x), or Y ==f (x), or y == (x), and so
forth. The set of values of the function y (x) is calledthe range
of the given function.
In particular,the functions defined by the set of natural numbers 2,3,
are callednumerical sequences.They are written
in the following way: Xl' X2 ,
Xn
or {xn }.
t
Given the function f (x)= (x+ 1
Find 1(2x),21(x),
2
2
f (x ), [f (x)]
Solution.)
cp

1,
\"',
.2.1.
.
f

\"', .

(2x)= ;;\037 :; 2f (x)= 2 ;+: ;

f (x ) =
2

1.2.2.
(a) Given the

;: :
\037

; [f (x)J2 =

Xl'

;+:r

\302\267

function
f (x)== log

Show that at

...
)/(x-I).

x2 E

I-x
I

+x

\302\267

(-1,1) the following identity holds true:

f(x1)+f(x2)=f( t+\037:;2 )')))

Ch.

16)

Solution.At

J. Introduction

I-Xl+
I+XI

1-+I +X

2
+X1X2

(I

== log

XIX2

I
XIX 2
I +XIX 2

2
2)

l+x2

== log

+X2

++
Xl

l-x == og (I- l)(l-X) '


og

Analysis)

and
xE(-I, 1) we have (l-x)j(l+x\302\273O

f( Xl ) + f( x2) = og
On the other hand,)
Xl

to Mathefnatical

(l+Xl)(l+X

-Xl

hence

(1)

X2

+XI +X2

-_

XIX 2)

(I-Xl)(l-x2)

og (J+Xt)(l+x2)

'

coincideswith the right-hand member of expression(1).


(b) Given the function f (x)==(aX + a-X )j2 (a > 0). Show that
f (x + y) + f
y) == 2t (x)t (y).

which

Given
1.2.3.

(x-

f (a + 1); f (a) + 1.
t

-1).

the function t (x)==(x+l)j(x:1

.2.4.Given the

function t (x)= x 3
(a)

t
(bt\037\037

Given the
1.2.5.

(b

3-x 1,

(-I),

II
a\037

-1

x
x

\037

(xj2),
xj(x2
tan

f (x)==

\302\260

-2),

\037

< 0,
< n,

n\037x\0376.

(-1)

The points

Hence

x ==n/2,

x ==2nj3

(-1);

).

Find f
f (nj2), f (2nj3), f (4), f (6).
lies within the interval
Solution.The point x ==
Hence
==3 - (-]) 1 == 2.
t

-1

Find

a) and t (

function

- 1.

Find

belong to the interval

[-1,0).
[0, J1).

(nj3)-=::V3.
The points X == 4, x == 6 belong to the interval [n, 6]. Hence
t

(nj2)==tan (nj4) =

1;

f (2Jtj3) ==tan

6
3
2
4
f (4) = 16_ 2 == ; f (6) == 36 2 == 17
The function t (x) is defined over the \\\\Thole number

1.2.6.

by the following

law:

(x) --

2x-5,

2x3+1.
1j(x

2),

if
if
if

x\0372,
2 x
x
j.)))

< :s;3,

>

scale

\037

1.2.Function.

Domain of Definition)

17)

Find: f (Jlr2),t (V8),f (Vlog2 1024).

1.2.7.

In the square ABCD with side AB== 2 a straight line


MN is drawn perpendicularly to AC. Denoting the distance from
the vertex A to the line MN as x, expressthrough x the area S
of the triangle AMN cut off from the square by the straight
line MN. Find this area at x == V 2
and at x ==2 (Fig.
Solution. Note that AC ==2 V2\";
If x\037
hence O\037x\037 2V'

2j

1).

2.

B)

V-2,

then)

S (x)==S6AMN

==x2

> V2, then


S(x) ==4-(2V2-X)2 == x2 +
+4x V2
If

- -

4.)

n)

Thus,)

Fig.

S(X)= {

1)

0\037x\037J/r2,

X2,
2

-x +4xV2-4, V2< x\0372J/2.

== (V2/2)2==1/
Since V2j2<V2,S(V2j2)
2, Since 2>V\"2,
S (2)= 4 + 8 V-2 4 == 8 (V-2 I).)

1.2.8.
Bring

the number an, which is equal to the nth decimal


the
expansionof V2 into a decimal fraction, into corplace
respondencewith each natural number n. This gives us a certain
function an===cp(n). Calculatecp(I), cp(2),cp(3),cp(4).
Solution. Extracting a square root, we find V2== 1.4142
Hence)
==4
(P ( 1)
(2)-== 1; (3)==4; (4) ==2.
in

..

. .

\037

cp

cp

cp

1.2.9.
Calculate f (x) ==49/x +
2

x2

at the

points for which 7jx+ x==3.

2
but 7jx+x==3,hence
Solution.t(x)==49jx
+x2 ==(7/x+x)2-14,

f (x)

-==

9-14 -5.
==

Find
1.2.10.
known

+bx+c,if

2
a function of the form t(x)==ax
==
that f (0) ==5; t
t (I)

Solution.)

(-I)

===

10;

6.

. + b.0 + c,

f (0) ==5 ==a 02

10=-=a-b+c,
1(-1)==
f(I)==6==a+b+c.)))

it

is

i. introduction

Ch.

18)

to Mathematical

Analysis)

Determinethe coefficientsa, b, c from the above system. We have:

a=3;b=-2;c=5;hencef(x)=3x2 -2x+5.
1.2.11.
Find a function of the form
(c > 0),

+ bcx
f (0) == 15;f (2) ==30;f (4) == 90.
f (x) ==a

if

1.2.12.
Find cp['i'(x)]and

'i'[cp(x)]if
2
(x) == x and 'i'(x) =='2

cP

Solut ion.)

.)

(X)]2 == (2 )2 = 2
['i'(x)]==
['i'
'i'[cp (x)]== ==2x2.
X

q;

2\037

1.2.13.
Given the

2X

(X}

function
f (x) ==

5x2

+ .
1

2-x)

Find f (3x); f (x3); 3f (x); [f (x)]3.

1.2.14.
Let)

at
at
at

3X

f (x) ==

-l<x<O,

{ 3x-l

\037

\037

x < 1,
x

3.

\037

Find f (2), f (0), f (0.5),f (-0.5),


f (3).

1.2.15.
Prove

that if for an exponentialfunction y ==aX (a > 0;


1) the val ues of the argument x ==X n (n == 1,2, .. .) form an
arithmeticprogression,then the correspondingvalues of the function Yn ==aXn (n == 1,2,.. .) form a geometric
progression.

=1=

1.2.16.
f (x) ==x2 + 6,
Find f (x)
1.2.17.

cp

if

(x) ==5x.Solve the

cp

(x)

I.

f(x+ 1)=x2-3x+2.

1.2.18.
Evaluate the

functions
+ 1/x2 and

2
f (x) ==x

for the

equation f (x) ==

cp

(x) == x

points at which l/x+ x ==5.

1.2.
19.f (x) ==x+ 1;

+ 1/x4

x-

2; solve the equation


(x) ==
==
f (x)
f (x)
(x)
(x)
A rectangle with
altitude x is inscribed in a triangle
ABC \\vith the base band al t itude h. Expressthe perimeter P and
area S of the rectangleas functions of
I

1.2.20.

cp

cp

+
x.)))

cp

\302\267

1.2.Function.

1.2.21.
Find the domains o

Domain of Definition)

definition of the following functions:

x-I+ V6-x;
(b)f(x)=Vx-x-2+V3 + 2x
(a) f (x) == V

1/' xx

(c) f (x) =

x2

(d) f (x) ==

(e) f (x) ==

sin

\037

x2

19)

1;

log

5x-x\037

(f) f (x) == logx 5;

x2-5x+6;

(g)f(x)==logx 2 _1_4X+6
(h) t (x) == arc

(i) f (x) =
CD

log

x-3
sin2

(:_ x) + V x + 2 ;

f (x) == log cosx;

(k) f (x) == arc cos4 +


1

(1) y==

JI

log (4-x);

3
2'

sIn x

Ixl-x

Solution. (a) The domain

of definition of the given function

consists of those values of x at which both items take on real


values. To en.sure this the following two conditions must be satisfied:)

x-I\037O,

{ 6-x

\037

O.

I;

6.

x
I-Ience,
By solving the inequalities we obtain x
the domain of definition of the function will be the segment
(e) The funct ion is defined for t he val ues of x for which
5x xJ
log 4
\037

\037

\037

[1,6].

0.

This inequality will be satisfied if


5x

x\037

\037

1, or

x2

5x + 4

< O.

Solving the latter inequality, we find 1 x 4. Thus, the segis the domain of definition of the function.
ment
The
is defined for all positive x different from unity\037
function
(f)
which means that the domain of definition of the function consists
of the intervals (0, 1) and
00).)))
\037

[1,4]

(1, +

\037

i. introduction

Ch.

20)

to

\0371athematical

is defined for the values of x for which

(k) The function

-1\037
sin
4+2'
Since 4+2sin x > 0

at

the inequality)

x)

\0371.

x, the problem is reduced to

any

sin
4+2'

Whence)

Analysis)

x)

\037

solving

1.

Le.sinx\037-1/2'
3\0374+2sinx,

By solving the latter inequality we obtain


n
7n
-6+2kjt\037x
\0376+2kjt(k==O,+ 1,+ 2, .).
\"

(I) The function

is defined for the values of x for which


x O.
r

1-

x x > 0, whence x > x. This inequality is satisfied at


Hence,the function is defined in the interval (-00,0).
I

1.2.22.
Find the
(a)

(b)

<

domains of definition of the following functions:

f (x) == Varcsin (log2x) ;


f (x) == log2 log3 log4 x;

(c) f (x) =

\037

+ 2arc

sin

(d) f (x) = log 14

\037;
+ Yx-2

x2 ;
\\

(e) f (x) = V cos(sin x) + arc sin

tt

Find the ranges of the following functions:

--

(g) y

(f )

2 cos3x
x

'.

I-r-x2
Solution.(a) For the function f (x) to be defined the
inequality must be satisfied
arc sin (lOg2x) 0,

folIo\\ving

\037

whence 0 log2 x\037 1 and 1 x\037 2.


(b) The funct ion log2 log3 log4 X is defined for log3 Iog4 x > 0,
whence log4 x > 1 and x > 4. Hence,the domain of definition is
\037

\037

the interval 4 < x<+ 00.


(c) The given function is defined

satisfied simul taneously:


x

but the

is

why

*0; -1
-1

\037

\037

if

the following inequalities are

1 and

x> 2,

that
x\037 1 and x > 2 are incompatible,
inequalities
the function is not defined for any value of
\037

x.)))

\037

1.2.Function.

Domain of Definition)

be satisfied simultaneously:

(e) The following inequalitiesmust

cas(sin x)

\037

21)

and

!xX

\037

1.

The first inequality is satisfied for all values of x, the second,


for x 1== Hence,the domain of definition of the given function
consists onl y of two points x ==

1.

+ 1.
cos3x ::=.:2y-l .

(f) We have)

y)

Since)

-1

\037

cos3x

\037

1,)

-1 2y-1

have

\\ve

\037

\037

y)

whence, taking into account that

> 0, we obtain

(g)

Solving

-y\0372y-l\037yor 3\037Y\037
with respect to x, w e obtain
X

= :!:V2y
I

The range of the function

1-

4y2

1.

be determined from the relation

y will

4y 2

1,

\037

0.)

Whence)

-2\037y\0372'

1.2.23.
Solve the equation
arc tan

Solution.Let

tion on the left

Vx (x + 1)-t- arc sin V x2 + x + 1 -= nj2.

us investigate the domain of definition of the funcside of the equation.This function will be defined

for)

x2

+ x 0, 0 x2 + x + 1
\037

\037

\037

\037)

whence + x ==O.
Thus, the left member of the equation attains real values 0111y
at X 1 = 0 and X2 ==
By a direct checkwe ascertain that they
are the roots of the given equation.
This problem shows that a study of donlains of definition of a
function facilitates the solution of equations, inequalities,etc.
t .2.24.
Find the domains of definition of the fol1o\\ving functions:
x2

- 1.

2x-3 ;
V X2+'2x +3

(a)

(b)

2
y==log sin(x-3)+V16-x
;)))

Ch.

22)

(c)

i. introduction

to Matlwmatiral

Analy\037is)

x-2

y= V 3-x+arccos\037;)

(d) Y= log(l+x)'
x

.2.25.

I].

t
The function f (x) is defined on the interval [0, What
are the domains of definition of the following functions:
(a) f (3x2); (b) f
(c) f (tan x)?
Solution.The given functions are functions of functions, or sue. composite functions.
perpositionsof functions,
an intermediate argument tl ==3x2 Then the
a) Let us introduce
funct ion f (3x2) ==f (u) is defined if
u
e. 0 3x2 1,
whence
3 \037x\037I/V
whence
(c) Similarly:O\037 tanx\037

(x-5);

i.

\302\260

3.
1,

-1/V

\037

\037

1, i.

\037

...).

kn\037x\037n/4+kn (k==O,+ 1,+ 2,


The function f (x) is defined on the interval
1.2.26.
are the domains of definition of the functions
(a) f(sinx); (b) f(2x+3)?)
\037

\037

[0,1].What

1.3.Investigation
of Functions

A function f (x) defined on the set X is said to be non-decreasing


on this set (respectively, increasing,non-increasing,
decreasing),if
for any numbers Xl' x2 EX, X I <X2 the inequality f(XI)\037f(X2)

'

<

>

f (Xl)
f (x2), f (Xl)
f (x2), f (Xl)
f (x2 is satisfied.
The function f (x) is said to be monotonic on the set X if it possessesone of the four indicated properties.The function f (x) is
said to be bounded above (or below) on the set X if there existsa
number M (or ffl) such that f (x) M for all X E X (or m f (x)
for all X E X). The function f (x) is said to be bounded on the set X

(respecti vel y,

\037

\037

\302\273)

\037

is bounded above and below.


funct ion f (x) is called periodic if there exists a number
T :> such that f (x + T) -= f (x) for all x belonging to the domain
if

it

The
\302\260

of definition of the function (togetherwith any point x the point


x -t- T must belong to the domain of definition). The least number T
possessing this property (if such a number exists) is calledthe
period of the function f (x). The function f (x) takes on the maximum value at the point Xo E X if f (xo) f (x) for all x EX, and
the minimum value if f (xo) t (x) for all X EX. A function t (x)
defined on a set X which is symmetric with respectto the origin
of coordinatesis calledeven if f
x)
x) = t (x), and odd if t
\037

\037

==-f(x).
In

analysing the behaviour of a function

termine the

(-

(-

following:)))

it

is

===

advisable to de-

1.3.Investigation

\037

1.The domain of

23)

of Functions)

definition of the function.

2. Is the function even, odd, periodic?


3. The zeros of the funct ion.

4. The sign of the function in the intervals between the zeros.


function bounded and what are its minimum and ma-

5. Is the

values?
The above items do not exhaust the analysis of a function, and
later on their scopewill be increased.
Find the intervals of increaseand decreaseof the funcand maximum values.
tion t (x) ax2 + bx + c, and its minimum
Solution. Isolating a perfect square from the square trinomial,

ximum

1.3.1.
===

we have)

f (x) = a

b
4ac b
x
+
+
4a
2a
)
(
2

a > 0, then the function f (x) will increaseat those values of x


-b/(2a),and
satisfying the inequality x+ b/(2a) > 0, e. at
when x+ b/(2a) < 0, e. at x < -b/(2a).
decrease
Thus, if a > 0,
in the interval
and incthe function f (x) decreases
)
reases in the interval
b/(2a), + 00). Obviously, at x
b/(2a)
value)
the function f (x) assumes the minimum
If

i.

i.

- -

(-00, ;a

(-

===

fmin==f

>0

x>

2a)

= 4ac-b2
4a
\302\267

the function has no maximum value.


Similarly, at a < 0 the function f (x) will increasein the interval
;a) and decreasein the interval (-b/(2a),00); at
function f (x) takes on the maximum value
the
b/(2a)
At

x=-(-00,

f max ==f

whereas

it

2a)

= 4ac-b2
4a '

has no minimum vaJue.


the minimum value of the function

1.3.2.
(a) Find

==3x2

+ 5x

-.
1

(b) Find the rectanglewith the maximum area from among all
rectanglesof a given perimeter.
a = 3 > 0, b == 5,
Solution.(a) Apply the results of Problem 1.3.1:

c=

-1. -

The minimum

point x ==

value is attained

5/6)
Ymill

(b) We

2
= 4ac-b
=
4a

by

the function at the

37
0

12

\302\267

denote by 2p the length of the perimeter of the required


by x the length of one of its si des; then the area

rectangle,and

S)))

Ch.

24)

I. Introductiun

to Mathematical

Analysis)

as
rectanglewill be expressed

of the

2.
S==x(p-x)or S==px-x

Thus, the problem is reduced to the determinationof the ITlaximum


value of the function S (x) == x2 + px. Apply the results of Proba == 1 < 0, b ==p, c ==0. The maximum value is attailem t
ned by the function at the point x == b/(2a)= p/2. Hence,one
of the sides of the desired rectangle is p/2, the other side being
==p/2, e. the required rectangleis a square.
equal to p

.3.t:

-x

i.

.3.3.Show that

3
(a) the function f (x) -== x + 3x + 5 increa'Sesin the entire domain
of its definition;
in the interval (1,+ 00).
(b) the function g (x) = x/(I+x2) decreases
Solution. The function is defined for all points of the number
scale.Let us take arbitrary points Xl and x2 Xl < x2 on the number
scaleand write the following difference:

'

)-f (Xl) = + 3x2 + 5) + 3xI + 5) ==


= 2 -XI) + I 2 + xi + 3) ==
2
= (x2 Xl)
+
+ 3] .
2 +
+
)
l(
Since x2
> 0 and the expressionin the brackets is positive
and x2' then f (x2) f (Xl) > 0, i.e. f (x2) > f (Xl), which
at all
f (x2

(x\037

(x\037

(X

X X

(x\037

Xl

Xl

Xl

X\037

\037

means that the function f (x) increasesfor all values of x.


for the folloFind the intervals of increaseand decrease
t .3.4.
functions:
wing
(a) f (x) -== sin x+cosx;
(b) tan (x+ n/3).
Solution.(a) Using the familiar trigonometricformulas, we find
f (x) = V2 cos

(x-n/4).

It

is

known

cosX decreasesin

tha t the function


2nJt

\037

and increasesin the intervals


Jt
x\037 2nn

(2n-I)

\037

\037

the intervals

(2n + 1)Jt

(n==

0, + 1, + 2, .).
\"

Hence,the intervals of decreaseof the function f (x) are:


(n==O, + 1,
n/4+2nn\037x\037n/4+(2n+I)n
and the intervals of increaseof the same function are:
(n=O, + 1,
n/4+(2n-l)Jt\037x\037n/4+2nn
Find the
1.3.5.

...),
...).

and maxilllum val ues of the function


a coSx -t- b sin x (u i + b2 >

minimum

t (x)

==-

0).)))

1.3.Investigation

\037

Solution.The given
f

25)

of Functions)

on can be represented as:

functi

(x-a),

2
(x) ==V a'l. -t- b cas

where casa ==al


+ , sin a ==bl Va + b'l.Since
the maximum value of f (x) equals + Va2 -t- (at
the minimum
value of f (x) being equal to
\037/

b2

a2

0\"\"

-1).

cos(x-a)==

1.3.6.
Find

cas(x-a) I,
cos(x-a) 1),

\037

-==

V a2

+ b2

(at

value of the function


f (x) == 3(X -2)3+8.

the minimum

Solution.We denote by
cp

i. e.

(x) the exponent,


2
2)3
(x) (x
cp

\037

+ 8.

The function f (x) =- (x) takes on the minimum


same point as the function (x).
Hence)

value at the

3<1>

cp

-6x -t-12x==x ((x -3)2+3].


2

Whence it is clearthat the function


value (equal to zero) at x ==O. That is
the function f (x) is equal to 30 ==

why

cp

(x) ==x

1.

1.3.7.
Test

x+ cot x, where
decrease.

(x) == tan

increase and
1.3.8.
Given: n numbers

of x at which the function


f (x) ==

takes on the

attains the minImum


the minimum value of

the function
f

for

(x)

cp

< x < '!t/2,

...,

aI' a 2,

an' Determine the value

(x-a1)2 + (x-a)2 + ... + (x-a )2


2

minimum

Solution.Rewrite

value.

the function f (x)

the following way:


2 (at + a 2 +
f (x)
+ an) x+ (af + -t- +
wherefrom it is clearthat f (x) is a quadratic trinomial ax2 + bx + c,
where a ==n > O. Using the results of Problem t t, V.le find that
the function assumes the minimum value at x ==-b/(2a), e. at
x == (a}+ a 2 +
+ a,Jln.
Thus. the sum of the squares of deviations of the value of x
from n given numbers attains the minimum
value when x is the
mean ari thmetic value for these numbers.
==nx 2

...

in

a\037

...

.3.

...

1.3.9.
Which
which

of

thenl

is

of the given functions is (are) even,


(are) nei ther even, nor odd?)))

a\037),

i.

odd; and

Ch.

26)

/. Introduction

to Mathematical

Analysis)

(a) f (x) -== log (x + VI + x2);

I-x

(b) f (x) == log I +x ;


3
(c) f (x) == 2x

(d) f (x)

-x+1;

= X aX _+
aX

Solution. (a)
f ( -f- x)

I
\302\267

It can

be seen that

f(+x)+f(-x)=-=O. Indeed,

+ t (--x) == log (x + VI + x ) + log (-x+ VI + x2) ==


== log (1+ X2 _X 2) 0,
2

hence,f (x) ==
(b) t

- (-I+x=

===

x) for all

(-x)==log I-x

(- (-1,1).Hence,

x,which meansthat
l -X -1

the function is odd.


I=-log x.

log ( I+x)

f (x) for all

I+x

Thus, f

x) ==

1.3.

-J/l+ + - - +
+ '.

from

the domain of definition

the function is odd.


to. Which of the following funct ions is (are) even and which
is (are) odd?
2
(a) f (x) ==4 2x4 + sin x;
(b) t (x) == kx x x
a
I
(c) f (x) ==
kx
==
sin
cnsx;
x
(d) f (x)
(e) f (x) ==const.

I-a

J;r 1

x 2;

1.3.11.

Prove that if f (x) is a periodicfunction with period T,


then the function f (ax + b), where a > 0, is periodicwith period T

la.

Solution.Firstly,
f

=f(ax+b),
(a(x+Tla)+b]==f [(ax+b)+T]

since T

is the period of the function t (x). Secondly, let T 1 be a


positive number such that
f [a (x+T1) + b] ==f (ax + b).
Let us take an arbitrary point x from the domain of definition

x' == (x b)la. Then


+ ) = f (x) = f [a (x'+ T 1)+ b] =
== t (ax'+ b + aT t ) == f (x+ aT l).

of the function f (x) and


x b
b
f (ax' b) = f a a

put

l' i.

e. Tt Tla and
Whence it follows that the period T aT
T la is the period of the function f (ax + b).
Note. The periodicfunction t (x) == A sin (wx+cp), where A, (0, q>
are constants,is calleda harmonic with amplitude I A I, frequency
\037

\037

w)))

\037

and initial
function A

phase
sin (U)X

.8.Investigation

of Functions)

27)

Since the function sin x has

cpo

+ cp)

Indicatethe
1.3.12.

has a period T == 2n/ffi.

period 2n, the

amplitude I A I, frequency ffi, initial phase


T
the
of
following harmonics:
period
==
(a) f (x) 5 sin 4x;
(b) f (x) ==4 sin (3x + n/4);
(c) f (x) == 3 sin (xj2)+ 4 cos(x/2).
t
Find the period for each of the following functions:
(a) f (x) ==tan 2x;
(b) f (x) ==cot (x/2);
(c) f (x) == sin 2Jtx.
Solution.(a) Sincethe function tan x has a period n, the function
tan 2x has a period nj2.
cp

and

.3.13.

.3.14.
Find the

x+ cos4 x;
f (x) == I cosx I.

(a) f (x)
(b)

period for each of the following functions:

==sin4

Solution.(a)

sin4x+cos4x=(sin2x+cos2x)2-2sin2xcos2

= l-\037
sin2 2x ==
2

I-cos
1-..!..

4x) == 4

4(

whence T ==2n/ffi == 2n/4 == n/2


(b) f (x) ==
cos2x has a

cosx

2X
==V cos
V(I
==
period T n; hence,the

\037

+\037
4

sin

( 4x+ 2 ) '
\037

\302\267

+ cos2x)j2; but

\037=

period.

the funct ion


given function has the same

Prove that
.3.15.

the function f (x) ==cosx2 is not a periodic


one.
the function has a
contrary. Suppose
period T; then the identity cas(x + T)2== cosx2 is val ide
By the conditions of equality of cosinesfor a certain integer k
t

Solution.IJet

us prove the

we have)

x2 + 2Tx+T2 + x2 == 2Jtk.

But this identity is impossible,since k may attain only integral


values, and the left member containsa linear or quadratic function
of the continuous argument x.

1.3.16.
Find the

greatestvalue
f (x)=

1.3.17.
Which

odd:

of the

(a) f (x) == V (1

X)2

of the function
2

V2x 2

follo\\ving

+ V'(1+

-4x+3'

functions are even, and which are


X)2;)))

Ch.

28)

I. Introduction

to Mathematical

Analysis)

-Ixl;
x-

(b) f(x)=x2
(c) f (x) ==x sin2 x3;
X
(d) f(x)==(1+2
)2j2X ?
1.3.18.
Find the period for each of the following functions:
(a) f (x) == arc tan (tan x);

X-j'[

(b) f(x)==2cos3
t 9. Prove
1.3.

f (x) ==

\302\267

that the functions


(b) f (x) == cosV x

x+ sin x;

(a)
are non-periodic.)
\037

1.4.InverseFunctions

Let the function y == f (x) be defined on the set X and have a


range Y. If for each y E Y there existsa singlevalue of x such that
defines a certainfunction x g (y)
f (x) = y, then this correspondence
calledinverse with respect to the given function y ==f (x). The suf\037

of an inverse function is a strict


ficient conditionfor the existence
monotony of the original function y ==f (x). If the function increa,cs
(de\037reases), then the inverse function also increases(decreases).
The graph of the inverse function x ==g (y) coincideswith that of
the funct ion y == f (x) if the independentvariable is markedoff along
the y-axis. If the independent variable is laid off along the x-axis,
e. if the inverse function is written in the form y ==g (x), then
the graph of the inverse function will be symmetric to that of the
function y ==f (x) with respect to the bisectorof the first and third

i.

quadrants.
t .4.1.
Find the inverse to the funct ion y == 3x+ 5.
Solution.The funet ion y ==3x+ 5 is defined and increasesthrough-

scale. Hence,an inverse function existsand increases.Solving the equation y ==3x+5 with respect to x we obtain
x ==(y-5)/3.
out the number

.4.2.Show that

the function

== kjx (k

0) is inverse to itself.

Solution.The function is defined and monotonic throughout the


entire number scaleexceptx == O. Hence,an inverse function exists.
The range of the function is the entire number scale,excepty ==O.
Solving the equation y ==kjx with respect to x, we get x ==kjy.
1.4.3.
Find the inverse of t he fu nction

2
+ I), (a>O, a I).
=10ga(x+Vx
Solution. The function y == loga (x+ Vx 2 + 1) is defined for all x,
sinceVx2 + 1 > x 1, and is odd [seeProblem t .3.9
(a)J.It increases)))

=1=

\037

.4.Inverse

Functions

29)

x, hence,it increaseseverywhere

for positive values of

and has an

inverse function. Solving the equation


y== loga

respect to x, we

with

(x+V x2 + 1)

find

a-Y == x+ V + 1,)

aY ==x + V x2 + 1;
whence)

x=

(aY

\037

.4.4.Show that

-a- ) = sinh
Y

X2

(y In

a).

the funct ions

-x+l,x\037I/2 and cp(x)=1/2+Vx-3j4


inverse, and solve the equation
x -x+1 1/2+ x-3/4.
2

f(x)===x

are mutually

==

V\"

increases
Solution.The function y=x -x+I-==(x-I/2)2+3/4
the interval 1/2 x < 00,and with x varying in the indicated
2

in

\037

we have 3/4 y < 00. Hence,defined


x ==g (y), x
y < 00 is the inverse function
from the equation

interval

3/4

\037

found

\037

x2

the interval

1/2,which

respect to x, we obtain

x==g (y) = 1/2+ Vy

3/4 =

cp

(y).)

solve the equation

now

x2

-x+1

==

1/2+Vx-3j4.

Sincethe graphs of

the original and inverse functions can


2
only on the straight line y ==x, solving the equation x

we find
t

is

-x+(l-y)= O.

Solving the equation with


Let us

in
\037

x == 1.

.4.5.Find the

intersect

-x+1

==x

inverse of y = sin x.

Solution.The domain of definition of the function y ==sin x is


the entire number scale,the range of the function is the interval

[-1,1].

But the
not fulfilled.

condition of existence
of an inverse function is

Divide the x-axisinto intervals /1n n/2 x nn + nj2.If n is


even, then the function increases on the intervals nn nj2 x
nn + n/2; if n is odd, the function decreases
on the intervals
n/2 x nn + n/2. Hence,on each of the indicated intervals
there existsan inverse function defined on the interval
1, 1

nn-

\037

\037

\037

\037

\037

\037

\037

[-

J.)))

Ch.

30)

/. / ntroduction

to Mathematical

Analysis)

In particular,for an interyal
n/2 x n/2 there existsan Inverse function x == arc sin y.
The inverse of the function y==sinx on the interval nJt-n/2\037
x nn + n/2 is expressed
through ate sin y in the follo\\ving \\vay:
x == l)n arc sin y + nJt (n ==0, + + 2,
\037

\037

(-

1.4.6.
Find the

\037

\037

1,

...).

inverse of the given functions:


(a) y == sin
1) at (n/6+ 1/3) x (n/6+ 1/3);
x\037 3;
(b) y ==arc sin (x/3)
(c) y ==51og x;

(3x- at-3

\037

\037

\037

(d) y==2X (X-l>.

1.4.7.
Prove

that

the function

itself.)
\037

y==(l-x)/(l+x)is

inverse to

1.5.GraphicalRepresentation
of Functions
1.5.1.
Sketchthe
(a)

(b)

graph of each of the following functions:

-2x + 3;

2
4
f (x) ==x
2x
f (x) == I x2 ;

(c) f(x)==sin2 x-2sinx;


(d) f (x) = arccos(cosx);
(e) f (x) = V sin x;

1
(f) f (x) == x /l og x.

Solution. (a) The domain of definition of the function f (x) is the


entire number scale.
The function f (x) is even, hence its graph is
symmetrical about the ordinate axisand

!I

it

is

sufficient to investigate the function at x O.


Let us single out a
square f (x) ==
= (x2 1)2+ 2. Since perfect
the first summand
2
0, the minimum value of the func(x
tion, equal to 2, is attained at the points

-1)2

\037

\037

-2 -1

.r

x ==

:f::1 (see Fig.2).

The function f (x) decreases


from 3 to 2 on
the closedin terval
x 1 and increases
unboundedlyon the open interval 1 < x < 00.
(b) The domain of definition of the funcFig. 2
tion f (x) is the entire number scale.The function f (x) is odd, therefore its graph is symmetrical about the origin
of coordinates and it is sufficient to investigate the function at
\302\260

\037

0.

Sincef (0) ==0, the graph passes through

that

\037

\037

the origin. It is obvious


there are no other points of intersection with the coordinate)))

\037

axes.Note

that

whence)

31)
1.5.
Graphical Representation of Functions)
1+x2\0372Ix!,
It(x)l\037 1.Indeed,(1-lxD2\03700r

+ x2 ::= f (x)I.
f(I)==I,in the

2\\x\\
\037

Sincef(x)\037Oat

x\037O and
value of the function f (x) equals

maximum

being zero (see Fig. 3).)

interval

the
[0,00)
value

1,the minimum

g)

x)

-)-----------------Fig.

\037

3)

Let us prove that the function increases


on the
Then)
Let 0 x] < x2
x\037

1.

\037

\037

f{x2)-f (Xl) =

1.

2x 2 + 2X2Xl2

2x 1
I

1\037:\037)

closedinterval

- 2Xl

\037

:!

2XtX2

(l+x\037)(l+xi)

+x;)

= 2 (x2-xd(l-Xt 2) > 0
X

and f (x2)

(]

> f (Xl)'

Similarly, we can show

decreases.
Finally,

on the interval

that

(x) ==2xj(1

+x

\037)

(I -t- xi))

(1,00) the

function

+ X2) < 2xjx2 == 2/x,

whence it is clear that f (x) tends to zero with an increase in x.


(c) The domain of definition of the function f (x) is the entire
number scale.The function has a period 2Jt, that is why it is quite
sufficient to investigate it on the interval [0,2Jt], where it becomes zero at the points x ==0; x = Jt; X == 21t.
Writing the given function in the form
f

(x) ==

(I-sinx)2-1,

that it increaseswith a decrease


in the function sin x and
on
decreasesas sin x increases.Hence,the function f (x) decreases

we note
[Jt/2,

-(Fig.

[0,Jt/2] and [3Jt/2, 2Jt], and increaseson the interval


3Jtj2].Sincef (Jt/2) == 1,and f (31t/2)==3, the range of the

the intervals
function is

\037

f (x)

\037

4).)))

Ch.

32)

I. Introduction

to Mathematical

Analysis)

(d) The domain of definition of the function is the entire number

scale. Indeed, cosx

1 at any x, hence, arc cos (casx) has a


meaning.The function f (x) is a periodic one with the period 2Jt,
hence, it is sufficient to sketch its graph on the interval [0,2n].
I

\037

on this interval the following


equality is true:

But

!J

f (x)=

Z)

x,

\037

\037

:11,

2Jt-x, Jt\037x\0372Jt.

\037

\\

first assertion follows


the definition of the function
arc cosx, whi Ie the secondone can be
proved in the following way. Let us

Indeed, the

1)

from

-x,

x' = 2Jt
n
x 2n; then
x' nand
= arc c()s(casx') = x' = 2n-x.
f (x)= arc cos[cos(2n-x')]
we draw the graph (seeFig.5).
Taking all this into consideration,
=
The
function
sin
x
is
a
}I.'
(e)
y
periodicone with period 2Jt;
put

Fig. 4

that is

why

\037

\037

\037

\037

we may confine ourselves to the interval

-21t

[0,2n].But)

x)

-'It)

o)

1C)

Fig.

21C)

5)

the whole interval [0,2n],it is


the interval (Jt, 2Jt) the
radicand is negative.The graph is symmetrical about the straight

the function is not defined


defined only in the interval

in

[0,n], as in

y
1)

-21C)

-TC)

1(;

-2)

x)
1C)

Fig. 6)

line x:=n/2, as well as the graph y = sin x (see Fig. 6). Herewe
have an exampleof a periodicfunction which does not exist in
the infinite set of intervals.
(f) The domain of definition of the function is
o < x < 1 and 1 < x <
00.)))

\037

1.5.
Graphical

Reduce the formula to the

33)

of Functions)

Representation

form

0
I
x
f (x)= X IIog = Xl og:J: = 10.
1

graph of the given function is the half-line y = 10


in the right-hand halfplane with the point x = 1 removed (seeFig.7).
t
Sketch the graphs of funG-tions defi!I)
ned by different formulas in different intervals (and in those reducibleto them):
10)
Jt
X
sin x at
0,
I
I
x
2
at
(a) y=

Hence,the

.5.2.

-<
\037

\302\260

{ I/(x-l) at

(b) y=

-2 at x>O,
=

1/2 at x
x3 at x
Vx2;

\037

\037

1,

1<x\0374;

I
I)

0,

o)

:c)

1)

< 0;

Fig.

(c)y=x +

(d) y=2/(x+Vx2).

Solution. (a) The domain


n, 4].The gra ph

interva
of the

[-

7)

of definition of the function is the


of the function consists of a portion

sinusoid y=sinxon the interval -Jt\037x\037O,straight

line)

If)
!f)

1/2I

o1

=- :c)

-1

x)

-'It)

-2)
Fig.

Fig.

8)

9)

= 2 on the interval (0, I] and a part of the branch of the hyperon the interval (1,4](see Fig.8).
bola y=
(b) The graph of the function consists of a portion of a cubic
parabola.an isolated point and a half-line (see Fig.9).
(c) The function may be given by two formulas:

1/(x-l)

2x,

if

y= { 0, if

x
x

\037

<

0,
o.)))

Ch.

34)

i. introduction

to Mathematical

Analysis)

Thus, the graph of our function is a polygonal line (seeFig. 10).


(d) From (c) it follows that the function is defined only in the
interval (0, + 00), y being equal to l/x (x 0). Thus, the graph
of our function is the right-hand part of an equilateral hyperbola

>

(see Fig. 11).)

9)

2
1)

1)

!f=0)

x)

o
Fig.

1.5.3.
Sketchthe
(a)

1)

10)

Fig.

11)

graphs of the following functions:

y=cosx+lcosx/;
=x 2

(b) y

:c)
1)

+ Ix.

2 cosx at

cosx 0,
\037

Solution.(a) cosx + cosx 1=


0 at cosx < O.
{
the
of the graph for the funcordinates
non-negative
Doubling
tion y = cosx (the broken line in Fig.12) and assuming y == 0 at)
I

_ 31r

/2
',-Tr
----\",

!\037

2
Fig.

12)

the points where cosx < 0, we can sketch the desired graph (the
solid line in the same figure).
(b) The function Ix+2lxmay be given by two formulas:

(x+2)xat x\037-2,

y= { -(x+2)xat x\037-2.
Plotting separately both parabolas:y=(x+2)x=(x+l)2-1,

y=-[(x+

- 1],

retain only the parts correspondingto


and
1)2
the above indicated intervals. Drawn in a solid line in Fig.13is
the graph of the given function, the broken line showing the deleted parts of the constructed parabolas.)))

\037

1.5.
Graphical

Sketch the
1.5.4.

35)

of Functions)

Represpntation

graph of the function

y=2Ix-2\\-1x+ II+x.
Solution.At
At

2
Y= 2

\037

2x-5.

-1< = - (x-2)-(x+1)+x== x \0372


-.;;;:::
-.;;;:::)

(x-2)-(x+I) + x

2x+3.

!J)

I
I
I
I
I
I
I
I
I
I

x)

:c)

-1 0
-1)
Fig.

Fig.

13)

14)

Finally, at x \037-l
Y

Hence,the
Y=

-2(x-2) + (x+I) + x =5.)

given function can be rewritten

x\037-I,

5,

in

the following way:

-2x+3, -I \037x\0372,
{ 2x-5, x 2.
\037

Therefore the graph is a polygonal line

(see Fig. 14).


Sketchthe graph
1.5.5.
Y

=;:=

2x

of the function

-2- .

:c)

-1)

Solution. Draw graphs for the functions


= 2X and Y2 = 2-X (broken lines in
Fig. 15), and add graphically their ordinaFig.
tes. In doing so bear in mind that Y2 < Y <
that Y2 tends to zero with an increasein x, whereas YJ
< Yt, and
tends to zero with a d\037crease in x (the solid line in Fig.15).

Yl

15)))

.5.6.Sketchthe

graph of the function


y = x sin x.)

Ch.

36)

/. Introduction

Solution. Being the product


=

sin x, the function


analyse it for x 0.
We draw graphs for

Y2

to Mathematical

\037

of two
is an even

Analysis)

odd functions

one, that

is

Yl

why

=x

and

we shall

= sin x (the broken lines in


the points where Y2 = sin x = 0, Y = Yl Y2 = 0, and at the
At
1,Y = :f:Yl = :f:x. The latter equality)
points where Y2 = sin x =:f::
Fig. 16).

Yl

==x

and

Y2

\302\267

3:)

,)
Fig.

16)

indicatesthe expedience
of graphing the auxiliary function Ya = x.
Marking the indicated points and joiningthem into a smooth

--1.

curve, we obtain the required graph (the solid line in Fig. 16).
Sketch the graph of the function Y = x (x 2 1) by multithe
ordinatesof the graphs YI = x and Y2 = x 2
plying
Graph the following functions:
(a) y=x/(x2
(b) Y= l/arccosx.
Solution. (a) Since the function is odd, it is sufficient to investigate it for x O.
Let us considerit as the quotient of the two functions:
YI=X and Y2=x2
Since at x = 2 the denominator Y2 = 0, the function is not defined at the point 2. In the interval [0,2) the function Yt increases
2 decreases
from
to 2, the function Y2 is negative and Y21 =
==:.
from 4 to 0; hence, the quotient f (x) Yl/Y2 is negative and increasesin absolute value, Le. f (x) decreasesin the interval [0,2)
from
to
In the interval (2, 00) both functions arepositive and increasing.
Their quotient decreases
since from 2 Xl x 2 it follows that

1.5.7.
1.5.8.

-4),
\037

-4.

\302\260

\302\260

4-x

-00.

- x:-4- -

Y2-Yl _

Xl

x\037-4

\037

<

_ 1-X2)(X 1 2+4) <0


(X

(x\037-4)

(xr-4)

\302\267)))

1.5.Graphical

\037

Representation

of Functions)

37)

The indicatedquotient tends to zero asx-+ 00, sincey


The general outline of the graph is presented in

solid I in es) .
(b) Denote Yl
tion

\037

I.

At

== arc cosx.
x == 1 we
If)
I)

=
-+ O.
Fig. 17 (three
1

The domain of definition of this funchave Yl:=0, hence, Y == l/Yl ---)-00 at

\"

/)

,
I,
:
'\\

I \\

x)

I
I
I
I
I
I
I

y
1r

'

I
I
I

\"

\\

,)

-1)
Fig.

\037\037\037xa

Fig.

-+ 1, i. e. x == 1 is

,I,:
1

17)

I
I
I

:0)

18)

a vertical asymptote. The function Yl decreases


of definition
1), henceY == I/Yl incre==
at x
Accorn
attained
is
ases. The maximum value Yl
The
solid
line
is
function
of
the
value
minimum
the
l/n.
dingly,
in Fig. 18represents the generaloutline of the graph.)

on the

[-1,

entire interval

\037

- 1.

Simple Transformations of Graphs


I. The graph of the function Y:=f (x+ a) is obtained from the
graph of the function Y ==t (x) by translating the latter graph along
the x-axisby a scaleunits in the directionoppositeto the sign
I

of a

(see Fig. 19).

II. The

function

graph

y:=t(x)+bis

Y ==f (x) by

scale units
b
Fig. 20).

by

obtained from the graph of the


latter graph along the y-axis
the
translating
to the sign of b (see
in the directionopposite

>

I.

II The graph of the funet ion y ==t (kx) (k


0) is obtained from
the graph of the function y:= t (x) by \"compressing\"the latter graph
1 and
against the y-axis in the horizontal directionk times at k

>

by \"stretching\"

times at k

it

in

the horizontal directionfrom the y-axis I/k

< 1 (see Fig.21).)))

Ch.

38)

I. Introduction

to Mathematical

Analysis)

>

IV. The graph of the function y = kf (x)(k 0) is obtained from


the graph of function y = f (x) by \"stretching\" it in the horizontal
directionk times at k I and \"compressing\" it against the x-axis
(i. e. vertically) Ilk times at k < I (see Fig. 21).

>

!I)
!I:sf(x)+b)b >0

I/=f(x+a)
!/>O)

!I=f(x)

\037o
\037\"

y=f(x)+o)b<O)

,,

:c)

o)

:c)

Xo

U=f(3J))

Fig.

19)

Fig.

20)

V. The graph of the function y =


f (x) is symmetrical to that
of the function y = f (x) about the x-axis,while the graph of the
function y f (---x) is symmetrical to that of the function y = f (x)

about the y-axis.)

If)

!I)

y=fflxlJ)

II
I
II

:c
!f=f(x)

Fig.

:'

x)

0)

Fig.

21)

22)

graph of the function y = f ( x I) is obtained from the


of
the
function y = f (x) in the following way: for x 0 the
graph
graph of the function y = f (x) is retained,then this retained part
of the graph is reflected symmetrically about the y-axis, thus determining the graph of the function for x 0 (see Fig. 22).
The graph of the function y = f (x) is constructed from
the graph y = f (x) in the following way: the portion of the graph)))

VI. The

\037

VII.

\037

\037

1.5.
Graphical

Representation

39)

of Functions)

of the function y ==f (x) lying above the x-axisremains unchanged,


its other portion located below the x-axisbeing transformed sym-

metrically about the x-axis(see Fig. 23).


VIII.The graphs of the more complicatedfunctions
y

===

(kx+a)+ b

'At

are drawn from the graph of y ==f (x) applying consecutivelytransformations I to V.

1.5.9.
Graph

t he

function

y ==3J/. 2 (x

+ 2.5)-0.8

transforming the graph y == Vx.


z)
of the function
o)
I
==
I
is
the
of
the
x
branch
V
y
(which
upper
I
parabola y2 ==x) (Fig.24, a), and transform
=ftx)
(cJi'
fI
it in the following sequence.
Sketchthe graph of the function y==3V'2x
Fig. 23)
by enlarging 3 V2' times the ordinates of
the points on the graph of the function y = V x and leaving their
abscissasunchanged (see Fig. 24, b).
Then sketch the graph of the function y = 3V 2x which will
be the mirror image of the precedinggraph about the y-axis (see
by

Solution.Sketchthe graph

'

Fig. 24, c).)

!J)

!f)

+y

y=3 V- 2{x+2.5)-0.8)

3)

2.2)

o)

-1/2 0

\037)

0)

(c))

\302\253(1,))

Fig.
By shifting the obtained graph
downwar d draw the

0.8 u nit

-3 -2.5 _ 0
0.0

\037

\302\267)

(d)

24)

2.5scaleunits

leftward and then

desired graph of the function


(see Fig. 24, d).

-2(x+2.5)-0.8

y==3V

1.5.10.

Graph the function


the cosinecurve.)))

3 sinx by
y=3cosx-V

transforming

Ch.

40)

I. Introduction

to Mathematical

Analysis)

Solution.Transform the given function


3
Y=3cosx-VV: cosx-\037sinx)=
Sinx=2V\0373(

= 2 V3cos x +
(

\037

Thus, we have to sketch the graph of the function


y=2

V3cos(x + 'It/6),

which is the graph of the function y = 2 3 casx translated by 'It/6


leftward. The function has a period of 2n, hence it is sufficient to
'It \037x
'It
draw its graph for
(see Fig. 25).)
V\037

\037

:c)
:x)

-I)

-2VJ)

F j g. 25)

Fig. 26)

The graph of any function of the form y = a cosx + b sin x, where


and b are constants, is sketched in a similar way.
Graph the following functions:

1.5.11.

= x+3
x+ ;
(b) y = x2 9;

(a)

1,
y=
{ (x-l)j(x+1),

(c)

X2+X+
sin2 x,

(d)y=x+l/x;
(e) y=X 2 _X 3;
(f) y=x+sinx;

x;
(g) y =
3
in
s
(2x
(h) y
(i)

if

-1

\037x

< 0,

if

O\037X\037'It,

if

'It

<

l/cos
--4);
=
y=2V-3(x+1.5)-1.2;)))

\037

5;

\037

(j)

1.6.Number

Limit
Sequences.

of

41

Sequence

r)

y=lx2 -2x-ll;

(k) y = II X

1-

1 I;
(1) y = cos(sinx);

on
y=lsinxl+sinx

(m)

the interval

[O,3n];
> 0,
0 at x= 0,
I at x

(n) y

===

x 2 signx, where

signx ==

1 at x

< O.

t 2. The function y = f (x) is given graphically (Fig. 26).


t .5.
Sketch the graphs of the following functions:

f (x 1);
y
(b) y = f (x12);
(c) y=== If (x) I;
(d) y == (I f (x) I f
(e) y=== f (x) Ilf (x).)

(a)

===

(x\302\273/2;

Limit of a Sequence
g 1.6.NumberSequences.

..>

\"',

The number a is called the limit of a sequencex\" X2,


Xn ,
as n -+00,a = lim Xn if for any 8 > 0 there existsa number
n-+

\037

Ixn-a1<

8 holds true for all


N (e) 0 such that the inequality
N (e).
n
A sequencewhich has a finite limit is said to be convergent.
A sequence{xn} is calledinfinitely small if lim X n ==
and infi.
X
if
lim
nitely large
n
Given the generalterm of the sequence{xn

>

===

0,

00.

1.6.1.

}:

x=
n

Write

the

first

five

sin (nn/2)
n)

terms of this

Solution.Putting consecutivelyn

term

xn ,

we obtain)

Xl
X

sequence.

==

1,2, 3, 4, 5

= sin t/2) = 1;

2-

sin (231:/2)

--,.
O

= sin (331:/2)
= \"31 ;
3
sin
= 0;
X4 =
sin (531:/2) =
Xi =
5.)))
5
xa

(\037:rt/2)

In

the general

Ch.

42)

1.Introduction

to Mathematical

Analysis)

1.6.2.
Knowing the first several terms of the sequence,write one
for the generalterm:
possibleexpressions
2 5 10 17 26
(a)
'8' 13'18' 23;

of the

3\"

3'

5'
I

3, 4\"' 4,
(b) I, \"2' 2,
Note. A knowledgeof the first several terms of a sequenceis not
That is why this problem should
sufficient to define this sequence.
be understood as one of finding a certain simpleinductive regula1

rity

the given

with
compatible

terms.

Solution.(a) Note that the numerator of each of the given terms


of the sequenceequals the square of the number of this term plus
2
unity, Le. n + 1,while the denominatorsform the arithmetic progression3, 8, 13,18, with the first term a ==3 and the com-

mon difference
an

...

d ==5. Hence,

=a1+d(n-I)== 3+5 (n-l)==5n-2,

thus we have)
Xn

n2

=5n-2')

(b) Herethe general term of the seq.uencecan be written with


the aid of two formulas: one for the terms standing in odd places,
the other for those in even places:
Xn

==

{ I/(k+I)

atn==2k-l,

at n = 2k.

is also possibleto expressthe generalterm by one formula,


for instance,
which will be more complicated,
It

Xn

= nt 1 [l-(-l)n]
+

n\0372

Find the
1.6.3.

[1+ (-l)n].

first several terms of the sequence


is given by one of the following formulas:
(a) X n = sinn(nn/3);
(b) X n = 2- casnn;

term

if

the general

(c) Xn =(1+ I/n)n.

1.6.4.
Using the

(a) lim x n = 1

if

definition of the limit of a sequence,prove that

xn =(2n-l)/(2n+
1),

+ I)/(5n2-1).
Beginningwith which
< 0.01fulfilled?
xn-3/51
Solution.(a) For any 8 > a let us try to find a natural number
N (8) such that for any natural number n > N (8) the inequality

(b) lim Xn == 3/5 if


n is the inequality

xn-II< E

is

Xn

fulfilled.)))

== (3n2

1.6.Number

\037

For this purpose let us

of

absolute value of the difference

2n+-1

the

find

-1/<
1

43)

Sequence)

- - 2n+1
-2 _
-2n+I'

2n
1

Limit
Sequences.

<
1/8-

8 is satisfied if 2n 1 8, whence
1
n > I/E-l/2'Hencethe integral part of the number
/2 may
be taken as N (E), Le. N = E (l/E_1/2),
So, for each 8 > 0 we can find a nUITIber N such that from the
< 8, which means that
inequality n > N it will follow that
Thus, the inequality

Xn

\037

xn-11
.11m 2n. = 1.
n-+oo 2 +

n)

(b) Let us

the absolute value of the difference

find

3n2
5n 2

Let 8

8
+ 3
-1
5' =5(5n-1)'
2

8
5 (5n2
is fulfilled.

Solving this inequality, we

so that the inequality

-1)< 8.)

fin

1.

8
> 258
+ '
\"5

> 2-,/8+58
e
5 V

\302\267)

Putting)

N=E (
we

concludethat

at n

-3/5I:

> 0 be given. Choosen

n2

I Xn

\037

>N

).

8\0375g

Ixn -3/51< 8,
the
which completes
If E =
then

proof.

0.01,
N

=E (

) =E( Y 805) =5,


the sequence,beginning with the 6th, are contai\037

8\03758

\037

and all terms of


ned in the interval
Given a sequencewith

(3/5-0.01;
3/5+0.01).

1.6.5.

known

that

lim

n-+oo

Xn

the generalterm xn =
1/3.Find the number of points Xn

side the open interval


L=

. It

is

\037\037+\037

-}l

o\037o

;
\037

\037oo

).)))

lying

out-

Ch.

44)

/. Introduction

Solution. The distance


equal to
X ll
I

to Mathematical

Analysis)

from the point

--}1=1---

Xu

to the point 1/3 IS

1=3 (9:/+4) '

3 (9\0379+4)

Outside the interval L there will appear those terms of the seLe.
quence for which this distance exceeds0.001,
19
3 (9n 4)

+ >

I
I

000

')

whence)
1

\037

== 703 7
< 18988
27
27

\"',

Hence,703 points (xl' X 2,


L.

X 703)

val

\302\267

are found outside the inter-

- -

Prove
1.6.6.

that the number l ==0 is not the limit of a sequen2


Xn == (n 2
2)/(2n 9).
Solution. Estimate from below the absolute value of the diffe-

ce with

the generalterm

rence)

n2-2

2n 2

-9

n2-2\\ '
= 2n2
9
I

3 the absolute value of the difference remains


At n
than the constant number 1/2; hence,there existssuch E
E == 1/
2, that the inequality
\037

greater

> 0, say,

-0 > +

2:22-=-\037

holds true for any n 3.


The obtained inequality proves that l = 0 is not the
ven
gi
sequence.
\037

.6.7.Prove

that the
I

sequence
2

the generalterm

xn == I/n,
has no

of the

2' 3'3'5' 4' 7' 5' ...)

1,
with

limit

{ nl(n + 2),)

limit.

= 2k

- 1,

if

if

n ==2k,)

Solution.I t is easy to show that the points x/l with odd numbers concentrateabout the point 0, and the points Xll with even
numbers,about the point 1.Hence,any neighbourhoodof the point 0,
as \\vell as any neighbourhoodof the point 1, contains an infinite
set of points Xn' Let a be an arbitrary real number. We can ahvays
choosesuch a small > 0 that the e-neighbourhoodof the point a \\vi
\302\243

11)))

1.6.

Sequences.Limit

/\\'ullzber

.\037

of

45)

Sequenf.-'I!)

not contain at least a certain neighbourhood of either point 0 or


Then an infinite set of nUlnbers Xn will be found outside
point
this neighbourhood,and that is why one cannot assert that all the
numbers Xn , beginning with a certain one, will enter the e-neighbourhood of the number a. This means, by definit ion, that the
number a is not the limit of the given sequence.
But a is an arbitrary number, hence no number is the limit of this sequence.)

1.

1.6.8.
Prove that Urn
1.6.9.Prove that lim

== 1

Xn

if X n

+ 1)/3n .
X n == (2n + 3)/(n + I).

== (3n

Find the
inequality
0.001,is fulfilled.

== 2 if
beginning with
< B, where E ==
I (2n + 3)j(n+
Prove that the sequence

the

of

number

1.6.10.

Xn

term

1)-2/

the

which

0.1;0.01;

2'2' 4'4' 8' 8\"'\


]

with

the general term


(
Xn

== {

2(n + 1 )/2

n IS

if

n IS even,

2n / 2

I
\\

odd,

if

has no limit.)

1.6.11.
Prove

xn ==an In!

Solution.Let

=
\037\037

\302\267

\037

\302\267

\302\267

that

=
\037

\302\267

\037

\302\267

\302\267

\302\267

\037

n- k

if

> 2a.Then

)(

\302\267

k\037

< (+ ) = (2a)k (
ak

a>O limxn=O

arbitrarily large

a natural number k

\302\267

\037

at any

k\0372

n
\037

at
\302\267

n
\302\267

\037

>k
)<

Sincelim (1/2)n==0 (prove it!),then at a sufficiently large


n
have:
< \037
and, hence, anInI
. < E, which means
2
)
(
(2a)k
lim (an In!)==O.
Test the following sequencesfor limits:
1.6.12.
(a) xn == 1/(2Jt);
1

(b ) x n

---{ Iin
I

for an even n,
for an odd n;

nn

-(-I)ll]

(c) Xli ==n COS

\"\"2

(d) xII ==n [ 1

.)))

we
that

Ch.

46)

Prove
1.6.13.

/. / ntroduction

to M athemalical

Analysis)

that the

sequencewith the general term


x n = l/nk (k>0)
is an infinitely
small sequence.
Solution.To prove that the sequencexn is infinitely small is to

pro\"

lim

that
n

\037

Xn

= O.

(S)

Take an arbitrary e > O. Since x n


inequal ity)
I

lInk

>. V lIe. HenceN may


V l/e,1.e. N = E (Vl/e).

whence n
of

= link,

we have to solve the

< B,
as
expressed

be

1.6.14.
Prove that the sequenceswith
=
, (b)
(a) x = 1-<;I)n
Xn

the integral part

the general terms

sin
\037

\037

--.

00.
are infinitely small as n
the sequence
Show that

1.6.15.

[(2n-l)]
the

with

general term

Find a numis infinitely small as n--.oo.


xn=(-1)n2/(5Vn+l)
ber N beginningwith which the points X n belong to the interval

(-1/10,
Ilia).
Solution.Take an

Ixnl=5
That is

why

1 Xn

I\"

take now

<5

-t- 1

V-n

<2

estimate X n I:
I

-'

V-n V n

1<e a; soon as n > 1/B3.HenceHm

Xn

= O. i.e.

fl\037oo

small.

the sequenceis infinitely


We

221

arbitrary e > a and

e= 1/10.Since Xn 1<I/Vn , Xn will necessarily


1/10if I/Vn < 1/10or n > 1 000.HenceN may
I

be smallerthan
be taken equal to I 000.But we can obtain a more accurateresult
by solving the inequality
I Xn I

V-n+

< 10
I

\302\267

3 = 54.872.
= 3.8
Hence N may be
holds true at n > (19/5)3
taken equal to 54 \037 I 000.
t 6. It is known that if X n = a + an) where an is an infinite1.6.
simal as n \037 00, then Iim X n = a. Taking advantage of this rule,

It

find

the

-+

00)

limits:
3n + 1+ sin
(a) xn =
3n

(nnj4)

')

b)

Xn

= 2n+(-l)n
2n

\302\267)))

1.6.Number

\037

. (a)
Solutton.
an

infin

Xn

Limit
Sequences.

= 3n +1+3sinn

-+

-+

47)

Sequence)

where an

= sin

= 3.

Xn

(nnj4) IS
n

(S))

V n = 1.)

Hm
n

= 3 + an,

(nnj4)

itesimal as n ---+00,hence Iim

1.6.17.
Prove that

of

00)

Solution.Let us prove that the variable V n can be represented


as the sum 1 + an, where an is an infinitesimal as n ---+00.
Let us put iY n = 1 +an . Raising to the nth power we obtain

n _ (1

+ an)n _ 1 + nan + n (n2'

1) 2
an

wherefrom we arrive at the conclusionthat


lowing inequality holds true:
n

+ ... + an,n
for any

n> 1 the

fol-

2
> l + n(n-l)
2r
an)

(since all the terms on the right are non-negative).Transposingthe


1 we obtain
to the left and reducing the inequality by

n-

unity

> \"2 a'H


2/n >
2

or
whence it follows that
==
lirn
also
lirn
V 2/n 0,
equals zero,
an
n-+oo
n-+(S)
that)
Henceit follows
a\037

Ii In
f1

Prove that
1.6.18.

-+

the sequencewith

large as

infinitely

us

the inequality)

Since

infinitesimal.

n = 1.)

iY

== 3 V

-.00.
Xn)

Solution.Let

> an > O.

00)

is

V 2/n

i. e.an is an

,.-

the general term

take an arbitrary positive nurnber


3V

M and solve

;;>M.

Taking the logarithm, we obtain

Vn>log.1

1\\;1,

If

we

now

lity 1 Xn
infinitel y
I

>M

take
will

large.

> (logaM)3.

== E (log3M)3, then for all n > N the inequabe fulfilled, which means that the sequenceis

LV

1.6.19.
Prove that)
liI]]
n-+oo)))

va

(Q

> 0).

Ch.

48)

\037

i. introduction

to Mathematical

Analysis)

1.7.Evaluation of Limitsof Sequences


If the

sequences{xn} and

(1) Hm (xn + Yn) = litll

Xn

are convergent,

{Yn}

:f:lim

(2) Iim (xnYn) == Iim Xn lim Yn;


lim Xn
Xn
(3) I 1m Yn = r1m Yn (I1m Yn =1= 0)
If X n
Iim Yn'
Yn then Iim X n
e

then

Yn;

'

\302\267

\037

\037

.7.1.Find

Iim
n-+oo)

X n if

+5n+4; (b) Xn ==5n 3+2n 2-3n+7 ;


II
4n3-2n+
4n 2 -4n+3
(c) xn ==2n 3 +3n+4; (d) xn =12+22+...+n2.
5n3+n+
(a)

(e)

Xn

Xn

== 3n

== 1

2+n2

+ 2+ ..+n .
n

')

\037

Solution.(a) xn ==

3+\037+\037
n
n2

+
lim
n
xn =
n2

Ii m
n -+ 00

00.

(d) Recall that

+4jn 2)
== 3.
2
)
_+
)
(n

(3-1-5/n

-+

hm
-+

00

2)

+ 22 + 32 + ... + n 2 = n (n+

12

I)
1\037(2n+

Hence)
Xn ==

(n+ 1)(2n+ 1)

6(5n3 +n+l)

Hm

1.7.2.
Find
(a)

!imXn ,

+n

xn = 1/15.)

30_L\037
I
n2

\037'
/1,3

if)

+n-2 3.'

( 4n2+2n+7)
(c) Xn == iY 5n;
(e) Xn == V n 5;

S'

+n+fi2
+n+I)- +

2n 3 +3n2
6(5n3

n-+oo)

3n 2
Xll\037

n-+oo)

.)

n/-

olutton. (a)

2n3+2n2+1 4 ;
(b) Xn == ( 4n 3 +7n2+3n+4
)
(d) Xn == iY
Xn

== vn/

n\037;

6n+3.

(f)
3
. 3n2+n-2
==
4n2+2n+7
)
(
' 3n2+n-2 3n2+n-2 3n2+n-2
=

}\037\037

-= .
nl\037\037

( 4n2+2n+7) ( 4n2+2n+7) ( 4n\037+2n+7 )

=
(}\037n:.

\037\037\037\037\037\037\037\037\037:

r=( r=
\037

\302\267)))

\037:

J .7. Evaluation

\037

49)

of Limits of Sequences)

(c) In solving this example,and also the rest of the examples


take advantage of the following equalities(see
of Problem
and t
Problems

1.7.2,
1.6.17 .6.19):
liln
n -+

1 and

iY n

iY a

lim
n

00)

-+

I.)

(1))

00)

We have)

but

follows

from (1) it
== 1 1 ==

. I.
1.7.3.
Find)

Iim

Iim iY n,
V'5n-+oo)

iY 5n == Iim

lirn Xn == Iinl
n-oo
n-+oo

n-+oo

lim iY 5

that

I and lim

V\" n

I; hence

Xn

. (2n3 + -5n2
2n2+3 5n+ )
1

Solution.Summing

fractions, we obtain

the

2n 3

xn == lOn 3
Whence)

Xn

Note. If we

==

put)

2n 3

3
== 2n
2

Yn

2 +3
-13n

+2n2 + 15n+3')

lOn 3

nl:\037

}\037\037

\302\267

\\

}\037\037

2n

2 +3
-13n

==

+2n2+ 15n+3

51

2
1-5n

zn==5n+I')
Jim (Yn + zn) == 1/5, though
3

;)

each of the
large quantity. Thus, from the convergenceof a sum of sequencesit does not, generally speaking,follow
that the summands converge too.
then

the limit of their

summands is an

surn

infinitely

1.7.4.Find nIim
-

Xn

if)

00)

2n+3-Vn-l;

(a) xn ==V

I- -n+1;

(b) Xn = Vn2+n+

Vn

(c) xn==n 2 (,.n-Vn 2 +1);

(d) xn=V n2-n3+n;


Vn2+ 1+Vn ;
X ==
3

(e)

..r-

V n 3+n- r

I-2+3-4+5-6+...-2n
.'
1

(h)

V(n+ 1)2-V(n-l)2;
Y4n2-1
Vn2+1+
xn ==D + 2.3 -t3.4 + . + n (n + I)

(f) Xll ==
(g) xn ==

\302\267

\302\267

\302\267)))

1.Introduction

Ch.

50)

to Mathematical

Analysis)

Solution. (a) xn =Vn(V2+3/n-Vl-l/n)---.


+00as fl---+00,
since the second m ultip lier has a positive limit.

- (n-y
n-

(c)

n2

n2

--n+-n +1

+1)

n2

(n2_ n 3)2/3_n

-+

I
\302\267

(d) Xn =

Y n2

+V

V n 2-n3 +n

00 as

-+00.

\0372)

2)

--I) -( --I)
(
1

2/3

It

means, x n ---.1/3.

(e) Factoringout the terms

of the

- n( V/

Vii

iIn 3+n-V

n 3/ 4

n)

( V I+\037- V

)
\"/l+\037+\"/\037
V
V

the numera-

\037

-++
V

n\037

\037

Find
1.7.5.

in

\302\267

+1

I+\037+ V\037)

= n 1/4
Iim

1/3

n)

highest power

tor an d denominator, we have:

Vn2+1+
xn-

fl

I+\037n2

00 as n --+ 00.)

n)

X n if

n\037oo)

vn
V n+l+vn

(a) X n =
(c) xn

(b)

Xn

=:.;)V, n

(d) X n =

2n
n
n2

+4n

n3

=Vl-n +n;)

-3n2 ;

cosn 3

n
+
(e) xn= 2n2_l coS2n_I I_2n (-I)n
+1 ;
1+ + +\",+ 2n
= 2 4
2n

(f)

Xu

-+
3n

6n

\302\267

1+3+9+\"'+
3
n)

\037

1.8.TestingSequences
for Convergence
Bolzano-Weierstrass' theorem.A monotonic boundedsequencehas

limit.
HIn Xn = Iim

a finite

Theorem on passing to the limit

and

fl

--+

oc)

+ 00 or

00

\037

Zn

==c, then

n\037oo)

00)

but

in

lim

not

00).)))

inequalities.If
Yn

=c

Xn

\037

Yn

\037

zn

too (c is a number,)

\037

= (2n

prove

that)

1.8.
t. Prove
1)/(3n

1.8.Testing

Sequences tor Convergence)

the sequencewith

that

+ 1) is an increasingone.

Solution.We have to prove

that

X n +t

+ 2n
+4 > 3n +

2n
3n

51)

the general term

>

Xn

Xn

==

n, Le. to

for any

I
\302\267

The latter inequality is equivalent to the obvious inequality


6n2 +5n+ 1 > 6n2 + 5n-4.)

Hence,Xn+l
t

> Xn'

.8.2.Given a sequencewith

the general term


Ion

Xn

==!if
at n
sequencedecreases

Prove that this


Solution.)

X n+l

Since

n\037

<I

==

at n

IOn

\302\267

10.

liOn
0
10
=-e-==X
I

(n+l)f

n+1

nJ

10, then

\037

\037

xn+l

<x

n+l'
beginning

with

at n
number, which means that the sequencedecreases
Test the following sequencesfor boundedness:

\037

1.8.3.

(a)

Xn

this

10.

5n 2
= n;a+3
;

2n

(b) Yn=(-l)n
n+1sinn;

(c) zn = n casnn.
Solution.
(a) The sequence{xn} is bounded, sinceit is obvious that
5n 2
0< n 2+;j<5 for all n.
(b) The sequence{Yn} is bounded:

IYnl=I(-I)nl' lsinnl< n\037l <2.


n\037l

(c) The sequence{zn} is not bounded, since

1.8.4.
Prove
Xt

(a >

-+

x0

\302\267

')

Xo

that the
x2 ==a

Xl

Zn

== Incusnn ==n.

+Xl

')

1, > 0) converges.
Xo

Solution.Let

sequence)
X3

us prove that
x n l as

bounded. Firstly, x n

< -

X\"

a + X2
this

Xn-l
xn = a +
Xn-l)))

. ... .

')

,)

Xn

== Xn-l

Xn

'

\302\267

-1)

sequence is monotonic and

< Xn-l'

CIl.

52)

I. Introduction

to Mathematical

Analysis)

Hence,the given sequenceis a decreasingone.Secondly, all its


terms are positive (by condition a > 0 and Xo > 0), \\vhich means
that the sequenceis bounded below. Thus, the given sequenceIS
monotonic and bounded, hence it has a linlit.
Prove that the sequencewith the general term

1.8.5.

Xn

(i.e.Xl=

5\0371

+..+

+ + + + 53+
; X3=
+
X2=

==

5\037

52\0371

5\0371

\302\267

5n -1-1

+5Tn+

5\0371

53\037.I

...)

converges.
sincexn+l ==Xn + 1/(5n 1 -:-1)
Solution.The sequence{xn } increases,
it
isbounded
and, hence,xn+l > Xn' Besides,
above, since1/(5n + 1)<
< 1/5 at any nand
4-

fl

xn = 5

]
+ +

+ 53+ + + 5n + <
1/5 /5n +
=
+
+\"'+
2
n
3
5
<S+ 5 5
1-1/5 =\"4(

+I

5\037

\302\267

\302\267

\302\267

1- )
1

5n

<\"\"4'

the sequenceconverges.
of a
Taking advantage of the theorem on the existence
the following
limit of a monotonic bounded sequence,prove that
sequencesare convergent:
Hence\037

1.8.6.

-1

(a)

Xn

= n 2n 2

(b)

Xn

== 2

+ 21 + 3f +

Prove
1.8.7.

that

limits:

\302\267

\302\267

\302\267

+ n!

\302\267)

the following sequencesconverge and

(a) x 1= V2; x2 ==V2

find

+V2;

X3

-(2 + V 2 + V2;

Xn

(C)

xn=E(n y ) ;

2;
+ V2 + v + V-)
\302\267

\302\267

\302\267

n radicals)

...;

n)

...
< ...,

(d) the

sequenceof successivedecimal approximations 1; 1.4;

1.414;
1.41;
==
Il + 1

that

it

r-2;

of the irrational number J;

(e) X n n!jnn.
Solution. (a)

prove

Xn

= (n +2n2)1 ;

(b)

<X

...; 12'-)

their

It is obvious that Xl < x2 < xa <... < x n <


Le. the sequence is increasing.It now remains to

is

bounded.)))

\037

...

1.8.Tesfin[!, Sequences

53)

for Convergence)

...

=V2<2
.

have xn ==J 2 -txn -1, n== 2, 3,


Since x1
x
< V2-r-2==2,Xa== V2 + 2 < V' 2+2===2, Let it
be proved that Xll -1 < 2. Then xn==V2+Xn_l< V2+2==2.
Thus,
with the aid of mathematical induction we have proved that xn < 2,
Le. the sequenceis bounded. Hence,it has a finite limit. Let us
it. Denote)
find
We

X 2 == \037/-2-t-Xl

Iim X n ==y.
12-00)

Then, x n = V2-j-x
n _ 1; raising to the second power, we obtain
x\037==2+Xn_l'

Passing to the limit, we can rewrite this equality as follows

(2+xn _ 1),

y2=2+y.

or

lim
12-00)

Iimx\037=
n-CYJ

The roots of the obtained quadratic equation


Yl

= 2;

Y2

-1.

are:

The negative root doesnot suit here,sincexn>O.Hence,HIn Xn =Y1=2.


'l-oo)
E (ny)
E (ny) ny or y-\037
(c) We have
y. But the
n
n

ny-l <

and
{y}
x ==y.

sequences
why

Hrn

{y}

\037

<

\037

converge,their

\037

limit

being y, that is

n-oo n
(d) This sequenceis non-dec-reasing,since each following term
is obtained from the precedingone X n by adding one more
xn+l
significant digit to the decimal fraction. The sequenceis bounded
above, say, by the number
Hence,the sequenceconverges,its
limi t being V2.
(e) The sequencedecreases
monotonically. Indeed,)

1.5.

xn+l

(n

+ I)! ==

(n+ l)n+l

n!

(n+ I)n

= nnn!

nn
\302\267

(n+ l)n

= (n+nn l)n

Xn')

nn
Since(n+
< 1, xn+l < Xn'
l)n
Then, sinceXn > 0, the sequenceis bounded below, hence Iim

exists.Let
show

that

us
l=

1.Obviously, l =

denote it

0. Indeed,

(n+l)n =
nn

n+1 n =
n

lim
n-+oo)

Xn

\037

0.

_
Now let

1+nJ..=2.
n:;::;-

( l+\037)

n\037

n)))

Xn

00)

us)

I. Introduction

Ch,

54)
nn

to Mathf'matical

<

Hence,(n+ l)n <\"2and

xn+l

I
\"2

Analysis)

Passing over to the limit.

xn'

we obtain)

which, together

with

0, brings us

\037

1.8.8.

2'l,

\037

to the conclusion:

O.

===

limits of the sequenceswith the following gene-

Find the
ral terms:)

x==
n

+n '

z===
n

\302\267

V n2

V n2 +

Yn

===

Solution.Let

Ixn

Jl n 2

+1 + V n2+2 +

us prove that

-1/=
_

We can prove

n2

Iim
n-+(S))

Xn

+ ..r
, n 2+n
1

\302\267

\302\267

\302\267

\302\267

1.Indeed,)

===

= n-1
+n

Vn 2

+.'l _

y n2+n

<\037.
2n

JI n.2+n (n+ JI n2+n)

similarly that
Iim

Zn

1.

===

n-+OI)

Then,)

Yn<

Jln2 +1 + Vn2+1
1

+...+ +1
-'\"

Vn 2

\037

n\037

===

Z n)

n)

On the other hand,

> V n2+n + y n2+n +


1

Un

l.

'-)

\302\267

..

\302\245

Thus,)
Xn

< Yn < Zn'

n2+n
.b.--

\037
n)

lirn X n
n-+oo

==

Iim

V n2+n

Zn

n-.oo)

.8.9.Using the

-+

Yn==

Iim
n-+fJ'))))

in

inequalities

1.)

f7i)

theorem on passing to the limit

prove)

n)

== 1

and accordingto the theorem on passing to the limit


lim

==x..

\037)

,-

n
if a

===

(a > 0).

In

inequali ties

\037

1.9.The Limit

1.8.10.
Prove

(a> I)

and

the existence
of
calculateit.

of

55)

Function)

the limit of the sequenceYn ==a 1/ 2n

1.8.11.

Taking advantage of the theorem on the limit of a


monotonic sequence,prove the existenceof a finite limit of the
sequence)
Xn

== 1

+ 22+ 32+

\302\267

\302\267

\302\267

+ n2

\302\267

1.8.12.
Taking

advantage of the theorem on passing to the limit


inequal i ties, prove that

in

+I-n}.

2
xn==1 if xn==2n(Vn

lim
n

00)

.8.13.
Prove that

sequence

the

=: a + V a;
Xl = Va; xa
v\"
xa

-(a +

Jir a

+ Va;

...,

xn =

+ ... + V
-(
'-)a + Va-.-\
a)

n radi

(a > 0)
has the

lirnit

that the
Xn

fini

te limi

t.

==

sequencewith the generalterm


I

3+1

\037t-

32

+...+ +n

+2

1.8.15.

3n

Prove that a sequenceof lengths of


2n -gon\" inscribed in a circletends to a limit
circumference).)
\037

Is)

b==(V4a+I+I)!2.

1.8.14.
Prove
has a

ca

perimetersof regular
(calledthe length of

1.9.The Limit of a Function

point a on the real axis is calledthe limit point of a set X


any neighbourhood of the point a contains points belongingto X
which are different from a (a may be either a proper or an improA

if

per point).
Let the point a be the limit point of the domain of definition
X of the function f (x). The number A is calledthe limit of the
function f (x) as x \037 a, A == Iirn f (x), if for any neighbourhood V
x-a
of the number A there existsa neighbourhood u of the number a
such that for all x E X lying in u, f (x) E V (the definition of the
limit of a function after Cauchy). The number A may be either
finite or infinite. In particular, if the numbers A and a are finite
we obtain the following definition.)))

Ch.

56)
A number
A == lim f

I. Introduction

to Mathematical

is called the

Analysis)

of a function f (x) as x \037

limit

a,

> 0 there existsa number l) (8) > 0 such


x-+a (x),
that for all x satisfying the inequality 0 < x-a < I) and belonging to the domain of definition of the function f (x) the inequality
defini tion\.
f (x) A < 8 holds true (the
=
A is
A
if

for any

- + 00,
i

\"E-I)

the definition is as follows.

If

called

number

the limit of a function f (x) as x -++ 00,A = litn f (x), if for any
x -+ +
0 such that for all x satisfying
E > 0 there existsa number M (8)
the inequality x > M (E) and belongingto the domain of defini tion
of the function f (x) the inequality f (x) A < E holds true (the
00

>

definition\.

\"E-M

The notation Iim f (x)= 00 means that Hm f (x)1=+ 00. The


x-+a
x-+a
rest of the casesare consideredsimilarly.
The definition of the limit of a function after Heine.The notation lim f (x)= A means that for any sequenceof values of x conI

x-+a

verging to the number

...,...)
...; ...

a)

Xl' x 2,

Xn ,

(belongingto the domain of definition of the function and


sequenceof values of Y
a) the corresponding
= f (X n ),
= f (Xl); Y2 = f (X2);
Yn
Yl

from

has a
t

limit, which is

.9.t.

differing

the number A.

Taking advantage of the definition of the limi t after


of the theoremson the limits

Heine (i.e.in terms of sequences)and


of sequences,prove that)
. 3x I
hm

x-+25

X)

+ =
+4

2'
1

'

......

Solution. Let us consider any sequenceXl' x2


satisfying
the following two conditions:(1) the numbers Xl' X2,
belong
to the domain of definition of the function f (x) = (3x 1)/(5x 4)
(Le. Xn =1=-4/5);(2) the sequence{xn} convergesto the number

Le.

IiIn X n ==

n-+oo

2.

2,

To the sequence{xn } there corresponds the sequenceof values


of the function)

+ .' 3x2 + .'


+ 5x2 +4

3x} 1
5xI 4

.
...,)

1.7),
1
3xn+ == (3xn + 1)= 6+
1m (5X +4)
5Xn +4 1'
=
210+4

proceedingfrom the theorem on the limits (9

.
hm

-+

00

f(xn )== nIUTI


-+

00

lim

n)))

\037

1.9.The

Limit

of

Function)

57)

Thus, independently of the choice of a sequence{xn } which


the correspondingsequences
converges to the number 2 (xn=l=-4/5),
of values of the function f (xn ) converge to the number 1/2,which,
according to the definition of the limit of a function, means that
I

3x

x:n;5x+4=2\"

\302\267

Note. The definition of the limit after Heine is conveniently


applied when we have to prove that a function f (x) has no limit.

is sufficient to show that there exist t\\\\'0 sequences


== Iim
= a, but the corresponding
such that litn
n_oo
n-oo
and {f
do not have identical limits.
sequences{f
Prove that the following limits do not exist:

For this
and

it

{x\037}

x\037

{x\037}

(x\037)}

x\037

(x\037)}

1.9.2.
Hm 21/%; (c) Hm sinx.
(a) Hm sin \037;
.-:-1 x- (b) .(-0
.(-00
Solution.(a) Choosetwo sequences
xn = 1

for

and

n\037

x\037=

which)

Hm X n
n-oo

(n= 1,2,

(4n\037 1)1&

Iim

x\037

...),

= 1.

n-oo)
The correspondingsequencesof values of the function
f (xn ) = sin 1

and

'

'

f (X n ) == SIn

I/(\037Jt)

+ 2/[(4n+1)n]-I

Hence,)

lim f (Xn ) == 0
XII

follows that

and Iim f
X'n)

Le. the sequences{f (xn )}


it

_I = sin n3t = 0
. 4n + n = SIn. 2nn +
= SIn

and {f

(x\037)}

.....1

(x\037)

...),

Hm

n_oo

Xn

Urn
rl

.....

Hm sin
n-oo

... sin x

lirn

(X.)))

Iim

sin xn =

x\037

does not exist.)

x\037

n-oo)

00

and)

=1

\302\267

= 1,

have different limits, whence

x-I x-

for which

11
-\037;f

sin \037
] does not exist.

Iim

(c) Choose two sequences,Xn = nn and

2,

are:

lim
n-+CIJ)

= 00.Since)

Jim
n

sin nn

x\037

==2nn

== 0,

(S))

sin (2nn + n/2)=

1,

+ n/2

(n ==

1,

i. introduction

Ch.

58)

to Mathematical

Analysis)

Note. The above examples


show that one cannot draw the con-

clusionabout the existenceof the limit of a function proceeding


from the sequenceof values of x of a particular form (for example,
proceedingfrom X n == 1 + 2/((4n + 1)n) in the item (a) of this problem), but it is necessary to consider an arbitrary sequenceXl'

...
...,
.9.3.
xn ,

x\037!'

having a given limit.)

Proceedingfrom the definition

after Cauchy

(Le. in

the

-5;
. 5x+ 5
(b) x-+oo3-L9=3;
.
(a) Iim
x

\037

of the limit
\"E-M\" ,

\"E-6\";

of a function
that

etc.),prove

(3x-8)

===

hm

(c) x

terms of

(l-x)2= + 00;
1

l\037\\

(d) Iirn logax == 00 (a


x_
(e) Hrn arc tan x n/2;)

> I);)

00)

===

X -+

(f)

ex>)

Hrn
x 3t/6)

sin x =

1/2.

Solution.(a) According to the

\"E-6\"
definition we are to prove
there exists6 > 0 such that from the inequality
it follows that If(x)-(-5)/==lf(x)+5/<E.
In other words, it is necessary to solve the inequality
3x 8 + 51==31
< E.

that for any

Ix-I/<6

>0

x-II

The latter inequality showsthat the required inequality If (x)+51<E

is fulfilled as soon as

== -5.
Hm (3x-8)
x-II< E/3 ==6. Hence,x-I

(b) According to the \"E-M\" definition of the limit one has to


show that for any 8 > 0 it is possibleto find a number M > 0
such that for all x > M the inequality)
5X
1

+_ <
+ 3
1

3x 9

8)

be fulfilled.
Transforming this inequality, we obtain
5X + 1
5 =
14

will

3x+9-3
I

/3x+9/< 8.

Sincex > 0, it remains to solve the


14

3x+9\"

8,)))

inequality)

(*))

J.9.The Limit

\037

whence)

hence M =

X>

of

Function)

14-98
38)

14 98

\302\267

3E

59)

14 98

Thus, for E > 0 we have found M = 38 such that for all


values of x > M the inequali ty (*) is fulfilled, and this means that

\037 00

+ =5
+ \"3

5x I
3x 9

then M=
Let, for example,8=0.01;

\302\267

-0.09

14

32

0.03 =463 ,
(c) We have to prove that for any K > 0 there exists8 > 0
such that from the inequality

Ix-II<8
there always follows the inequali ty
I

(l\037x)2

= (I

> K.

\037X)2

Let us choosean arbitrary number K > 0 and solve the inequality

/'
- ':.
I

(I

X)2

**
( )

,)

whence)

II-xl<YK
I

Thus,
soon as

if

we

put

l) =
\037K

' then

x-II< l), which means

(K>O).
the inequality (**) holds true as

that
!i\037nl

(I \037X)2

= + 00.

(d) We have to prove that for any K > 0 there existsM > 0
such that from the inequality x > M there always follows the inequality logax > K. Let us choosean arbitrary number K > 0 and
consider the inequality logax > K. If we put aK = M, then at
x > M the inequality loga x > K holds true. Hence,
Hm
X-++OO)

logax== + 00.

1.9.4.
Prove that Hm cosx does not exist.)
1.9.5.
of the roots of the equations sin (l/x)=1
Using the sequences
==

and sin
limit

(I/x)==

as x\037

O.)))

-I,

X-+oo

show

that

the function f (x) sin (l/x) has no

Ch.

60)

Introduction

,__

1.9.6.Proceedingfrom
ion prove

funct

(a)

that:

(b)

- =3;
>
+
--.

t-lirn+

aX
00)

sin x

(g ) I 1m
x _

x)

00

\037

00 (a

-==

x-I

(d) lim
X-.O)

I
(e) t:\03700
3x+2

(f)

of a

- y_x- = 2;

lim
x

limit

cosx = I;

(c) tIim
- 0 sinx=O;
2x

Analysis)

Cauchy's definition of the

==
Iirn
x-I)(3x-2) 1;

to Mathematical

1);)

1.10.Calculation
of Limits of Functions
I. If

the limits

lirn u

x-a

(x) and

theorems hold true:


( 1)

Ii rn

x-a[u (x)

liIn v (x) exist, then


t-a

the following

Ii m u (x) + Ii nl v (x);
+ v (x)}= x-a
x-a

= Hm u .limv
(2) lim
x-a[u (x).v(x)] t-O (x) t-a) (x);
(3)

I 1m

x_aV

' v (x)
-au (x) (I1m
= r1m
x
()
()
x_a
x-a)
(x)
X

Urn
x

=1=

0)

I.

I For all ma in elementary functions at any point of their domain of definition the equality Iim f (x) = f (lim x) ==f (a) holds true.
x-.a
x-a
I I If for all val ues of x in a certain neighbourhood of a point a
(exceptfor, perhaps, x = a) the functions f (x) and (x) are equal
and one of them has a limit as x approachesa, then the other one

I.

cp

has the same limit.


IV. The following limits are frequently used:

= 1;
1m
(1) x0 x
.
Iirn (1+ l/x)X == lirn (1+ a)I/'X = e = 2.71828.
(2) x-oo
a-O)
toga (l +x)=
Iirn
logae (a > 0; a =F 1);
1

(3)

sin x

t-

In

(4) lim

X-.O

.;

(I

+x) = 1;

aX-l =lna

(5) lim
x-.O

x)))

(a>O).

\037

t.to.t. Find
(a)
(c)

\037\037

+ 9x+7
+x3 + ;
x+
'
V + 3 + 3x

.t

2X

x-I

- _X+
+

3 _X2
X3 3

_2

-_x3-x-6
I

. .

( J) I 1m

x-

(p an d q

integers);

x- ;
10-;-2

.t-3[toga

61)

+3x2 -9x-2.'

(h) lim

2 ;

X)

xP
xq

(f) Hrn

-3 ;
r
x+6-2V3x-S
v\037-3

xl\037

!irn
x-+ 2

6x\037

V9+Sx+4x2_3
;

.t-O

. .

(d )

\302\267

x3

lirn
(b) x-)
2

--

(1) hm

limits:

the

3X6

(e) I 1m
(g)

Limits of Functions)

of

4XD

Ii nl
x

.10.Calculation

x-3

..r'

V x

+6-3 J ;

x+8-Y8x+

S-x-r 7x-3)

Solution.(a) Sincethere exist I imits of the numerator and denominator and the limit of the denominator is different from zero,
we can use the theorem on the limit of a quotient:)

+9x+7) 4+9+7
=
=
=4.
3x6 + x 3 +
lirn (3x6 + x 3 + 1) 3 + +
x...

Iim 4x6 +9x+7


x -+ 1

(4xD

Urn

x-+

1)

(b) The above theorem cannot be directly used here, since the
of the denominator equals zero as x-+2. Herethe limit of
the numerator also equals zero as x -+ 2. Hence,we have the
limit

indeterminate form
\037

. For x

=1=

2 we have)

-9x-2
(x +5x+1) _ x +Sx+1
-x-6 _ (x-2)
(x-2)(x +2x+3) x2+2x+3')

x3 +3x2
x3

Thus,

in

unct ions)

2
2

does not contain the point x = 2 the

any domain which

f( X ) =

x3 +3x2
x3

-9X-2and

-x-6

cP

(x)

are equal; helice, their limits are also


function (x) is found directly:)

2 +Sx+1
= xx2+2x+3)

equal. The

cp

+ +

x2 5x

15

cp(x)=-hm
x-+2x2+2+3=11;
x-+2

hm

X)

hence,)

. x3+3x2_9x-215

f (x==lm
) I
x-+2
x-+2
I 1m

3--6 =0.

X)))

limit

of the

62

J. Introduction

Ch,

to Mathematical

Analysis)

(c) Just as 10 (b), we remove the indeterminate

form
\037

transforming

\302\245

-.

-+

6x\037

.==

x -+

1.10.2.
Find the
(a)

2
x+1 = lim (x+I)(V6x
+3-3x)
2
3
3x
+3 +3x x _ t
2
Hm Y6x +3-3x_ 1
-1 3 (1-x)

Hm
x -+

by

limits:

CX:\0374-3X\037\037
2
Hm

!\037\037

);

(b) .t-++oo) (V9x + 1-3x);

(c)

I 1m

x-++oo

Hm (V2x2
(d)x-+-oo

-3-5x);

1-

x(Vx 2 +

(e) lim

x-++oo)

(f)

3/'

yx+3V-;-+5V-;.
Y 3x-2
+ V 2x-3
x);)

+ 3 and r
4x+2

+
4x+2 ;

Y2x2

\302\2452x

x2\037CX)

x\037\037<X>

2x
(g) Hm 5 /(x+ 3).
x-+oo)

Solution.(a)

3X2\037

!\037\037

x
3X

2)

\302\267

Herewe have the indeterminate form 00


fractions

I 1m
X-+oo

- -3x+ ) = .

X3

( 3x2

11m

X-+oo

9x3

00; let us subtract the

2X3+4x2
=
+6x2-12x-8
2+4/x
= Hm 9+6/x=
12/x2 -8/x3
x-+
00

9'

\037

N ate. We see that in such examples


the limit is equal to the
ratio of the coefficients at the superior power of x (provided the
polynomials are of the same degree).

(b )

I 1m
A

-+

00

( Y9x2

+
I

- _- .
3x)

In hand ling such

1m

1
-+

00

V 9x2

+ +3x
I

-.
0

in mind that the function


a
(x), where Pn (x) is polynomial of degree 11, tending
to infinity in the same way as the function V x n This allows us
to singleout the superior power of x and divide both the numerator and denominator by this power of x. In the given example)))

(c)

f (x) == V Pn

examplesbear

1.10.
Calculation

\037

63)

of Limits of Functions)

Vx; then we obtain:


.
Vx+3VX+5 Vx = hm
3 2x-3
3x-2
x-++
+
V
V
V

the divisor is

hm
x + 00

3-2/x+V 4/x-12/x+9/x
2
-Y3IS

00

\037

2 +3/V-X-+5/\\yx 3
2

f)

3)

\302\267

(d) Sincethe sum of two positive infinitely large quantities


also an infinit ely large quantity, then
!irn
x-+-oo)

we have

I lIn

V x2 (2
x

x\037+oo

x< 0

At

2
Y x

t-+-oo

:==

VX2

x, therefore

x Y-2 3/x2
x-++oo x (4+2/x)
I

(4+2/x)

= Y2
4 .

-x and, hence,

(2+3/x2) =

(4+2/x)

Note. From this

+ 00.)
-3+{-5x)J==-

.
= 1m

3/x2)

VX2 ==

we ha ve
]1m

not

2
(J 2x

X\037-oo)

>0

(f) At x

(V2x2 --3-5x)==IiIn

. -xY2+3/x2 = Y2

I 1m

x-+-oo x(4+2/x)

x-+oo

exist.)
2x/(x+ 3)

lirn

52x/(x+ 3) == 5x
(g) Iim
-+
x

\037

=:52 ==25.

00

00)

1.t 0.3.Find the limits:


. 2x-2
(a) hm .
1

x\037

(b)

V 26+x-3

. + Vx ;
hm

tim V
(d) x-+O

V-

(g)

I 1m

X-+

()
f

x)

r
X!.\0372

si x
. x-+3'
sIn +

2 sin 2 x
2 sIn 2
1(/6

I 1m

-l

teger);

..r3-2cosx)

x-+1T/6 J'

x)

(x-n/6) .,

sin

x+
;
x+
V 17-2

Hm 4
x\037-l

k/ I +x

(c) x-+-ll+
(e)

\302\267

+ 3 does
!im Y-2x
4x+2)

follows, incidentally, that

it

(k

positive In-

cosx

V (1-sin X)2

;)

\302\267

Solution (method of substitution).(a) Let us put 26+x==Z3.


x==-z3-26and z\0373 as x\037 1; hence

Then

I 1m
x\037l

2x-2

== I 1m

. 2 (z-3)(z2+3z+9) =
2Z3-54 = I 1m

V26+x-3 z-+3 z-3

z-+3

z-3

= lim 2 (Z2 + 3z + 9) 54.


==-

z-+3)))

I. Introduction

Ch.

64)

(d) Let us

Hence,

to Mathematical

l+x=zk; then x==zk-l and z-+1as x\037O.

put

- (see

{/I+x-l= Hm z-I
=
k -1 k
X
I

lim

2-+1Z

X-+O

(e) Let

obtain

.
z
(x-n/6) _ 1m
..r..r3-2cosx2-+0\"
3-2cos(z+nj6)

sin

I 1m

x-+JTj6\"

. _ _sinz
= 1m
3- 3 cosz
z 0
I

t.l0.1(d)).

Problem

x-nj6==z;then x=z+n/6 and z-+O as

us put

x--+nj6.On substituting we

-+

Analysis)

+ sin z

sin

.
= 1m

2sin (z/2)cos(z/2)

V 3 sin 2 (z/2) -t-2 sin (zj2) cos(z/2)

2 -+ 0 2

= Hm _ cos(zj2)
1.
z 0 V 3 sin (zj2) + cos(z/2))
==

Find
1.10.4.

the

1-

(a) lim

X\037os

X-+O

-+

limits:
lim
(b) x-+O

tan

;
x-;sinx
X

cos(nx/2)

(c) 1 1m

X-+ I

-x

\302\267

1-

= Iim 2 sin 22(x/2) = 2 Iim sin


(
. sinx(J-cosx)
.1m tanx-sinx= 1m
x.x =
x3

Solution.(a) Iim
X-+O
I
(b) x-+

X\037os

I
x-+

X-+O

3)

COS

sin

l-x=z.Then

put

.::= .

cosx)

x -+ 0

.-.
X

x-+O

= Iim
(c) Let us
Hence,)

x= l-z and

x)

cos 2 x

-x

Find the
1.10.5.

limits:

(a) lim (1+ Ijx)7 ;

(c)

.
1m

:\037m(J)

r
+ .

\037x

In(a+x)-lna
;

(h) Iim

x)

' . x-I
( ) 1m
In

x -+

(1+ kjx)mx;)
(d) XHm
-+

'

In

x-+o

lim (] + X)I /(3x);


(b) x-o)

.
(f) hm

(l
(e) I
x-+o 3x_
(g) Iim

\037

x-e)

CX))

e4X

x_ 0
X -+

t an x
eX

-:-e-X ;

SIn

x)))

z--+O as x--+1.

Z)

t-+CX>

\037

I 1m

2 ;

1-cas -2')
I.

cos 2 2 z
sin z
2 =n
)- = I UTI
(
Z
2-+0
1-+0
x-+I 1
Note.For a simpler method of solving similar problemssee
I 1m

x2

x)

..::

\037\037/2)

2'

\037

1.12.

l.ln.Calrulation

\037

Solution.(a)

x\037o

(i) Put
\\ve

tuting

- - x-

(1+x)_
3x 1

In

(e) Ii rn

In

x-

X-.f

Find)
1.10.6.

-. (

x-+I

(b )

'::J:i

-f--

(I -

+X

x:!)

7
;)

e. On substi-

-.
1

e)

\037

== I i m
x

1 )/

(x

1 -t-

l(

00)

x2

l/x

X2

eO

==

1.

limits:

the

\037/\037)/(

( 2+x)

-]
x\"'-x- )
( 2')
+ 2x

X2

1 1 nl
t\037

00

1.10.7.
Find

Soluti 0/1. Ii 111

rr

=e

.
(1 +z) ==

In

X-+OO

HIll

In3

\037

x
.
+
)
(

1 i nl

(a)

-.

(x/e)

In

\037

(1+

e (z + 1); z \037 0 as x \037

x=-==

U\037\037

==
un
un
x-e == x-+ee(x/e-I)
e z-+o

1m

\037

- _-

.3 x

\037

then

obtain

\037n\037

(1+x)

In

=
} [( + + rJ
=
1

)7X

\037

Ii lll

xje-1 = z;

65)

of Functions)
7

( +
1

}\037\037

of Limits

-X);

(2x +

-)
1

'2

\302\267

Solution.(a) Denote:

+ x)/(2+ x);

f (x) ==(1

cp

Ii fn

x-+

(x)=

I-\302\245x

I-x

f (x)

x-I

) == I 1m
Imcp(x
]
x-+

I
x-+

limits

But at finite

1)

Hm f (x)== A

relation holds true:

x-a)

lim

Iinl [f (X)] (x) ==eX a


x-+a)
<:p

Hence,)
1

}\037\037

Note.
out

that

If

in
liln

48)))

+ = ;
3

<:p

> 0, Jim
x-a)

cp

(x) III f (x)

x-+a)

cp

==eBIn

1
\037

(x)== B the

(3)

/2

\037

===

fol1o\\ving)

AB

/ 3.)
2

the form lim [f (x)]CP(x) it turns


x-a
(x)= 00, then the following)

examplesof
linl

\302\245x

+x (I - llx)..'(1-x) =

f (x)== 1 and

\037

-I-x =\"2'

( 2+x)

handling

x-+a)

3_3 1

x
== Iinl
2+-x
I

/. Introduction

Ch.

66)

to Mathematical

transforrnation may be recommended:

(x)] (x)

Ii 01 [t

(I'

x-a

== Ii n]
X-Q)

{[1

+ (t (x)

t.t 0.8.Find the


.
-!-3
(a)

+3

(, 2x2+5 )

!\037n;,

Ii!n ) + [t (x)
(x)- )}
(x) {

a)

\302\267

\037

1) ] 1 /

<f

limits:

8X2

2x\037

==

I lIn

]+

t 0
__\037

(A)

(J,

lim (1+ sin Jtx)cot ;rx;


(c) x-+
I

t an x

sin

--:-l/(X-a) (a=l==kJt, .
SinX

\037

(d) BIn
x-+a ( sIn )

(x)= 2X2 +
+ 5;
2X2

lien

<p

cp

Use the formula (*):


+3

hOl 2X2 +
( +
2X2

f (x)

lim

1J

==eX a

\037In

cp

(X)

U (x)

-+

-]
1

integer).

(x) 8x2 -1-3;


\037

1 ==

2X2

3
5

-f--

-1 _e

cp

==

2X2

I 1m

(x) U (x)

-1-3

2
2x'\037+

5;

2 (8x2 + 3)

X-oo

2X2

+3

_--e_

8X2

-I].
,

==

-1] -lull.
2x\037+

cp

+5 = 8.
\037

( 2X2-+- 5 )
f (x) is g i ven wit h the aid 0f
.
I
.
t (x)== lUll
+I
n-+oc

x--oo

.to.9. The fun ct ion

2X2

xl\037noo

\037,8X2
h
OJ)

lun
r-oo (x)[f (x)
Therefore)

\037=

!\037.n;,

x-+oo

X-+OO

(x)

(f

]}

+ =
(x) =
2xLt 5 1;
2
lieT] (8x + 3) == 00.)
(x)== x_oo)
f

!\037\037

denote:

us

x)

k an

wIth

a)

Solution.(a) Let

Lf

l'

(b)

Analysis)

x\"!.ll

the

itn i t

X-!.1l

Investigate this function and graph it.


Solution. Consider three cases:
(1) x > 1.Since in this caselirn X21l :=:00,then)
I

n-+oo)

(2) I x

< 1.In

(x)

. ] _ ]Ix = 1.
lun
] -t- ]/
2n

-==

n-+

this case liln


f/. -+

X
00)))

x\037n

if:

211

==

-1.

0; thprefore f (x)==

( )

\037

J .JO.Calculation

67)

of Limits of Functions)

1.

In this caseX211 == 1 at any n, and therefore f (x)==O.


(3) X -== +
the
function
under consideration can be written in the
Thus,
following way:)

-1a
I

f{x)=
or, briefly,

f (x) ==sign (\\ x

Ixl>1

if

Ixl<l

if

x==+ 1
{
t 1 (n)).
1) (see Problem 1.5.

1-

if

Fig. 27.

The graph of this function is shown in


The population of a couincreases by
ntry
per year.
)1

1.10.10.

ty

-1

2\037)/o

ho\\v

By

times does it increase

many

a century?
Solution.If we denote the initial number of inhabitants of a
given country as A, then after a
year the total population will amoun t to
in

A+

l\0370

\037o

100years

it

wi

11

[(I +

:
i
:

I
I

-1

s\037

amount to

times.

-1

--\037x

Fig.
will

T1

reach the total of

r'f
= e, we can

have increased

)A.

years the population

two

After

.2=(I+

, (:

Taking

I)

27)

+ 50) . After
1

+ Sloroo,i.e. it
into

will

account that

:::::;
e.
approximatelyconsiderthat (I +
Hence,after lOa years the population of the country will have
increasede'!. 7.39times.
Of course,this estimation is very approximate,
but it gives an
idea as to the order of the increase in the population;
(the quan( 1 I 1 = 7.245to within three dl'cimal
+ 50)
tity
places) .
,

( +*
I

\037i\037\037

\037

ijO

\\

\\

1.10.11.
Find the limits:
.
cosx +4 tan x .
(a ) 1m 2 -x-2x4 ,
. + Sx 7
(b)
3x\037-x-2
.
YS-x-2;
(c)
I

x\037O

2X2

x\037\037\\

I UTI

x-I

(d)
3*)))

-.m

11

00)

2-x-1
V-

2x'!.-Sx-\037-4
Sx2 2x 3

SIO

Ch.

68)

/. Int roduction

to Mathematical

Analysis)

(e) xlin1(Vxt+l-\037/x\037-1);)
(f)

00)

--\037

It 01

f- ( V

oc

+ 8x) 3

.to.12. Fin d

(a)
(C)

lun

x-a

+ 2-x,

2x

the Iin1

-2;

Yx-t-4
.
Sin

It In

a\"\"

x)

.),\037

I
(e) x-+Jt/3
6
cos(x +'It/))

1.10.13.
Find
( a)

a2X

11m
(c) x-a

x)

I 1m
X-+ I

(d )

,., x

I)

tan 2x tan (J(/4

Ii m
X-JI/4

the limits:)

. e-

11m
(b) X-+
0

(d)

;
2

(I-x);
..r-

sin

x);

\302\267

(1+ 4/x)X + 3;)


x_oo)
Ii rn

(b)

Sill

\302\267

its:

.
- aJ[
. tan3x-3tanx
1m

(X-Jt

(e)
(sin 2x)tan 2x;
Ihn (tan x)tan 2x;
(g) x-n/2

(f)

lim
x-+O)

l\037\037

X\037\037\0374

(h)

X)

(1+ 3 tan 2 x)cot


( 2x
\\

\\ x

2x+I )

(sin x)tall

Iin1

x-1t/2

x;

;
x;

. 3X2+2X+ (6X+ 1)/(3x+2).


' 1m
'
x- ( 2+ +2 )
. 1+3X (1-VX)/(l-X).
'
'
(J )
( 2+3x)
. .- .
(k )
.

(t )

00

x\037n;

It rn
f

e7.X

..0

e\037x

X)

Find
1.10.14.

(a) lim

the

limits:

-x) ;

arc
X
cV\037

.t-+O

(b)

I 1m

X-;T./4

In tan x
1

cot ;
x)

(c) li1n\037 In (1+asinx).


x-+O

SlI1X)

and InfiniteFunctions.
* 1.11.
lnf-initesimal
Their Definitionand Comparison
The function ex (x) is called infinitesimal as x --+- a or as x --+ 00
Ii n1 ex (x)== 0 or lirn ex (x)== O.
x-+
The function f (x) is called infinite as x -?-a or as x --,.00 If
HIll f (x):=:00 or Jini f (x)== 00.
if

00

x-\037a

x-a

x-

00)))

\037

1.
1.1

inverse to an

quantity

simal.

and Infinite

Infinitesinzal

quantity is

infinite

69)

Functions)

calledan

infinite-

Infinitesimal functions possess the following properties:


sum and the product of any definite number of infinitesimal functions as x -+ a are also inftnitesimals as x
(2) The product of an infinitesimal function by a boundedfunction
is an infinitesimal.
Comparisonof Infinitesimals.Let the functions a (x) and (x) be
If)
infini tesimal as x -+

(1) The

\037

a.

a.

\037

\0371\037

ex

\037

(x)
(x)

= c,

where c is a certain finite number different from zero, then the


functions a (x) and (x) are calledinfinitesilnals of the same order.
If c ==
then the functions a (x) and (x) are called equl'valent;

1,

\037

notation:a (x)

\037

\037

(x).

of a
0, then the function a (x) is calledan infinitesimal
==
a
0
is
thus:
written
to
which
order
relative
(x),
(x)
higher
(x),
and (x) is called an infinitesimal of a lower order with respect
If c ==

\037

(\037

\037

\037

to

a (x).

= C, where

If
:\037n;

<

< + 00,

then

the function

ex

(x)

r;(\037\037n

is calledan infinitesimal of the nth order as compared \\vith the


function (x). The concept of infinite functions of various orders
is introduced similarly.
t
Prove that the functions
\037

1.1.1.

2x-4
(a) f (x)= x 2 +5 as x

2,

\037

I
(b) f (x)= (x-l)2sin3 x I as x -*1 are infinitesimals.
Solution.(a) It is sufficient to find the limit

lim f
t 2

= O.

(x)= xlim2

2\037\037:
X\"'-r)

-)-

\037

(b) Firstly, the function cp(x)==(x-I)2is infinitesimal as


indeed, Iim (x-I)2==O. Secondly, the function

x-?1;

x\037l

'\"

\037 ;
(x)==sin3 x-)
I

is bounded:)
1

,,':1x-1 I
sin

\037

x =I=-

1,

'1

\302\267)

Hence,the given function f (x) represents the product of the


bounded function\",(x) by the infinitesimal fP (x), which means that
f (x) is an infinitesimal function as x
\037

1.)))

Ch.

70)

1.t 1.2.Prove
(a)

f (x) =

(b) f(x)=
are

-+

1.Introduction

to Mathematical

Analysis)

that the funct ions


12
--+

3x

2X2

sin x
x

as x

4;

as X--+OO

infinite\037jmal.)

I.t t .3.Find)

lim x
x-+o)

sin ( 1/x).

Solution.Since x is an infinitesimal as x --+0 and the function


sin (1Ix) is bounded, the product x sin (1Ix) is an infinitesimal, which
means t hat Ii n1 x sin (1Ix) = O.
x-+o)
t
with

.11.4.
Compare the

(a)

the infinitesimal
f1

(x)= ta n x 3;

following infinitesimal functions (as x ---+0)

(x) = x:
(b) f 2 (x) =
cp

V sin2 x ;

(c) t\037(x)=V9+x-3.
Solution.(a) We have

' 2-O.

tanx3
-tanx3 2 = I
x
an x3 11 lnx
X3
[
x-o
x-+o
x-o
x-o
Hence,tan x3 is an infinitesimal of a higher order relative to
(b) We have
tanx3

I In1

2
V sin x
x
0

Hence)V sin2x

wit h

I 1m

3'-

1lln
x

\037

I 1m

X -+

-3=

Y9+x

I Irn

x.

sin2 x
x2

I 1111
X -+

= 00.
VX ]
lower orderas compared
I

--

is an infinitesimal of a

x.

(c) We have

Y9+x+3

\302\267

V9+x-3

and x are of the same order.


infinitesimals
Determinethe order of smallness of the quantity
tt
with
to the infinitesimal (x.
respect==
(a) B cas(X cos2(X; (b) = tan (X sin (x.
2ex.= 2 sin ex sin
Solution.(a) = cos

Hence,the

1. .5.

ex.

\037

-cos

_ 1. 2
L
(X-+oa (X-O

Whence)

\"

I 1111

Hence, is
\037

an

\037

infin

I ill

sin

\037

(3aj2)sin (aj2)_
a) 'l

\037

\037

\302\267

itesima] of the saIne order as

secondone with respectto

a.)))

(X2,

i.e.of the

\037

1.11.6.
Assuming

1.12.
Equivalent

00,compare the

x \037

quant ities:

2
+2x+5 and
(a) f(x)==3x
2
==
2X
and
+ 3x
f
(b) (x)

f(x)==V/x+a

(c)

71)

Infinilesimals)

following

infinitely

large

+2x-l;

3
(p(x)==2x
cp

(x) == (x

+ 2)2;

cp(x)==Vx .

and

is of a lower order
Solution.(a) The infinite function 3X2+2x+5
as compared with the infinite function 2x3 +
1, since
2 +5/x3
3x2 +2x+5
3/x+2/x
rIn1
r1m
0
X-oo 2x 3 -t- 2x-1 x-oo) 2.+ 2/ X2_I/X3

2x-

-.

1.11.7.
Prove that

the infinitesimalsa==xand \037==xcos(ljx)(asx-\037O)

their ratio has no limit.


i.e.x cos
comparable,
(l/x) == lin1 cos
Solution.Indeed,lirn
(ljx) does not exist(prove
x-)oo
x-.o

are not

it!),

means that these infinitesimal functions are not con1-

which

parable.

1.11.8.
If x -+ 0, then

which of the following infinitesimals is


of a lower orderthan x; of the

(are) of a higher order than x;


same order as x?
(a) IOOx; (b) x 2; (c) 6 sin x;

1.11.9.
Let x

\037

sin3 x; (e) V tan 3 x.


O. Determine the orders of the following infini-

Vl+x3-1;

tesimal functions
(a)

2 si n 4 x

(c)

1x

x-

(g)

respectto x:

xC>;

2 cos

(e)

with

(d)

x+ ) ;
\037

(b) V sin2x+ x4;


(d) sin2x-2sinx;
(f) 2 V sin x ;
(h) tan x

+ x2;

cosx-Vcosx; (j) eX-cosx.


t.lt.to. Assuming the side of a cube to be an

(i)

infinitesimal, deof
termine the order of smallness of the diagonal of the cube
the area of its surface (S); of its volume (V).)
(d)\037

Equivalent Infinitesimals.
* 1.12.
Applicationto FindingLimits
If

the functions

a (x)

and

(x) are infinitesimal as x

-+ a

and

if

l'(x), (x) 6 (x), then


(x = !irn
x-a tj (replacingan infinitesimal by an equivalent one).
x-)oa p (

a (x)
tIn1

\"'\"

\"'\"

\037

\037

\037

\302\273

X)))

\037

Ch.

72)

I. Introduction

to Matlzcfnatical

Analysl\037)

If)

limf(x)=k, O<lk/<oo,
x....
a)

then)

f (x) a (x)--- ka (x).)


If)

a (x)--- V (x),
p (x)--- V (x),)

then)

a (x)--- (x).
\037

For two infinitesimal function) to be equivalent it is l1e\037essary


and sufficient that their difference be an infinitesimal of a higher
order as compared with each of the two.
Listed below are infinitesimal functions:
(a(x) is an infinitesimal as x -+0)
(I) sin a (x)--- a (x); (2) tan a (x) --- a (x);

1- a

cos (x)

(3)

\037

[a (x)]2/2;

(4) arc sin a (x)--- a (x); (5) arc tan a (x)--- a (x);
(6) In [I (x)]--- a (x); (7) a(.t(X)_1--- a (x)lna
(a > 0), in particular,el) (x) 1 --- a (x);

+a

(8)

[1+ a (x)]p

ii 1 + a (x)_ 1

1 --- Pa (x), in par tic u I a r,


Prove that as x\037o

1.12.1.
(a)

1-Vl+x
I

--- _1 x;
2

(b)

1-1+
1

___

a (x}
n

--- x;

(c) sin V' x Vx --- x2 + V x 3 .


Solution.(a) By formula (8) at P == 1/2 we have
--- 1. x.
\037= \037 (J/I+
2
1

VI

+x

VI

x-I)

+x

\037

(c) By formula (I) we have

sin1/xVx ---IxVx=

X 3/ 4,

V x2 + J/,-X3 =x 3/ 4 Vl + ,....3/ 4 ,
whencesin -V x x --- 11x2 +JIx 3
1.12.2.
Replaceeach of the following infinitesimals with
Xl/\037

\302\267

\037/

one:
+ 5a4 -t 6a5
(a) 3 sin a 5a3; (b) (1 cosa)2 + 16a:{

valent

.)))

an

equi-

J.J2. Equivalent

\037

Solution. (a)

73)

the sum of two infinitesimals

that

l'\\ote

I nfinile\037'itnals)

and

\037

orders is equivalent to the summand of the lower order,


since the replacement of an infinitesimal \\vith one equivalent to it

of different

is tantamount to the rejection


of an infinitesimal of a higher order.
In our example
the quantity 3 sin has the order of smallness
:1) the order of sma IIness

(-5a
(b)

3 sin a +

\037

\"'-/

the aid of the principle of substitution of equivathe limits:)


cos
I
sin 5x
(b) xliin) 0
In (I + 4x)
I
cos x2

- .'
- -

find

x)

;)

\037

\037\037

.
cosx
VI
; (d) Jim
2
0
+x
.1m sinV 2x+
arc sin 2 x-arc tan 2 x
;
3x
. 3 si x x22+x3 4 ;

- +
x+

+x2

In

(c) xI lITI 0
I
(e) x\037o

\037

+\037

sin 4x)

\037

-1

(f)

tan

I 1 ill

\037

' .n
()
I 11

\037

sin

5x

(1-

+ cas 2X)4 + Xo
x+sin6 x+2sin5 x
3/V x In (I +3x)
.,
--

7tan

x\037TIo

tan X)2

(g)

2 sin

(sin x-

!l\037\037

3a.

3 sin a \"'-/

\"'-/

quantities

(h)

16a3 is of the lower order,therefore


(1 casa)2 + 16a3 + 5a4 + 6a5 16a3.

1.12.3.
With
(a)

(-5a)

l\037ence

16a3 +5a4 +6af>==4sin4 + 16a3 +5a4 +6a5.


(1-cosa)2+

The summand

lent

1,

('/..

3,

..

3.
II - )

- - x-'.)
5

r-( arc tan r x ). ( e .


1
SIn 3
cosx+2 sIn
t an 3 x 6 sin - x +
2

.'

x-x2 +3x4

x-x')
5
'J

\302\267

have sin 5x \"'-/5x; In (.1+ 4x) \"'-/ 4x (see the list


of equivalent infinitesimals on page 72). Therefore

Solution. (a)

We

(c) xJim0
--+

l\037r\037

V + 1n
1

cosx
x\037

sin
In

Jim
x

5x

( 1 -t- 4x)

In

= 45

\302\267

x-I)]=

II +
x

(\037\037:

==4 lim

x-o

5x
4x

l-?'\037

cas
x\037-I

= _ 4 lim

x2\0372

t\037O

(d) From the list of equivalent infinitesimals we find:


2
sin 4x 4x.)))
II 1 x x 2 1 (x -r- x )/2

+ +

\037

\037

x;2,

\037

X)

-2.

/. 1nt roduction

CIl.

7\037)

to Mathematical

Therefore)
}if 1

Ii m
x

+ +x2
sIn

\037

sin 2x + arc sin2


Hence,)
.

sin 2x -f-

x-a
(h)

==

x-arc tan

arc sin 2 x
3x

2x

arc tan

\037

sin

-1

\"\"

(arc t an
Find the

/\037\037

1.12.4
.

x-o3
X)

2 5
'1r-::
r x ) ( e V-;- )

\037J\037

2x.-.,.,
2x.)

2
== 1 un 2x ==

Vx Vx; e5 V-;
V x In (1+ 3x) == l'

arc tan

I'

sin

\037

sin V-X.-.,.,Vx; In (1+ 3x) \"\" 3x;

x/2
==\037
8)
0 4x

Ii m
x

of equivalent infinitesimal functions given on

(e) Using the list


page 72 \\ve obtain

I un

4x

\037

AnaltlSis)

V x;
-x.3x

\"\"

x.5V\037

\302\267

-_

\037

approximate values of the roots


the
absolute error.
E
stimate
VO.994.
Solution.Use the approximat e formula

close

/1+ 0.02

\"\"

\"\"

estimatethe

1-

O.\0372

O.\03706

( *)

1.01;

= 0.997.

error we note that

; -(VI +x-l)= (x-2V I +x+ 2)=

= J..
2 (X + 1

1.02and

V1-0.006
To

V 1 x .-.,.,1 x/2
to zero). In our case

.(for x sufficiently

\302\267

\037

2 V x + 1+

1)==\0372 ( V x

Hence,the absolute error


2

of

8.

+1

1) 2 .-.,.,2
\037

\037

!=

x2

( 2) 8
the approximate formula (*) is esti\302\267

mated by the quantity


Using this estimate\\ve find that the absolute error of the root
02)2
and the absolute error of
Vl.02 1.01is (0'8
=0.00005,
\037

\037

;-

.J;

0.994 0.997amounts
\037

1.12.5.
Prove
(a)

J /
V

to

\037

(O.006r\037

that, as x.---?0,
I

1+x--l\0373x;)))

\037

0.000005.

1.13.
One-Sided
Limits)

\037

(b) arc tan

mx

(c)

-...-mx;

1-cos x\", ;

1.12.6.
For

sin2 x.

x---+-O determine the order of smallness,relative


(x) ==x, of t he following infinitesimals:

to the infinitesimal

\037

(a) V-sin 2 x+ x 4; (b)

1.12.7.
For

the infinitesimal

2
3(x-2)2+2(x
-4);

(b)

In

sin

\037

In

(g)

xt\037o

\037

In

(l-x+

Find
1.12.9.
\037

\037

(1+sin4x).
. 5
'

In

esm

x_I

-7x:)
J

\037rc

+ 3x-1

YI, sin
(f) Iim In (l + t an 2x) ,
x_ 0

(.

2x\037

I tIn

\037

+ I)
2 +4x3)
n (1+2x-3x
.

In

Vsinnx.

'.
(d) xIiln 0 arct\037n3x
Sill 2x

\302\267

In

(b)

of the method of

. sin 3x ;
(I +5x)
. e 3x _
(c) lun
(I-t-tan 2x)'
0
.
(2-cos2x)
;
(e) lun
sin 3x
x\037

x)

replacingan infinitesimal
one, find the following limits:)

an equivalent
!\037I\037

1+(lV\037

x-

\037

1.12.8.
Making llse
(a)

x2

x\0372 deterrnine the order of smallness,


relative to
2, of the following infini tesimals:
(x) -==

(a)
with

75)

\302\267

(h )

,)

I III]
x
0
\037

V 1 -j-x 2
I

--coS .
I

x)

approximate value of the root V 1042.)

an

1.13.One-Sided
Limits
A

number

as x ---+- XO

is

(A ==

calledthe

lin1
f
x Xo + 0

(x)

\037

() (e)

> 0 such that

for all

===

Ii/nit

f (xo

to the right of the function f (x)


if for any
0 there exists

+ 0))

x sat isfying

the inequal i ty

\302\243

>
0 < x-X < () (c)
Q

belongingto the domain of definition of the function f (x) the


inequali ty 1 f (x) A < e holds true. The I imit to the left of the
as x \037 xo
is defined in a similar way. If Xu ==0,
function f (xo-O)
then we write simply x-++O or x\037-Oand, respectively, f(+O)
and

and f

-0

(-0).

Find
1.13.1.

the one-sidedlimits of the functions:


+ 3 if x 1,

-2x

ifx>l asx-d;
(a)f(x)=\\3x_5
2
J

-1

\037

x
(b) f(x)== lx-II as x-rl;)))

Ch.

76)

(c) f (x) ==

/. Introduction

1-cos2x
x

(d) f (x)= 3 +

to Mathematical

Analyslc,)

as x --i- 0;
as x ---+1;

1+7 /(1-x)
1

(e) f (x) ==cos(n/x) as x \037 0;


(f) f (x) 5/(x 2)3 as x --i- 2.
Solution. (a) Letx\037l.Thent(x)==-2x+3.
Hence,f(I-0)=
-==

t (x) 1 is the]imit to the left.


1-0
If x > I, then f (x) 3x-5; hence
f (1+ 0) == li
t (x)== 2 is the Ii i t

liill

x\037

==

111

x\0371+0

to the

0'

ITl

(see Fig. 28).

right

-V
x--

(C) f ( )

- -_
cos2x

sin\037

_ Y2,si

11 X

x)

x)

but

\\

sin x 1=
{)

Fig.

sin x,)

if

-SInx,)

if

0 <:x

<:IT/2,
-JI/2<: <:o.)
x

Hence,)

28)

lim
(--0)= lirn-0t (x)==K-+-O
)
( V-2
x
==
f (+0) lirn f (x) == IiIn ( V 2
) V2.
X-++O
x-++O
(d) The expressionI/(l-x) tends to + 00, when x tends to I,
remainingless than I, therefore
= 0 1-0
IinJ 7 /(1-X)== + 00, liIn
) 3.
1+71/(1-X)'f (
Further, as x \037 I -1-0 we have 1/(I-x)\037-00. Therefore
lim 7 1/(I-X)== 0,

x\037

===

Si\037

\037/2,

-==

Si\037

=--=

x\037l-O

X\0371-0

\037-+I+O)

f(I+0)=x-+l+0(3+1+7)
Jil-n

(e) Let

us

terms)

y.)))

=3+1=4.

choosetwo sequences,{xn } and

{x\037},

2
and x ==2n

1,2,

xn ==
i vel
respect

1\037(I_x)

2n

\037

(n ==

with

\302\267

\302\267

.))

the general

.9

Then lilTl

1.14.
Continuity
==

Xn

lirn

of

Points of Discontinuity

Function.

77)

== 0 and)

x.\037

n\037oo)

n\037\037

liIn

f (xn ) == lien

lim f

cos2:n:n == 1;)

fl\037;$))

n\037;$)

== lirn

(x\037)

11-'\037

I1\037OO

cas(2n + 1) = O.
\037

function f (x) has no 1 imit to the right at the point 0;


that f (x) is an even function, we conclude that
into
account
taking
it has no 1 imit to the left either (see Fig. 29).

Hence,the

!f)
1)

;c)

-1
Fig.

Prove that,
1.13.2.

as

f (x )

29)

x-+l,the

I
'\\

+1
3x + 2

function

at 0
at 1

< x < 1,
<x <3

5.

has a limit to the left equal to 2 and a limit to the right equal to
Find the one-sided limits of the following functions as
x

1.13.3.

-+0:

(a) f (x)== 2_21 / X ;

(b) f (x)==ellx;

(c) f (x) ==

sin x
x)

1.14.Continuity of a Function.
and Their Classification
Pointsof Discontinuity

\037

Let the function y ==f (x) be defined on the set X and let the
point of this set.The function f (x) is said
Xo E)( be the limit
to be continuous at the point Xo if lim f (x)= f (xo)' The latter conXo
dition i.s equivalent to the condition lim
(xo) == Hm [f (xo +
point

)\302\243

\037

-.0

\037y

-f(xo)] =0.)))

l!1x

fix

-.0

\037x)

Ch.

78)

/. Introduction

to Mathematical

Analysis)

The function f (x) is cont inuous at the point Xo if and onl y if


f (xo 0) == f (xu + 0) ==f (xo)'
The function t (x) is continuous on the set X if it is continuous
at every point of this set.
Points of Discontinuityof the First Kind. Let the point Xo be the
limit point of the domain of definition X of the function t (x).
The point Xli is calleda discontinuity of the lirst kind of the fun1 imits
to the right and to the left and
ction t (x) if there exist
== t (x o -1-0) =1= f (xo)' then XII is called
they are finite. If t (x,,-O)
a remova\037le discontin uit y. Further, if t (xo 0)=1= t (xo+ 0), then Xu
is a non-removable discontinuity of the first kind, and the d.if1erence
is calleda jump discontinuity of the function
f(xo+O)-t(x'I--O)
f (x) at the point XII'
Points of Discontinuity of the SecondKind. If at least 011(-' of
the limits of t(xo-O)and f(xo+O) is non-existent and infinite,
then point Xo is calleda discontinuity at the second kind of the funth\037

--

ction f (x).

1.14.1.

Using only 1 the definition prove discontinuity of the function t (x) 3x4 + 5x:
+ 2X2 + 3x + 4 at any x.
Solution.Let Xo Be an arbitrary point on the number scale.Fir\037t
\037-=

Iim t (x):)

find

x
lirn
1C

.......

X0

.......

Xo)

f (x) == I.iln
-+

X0)

(3x4 + 5x;}-1-2X2 + 3x + 4) ==

3x\037

+ 5xo+

2x\037

+ 3xo-1-4.

Then compute the value of the function at the point xo:


+ -1-3x0 + 4.
f (xo) 3-\\ +
\037

2x\037

5x\037

\037

Comparing the results thus obtained,we see that


Ii
x

.......

11

t (xI))

--==

t (XI)')

xo)

Hence,the

function f (x) is cont inuous at the point Xu by definition.


is an arbitrary point on the number scale,\\\\'e have proved
continuity of the function for all values of x.

SinceXo

. .2.G

1 14

i v en

the

f un

ct j OilS:)

+ (2x + 3)
2

(a) f(x) ==<)


\037

6-5x

x-3

-2.\\2
(b) f (x) = J 3x
\\

(c) f (x)=

I
.)

\037;=\037

for

for
for
for
for

-oo<x\037lt
< x < 3,
3 x < 00;
x\0373,
x>
1

\037

3;)))

\037

1.14.
Continuity

of

Points

Function.

79)

of Discontinuity

Find the points of discontinuity (if any). Determine the Jump


discontinuitiesof the functions at the points of discontinuity of the
fi

rs t kin d

Solution.(a) The domain of definition of the function is the entire


number scale (-00,00). In the open intervals (-00,I), (I, 3),
(3, 00) the function is continuous.Therefore discontinuitiesare pos-

1, 3,

x == at which analytic representation of the function is changed.


Let us find the one-sided I im its of the f unct ion at the point x
sible only at the points x

===

-=\037

0) =

1:

+ (2X2 + 3) 1;
f(I+O)== lim (6-5x)==I.

f (1

Ii m
x\037l-O

\037

X'-+l+O)

The value of the function at the point x == 1 is determinedby the


e. t ( 1)==(2 + 3)/5== Since
anal yt ic representa t ion,
f
0) t (I + 0) ==f (1),
the function is continuous at the point x ==

first

(1-

i.

1.

===

1.

Considerthe point x = 3:
f

(3-0)==x-3-0(6-5x)== -9;
li\037

f (3

Ii
+ 0) ==x-+3+0)
(
111

3) = O.

We see that the right-hand and the left-hand limits,though finite,


are not equal to each other, therefore the function has a discontinuity

of the

first

at the point x ==

kind

3.

The lump of the function at the point of discontinuity is


==
9.
f
f (3 +
(c) The function is defined and continuous throughout the entire
number scale;exceptat the point x== 3/2. Since 2x 3 > 0 for

0)- (3-0) 0-(-9):=


x>3/2and 2x-3<0for x<3/2,

> 3/2,
x<3/2.)
==
f (3/2+ 0) == 1,f (3/2-0)
f

Hence,)

(x) ==

I at x
at

{ -1

-1.

Therefore, at the point x ==3/2 the function has a finite disconof the first kind. The jump of the function at this point
tinuity

is equal to 1-(-1)==2.
f(3/2+0)-f(3/2-0)
t.14.3.
Test the follo\\ving functions for continuity:

'
\"

(a)

(x) =

(
<

--

sin x

for

x =1=

..\\

fur

0,

==0;)))

Ch.

80)

/. Introduction

to Mathematical

Analysis)

(b) f (x)=sin(ljx);
XSin (ljX) for X=F O,
o
for x == 0;
x
for x
3
0,
f
(d) f (x) = 2a x
for
0;
(e) f (x) == arc tan (ljx); (f) f (x) ==(x a
1).
Solution.(a) The tunction is continuous at all points x
the point x -== 0 we have)
'
- sin x
sin x
I IIIl
1
f (0)== 1;) 11 ln
x

(c) f(X)=={

4.

<

x;;;::

\\

x--o

Hence,at this

means that

is

it

+ l)j(x+

-= -= .
x-+o

-=I=-

O.

At

x)

point the function is continuous

continuous for all val ues of

as

\\\\'hich

\\\\'ell,

x.)

!I
1
I
I
I
I
I
I
I
I

I
I

37r

-n:21

1-1

-3n2

I
I

2
1[,

2\0371
I

I
I

I
I
I
I
I

-1
Fig. 30)

(b) The function is defined and continuous for all x =F O. There


qre no one-sided limits at the point x = (cf. Problem I.t 3.t
(e)). Therefore, at the point x ==0 the function suffers a discontiof the second kind (spe Fig. 30).
nuity
==f (+0)==f (0)
== 4, and f (+0)== 2a;the equality f
(d) f
e. the function f (x) will be continuous at the
will be fulfilled,
poj n t x == 0 if we put 2a 4, a ==2. 2
+ 0) == lirn (x x + 1)==3, e. both one0) == f
(f) f
\302\260

(-0) i.

(-1-

(-1

sided limits are finite

===

a.nd

x--1
coincide.But

(-0)

i.

at the point

-1

x:=

the)))

1.14.
Continuity

.\037

at

Points

Function.

of DlSf.-Ontinuity

oJ)

is not continuous.The graph


-x
the point M (-1,3)
+1
=- then
function

function is not defined and, therefore,


of the function is the parabolay ==x 2

with

(-1)

we redefine the
it
3,
putting f
the function has a 1'<:1110\\\\'ill become continuous.Thus, 3t x ==
vable discontin u i ty.
removed.

If

-1

Test
1.14.4.

the following functions for continuity:


==
E
f
(x)
(x). It should be borne in mind that the function
(a)
E (x) is defined as the maximum integer n contained in the numas a number satisfying the inequality n x.
ber x,

i.e.

\037

(b))
A

(x) =

J
'\\

if

if

x is rational,
x is irrational.)

(x)is calledthe Dirichletfunction. For instance, (0) 1;'A( -1/2)== 1;


(V-2)==0;'A(n)==O, etc.
Solution.(a) The function E (x) is defined throughout the entire
number scaleand takes on only integral val ues. This function is
'A

'A

===

- 31).-

discontinuous at every integral val ue


riable, since E (n 0) ==n 1;
E (n

of the

+ 0) ==n (see Fig.

!I

(b) Let us choosean arbitrary

on the x-axis;two cases


are possible:( 1) the number Xo
is rational; (2) the number Xo is
point

Xo

irrationaL

1.

there are irrational points, where


(x) ==O. Hence,in any vicinity
of Xo there are points x for which
==

-------\037
:

In
In the first case'A(xo)==
a
of
rational
point
any vicinity
'A

independent va-

1
-1
\037-2
I

I
J

I
I)

x)

\037

-3)

'A (xo)-'A(x)
1==1.
Fig. 31)
the second case 'A (xo) ==O.
In any vicinity of an irrational point there are rational points
at which 'A (x)== Hence,it is possible to find the val ues of x for
I

\037y

In

1.

which)

= 1.
l\037yl=I'A(xo)-'A(x)1
Thus, in both casesthe di fference l1y does not tend to zero as
I1x\037 O. Therefore, Xo is a discontinuity. SinceXo is an arbitrary
point, the Dirichlet function 'A (x) is discontinuous at each point.
The graph of this func\"tion consists of a set of points with irra t ional abscissas on the x-axis and of a set of points with rational
abscissas on the straight line y == 1, that is why it is impossible to sketch
it.)))

of

/. Introduction

Ch.

82)

to Mathematical

Anal.usis)

1.14.5.
Using the

definition of continuity of a function In terms


test the following functions for continuity:
(x) = ax + b (a #:0);

\"\302\243-6\",

(a)

x2

x is rational,
if x is irrational.
Solution.(a) Choosean arbitrary point Xo. According to the
<lefini tion it is necessary to show that for any preassigned,arbitra0 it is possible to find a number 8 0 such
rily small number
8 the inequality t (x) f (xo)
c holds true.
that at X-Xo
Consider the absolute value of the difference

(b) f (x)=

if

-x2

\\

\"\302\243--8\"

\302\243

.1

<

(x)-t o)

>

>

<

o -i-b)
(ax+b)-(ax
ax+b-axo-bla
require that If (x)-f(X o) 1< This requirement
all x
(X

==

==

===

Let us
fulfilled for

\302\243.

II

X-Xo I.
will

be

satisfying the inequality

or Ix-xol<\302\243/lal (a=#=O).
o
}allx-x
a I, then at X-Xo 1<8 the inequality
Hence, we take 8
f (x)-f (xo) 1< is fulfilled. Continuity is thus proved for any
x ==
/<\302\243

if

\037

\302\243/1

\302\243

point

Xo.

of rational
(b) Choosean arbitrary point Xo. If {xn } is a sequence
If
is a sequence
numbers tending to xo, then lim f (xn ) ==
X'I-+ Xo
== -x\037. At Xo =F 0)
of irrational numbers tending to x o, then Iill1 f
,
x\037.

{x\037}

(x\037)

X.'1-+ Xo

the indicated limits are different and hence the function is discontinuous at all points x =1= O.
On the other hand, let now x ==O. Find the absolute value of
the difference If{x)-f{O)[:

{x)-f(0) == + x2 -O
is obvious that x2 < at x < VB.
If

1==-

x2

If
> 0 is given, then,
we
obtain
putting 8\037J/E and Ix-O/==Ixi<8,
l\037f(O)I==X2<\302\243.
lienee,9t the point x ==0 the function is continuous.And so,the
point x ==0 is the only point at which the function is continuous.Note
the
that the function under considerationcan be expressedthrough

It

\302\243

\302\243

Dirichlet function (see Problem 1.14.4


(b)):f(x)==x2 [2A(x)-1].
1.14.6.
Determine which kind of discontinuity the following

functions have at the point x ==Xo:


x+ 2 for x < 2,
(a) f (x)= J 2
1 for x
x
2; Xo == 2;
\\

\037

(b)
x-s xo==5; (c) f(x)==1+21 /x ; xQ==O;
==
tan
x; Xo == JIj2;)))
(d) f (x)
t (x)==arctan

\037

1.14.
Continuity

of

Points

Function.

83)

of Discontinuity

n 2 , where n is a natural number.


(e) t (x) == Vx-E(V-X); xo:::::
Solution.(a) Find the one-sidedlimits at the point Xo == 2;

lim (x + 2) ==4;
(2-0)==X-+2-()

t (2 -l-0) == !in]
x-\037

Herethe limits to the right


coincide,therefore the

do not

2+

(x2

0)

-1) 3.
==

and to the left exist, are finite but


function has a discontinuity of the

at the point Xo ==2.


The
function E (J,/-x)has discontinuitiesof the first kind at
(e)
every point x == n 2, where n is a natural number (see Problem
1.14.4(a)), whereas the function V-X is continuou\037 at all x O.
E (V x ) has discontinuitiesof
Therefore the function f (x) ==V
the first kind at the points
n 2,
4, 9,
first

kind

1,

1.14.7.
Test the

x-

...

\"',

\037

following functions for continuity

= eX-I

-X-;
for x 0,
(
(b) f (x)= { -X
==
(a) f (x)

eX

=1\"=

3 for x 0;
l /X for x
e
0,
(c) f (x)= f
-==
0;
t 0 for x
(

=1\"=

(d) f (x)=

(sIn X F \"; (e) f (x)=


(f) t (x)==E (x) +-E ( x).

I i III

n\037

(\302\243)

1.14.8.

'::
I

\037

\037

For each of the following functions find the points of


discontinuity and determine the jumps of the function at these

points:

4
t (x) = x:J. _ 2x + I ;

(a)

x+-2 ;
(b) t (x)==x+ X-1-21

x-] .
2 -x3 '
I

(c)

(d)

(
t (x) == <

(x)

\037-=

x
2

x-

for

for

1.14.9.
Redefine the

to make them

(a) f (x)=

\037

> 1.

following functions at the point

continuous:
x

11:
;)))

1,
x=O so as

Ch.

84)

(b) f (x)=

(c) f (x)=
(d) f (x)=
\037

I. Introduction

to Mathematical

Analysis)

5X2

;-3x ;
x

;
V\037-I
. .)
)

S10-X
cos

x)

/.15.Arithmetical Operationson Continuous

Functions.
Function
Continuity of a Composite

the functions f (x) and g (x) are continuous at the point x ==xo,

If

then the functions

(l) f (x)+ g (x); (2) f (x).g (x); (3)


(g (xo) 0)
are also continuous at this point.
the point x ==Xo and
If the function u == (x) is continuous at
the function y = f (u) is continuous at the point U o == (xo), then
the composite
function y == f [cp (x)] is continuous at the point x ==xu'
\037

\037

\037\037)

cp

cp

1.15.1.
Test

the following functions for continuity:


2x 8x2 II
f(x)=x4 +4x3 +8x2++8X+4 ;
3 sin 3 x+ cos2 x+ .
'
( b ) f (x) =
4 cosx 2
3
x cosx + si0 x
.
(c) f (x)=D

(a)

cos (IjSln

\0372

x))

Solution.(a) A function representinga ratio of two continuous


functions (polynomials in this case) is discontinuousonly at points
for which the denominator becomeszero.But in our case

+ 4x3 + 8x2 + 8x + 4 ==(x2 + 2x + 2)2,


since x2 + 2x + 2 ==
1)2+ 1 > at any x, the
x,l

and
denominator
C\037+
never becomeszero.Hence,the function f (x) is continuous throughout the entire number scale.
(b) The function f (x) suffers discontinuities only at points for
at points which are the
the denominator equals zero,
\\vhich
roots of the equation
== or cosx = 1/2,)
4 cos
whence)
x = X n == + n/3 + 2nn (n == 0, +
+ 2,
\302\260

i.e.

x-2

\302\260

1,

...).

Thus, the function f (x) is continuous everywhere, e:xceptat tRe


point

Xn')))

\037

1.15.
Operations

on Continuous

Functions)

85)

the preceding example,the numerator is continuthe entire number scale.As far as the denominator
is concerned,accordingto the theorem on continuity of a composite
function, it is continuous at points where the function u == I/stn x
is cont inuous, since the function cas u is continuous everywhere.
Hence, the denominator is continuous everywhere, exceptat the
we must excludethe points
points x ==kn (k an integer). Besides,
at \\vhich
cos(l/sinx)==0,
the points at which l/sinx ==
== (2p+ 1)n/2 (p an integer), or sin x == 2/[(2p + 1)
Thus, the
function f (x) is continuous every\\\\t'here
exceptat the points x ==k'JI
and x =
1)'1arc sin
+nn (k, p, n =0, 1,+ 2,
11:

(c) Just as

in

ous throughout

i.e.

(-

n].

...).

:f:

(2P;I)

Test the
.15.2.

(a)

== cosxn,

cos log x;
r
2
y ==V 1/2 cos x.

(b) y
(c)

functions for continuity:


following composite
where n is a natural number;

==-

Solution. (a) We have a composite function y ==casu, where


1l ==x . The function y ==cosu is continuous at any point u, and
the function u ==Xll is continuous at any value of x. Therefore, the
function y ==cosxn is continuous throughout the entirenumb er scale.
2 where u==cosx.
2
The function
,
(c) Here Y==1
is defined and continuous on the interval
V
V2/2L the
Junction u ==cosx is continuou s througho ut the entire number scale.
2 x is continuous at all
values
Therefore, the function y == VI/2
{)f x for which
12

/ 1/2-u

(-

2/2,

Vl/2-u

-cos

cosx

r
\037

.15.3.For

_.

n/4 + 2nn
5n/4 + 2J1n

2/2, i.e.t

JI

\037

\037

\037

\037

each of the following functions

discontinuity and determine their character:


==

2 ' \\\\'here
U'l.+ll1

(a)

(b)

y=u 2,

(c)

==

where

1J.. ,
u2

\\\\'

u ==

forx\037O,
fx-l
x+ 1 for x < 0;

here

u == tan

u\037

I)

SoluiiO/l.(a) The

x-I;

function)
u ==<p

x.

(x) ==x-I)))

3rr/4 + 2nn,

7n/4 + 2nn.

find

the points of

Ch.

86)

/. Introduction

to Mathematical

Analysis)

1.The function

suffers a discontinuity at the point x==


]

y==f{u)=u 2+ u-)2
u2
find

suffers a discontinuity at points \\vhere


and U 2
Using these values of ll,
of x by solving the equations:)
===

1.

+u-2===0,i.e.u t ===-2
the correspondingvalues

-x-I')
-2=x-I' 1--'
I

whence x == 1/2 and x == 2.


Hence,the composite function is discontinuous at three points:
1/2,x2 == 1, x3 ==2. Let us find out the characterof discontiXl
nuities at these points.)
\037

hm

lim

===

x\0371

therefore

X2 ===

==

0,

\302\2431-00)

1 is a removable discontinuity.)

lim y==
x-l/2

hence,the points XI
kind.
Given
1.15.4.

lim
u-\037-2)
===

the

!im y

y===oo;

1/2,

Xa

===

===

lim

-. y

== 00;

1)

2 are discontinuities of the second

ion f (x)

funct

\037

===

1-x).

1/(

discontinuity of the composite function

FiJr1d

the

points of

y==f{I[I(x)]}.)
Solution.The

po-int

x =-= 1 is a

discontinuity of the function


1

v\037t(x)===-.
I-x)
x =I=-

If

1, then)

Hence,the
1 [I(x) J.

x-

u==f[f (X) ] = I-I/(I-x)=\037'


point x ==

\302\260

is a

discontinuity of the

functi-on

\037

If

x =I=-

0, x *-1, then
[I(x)]}= I-(x-l)jx==x
I

===

f {f

is continuous everywhere.
Thus, the points of discontinuity of this compositefunction are
x ==0, x == 1,both of them being removable.)))

1.16.
Funct.

\037

Cont. on ClosedInterval:

87)

Properties)

1.16.The Properties
of a Function Continuouson a
ClosedInterval.Continuity of an InverseFunction
I.
function f (x), continuous on the int0rval [a, b], possess-

\037

l\037he

es the

fol1o\\\\'ing

properties:

(x) is bounded on [a, b];


f
(2) (x) has the minimum and maximum values on [a, b];
(3) If In == min f (x), M == max f (x), then for any A sat isfy-

(1) f

a<x<b

a<.x<.b

A
M there existsa point Xo E [a, b] for
ing the inequalities m
which f (XI)) = A.
then we can find a point
In particular, if f (a).f (b)
c (a c b) such that f (c)==O.
I Continuity of an Inverse Function.If the function y ==f (x) is
defined, continuous and strictly monotonic on the interval X, then
there existsa single-valued inverse function x == (y) defined, continuous and also strictly monotonic in the range of the function
y -= f (x).
1 ==0 have a root?
Does the equation
\037

\037

< 0,

< <

I.

cp

1.16.1.

Solution.The function

sinx-x+

f (x)== sin

x-x+1

;s continuous over the entire number scale.Besides,this function


changessign, since f (0)== 1,and f (3Jtj2) == 3nj2.Hence,by prothe interval [0, 3nj2] there is at least one root
perty (3) within
of the given equation.
Has the equation -18x
1.16.2.
+ 2 ==0

interval

[-1, I]?

xC>

roots belonging to the

1.16.3.

Prove that any algebraicequation of an odd po\\ver


real coefficients)
a oX 2n + I + a I X 2n +
+ a2n x -t- a 2n 1 ==0
has a t least one reaI root.
Solution.Considerthe function
211
2n
f (x)==Q X +-1 + a 1 x +
-t-a 2n x-tQ2n+l'
\302\267

..

is continuous throughout

Let, for

determinacy
f

the number

sake,ao > O. Then

== -1-00,

x-+oo (x)
Ii rn

ith

(*))

...

\\\\hich

-t

\\\\

and

Ii In
X\037-oo)

scale.

f (x) ==

- 00.

find numbers a, b, a < b such that


f (a) < 0;
property (3), between a and b there existsa number
c such that f (c) ==0, which proves that the equation (*) bas at
least one real root.)))

Hence, we can

f (b)

> O. By

Ch. /.

88)

1.16.4.
Let

the
val

the function

equation f (x) ==

[a, b]. Arrange

Prove that

in

10 Malhematiral

Introduction
f

have a

\302\260

them

(x) be continuous on [a, bJ and let


number of roots on the interfinite

order:

the ascending

in

Analysis)

... < < b.

a < Xl < X2 < X:} <


each of the intervals

...
X'l

, (X1l' b)
(a, Xl)' (Xl' x2), (X 2, Xa ),
the function f (x) retains the same sign.
Solution.If the function changed its sign on a certain interval,
then \\ve could find one more root of the function, \\vhich contradicts
the condition.To determine the sign of the function on any of the

indicated intervals it is sufficient to compute the value of the function at an arbitrary point of the appropriate interval.
Given a function on the interval
2, + 2]:
2 2
x
x
if
<
0,
+
f
f (x)= _(x2 + 2) if
0 x 2.
Is there a point OB this closedinterval at which f (x)= o?
Solution.At the end-points of the interval
2, +2]the given
function has different signs:

1.16.5.

[-

-2

\037

\037

\037

[-

(-2)== +6; f (+2)==-6.


zero at any point
But it is easy to noticethat it does not become
of the interval [-2, +2].Indeed, x2 +2 > and -(x2 +2) ;:0
has a
at
this is due to the fact that
at
f

\302\260

any x;

the point

x == O.

Does the
1.16.6.

function

-sin

(x)

discontinuity

3
3
1tX
f (x)==x /4
within
the interval

take on the value 2


\037

[-2,2)?
-sin + 3 is

JtX
x
continuous
funct ion f (x)
the interval
Furthermore,at the end-pointsof this
interval it attains the values)

Solution. The

== 3/4

[-2,2].

within

f(

< 2+ < 5,

2) == 1.; f (2) ==

5.

property (3), withi.n the interval


x such that t (x)= 2+
2) there existsat least one

Since

[-2,

then,

by

\302\267

p\037)int

1.16.7.
Show that

the function
2X

t (x)=

2x

{2

for
for

-= 0, < 0,
1

\037

1 f()r 0

<X

\037

1,)))

\037

Funrt.
1.16.

Cont. on ClosedInterval: Properties)

[-1,1],has neither

defined and bounded on the interval

values.

nor minimum

89)

[-1,

maximum)

Solution.In the interval


0) the function increasesfrom 3/2
it increrlsesfrom 0 to
to 2 and in (0,
it doesnot attain either
the va lue 2 or O. Therefore the function is boundedbut never reaches
its upper and lo\\ver bounds. This is because there is a discontinuity
at the point x ==o.)

1]

.16.8.Show

1,

that on any interval [a, b] of length greater than


E (x) attains its minimum value but
the function f (x) ==
never reachesits maXilTIUm.
Solution. In any interval In, 11 1), \\vhere n is an integer,the
never attaining the maxigiven function f (x) increasesfrom 0 to
1
for
mum. lienee,0
f (x)
any
x. Sinceon the interval la, b] we
can find at least one internal integral point n, then f (n) == and
but
lim f (x) -=
f (x)=I=I for any
t

x-

unity

<

\037

\302\260

1,

X-+/l-Q

1,

x)

the function reaches -2 -1 a)


2) 3)
never
value but
its lllinimum
Fig. 32)
reaches its maximum. This is because there is a discontinuity at the point x ==11 (seeFig. 32),

x. It

means

that

1)

Prove that the function y == 2n+ V-X (n a naturai number)


1.16.9.
the number scale,consirleringit as a function
is continuous throughout
1.
inverse to =

X21l+

Solution.The .function y ==X 2ll + is continuous and increasesfrom


00 to 00 over the entire number scale.Hence,the inverse function
x + t/y is defined for all y, continuous and increasing.Denoting
J

21/

===

the independent variable again as

'
2n+l/v x possessesth e requIred

===

1.16.10.
Prove
y

that

= aox

2n+l

scale.

Solution.As

we find
propert lese

that

for any function of the


a l x2n-l a 2x2n-3

...,
...,
.

\\\\:here all' a p a 2,
an inverse function

ber

x,

the function

form)

+ . . +allx + an +1')

(*))

\302\267

all' a n + 1 are positive numbers, there

increasingand continuous throughout the

is known, the functions

x, x3, x5,

.. .,

X211+1

exists
nurn-

incrfase

entire number scale. Then, since the coefficients


throughout
n + 1) are positive, the function f (x)==aox + +
a i (i == 0, 1,
.
. .
.2IzI
-L
F ur th ermore,I t IS con+ a1x +
aI/x + a ll + a I so Increases.
tinuous. Therefore, for a function of the form (*) there exists an
inverse function increasing and continuous over the entire number
the
\"

sca

Ie.)))

211

Ch.

90)

/. Introduction

to Mathematical

Analysis)

of an inverse
Note. This exampleestablishesonly the existence
function x ==g (y), but gives no analytic expressionfor it. It is not
always possible to expressit in radicals.The problemsof the existence of an inverse function and of expressingit analytically should

be confused.

not

Prove
1.16.11.

x==x(y)

(-00 <

existsonly

there

that

< 00) which

one continuous function

satisfies the Kepler equation:

x-8sinx==y(0<8<1).

Solution.Let us show that

Xl < X2
y

(x2)

y (x) is an increasingfunction. Let


be arbitrary points on the number scale.Then
==(X2 8 sill x 2) (XI
e sin Xl) ==
Y (Xl)

- -- -

--

== (x2

X I)

--8 (s

in

Estimate the absolute val ue of the difference I SIn


I

SIrl

x2

-sin

XII

= sin
21

.
SIn

Since Y (x)

IS

-;

Coc
/2tXI

II
I

X2

\037

-- 8

(X2

XI

1,

whence)

---2

Xl

2
21
Since0 < e <
\037

XI)

Xl

sin Xl).
sin xII:

\037

X2

-X

sin xII< (x2

sin x2

xx2

== (X 2

-X ).
L

Xl)')

Y (XL) > O.
the interval (--00,00),the
inverse function x is a single-valuedand
continuous function of y.

8 (sin x2

sin Xl)

continuous function

==Y (.'<2)

in

-+

1.16.12.
Show that

the equation
X3
3x 1 ==0
has one root on the interval
2].Calculate this root approximatelyto within

two

[1,

decimal places.

1.16.13.

at

\\vhich

The function f(x) is defined


on the interval [a, b] and has values of
the same sign on its end-points.Can one
Fig. 33
assert that there is no point on [a, b 1
the function becomes zero?
Prove that the function

1.16.14.

f (x) ==

\\

x+ 1 at -l\037x\037O,
X

at

0 <x

\037

is discontinuousat the point x ==0 and still has the maximum


1] (see Fig. 33)
the m in i m urn val ue on

[-1,

\302\267)))

and

Additional
1.17.

\037

\037

Additional Problems)
1.17.
Prove the
1.17.1.
(a) n!< ( nt r for a natural
2n
3 5

Problems)

91)

inequaliti<.\037s:

(b) _
2

_ _ ...- <
I

\302\267

\302\267

2n

11

> 1;

'
V 2n i- I

1.17.2.

Prove the inequalities:


202
(a) 202303 > 303 ;
(b)

200.
200!< 100

1.17.3.
Solve the
(a)
(b)

inequalit ies:

I;
Ilxl-21\037

> 2;
-11
112-3xl
2

(c) (x-2)V- x + 1 > -1-2.


Can a sum, difference, product or quotient of irrational
numbers be a rational number?
x\037

1.17.4.

Do the
1.17.5.

equations

(b) Itanxl-=tanx+3
(a) Isinxl==sinx+3,
have any roots?
Prove the identity X+JXI
x-JX!

1.17.6.
Prove the
1.17.7.

r =x .

r +(

Bernoulli inequality
1 +X1 +X 2
+xn ,
(1+xt )(] +x2
(I+xn)\037
X n are numbers of I ike sign, and 1 + Xi
where Xl' X 2'

)...
...,

+...

...,

1,2, n).
Find the domains of definit
1.17.8.

(i ==

ion of the following

(a) f (x) ==VX 3_X ;


2

\"

(b) f (x) = -.Isin V-X

(c) f (x)

==-

(d) f (x) =

--

sin2

Jtx;

Y Ix\\-x

and g (x) =

1-3);

(e) f (X) ==arc sjn (I x


(f) f (x) ==arc cassIn x

Y x-Ix\\

-J- .

Are
1.17.9.

(a) f (x) =
\037b)

\037

the

follo\\\\ling

and

(x) = log

x2

cp

(x)

and

cp

functions identical?

:=];
(x) ==2 log

x;)))

>0

functions:

Ch.

92)

I. Introduction

to Mathematical

(c) f (x) =x and (x) = (V x


2
2
x;
x+cos
(d) f(x) == I and cp(x)==sin
(e) f (x) == log (x-1)+ log (x-2) and
cp

)\037;

(x)==log (x

- (x-

1)
2).
interval are the following functions identical?

1.17.10.
In what

(a) f (x) ==x and

Analysis)

cp

(x)

cp

1Olog x;

===

(b) f(x)=Vx J/x-1 and fp(x)==Vx(x-I).


1.17.11.
An isosceles
triangleof a given perimeter2p 12revolves about its base.Write the function V (x), where V is the volume
-==

solid of revolution thus obtained and x is the length of the


lateral side of the triangle.

of the

1.17.12.
Inves.tigating

(a) solve the inequa lity

the domain of definition of functions)

J/x-t2 + Vx-5

\037

prove that the inequality

(b)

V5-x;

(x-3)\037-5)

log2_x

has no solu t ions.

1.17.13.

The function y==signx was defined in Problem


Show that

1.5.11
(n).

(a) x ==x signx;


I

(b) x == x sign x;
(c) sign(signx) ==signx.
I

1.17.14.
Prove that

for a linear function

if

f (x) \037ax+b

1, . . .)
= f (x,J (n == 1,2, ...)

the values of the argument x Xn (n = 2,


form an arithmet ic
progression,then the correspondingvalues of the function
\037

Y 1l

also

form an

arithmeticprogression.

1.17.15.

Prove that the product of two even or t\\\\'O odd funct ions
is an even function, whereas the product of an even and an odd
function is an odd function.

1.17.16.Prove that

if

the domain of definit ion of the f unct ion

f (x) is symmetrical with respect to x ==0, then f (x) + f


even function and f (x)-f x) is an odd one.

(-

(-x) is an

t.17.17.Prove that any function f (x) defined in a symmetrical


(-i,I) can be presented as a sum of an even and an

interval

odd)))

\037

Additional
1.17.

Problerns)

function. Rewrite the following functions


an even and an odd function:
x -r- 2
(a) f (x) =: I TX'\".) ;
I

(b) y

=.=:

ax

in

93)

the form of a sum of

\302\267

1.17.18.
Extend the function t (x) ==X2+X defined on the intero
[0,3]nto the interval [-3,3]in an even and an odd way.
1.17.
19.The function {x}==x-E (x) is a fractional part of a
number x. Prove that it is a periodicfunction
period 1.
1.17.20.
Sketch the graph of a periodicfunction with period
T 1 defined on the half-open interval (0, 1]by the formula y.== x2.
Let us have two periodic
1.17.21.
functions t (x) and (x) defined
val

\\\\'ith

===

set.Prove

cp

that if the periods of these functions are


functions.
commensurate,then their sum and product are also periodic

on a common

1.17.22.Prove that the Dirichlet function


1.I 4.4(b)) is a periodic one but has no period. (x)
1.17.23.
the function
Prove that
A

(see Problem

if

sin x + cosax
is periodic, then a is a rational number.
t (x)

Test
1.17.24.

=-==

the following
(b) t (x) == x

funct

1-

ions for monotony:

x.
(a) t (x) == x I;
Prove that the sum of t\\\\'o functions increasingon a
certain open interval is a function monotonically increasingon this
interval. Will the difference of increasing functions be a monotonic
function?
I

1.17.25.

t.17.26.Give an exampleof a
1.17.27.
Determine the inverse

an inverse.)
if

finition

(a)

-- tan h

x;

(b)

1.17.28.
Show that

rea]

roots.

1.17.29.
Construct

if

x2

if

{ 2x

if

\037

non-monotonic function that has


function and its domain of de-

-oo<x<l,
1 x
\037

\037

4,

4<x<oo.

the equat ion x2 -t- 2x + 1 ==


the graph of the function
y

-==

(x-I)+ f (x +

I),)))

--1 + V x has no

Ch.

94)

/. Introduction

where)

f (x) =

k(l-Ixl/l)at

\\

1.17.30.
Knowing
Y

==f2

(x);

at

Analysis)

Ixl\037l
x
t.
I

>

the graph of the function Y==f(x), sketch the

graphs of the following


(a )

to Mathematical

(b)

== V

ions:

funct
r
f

(x) ;

(c)

==f

[f (x)]

t.t 7 .31.Prove that the graphs of the functions ==loga x and


x can be derived from each other by changing all ordinates
in the ratio I:l/n.
Prove that
the graph of the function y=f(x), defined
1.17.32.

== logan

if

throughout
axesx ==a

the number scale, is symmetrical about two vertical


and x ==b (a < b), then this function is a periodic one.
Let the sequencexn converge and the sequenceYn diverge.
be said about convergence of the sequences

t.t 7 .33.
can

What

(a)

+ Yn;

Xn

t.

(b) xnYn?

Xn and Yn diverge.Can one assert that


t 7 .34.Let the sequences
the sequencesXn + Y'l' XnYn diverge too?
t7
Let an be an interior angle of a regular n-gon (n ==3,
Write the first several terms of the sequenceall' Prove
4,

t. .35.

...). n.

tha t Iim an ==

.36.Prove
1.17

that from

Is the converse true?

Hln X n ==a it

n-oo

I.17.37.If a sequencehas an
sequenceis unbounded?And if
mean that

it

has an

unbounded but not an

t. .38.

t7
Prove
of an arbitrarily

that

infinite
infinite

follows tha t lim

Xn ,

n\037oo

-==

,a I.

infinite limit,does it mean that this


a sequenceis unbounded, does it
limit? Prove that Xn ==n(-l)1t is an

function.

the sequence{an},where an is the

nth

digit

chosen irrational number, cannot be monotonic.


t .t 7.39.Prove that if the sequence{an/bn } (bn > 0) is monotonic,

then the sequence)

will

\302\267

\302\267

also be monotonic.

I.t 7 .40.Prove

and

+ + ..+an
+ .. +bn }
{
a2
at
b i + b2

the
-

(a)

\037I

existenceof

them.)

find

2-, V 2

\037/

T,

limits of the following

2 V 2 J/ 2

...
;)))

sequences

AddU ional
1.17.

\037

(b) xn
(c)

Xll

95)

(c > 0, k > 0);


where
digit of the number n.
alJ is the nth
a,)n,
Prove that at an arbitrarily chosen x the sequence
ell /

===

==

P roblflns)

V n!

1.17.41.

\037nx)

} is bounded.

Prove that
1.17.42.

the sequence
J E (x) -i-E (2x )

.
\037-

n l)

\\

..

+ E (n x)

has the limit xj2.


Prove that)

1.17.43.

lirn
h
\037

Given
1.17.44.

the

===

{
\037

\037

Let)
1.17.45.

a oxn
buxlll

0,

I -1-x for x
o for x ==O.)

that)

P (X ) ==

> 0).
=-;1=

-==

f (x) ==

Ii in

Prove

1 (a

0)

unct ion

f (x)

Prove

ah

a 1xll
b
-t- 1Xlll
-+

1.

0)

- -t-t- .. .. .. -t- ball


1

-4

(au

=1==

X-+

1.17.46.
Find
(a)

P (x) =

the

00

=1=

0).)

(X), if n
Q(Jbo, if n

> In,

m,
. n<nl.)
0
,t
{
===

constants a and b

21 ax::1
-x -ax-b)
b

(
(J;'x2

}\0371\037

\037

from

-\037

00)

1
1

Il -+

XU

the condition:)

0;

==O.)
+1
Sketch t he gra phs of the follo\\vin2
1.17.47.
0);
(a) f (x) == Iim / 1 + (x
lilJ1

0; bo

/IJ

that)
linl

(b)

functions:

\037

00

(b) f (x) = !im


Il _

sin211 x.)

Prove
1.17.48.

that

00)

n-oo[(1+ x) (1+
lim

..

+x4) . (1 -i-x 2\]") =

x2) (1

\037

x)))

(I

x!< 1).

Ciz.

96)

Can
1.17.49.

I. Introduction

one

to Mathenlatical

replaceinfinitesimal

Analysis)

summands

by

equivalent

inHnitesimalsin computing a limit?


1.17.50.
Determine the order of smallness of the chord of an
small
circulararc relative to the sagitta of the same arc.
infinitely
1.17.51.
Determine the order of smallnessof the difference of the
regular n-gons relaperimeters of an inscribed and circumscribed
tive to an infinitely small side of the inscribed n-gon.
The volumetric expansion coefficient of a body is con1.17.52.
sidered to be approximately equal to the triple coefficient of linear
expansion.On equivalence of what infinitesimals is it based?
Does the relation log (1+x)\"'\" x hold true as x\037O?
1.17.;)3.
1.17.54.
Wi 11 the sum of t\\\\,O functions t (x) + g (x) be necpssari
ly
discontinuousat a given point Xo if:
(a) the function f (x) is continuous and the function g (x) is discontinuous at x==x
o,
(b) both functions are discontinuousat

x= xo? Give someexamples.

Is the product of t\\\\,O functions f(x)g(x) necessarily


1.17.55.
discontinuous at a given point Xo if:
(a) the function f (x) is continuous and the function g (x) is discontinuous at this point;
(b) both functions f (x) and g (x) are discontinuous at x = xc?
Give some examples.
Can one assert that the square of a discontinuous f unc1.17.56.
of a function is also a discontinuousfunction? Give an example
tion discontinuouseverywhere whose square is a continuous function.
t.17.57.Determine the points of discontinuity of the following
functions and investigate the characterof these points if:
I

(a) f (x) == 1 _ e x/( 1- x) ;

(b) t (x) ==- 2


(c)

- 2 1/( -X);
1

where A (x)
cp(x)=x[I-2A(x)],
1.14.4

Problem

1.17.58.

is the

Test the following unctions


their graphs:
E (x);
(a) y ==
f

x-

(b)

y=x 2 +E(x2);
2

(c) y==(_I)E(x);

(d) y=

tim
fl

-+

00

Dirichlet function (see

(b)).

1+(2n
81

x)))

)2n

'

for

continuity and sketch

1.17.
Additional

\037

1.17.59.
Investigate the
if

nuity

Problems)

97)

functions f [g(x)] and g [f (x)J for conti2

f (x) = signx and g (x) = X


Prove that the function

1.17.60.

2x

f(x)={ X+l/2

(l-x).

at

-1 x
\037

\037

0,

O<x\037l

at

is discontinuousat the point x = 0 and nonethelesshas both maxivalues on

mum and minimum

1.17.61.
Given the

[-I,1].

function

-_ { (x+1)2-(1/1

x 1+ l/X) if X

f (x)

1f

[-2,

*-0,.
0

Ascertain that on the interval


2] the function takes on
all intermediate values from f
to f (2) although it is discontinuous (at what point?).
Prove that if the function f(x): (1) is defined and monotonic on the interval [a, b];(2) traverses all intermediate values
between f (a) and f (b), then it is continuous on the interval [a, b].
Let the function y = f (x) be continuous on the interval
its
range
being the same interval a\037y\037b. Prove that on
[a, b],
this closed interval there exists at least one point x such that

(-2)

1.17.62.

1.17.63.

f (x) ==X.

Explain this geometrically.

1.17.64.
Prove

that if the function f (x) is continuous on the


X n are any values from this open
(a, b) and Xl' X2,
interval, then we can find among them a number S such that

\"',

interval

f (s)== n) [f (Xl)

+ f (x2) +

\302\267

\302\267

\302\267

+ f (Xn )].

1.17.65.

Prove that the equation x 2x = 1 has at least one positive root which is less than unity.

1.17.66.

Prove that if a polynomial of an even degreeattains at


least one value the sign of which is opposite to that of the coefficient at the superior power of x of the polynomial,then the latter
has at least two real roots.

1.17.67.
Prove
2

y ==(I

4--3 ,

-1

8)))

+x )

that

the inverse of the discontinuous function

sign x is a continuous function.)

Chapter2)
DIFFERENTIATION
FUNCTIONS)

OF

\037

2.1.Definitionof the Derivative


The derivative

f' (x) of

the function y ==f (x) at a given point x

is defined by the equality

'

f (X) =

I 1111
\037x

\037

11 y

== I lrn

0 I1x

\037x

\037

(x

+ I1x)

t (x)

I1x

If this limit is finite, then the function f (x) is calleddifferentiable at the point x; and it is infallibly continuous at this point.
Geometrically, the value of the derivative f' (x) represents the
slope of the line tangent to the graph of the function y ==f (x) at
the point x.
The number)

'

f + (x) =

Iim
\037x...

+0

(x+I1x)
I1x

(x)

is calledthe right-side derivative at the point x


The number)

f'- (x ) = r n:0
\037x

(x

+I1x)
I1x

(x)

is calledthe left-side derivative at the point x.


of the
The necessary and sufficient condition for the existence
derivative f' (x) is the existenceof the finite right- and left-side
deriva tives, and also of the equality (x) f (x).
If f' (x) 00, the function f (x) is said to have an infinite derivative at the point x. In this casethe line tangent to the graph
of the function y ==f (x) at the point x is perpendicularto the
x-aXIS.
2.1.1.Find the increment l1y and the ratio for the following
functions:
===

f\037

\037

===

\037\037

(a) y

===

I1x==0.000
1;
x= 1 and I1x==O.2.

V.x at x ==a

1
(b) y== x 2+ x-)))6 at

and

2.1.Definition

\037

Solution.(a)

==

\037y

\037I

of the Derivative)

99)

= 0.01;
x-t- \037x- VX- V0.0001
0.01
l1y =
= 100.
0.0001
\037

L\\x

2.t .2.Using the

definition of the derivative,

the derivatives

find

functions:
=
cos
ax; (b) y ==5x2
(a) y
Solution.(a) = cosa (x+\037x)-cos
ax=
= 2 sin ax+

of the following

-2x.

\037y

i i

-2sin ( ax+

!1y

sin
!1x
)

L\\x

. .

,
== 2 1 un sin ax a
r1m
Y = 1 1m
.1x 0
.1x 0 X
( +\"( ) -+ 0
In particular, if a-==l, then
and
\037

.3.

sin
\037

x;

.
')

.
sin

\037x

\037

-+

!1x

Llx)

- 2.1x
y=cosx y'==-sinx.

L\\x

.!J.q-

a
2

LlX

I1x

-=

a sin

ax.

2.t Show that the following functions have no finite derivatives at the i n diea t ed poin t s

the point x =0;


(b) y
V 1 at the point x == 1;
==
(c) y 3/ x 1+1 at the point x == O.
(a)

== V Xi at

!f)

x-

===

t.S:.

5/-

== V5/ (x + \037X)3_ V x3.


Solution. (a)
At x = 0 we have Lly=
V Llx3 !1y= VLW\" =
\037y

'l1x

L\\x

tion\037y

\037x

hence,)

\037

\037)

00,

-1 0
.
Fig. 34

is no finite derivative.
\037x

>0

atx==O

the increment of the funcwill

<0

+0

the increment of the function


3 (0 +
+ 1=
Lly =
\037x)

\037

1-

Lly

be

3Llx,)

L\\y

L\\x)

Sincethe one-sidedlimits are different, there


(see Fig. 34).)

4*)))

will

- :=_3.

Hm

.1x
the point x ==0

x')
1)

be: \037y==3 (0+\037x)+1-1=3Llx.


Therefore)
.I Iln l1y = 3.
I1x)
.1x_

At

\037

\037

i.e.there
(c) At

\\

(j.:)

L\\x)

= !im
=
= /\037
2 ; hence,y' (0) .1x
I1x2
0 V\037
v
1.

\\\\

IS no derivative at

Ch,

100)

II. Difjerentiation

2.1.4.Investigate the
1.

the point x ==

function y == Iln x for differentiability at


I

Solution.At x == 1
== Iln(1+
\037y

1.e.)

\037

y == Iln

(1+

In

(I

at
{ -In(I+\037x)

L\\x)

at

I1x

A
uX

{_ In(\037:I1X)

at

Iim

_+

\037y

LlX

= + 1 and

Jim

.1x_

\037

0,

\037x<O.

> 0,
,1x<O,)

whence)
I1x

I,)

\037x)

\037x

\037x)

1==

\037x)

In

=
I1x

11= Iln( 1 +
at
In (1+

I-I

\037x)

Therefore)
Ay

of Functions)

\037y

LlX

= 1.

Since the one-sidedlimits are different, there is no derivative.

Hence,the

function y == , In

x is not differentiable
x == 1 (see Fig. 35).
I

at the point

2.1.5.

Find the average velocity of


motion specified by the formula
s == (t 2
+ 2) m
from t 1 == 5 sec to t 2 == 15sec.
the definition of the
x derivative,Using
find the derivatives of
o 1
the following functions:
3
2
Fig. 35
(a) y ==x; (b) y l/x
Investigate the function y==lcosxlfor differentiability at
the points x == n/2 + nn (n an integer).)

-5t

2.1.6.

\037

2.1.7.

\037

2.2.Differentiation
of ExplicitFunctions
I. BasicRules of Differentiation
( 1) c' 0;
(2) (u + v)' =u' + v';
(3) (cu)'==cu';
(4) (uv)' '==u'u + uv', the product rule;
u
u'v-uv'
(5) ( v ) = v2 (v 0), the quotient rule.
-==

=#=

Herec==const, and u and v are functions of x which have derivatives at a correspondingpoint.


(6) If the function u == (x) is differentiable at the point xo, and
the function y = f (u) is differentiable at the point U o ==cP (xo), then)))
cp

\037

2.2.Differentiation

Functions)

of Explicit

101)

the compositefunction y ==f (cp (x)) is differentiable at the point Xo


(xo) == (u o) (xo), the function of a function, or chain, rule.
I I. Differentiation of Basic Elementary Functions

and

y\037

(I)
(3)

u\037

y\037

(un)' ==nun-tu'; (2)

(sin u)' ==cosu.u';

(cosu)'=-sinu.u';
u'

(4) (tan u)' ==COS 2 U ; (5) (cot u)' ==

u'

.
SIn2 u

u'

(6) (In u)' \037u;


(7)

8) (eT1)'=eUu';
(aU)'==aulna.u';

u.u';

(9) (sinh u)'==cosh

1+ =

(10) (coshu)' ==sinh u.u';


u'
(11) (arc sin u)' = y l-u.2 =
(12) (arc tan u)' ==

u'

(a) y ==5X 2/ 3

(b) y =

-3x/

x2
v-:-

(arccotu)'.

u2

2.2.1.
Find y', if:

5 2

(a rc casu)';

+ 2x-3;

s).
v- (a, b canstan
-1
t

Solution. (a) y' = 5.

-I; - .

X 2/ 3

_ 3.

Xb / 2

-I

\037

-2.3x-3-1=

\037.\037-

x Vx

:4
Find y', if:
2.2.2.
n x + casx
(a) y ==3 cosx+ 2 sin x; (b) y == sIn
cosx ;
3 arcsinx.
2
(d) y==x
(c) y==(x +I)arctanx;
==
3
Solution.(a) y'
(cosx)'+ 2 (sinx)'== 3 sin x+ 2 cosx;
cos
cosx) (sin
cosx)'(sin x+cosx) =
, (sin x+ x)'(sin
(b) Y ==
(sin x-casX)2
s\037

x-

x-

x-

x+sin x) (sin x+cosx) _


_ (casx-sin x) (sin x-cosx)-(cos
-)
(sinx-cosx)2

(sin

(d) y' = (x3)' arc sin x+ (arcsin x)'x 3 = 3x2 arc sin x+ y

2.2.3.
Find the

2
x-cos .

I\037

x)\037

x2

derivative of the given function and then comthe


particular value of the derivative at the indicated value
pute
of the argument:
2
at x==-8;)))
(a)
v x

f(x)==I-3/-+16/x

Ch.

102)

II. DiUerentiat

ion of Functions)

x )2/x at x==O.OI;
(b) f(x)==(1-V
-==
1
sin t) at t =:n /6.
c
( ) f (t) (cost) /(

Putting x ==

'

(c) f (t )

_
f'(x)==-32 x-t/3-I6x-2==32/_ x .
3V x

-8,

Solution.(a)

1\037

we obtain

r-

2
f'(-8)==-3 3V-8
-sint (i-sint)+cos2 t =

(1==

sin t)2

-12;

f' (n/6) 2.

Whence

16

(-8)2
sin t

2.2.4.Taking advantage of the differentiation formulas,


dprivatives of the following functions:
(a) y=2x3 +3x-5;
(c)

-_ -+ + .'
2X2

(b) y=V:X+

(e) y==

x2

1
x
x -t- I

cos<p+ s-in
I
cos

(d) y=

x+ sin x);)

(h)

=-=

;x +O.lx ;
1o

x+Yx

<p

(g) y ==eX (cas

the

x-2V-x ;

x
(f) y ==2e

<p

find

eX

n x;)

+ sin
xe)

'

2.2.5.Taking advantage of the rule for differentiation of a composite function find the derivatives of the following functions:
(c) y==5cosx;
(b) y==lntanx;
(a) y==sin:Jx ;
2
3
(d) y == Insin (x + 1); (e) y ==arc sin V 1 x ;
2
(f) y == In& (tan 3x);
(g) y -= sin 1 1/(I-x).
Solution.(a) Herethe roleof the external function is played by
the power function: sin x is raised to the third power. Differentiating
this power function with respectto the interrnediate argument

(sinx), we

obtain)

3
(SIll X ) sin

2 .
x ==3 SIn x,)

intermediate argument sin x is a function of an independent


variable x; therefore we have to multiply the obtained result by
the derivative of sin x with respect to the independent variable x.
Thus, we obtain)
.3 / . ' .2
,
.
Yx == (SIn x)sin x (SIn x)x == 3 sin xcosx,)
but the

=. -

2
,
' I 1
(b) yx ==(1n t an x )tan x (t an x )x == tan x coS2 X sin 2x .'
== 5cosx In 5
sin x)= 5C05x sin x In
x (cos
(c) == (5
COS

y\037

x)\037os

x)\037

(-

5;)))

2.2.Di{lerentiation

\037

(d)

(e)

10

Functions)

of Explicit

3)

[Insin(x3 + l)]'sin I +1)[sin(x3 + 1)]\0373+1 [x3 + 1];=


= sin. (x3 + I) cos(x3 + 1).3x2
-= 3x2 cot (x3 +
== (arc s-1n V 1 x2 ) _ x2 (V I
x2) _ x2 ( 1 x2); =
1
(X

y\037==

y\037

IV

I-(I-x) .2 Jf
1

Jf

2.2.6.
Find

\302\267

<

(-2x)==_ Ixl x 1-x2

I
l-x\037

,
1)'

(x =#: 0)

\302\267

\302\245

the derivatives of the following functions:

(b) y=(3-sinx)3;
2
s
+ 1/co x;

(a) y==(1+3x+

5x\037)4;

(c) Y == V sin2 x
== V2ex

(d) y
(e)

+ 2x + 1-t-Inx;
D

-+

x;
y=sln3x+cos(x/5)+tanV
2

(f) y

-==

sin (x

5x I) + tan (a/x);

(g) y ==arc cosVx;


In (arc tan
(h) y ==arc tan (In x)
2 arc tan
==
In
(i) y
(x/3);

(j)

x);

V x + -Vx + V x.

2
2
Solution.(a) y'==4(1+3x+5x
)3(1+3x+5x
)'=
==4(1+3x+5x2)3(3+lOx);
I
I
'
_=
;
(Vx ) ' ==
(g) Y ==

,1

VI-(Jfxr\037

(j)y'==

2 -V x + V x + Vi l

2.2.7.Find the
We
Y

1.e.)

2
2(1+x
)-4x:!
=

-./-I _
V

2x

\037

-1-x

\037

(l+X2)2

)
, ==

==
I

1 the derivative

2.2.8.
Find
(a)

-+

{
I

\302\245

y ==sinh

( 1+2 \037-x )] .

2x

y==arcsln 1 + x2

At

l+2 VI+

- Jfx(l-x)
1

2 Yx

derivative of the function

have

, ==

-Yl-x
_

2(I-x2)

x2

at

2
(l-x
)
2
2

=ll-x1(1-.-t-x)

Jf{l_X2)2(l+X2)
at

x2
2

\037

')

x I < 1,
x

> 1.

is non-existent.

the derivatives of the following functions:


5x cosh(xj3);)))

Ch.

104)

(b) y ==coth (tan x)

II. DiUerentiat

-+

+
esinh ax
bx-cosh bx

(sin6x);

\302\267

Solution.
(a) y' = (sinh5x)'cosh
(c) y' =

;+ = 5x( ;)'=
sinh

cosh

5 cosh5xcosh

\037

2.2.9.Find the
y=

(C)

==

+ -}sinh 5xsinh

x)'
cosh(sin 6x) (sin6x)'=
.)tanhtanh 2 x +

__

V (cosh2

x-sinh 2 x) /cosht x +

cosh

+ 6 cos6x cosh(sin 6x).

derivatives of the following functions:

i/-Vs=x
I-x .
x3 (X2 + I )

Vx

Ijcosh2 x

+ 6 cos6xcosh(sin 6x) =
(a)

ions)

tanh (cot x);

(c) y ==arc cos(tanh x) sinh


(d) y ==sinh 2 x3 cosh\037J x2;
(e) y ==sinh

ion of Funci

(u(x)>0);

(b) y=[u(x)]V\\XJ

SIn3 xcos2 x;

+ x2
(d) y=(\037/tanx)X+l.

Solution. (a)

the method of logarithmicdifferentiation.

Apply

Consider, instead of y, the fu nction


x 3 (x2 + I)

z=lnly)=InV V 15-xl=lnlxl+31n(x2+1)-T51nI5-xl.
I

\\

Taking into account that (In


zI
But

uJ

)' == u'ju, we have

-24x3 + 125x2 -14x+75


= x+ 3 (x2x
+
2
15
+ I) (5-x)
15x(x2 + I) (5-x)
1

z' ==(In

y r )' ==y'/y, whence

+1).
V

y '==y zl==

3 (X 2

V5

-24x3+125x'\037-14x+75
15x(x2

+ )(5-x)

\302\267

(b) Suppose the functions u (x) and v(x) have derivatives


given domain of definiti.on. Then the function
z == In y ==v In

also has a derivative

in

this domain, and

-.

z' = (v In u)' ==v' In u + v u'


u)))

In

the

2.2.DiUerentiation

,\037

Hence,the

of Explicit

105)

Functions)

function)
ln y

== eZ)

y ==e

also has a derivative

the indicated domain, and

in

y' ==eZz' == y z'

.)

Thus,)
V
y' = U (v' In u+ v ) =VUV-1u'+uv In u.v'.)
\037'

Show that
2.2.10.

the function y == xe-x2j2 satisfies the equation

(I-x)

xy' =

Solution.)

y.)

(1-x);

y' ==e-x2j2 _x2e-X2j2==e-x2j2


x2).)
xy' ==xe-x2j2

(1-

Hence,)

xy' ==y

2.2.11.
Show that the
xy' (I-x)y.
2.2.12.
Investigate the

(1-x).)
2

function y ==xe-X satisfies the equation

==.:

(a)

2.
(b) y=yl-Vl-x
y=arcsin(cosx);

sin x < O.
(k
f

(cosx)'
1cos
x
y
V
-1at where sin x > 0;

Sa1Ulan.
t. (a )
Hence,y' ==
-==

following functions for differentiability:

' ==

==

2X

points

points where sin x


+ 2,
the function,

At

...)

0, + 1,

===

sin

y'
0, i.e. at
though

sin

==

sin2

x
sin x

\302\267

== 1 at

points where

the points

continuous,is

x ==kn

not dif-

erent i ab 1e.
(b) The domain of definition of this function is the interval

-l\037x\037l.

-I-- (-2x)atx+Oandx+:i:1.
V_
1-0 x---+-l+0
y'\037+oo.
x 0, i.e.
e exists
- - . s.

-y'-2V

\302\267

VI

-'x\037

As X---+
or
out whether the derivativ
.
L\\x t
JI I VI
t her I 1m
L\\x
.1 0
,
1 \"'-/
SinceJI 1
X'

- -

-\037

. V
,1m

\037x

\037

i\\x

L\\x 2

we have
at the point

y'

whe-

then)

\037X2,

' r/
1m
1

==

Let us find

==

eXlS t

-\"2

\037X2

- -

x2

\037x'-+

r\"2

i\\x

i\\x 2

- -h
{

as
1

V2

\037x

as

\037

\037x

+ 0,
---+-0.)))

II. DiOerentiation

Ch.

106)

of Functions)

Thus, y'- (0) =1= (0), which means that the function under consideration has no derivative at the point x ==0, though it is cont inuous at this point.
of f' (x) and even of
Note. There are casesof failure of existence
i.
e.
when
the
at
a
and
graph of the funcgiven point,
(x)
(x)
tion has neither a right-, nor a left-side tangent at the given point.
y\037

f:\"\"

f\037

For instance,the function

at
f(x)= 0xsin(l/x) at
f

\\

is continuous at the point x ==


(x) = sin I
derlva t Ives, sInce\037
L\\x

. .

2.2.13.
Find the
(a)

\037f

0, but

x=;bO,
x ==0

does not have even one-sided

.)

derivatives of the

f (x)= sinh (x/2)+ cosh(xj2);

follo\\ving

functions:

(c) f(x)==2Vcoshx-l;
(b) f(x)=ln[coshx];
=
arc
sin
(d) f (x)
[tanh x ];
2
4x;
(e) f(x)=Vl+sinh
ax
=
sinh bx).
bx
e
+
(cosh
(f) f (x)

2.2.14.

Applying logarithmic differentiation


of the following f unct ions:
sin 3x
;
(a) y= (cosx)slnx; (b) y=.

find

the derivatives

V l-sin3x

(c)

Vx=I

'

Y=V (X+2)2Y(X+3)3

2.2.15.)

cos2 X

f (x ) = I +.sin 2 x
show

-3f'

that)

2.2.t 6. Show that

.
')

3.

(Jtj4) ::=
f (n/4)
the f unct ion)
y

= x-e-X\037
2x2

satisfies the differential equation


2
xy' + 2y = e-X .
Find t he derivativ es of the following functions:
2.2.17.
=
In
cosVarcsin 3- (x > 0);
(a) y
2\037

(b) y= Varc tan V

co\037

In 3

x.)))

\037

\037

2.3.SuccessiveDifferentiation.

Leibniz Formula)

107)

2.3.Successive
Differentiation
of ExplicitFunctions.

l.leibnizFormula

1)th order of a function y = f (x) is


If the derivative of the (n
already found, then the derivative of the nth order is determined

the equality

by

(x)= [y<n-l) (x)]'.


particular, y\" (x)== [y' (x)]',y'\" (x)== [y\" (x)]',and so on.
y(n)

In
If

u and v are functions differentiable n times,then for their


linear combination c1u+ c2v (c1, c2 constants) we have the following

formula:)

(c1U

and for their product


(uv)\\n

= u\\tl)v+nuu,-uv'

uv

+ C2V)

<n)

C1u(n,

+ C2')
V(n)

the Leibniz formula (or rule)

+n

I) u\\n-2}v\"

+)

(\037.-;

+ ... + uv(n) == k=O


I) ...(n k + I) _

h
were

u (0)

_-u, v (0)-van
_
d

( 1) (x )

==m (m
== aX Inn

n (n

-1).>..0).- +

binomial coefficients.
m (n)

_
n-

Ck

C\037u(n-

k)V(k),

- -kJ(n-k)!
-}.2.3...k
\037

n!

are

Hereare the basic formulas:


n 1)xm Il .
(m

a (a
In particular, (eX)(n) = eX.
(2) (aX)(lZ)
l)
==
(3) (In x)(n)
)'Z-l(n-n
x
==
sin
(4) (sin x)(n)
(x+n'Jtj2).
(5) (casx)ln) ==cos(x + nnj2).

!.

(-1

2.3.t.
funet

(a)
(e)

Find the derivatives of the

order of the following

nth

ions:
y
y

x; (b) y ==ekx ; (c) y ==sin x;


2
== sin x casx; (f) y == sin 3xcos
x;
==

(d) y == sin 5x cas2x;

In

(g) Y ==In

(x

+ x-2).

Solution.
I

;
; y\"'==1.2xy'=X-=x-; y\"=={-1)x4
1.2.3x.
;
(n-l)!x-\"=(-I)Il-:,,(n-I)!
y(n) =(_I)n-l
y(4) '-cosin (x+nj2);
(c) y' ==cosx==
==cos(x+ nj2)==sin (x+ 2'Jtj2).

(a)

...;

y\"

In

general,if we assume that


y{k)

for a given

= sin x+ k

\037

n ==k

.)))

Ch.

108)

then

it

will

II. DiUerent iat ion

of Functions)

out that

turn

y(k+O=COS
(x+k ) =sin[(k+1)
\037

\037

+x] .

Whence, by virtue of mathematical induction we concludethat for


any nat ural

n)

).

y(n)=Sin(x+n
(d) y = sin 5xcos2x=+ [sin 7x+ sin 3X] .
\037

Therefare

= + 7n sin 7x + n + 3\" sin 3x+ n


)
[
(
(
)] .
2x+
,
Y = x2
+x-2'
yln)

\037

\037

(g)

To simplify the computations let us transform the obtained


function:
Y

(x+2)+(X-l) + \037=(x_I ) -1+ (x + 2) -1


'= 2x+1
x-I x+2
+x-2= (x-I)(x+2) =\037
_

x2

\302\267

Whence)
y\"

(x-1)-+ (x+ 2)
==
y(n)== (-I)n-l(n-I)!
[(x-I)-n+(x+2)-n]
y'\"

..

232
\302\267

=-1
(X+2)-2;
(x-l)-2-I
= 1 .2
1.2 3

-_ _
(

ax+b'.
= ex
+d

fi

3;)

n.

_ 1) ' [(x _

1)n 1 (
nd

T (x + 2)1l
]

_L

1)1l

Y (lZ).

eel)

Solution.Transform the given expressionin the following \\vay:


ax b
= cx+
= e + ebe(ex +add) =!!.+ be
d
-\037

Whence)

y' == (

\037

-1)be-ad
-'ce (cx+ d) -

ad

(ex _L d )

-1.

,
2,

..............
-

y\"

y'\"

y(n)

= (-1)(

2
2) bc-;ade (ex+-d)-3,

= (-1)(-2)(-3)bc-;ade3 (ex+ d)-4,)

ad
- (ll + 1) ==
= (-I)nn!bee en (ex + d)

= (-I)'l

n' en

(ex +d)n+1

(be

ad).)))

.)

\037

2.3.Successive DifJerentiation.

2.3.3.
y==x/(x2 -1);find

109)

Leibniz Formula)

y<n).

Solution.Transform the given expression)


Y

- =2 [x+ + x- ]

= x 2x

(n)

')

2.3.2):

therefore (see Problem


Y

n!
= (-I)n
2
[(x+l)n+ll-(x_l)n+l')
]
I

\037

2.3.4.Using the Leibniz formula, find the derivatives of the


indicatedorders for the following functions:
2
(a) y ==x sin x; find y(25);
(b) y = eX (x2 1); find y(24);
find y<n).)
(c) y = e sin

CXX

\037x;

Solution.(a) y(25) ==(sin X'X 2)<25)= (sinX)<26)x2 + 25 (sinx)<24) (x2)' +


+ 25;2\\sin X)(23) (x2)\", since the subsequent summands equal zero.

Therefore
y(25)

= x2 sin x + 25 + 50xsin x + 24 + 600sin x+ 23 =


)
(
)
(
(
)
== (x2 -600)cosx+ 50xsin x.
\037

\037

2.3.5.
Compute the value of the
.
at the pOInt x==O.
Y=

\037

derivative of the function

nth

3x-\0372

x\037-2x+5

Let
Solution.By hypothesis we have y(x)(x2 -2x+5)=3x+2.

us differentiate this identity


then (for n
2) we obtain

times using the Leibniz formula;

\037

yn

(x)(x2 -2x+5) + ny(n-lJ (x) (2x-2)+

Putting

x==O,we have
5y<n)

(0)

2ny<n-u

(0).+n

Whence)
yen,

(0) =

\037

ny(n-lJ

(n

(0)

n (n

2 = O.)

(\037-I)y(n-2) (x).

1)y<n-21 (0) = O.)

-1)y(n-2) (0).

We have obtained a recurrencerelation for determiningthe n th


derivative at the point x ==0 (n 2). The values y (0) and y'(O)
are found immediately: y (0)==2/5;)))
\037

II. D'OerenttQ110n

Ch.

110)

Y (X)

- -2x+5)l
-

. .

(0) =

\037

;)

===

\037

FlInftt{)n.\037)

4x + I 9

\037X2

(x2

Then, successively putting n


the derivatives of higher orders
reIa t ion.
For example,)
= 2
y\" (0)
y\"l

oj

2, 3, 4,

152\037

...,
.)

25

find the values of


the aid of the recurrence

with

.3. \037;

' (0) == 19

2\037

3\0372

. = .
. =
.
\037

\037\037

1\03765

\037;:

2.3.6.
Find the derivatives of the 'secondorder of the
functions:
(a)

y=xVl+x2;

2.3.7.
Given the

1- ;
;esinx

(b) y=

(c) y=e-x

follo\\ving

\".

x\037

function)

y ==cJ e

2X

+ c2xe +
2X

eX.)

this function satisfies the equation


y\"
4y' + 4y ==eX.
2.3.8.Using the Leibniz formula give the derivatives of the Indicated orders for the following functions:)
a
find y(20);
(a) y= x sin x;
x sin
find y''';
x;
(b) y=efi nd y(nj;
(c) y ==eX (3x2 4);
2 cos
find y(2n).)
x;)
(d) y ==
)
Show

that

(1-x

2.3.9.Using the

functions
(a)

expansion into a linear combination of simpJer


the derivatives of the IOOth order of the functions:)

find

= x2

3x 2

(b) y= 1\037

Yl-x)

;)

2.3.10.
Show that

the function

[cI cos(In x) +c2 sin (1n x)]


constants) satisfies the equation
y:== x

(c1, C2,

x2y\"

2.3.11.
Prove

that

+ (I-2fl)xy' + (1+ n2) y ==O.


if

f (x) has a derivative of the

[f (ax+ b)J

(fll

\037

allf<n)

(ax+

b).)))

nth

order,then

\037

2.4. Inverse,

and Parametrically

Implicit

Represented

Functions

III)

2.4.Differentiation
of Inverse,Implicitand Parametri-

Functions
cally Represented
\037.

t. The

function
function

Derivative of an Inverse Function. If a differentiable


Y ==f (x), a < x < b has a single-valuedcontinuous inverse
x ==g (y) and =1= 0 then there existsalso
Y\037

x!J

-,
I

\302\267

Yx

For the derivative of the secondorder we have)


\"

Xyy

==

.
----,-Yxx

(Yx)3

2. The Derivative of an ImplicitFunction. If a differentiable


function y == y (x) sat isfies the equation F (x, y) ==0, then we have
to differentiate it with respectto x, consideringy as a function
F (x, y) = 0 with respect
of x, and solve the obtained equation
the equation should be twice differentiated with
to
To find
respect to x, and so on.
If
The Derivative of a Function Represented Parametrically.
the system of equations
d\037

y\037t

y\037.

3.

x==cp(t),Y=='I'(t),(X<t<\037,

where (t) and 'I'(t) are differentiable functions and cp'(t) =1= 0,
defines Y as a single-valued continuous function of x, then there
existsa derivative and
cp

-,--= -,.

Y\037

Yx='Pi
CPt

(t)
(t)

yt

Xt

The derivatives of higher orders are computedsuccessively:


\"

y xx

In

'\"

(Y\037)t

yxxx

\"

Xt

\"

an d

so on .

Xt)

particular, for the second derivative the following formula IS

,,,- \",)

true:)

= Xt ytt

\"

Yxx

\302\267

the function

==2x :J
(b) Y ==
(c) y == x
y

xu Yt

(Xt)3

2.4.t. For
(a)

= (Y:x)i

+ 3x5 + x;

3x-(cosx)/2;
+ ex;

Solution.(a)

We

find
find
find

have
,

Y\037

x\037;
X\037y;

X;y.

+ 15x4 + 1,hence,
]
=6x2 +15x4 +1')))

==6x2
1

XY=
Y\037

Ch.

112)

(c) y\037=

+ex ,

II. Di{Jerentiation
hence,

Y\037x=ex,

xY=]+ex ' x
1

eX

II

YY

2.4.2.Using

y\037

Yx

(c) At

x\037

=-(I+e)3'

the rule for differentiation of an inverse function,


the derivative
for the following functions:
2
(a) y=f/ x; (b) y=arcsinvrx; (c) y-==lnVI+x
Solution. (a) The inverse function x ==y3 has the derivative
== 3y 2. Hence,)

find

x\037

of Functions)

= 3y2] =

I
x\037

-1.

>0

== e2y IVe2y

the inverse function

V x2
x == Ve
3

\302\267

2Y

Hence,

yx

= V\037Y=l
= xYX2
2+
e2y

x\037

1 has the derivative

== 2 x
x

\302\267

2.4.3.For each of
rically
(a)

find

the following functions representedparametthe derivative of the first order of y with respect to x:

x==a(t-sint), y==a(l-cost);

(b) x=ksint--sinkt,y==kcost+coskt;

(c) x = 2ctIn cot t,

== tan t

y==e-ct .

(d) x==e ,

+ cot t;

Solution.(a) Find the derivatives


parameter

of

x and

with

respectto the

t:)

x;= a (I-cost);

y; ==a sin t.)

Whence)

a sin t
= a (1= t
cost) cot '2(t
2t
dx _ -2cosec
=_ 4 '
dy

dx

(c)

dt

=1=

2k'Jt).

cot t

sin 2t

4.cos2t .
2 t ==
= sec t -cosec
sin 2 2t '
4
si
cos
dy =
= cot 2t t
.
4 SIn 2 2t
dx
)
(

dy

\037

dt

k:rt

\037\037

=1=

\037t

2.4.4.The functions
(a)
(c)

are defined parametrically:

3
x==acos
t,
3
{ y ==b sin t;
==
X
a (cost

{y= a (sin

2)

(b)

+ t sin t),

t-t cost);

(d)

x==t3 +3t+I,

{y == t t3-3t-t- 1;
t,
X
e
t sin t.
e
y=
{
=-=

Find for tbem the second derivative of y

c\037\037

with

respect to

x.)))

\037

2.4. Inverse,

Implicit and Parametrically

Solution.(a) First
Yt

,
Yx

find

cost;

==3b sin2 t

= _ 3b3acos2tt sIn
c\037s

sin\037

Then we shall

find

y\037x

Represented

= _.!!...-

Functions

113)

Y\037.

3a cos2 t sin t;
t
a tan t ( t =1= (2k + 1) 2 )
t
using the formula
xi ==

.::.

--,

= (y;); ,

II

Yxx

Xt)

where)

' '=

(Yx )t
Whence)
\037

Yxx

- cos

2t

t-e

a cos2 t.)

= 3a2 cosb4 t sin t'

(-3a

cos2 t

sin t)

t-sin

t sin t ==et
(d) xi ==ett cas
(cos
t);
t
t
Y; ==e sin t +e cost ==e (cost + sin t);

, cos t + sin t
YX=cos/-sin/; '
t + sin I
cos
sin t ) t
(
= == et (cos1I-sint)

COS

\"

(y\037);

Yxx

2.4.5.
Find
(a) x

===e-t

y\037\037x:

find

xt=-e-;

y;=3t2

Then

find

\"

(Y\037)t

Xt)

find

finally,
y\037;x

- -_ - -e-+ =

y: = 3t 2/e-t =
the second derivative

_
yxxAnd

(y:\037);

2.4.6.Find the

(3et t 2 6tet )
t

-e-

derivative

===0;
X3-1-x2y+y2

(b)

y\037

,)

3et t 2

the third derivative


= 3e2t [2 (/2+2/) + 21+ 2J

Xt

(a)

y= tan t.

= t 3; (b) x ===sect;

Solution.(a) First
whence)

et (cos I-sint)3)

\037

3te2t (t

+ 2) .

= _ 6e3t (t 2 + 3t + 1).

of the following

lnx+e-/x ==c;

implicit functions:

(c) x22/,+y2-4x-IOy+4==0;
/s
(d) x

+ y2/3 ==a .
2

Solution.(a) Differentiate

a function of

x; we get:

with

respectto x, consideringY as

3x2 + 2xy + x2y' + 2yy' ==

O.)))

Ch.

114)

II. Differentiation

of Functions)

Solving this equation with respect to y'


Y

2.4.7.Fin d

if

y\037x

3x2

x2

+2xy
+2y

find
\302\267)

(b) eX-eY==y-x;
(a) arctan y-y+x==O;
(c) x + y ==eX Y
Solution.(a) Differentiate with respect to x, consideringy as a
f unet ion of x and determine y':

y'

+ y2

0
+ 1 ==,

Differentiate once again

wh

ence y ' == I +y2y2 ==Y - 2 +

respectto

with

==

y\"

\"

2.4.8.Find the
x3

value of

==

_ 2y-3y'.

2 (1

get

finally

y2)
\302\267

yo

at the point

y\"

x:)

Substituting the value of y' thus found, we


Yxx

x == I

if

-2xy2+5x+y-5==0and yIX=l== I.
2

Solution.Differentiating

respect to x, we find that


3x2 xy 2_4x2yy' + 5 + y' ==O.
Putting x == 1 and y == 1,obtain the value of y' at x == 1:
4-4y'+ 5 + y' ==0; y' ==4/3.
Differentiate once again with respect to x:

-4

with

3-

Pu t ting

-8xyy'-4x2y'2_4x2yy\" + y\" ==O.


6x-4y2-8xyy'
x == 1;y == 1 and y' ==4/3, find the val ue y\" a t x 1:
6

2.4.9.Find

y\037

(a) x + Vxy +

-- _
4

::--.:-.

_3

64 64
3

\"

== 0

'

\"

==

_ 8 2722

\302\267

for the following implicit fu nctions


2
y2;
y ==a; (b) arc tan (y/x) == In Vx

(c) eX sin y-e- cosx ==0;

(d) eY

+ xy ==e;

fi

nd y at

2.4.to. Find y;x ot


(a)

\037

t he

poi n t (0, 1)

the following implicit functions:

y==x+arctany;

find
(b) x 2 +5xy+y2_2x+y-6==0;

y\"

at the point (1,1).)))

2.5.Applications

\037

For each of
2.4.11.
trically

find

the
a si'n t

the following functions represented parame-

indicatedderivatives:

x=-I+bcost '
(b) x == In (I + t 2),
2
(c) x t +2 2,
(a)

115)

of the Derivative)

Y==

\037

e-

t
,
(d) x ==
==
(e) x 4 tan 2 (t/2),
2
(f) x==arcsin(t
(g) x ==arc sin t,

t-arctan t;

== t 3/3
==-

t;

find

y\037;

find

Y\037;

fi

n d y;x;

arc tan (2t + 1); find

+ b cost;
t;
y==arccos2

-1),

Show that
2.4.12.

===

c cos t

I+bcost;

==a sin t
==

1-

t2

V\037

the function

==f

Y\037;

find

y\037;

find

y\037;

find

Y;x'

(x), defined by the parametric

equations x ==et sin t, Y ==et cost, satisfies the relation


== 2 (xy'-y).)

y\"

(x

+ y)2 ==

* 2.5.Applications
of the Derivative
The equation of a line tangent to the curve of a differentiable
function y ==y (x) at a point M (xo Yo)' where Yo Y (xo)' has the

'

form

y-

Yo

= y' (xo) (x-xo)'

A straight line passing through


the point of contact perpendicularly to the tangent line is called the normal to the curve. The
equation of the normal at the
II
point M will be

y-Yo==

-'
\"ii

(x-xo)'

(xo)

y' (xo) =I=- O.)


The segments AT, AN are
called the subtangent and the
subnormal, respectively; and the 0 T
lengths MT and MN are the
so-calledsegment of the tangent

x)
A)

N)

Fig. 36)

and the segment of the normal,

respectively(see Fig.36).The lengthsof the four indicated segments)

are expressedby the

AT=I

2.5.I. Write

follo\\ving

formulas:)

:,/; AN=lyy'l; MT=I:,IVl+(y')2;


MN=ly/Vl+(y')2.

the

-+

equations of

3
(a) to the curve y ==x

3x

thp tangent line and the normal:


2 at the point. (2,
4);)))

Ck.

116)

II. DitJerentiation

2
(b) to the parabola y==2x
4

-x+5
-16atat x==-O.5;
the
of intersection

(c) to the curve y ==x2 + 3x2


with the parabola y == 3x .
Solution.(a) Find the derivative
y' ==3x2

of Functions)

-3,

points

at the point x ==

2:

y' (2):== 9.

The equation of the tangent line has the following form:


y-4-=9(x-2) or

9x-y-14=O.

The equation of the normal is of the form:


1

or x+9y-38==O.)
y-4==-g(x-2)

(c) Solving the system of equations


Y

{Y
we shall

find

Now we

==x4
:==

+ 3x2 16,

3x2,

the points of intersection of the curves

x1==-2,x2 ==2, Yt==Y2==12.


2
find the derivatives at the points x =
y' == 4x3 + 6x, y' (-2)== 44, y' (2)==44.

and x ==2:

Therefore, the equations of the tangent lines have the form


44 (x+2),
44

y-12=-

(x-2).

y-12

--\037

The equations of the normals have the form

- (x-2).)
-+
--

y-12==44 (x+2), y-12==


1

Find the
2.5.2.

line:

points on the curve

===

44

x3

3x

5 at which the

tangent
(a) is parallel to the straight line y == 2x;
(b) is perpendicular to the straight line y == x/9;
with the positive directionof the x-axis.
(c) forms an angleof
Solution.To find the required points \\ve take into consideration
that at the point of tangency the slope of the tangent is equa I to
the derivative y' -= 3x2
computed at this point.
(a) By the conditionof parallelism
45\302\260

-3

3X2-3:== 2,
3 , x 2 == 1/V3.The required points are:
whence x 1 ==-1/V
3f9).)))
3 , 5+8V3/9),M 2 (1/V3,
Mt

(-1/V

5-8V

\037

(b) By the

2.5.Applications

of the Derivative)

117)

condition of perpendicularity
3x2

-3

==

9,

(-

2, 3), M 2 (2,7).
2, x2 == 2. The required points:M 1
Find the angles at which the following lines intersect:
2
and the parabola y ==4
/2;
(a) the straight line y ==
==
==
cosx.
cosine
curve
and
the
sin
x
sinusoid
the
y
y
(b)
Solution.(a) Recallthat the angle betweentwo curves at the point
of their intersectionis defined as the angle formed by the lines tangent to these curves and drawn at this point. Find the points of
intersectionof the curves by solving the system of equat ions

whence Xl ==

2.5.3.

-x

4-x

Y=4-X

{ y = 4-X:/2.)

Whence)

MI (0, 4); M 2 (2, 2).


Determine then the slopes of the lines tangent to the parabola at
the points M 1 and M 2:
y' (0)==0, y' (2) == 2.)
The slope of a straight line is constant for all its points;) in our)
caseit equals
Finally, determine

-1.

the angle between the two straight


1

ines:

tan
tan

CPI

==

1;

CPI

==

!J)

J{)
45\302\260;

= -1+2
+2 = 3;
I

CP2

CV2

= arc tan

2.5.4.Prove

x)

18.5\302\260.)

\037;:;::::

o)

Xo

T)

the segment of
Fig. 37)
==
the tangent to the hyperbola y c/x
which is contained between the coordinate axesis bisected at the
point of tangency.
Solution.We have y' == c/x2; hence,the value of the subtangent
for the tangent at the point M (xo Yo) wi 11 be)
that

i.e.

:'
I

'

= x0 I,
I

OX o ==xoT (Fig. 37), which completesthe proof.


Whence follows a simple method of constructinga tangent to the
hyperbola y ==c/x: layoff the x-interceptOT ==2xo' Then MT will
be the desired tangent.)))

Ch.

118)

II. Differentiation

of Funct

ions)

2.5.5.

Prove that the ordinate of the catenary y ==a cosh (x/a) is


the geometricmean of the length of the normal and the quantity
Solution.Compute the length of the normal. Since

a.

y' ==sinh (x/a),


the length of th e norm al will be
MN == y VI + (y')2 Y V 1 + sinh2 (x/a)==y cosh(x/a)==y2/ a ,
===

whence

y2

a.MN, and y ==V a.MN, which

==

2.5.6.Find

completes the proof.

the slope of the tangent to the curve

x==t2

\\

2
y ==2t

-1).

at the point M

--

+3t-8,

2t

(2,
Solution.First determine the value

ven values of x and


two equations)

of t correspondingto the giThis


must
value
y.
simultaneouslysatisfy the
I
\\

- - - 1.
1.
2.

t2
2t 2

+ 3t

2t

8 ==2
5 ==

The roots of the first equation are t 1== 2; t 2 == 5, the roots of the
second equation i 1 ==2; t 2 ==
Hence,to the given point there
correspondsthe value t == Now determine the value of the derivative at the point M:
y
And

so,the

to 6/7.

' x =2 ==
1

Yt

xi

t =2

4t-2 = 6
= 2'
( +3 ) t = 2 7

\302\267)

slope of the tangent at the point M (2,

2.5.7.

-1)is equa)

Prove that the tangent to the lemniscatep ==a Vcos28at


the point correspondingto the value 80 == n/6 is parallel to the x-axis.
Solution.Write in the parametric form the equation of the lemniscate:)
x ==p cos8 ==a V cos28 eos0,
y ==p sin 8 ==a Vcos28 sin 8.)
\\\\'hence)

,
Xo

,
yo
x\037

-aVcos20
cos28
y-\037

a cos8 sin 28

.
sIn

a sin 8 sin 28

a cos20
y cos28 + V

(n/6)==

a V 2,

y\037

co\037

(:rtJ6) -:.=

O.)))

n,

0,

2.5.Applications

\037

Thus, the slope k:=:: ,


y\037

of the Derivative)

119)

(1[/6)

= O. Consequently,
. the

xe (31:/6)

the

lemniscate at the point

== a/V\"

2\"

80 == n/6 and

with

is para 11 eI to t he x-axis.

2.5.8.
Find

line tangent to

Po == aJir

cos280

===

equations of the tangent and the normal to the

the

following curves:
3
2
(a) 4x -3xy2+6x

-5xy-82+9x+14==0at the point (-2,3);


y

(b) X5 +y5_2xy==0at the point

(1,1).

Solution.(a) Differentiate the implicit function:

I2x2 -3y2-6xyy'+ 12x-5y-5xy'


-I6yy'+ 9 ==O.

(-2,

Substitute the coordinatesof the point M


3):
48-27+ 36y'
+ 109'
-48y'+ 9 ==0;)

-24-15

\\vhence)

y' ==

-9/2.

Thus the equation of the tangent line is

y-3==-29 (x+2)
and the equation of the normal)

lj-3

-==

2.5.9.
Through
== 4

\"9

(x

+ 2).

the point (2, 0), which does not belong to the


draw tangents to the la tter.
be the point of tangency; then the equation
Solution.Let (xo
of the tangent will be of the form:

curve

x,

'

x\037)

y-xg==y' (xo) (x-xo


))

y-x\037 == 4xg

(x-xo)'

desired tangent line passes through the point


of this point satisfy the equation of
the
coordinates
hence,
(2, 0),
By hypothesis the
the tangent

line:

-x\037==4x\037(2-xo);3xg-8x\037==0,
whence Xo ==0; Xo ==8/3.Thus, there are two points of tangency:
M 1(0, 0), M 2 (8/3,

4096/81).

Accordingly, the equations of the tangent lines will be


4096 2048
8
y===O,

y-81=2]
)
( x-a')))

II. Differentiation of Functions)


3
2.5.10.
+ 5x-7.Find out at which
f (x)==3xo-15x
Ch.

120)

of the pointsx
the rate of changeoJ the function is minimal.
Solution.The rate of change of a function at a certain point is
equal to the derivative of the function at this point
2
+5==15(x2 -1/2)2+
f'(x)==15x4 -45x
1/12].
The minilllum value of f' (x) is attained at x == + I/V2.Hencethe
rate of change of the function f (x) is at the point
minimum
r
x == I/V 2 and equals 5/4.
3.
A point is in motion along a cubic parabola 12y==x
2.5.11.
Which of its coordinateschanges faster?
Solution.Differentiating both membersof the given equation with
respectto t we get the relation between the rates of change of the

:i:

coordinates:)

12yi==3x
or)

xi)

--;--4'
X2

yt

Xt)

Hence,

-2

< x < 2 the ratio y;:x; is less than unity, i.e.the rate
(I) at
changeof the ordinate is less than that of the abscissa;
(2) at x==+ 2 the ratio y;:x; is equal to unity, i.e.at these
points the rates of changeof the coordinatesare equal;
(3) at x <\0372 or x > 2 the ratio y;:x; is greaterthan unity, i.e.
the rate of changeof the ordinate exceedsthat of the abscissa.

of

A body of
2.5.12.

-I+
seconds).Find the
the law 8

after

it

===

mass 6g is

In (t

in

rectilinearmotion accordingto

+ 1)+ (t + 1)3 (82 is

kin\037tic

begins to move.

Solution.The velocity

of the distance:)

v (t)

in

centimetresand

t,

in

energy (mv /2) of the body one second

of motion

= s; = t

\037

is equal to the time derivative

+ 3 (t + 1)2.)

Therefore
mv 2

2 and 2==2 ( 122 )

v(I)==12

(erg).
==468\"4
The velocity of rectilinearmotion of a body is proporti2.5.13.
onal to the square root of the distance covered (s), (as, for example,
in free fall of a body). Prove that the body moves under the action
of a constant force.
Solution.By hypothesis we have
v ==s; ==ex

Vs (ex ==const);)))

.\037

2.5.Applications

\",

whence)

121)

,
')
r- St == /2.

Sit ==Vt ==ex.

But

of the Derivative)

ex....

2 J' s
\"1

according to Newton's law the force

F ==kStt
Hence,)

F ==

kex.

(k ==const).)

/2 ==const.

2.5.14.
A raft

is pulled to the bank by means of a rope \\vh ich


on a drum, at a rate of 3 m/min. Determine the speed
of the raft at the moment when it is 25 m distant from the bank
if the drum is situated on the bank 4 m above water level.
Solution.Let S denote the length of the rope between the drum
and the raft and x the distance from the raft to the bank. By
hypothesis)
S2 =x 2 + 42
is

\\vound

Differentiating this relation with respectto


ship between their speeds:)

t, find

the relation-

2S8;==2xx;,)

whence)

, ==

Xi

-s,
x)

St.

Taking into considerationthat


we obtain)

s;==3;x==25;s==V252+42\03725.3,)
, == Y252

Xi

+ 42

25

\302\267

\037

.
3.03(m/mln).

2.5.15.
(a) Find

the slope of the tangent to the cubic parabola


the point X ==V3/3.
+X2)
(b) Write the equations of the tangents to the curve y== 1/(1
at the points of its intersection with the hyperbola y == 1/(x+1).
(c) Write the equation of the normal to the parabola y ==x2+4x+1
perpendicularto the line joining the origin of coordinateswith the
vertex of the parabola.
(d) At what angle does the curve y ==eX intersect the y-axis?
The velocity of a body in rectilinearmotion is determined by the formula v ==3t + t 2 What acceleration
will the body
have 4 secondsafter the start?
y

-==

x: at
3

2.5.16.

2.5.t 7. The law


of
2

100kg

is 8 ==2t

rectilinearmotion

of a body

with

+ 3t + 1. Determine the kinetic energy

the body 5 secondsafter the

start.)))

mass of

(mv

/2) of

II. DifJerentiat

Ch.

]22)

ion of Funct ions)

2.5.18.
Show that

if the law of motion of a body is s ==aet + be-t,


its acceleration
is numerically equal to the distance covered.
A body is thrown
2.5.19.
vertically with an initial velocity of
a m/sec.What altitude will it reach in t seconds?Find the velocity
of the body. In how many secondsand at what distance from the
ground will the body reach the highest point?

then

2.5.20.Artificial satellitesmove round the Earth in elliptical


orbits.The distance r of a satellitefrom the centreof the Earth
as a function of time t can be approximately expressed
by the follo\\ving

equation:

=a

where M =

?;(t

t == tirrle

[1

- HOSM

e; (cas2M I)J

in)

parameter

a ==semi-majoraxisof the orbit


c ==eccentricity of the orbit

P ==periodof orbiting
1
= time of passing the perigee
ty the satelli tee
Herea, P and t n are constants.
from the satellite
to the
Find the rate of change in the distance,
centre of the Earth (i.e.find the so-calledradial velocity of the
tn

\302\243,

satellite).)

* 2.6.The Differential
of a Function.
to ApproximateComputations
..4.pplication
If the increment
\037y

==f

of the function y ==f (x) can be

\037y

(x+

\037x)

-f (x)

== A (x) \037x

+ a (x,

\037x)

expressedas:

\037x,)

where)

!in1a (x,

.1x

\037

\037x)

0,

==

0)

is calledditJerentiable at the point x. The


of
this increment A (x)
linear
is calledthe ditJepart
principal
rential and is denoted df (x) or dYe By defini tion, dx ==
For the differential of the function y ==f (x) to exist it is necessary and sufficient that there exist a finite derivative y' == A (x).
The differential of a function can be written in the following way:
then such a function

\037x

\037x.

dy ==y' dx ==
1

IIi

te

The perigee of
centre of

to the

the
the

satellite orbit is
Earth.)))

f' (x) dx.)


the shortest

distance

from the

sate-

2.6.DitJerential

\037

of

u == cp

For a composite function y ==f (u),


retained in the form)

123)

Function)

(x) the differential

is

f'

dy == (u) du

(the invariance of the form of the differential).


With an accuracy up to infinitesimals of a higher order than
the approximate formula
dy takes place.Only for a linear
function y==ax+bdo we have \037y==dy.
DifferentiaIs of higher orders of the function y ==f (x) are successively determined in the following way:
\037x

\037y

d2y
If

==f (x)

d2y
But

==d (dy);

d3y

..

\"

ll
dny ==d (d

-ly).

\"',

2.6.t. Find

d2y ==f\" (u)du2 +f' (u)d2u, and

u=<p(x),then

y--:=f(u),

so 0n.)

d (d2y),

==-

and x is an independent variable, then


==y\" (dX)2; d3y ==y'\" (dx)3,
dny ==y(ll) (dx)n.
\\\\'r:ere

if

\037

the differential of the function


-== In
(1+ e10X ) + arc tan e5X
y

Calculate dy at x = 0; dx ==0.2.

Solution.

dy

(I + e10X )'

e10X

(e6x)' dx _ 5e (2e - I) dx.


_ l-f..
+e
]
6X

elOX

5X

10X

Substituting x ==0 and dx ==0.2,we get


dy Ix=o;

2.6.2.
Find

dx=O.2=

5
0 .2
2.

==

0.5.

the increment and the differential of the function


3 x
I
y == 3x

0.1.

==
at the point x == 1 at
Find the absolute and relative errors allo\\\\Ted when replacing the
increment of the function with its differential.
\037x

Solution.

i1y == [3 (x+

\037X)3

+ (x+

-1]-(3x+9xx-I)+ 9x
3

\037x)

==

2 \037x

==

Whence)
\037y

At

x== 1 and

+ 1)
-dy= 9x +
2
dy ==(9x

\037x==O.1

\037X2

we get

\037X2

\037x.)

3\037X3.

== 0.093,
0.09+ 0.003
==
1.093.)))
1;

\037y-dy ==
dy ==

\037y

3\037X3

\037x,

II. DifJerentiation

Ch.

124)

The absoluteerror /).y-dy= 0.093,


the relative
o .093
01
0 085 8 5 /0.
093\037.
I

===

of Functions)

or.

1.
2.6.3.
Calculateapproximately the

error i'J.Y--;,/Y 1=
I

increment of the function

y==x -7x+8
3

as x changesfrom 5 to 5.01.
2.6.4.Using the conceptof the differential)
value of the

funct

ion

f;

y==

Solution.Notice that

the approximate

r2-x at x=0.15.
2+x
we get
\037y==y(x+\037x)-y(x)

from

y (x

or, putting

find

\037x)

==y (x)

\037y,)

\037y\037dy,)

y(x+\037x)\037y(x)+dy.
In our

== 0.15.
Then
problem let us put x ==0 an d
4
1
,
2+X
(-4)
Y ==\"5
2 x ) (2 + X)2 ;
\037x

V( -

\302\267

y'(O)=- , dy=- .0.15=-0.03.)


\037

Hence,)

\037

(0.15) y (0)+ dy == 1-0.030.97.


==
(accurateto 10-4).
y (0.15)0.9702

y
The true value of

--==

\037

2.6.5.Find the

value of:
approximate
(b) log 10.21;(c) V 33; (d) cot
(a)
Solution.(a) In solving this problemwe shall use the formula (*)
of the precedingproblem.Putting x ==nj6, == n/180,
we compute:
V3
y (x)==cos
45\302\26010'.

cos31\302\260;

\037x

j'[

'

y (x) ==

cos31=cos( +

6=\037;
. n
SIn ==
;

-6-

\037

- -

(c) Put x = 32;

\037x

I\037O

\037

\0373

2\"

-+

l\037o

=0.851.

=::1.By formula (*) we get

'

1 =2-t u
xx=32.
)
V 33\037 V 32+( Jt5/-5

V324)))

=2+ 80=2.0125.
1

\037

2.6.Differential

of

125)

Function)

2.6.6.
All

faces of a coppercube with 5-cm sides were uniformly


As a result the weight of the cube was reduced by
down.
ground
0.96g. Knowing the specificweight of copper (8) find the reduction
the amount by which its side was reduced.
in the cube size,
Solution.The volume of the cube v ==x3, where x is the length
of the side.The volume is equal to the \\veight divided by the den3
== 0.96/8
== 0.12
(cm ).
sity: v ==p/d; the change in cube'svolume
that
Since approximatelyequals dv and taking into consideration
== 3 x 52 X \037x, whence
dv = 3x2 dx we shall have 0.12

i.e.

\037v

\037v

,1x=

\037..1225

= 0.0016
em.

cm.
Thus, the side of the cube was reducedby 0.0016
2.6.7.Find the expressionsfor determiningthe absoluteerrors in
the following functions through the absolute errors in their argumen

ts:

(a) y==

x;

In

(c) y==sinx

(b) y == log x;
(d) y == tan x (0 < x

(0<x<n/2);

< nj2);)

log (sin x) (0 < x < jt/2);


==
log (tan x) (0 < x < jt/2).
(f) y
Solution.If the function f (x) is differentiable at a point x and
is sufficiently small,then
the absolute error of the argument
the absolute error in the function y can be expressed
by the number
(e)

=7:

\037x

\037y

= y;
I

\037x'

i.e.

= (In x)' Ix ,1x= ,


the absolute error of a natural
(a)
logarithm is equal to the relative error in its argument.
M
where M == log e ==0.43429;
(b) == (logx)' == X
(e) == [log (sin x)]' ==M cot x

,1,

\037xx

\037x,

\037x

\037y

\037y

\037x

(f) ,1,= [log(tan x))' ,1x=


I

s\037\037x

\037x;

,1x.

From (e) and (f) it follows that the absolute error in log tan
is always more than that in log sin x (for the same x and \037x).
Find the differentials dy and d2y of the function
5
7x2 3,)
y == 4x

2.6.8.

assuming that:
(1) x is an independent variable;
(2) x is a function of another independent variable.
Solution.By virtue of the invariance of its form the differential
of the

first

order dy is written identically in both cases:


4
dx.)))
dy ==y' dx ==(20x-14x)

Ch.

126)

II. Dit7erentiation

of Functions)

the first casedx is understoodas the increment of the


independent variable dx (dx == \037x), and in the second,as the diffefl'ntial of x as of a function (dx may not be equal to \037x).
Since differentials of higher orders do not possessthe property
of invariance, to find d2y we have to consider the following two
But

in

cases.

be an independent variable; then


d2y ==y\" dx2 == (80x3
dx2.
(2) Let x be a function of some other variable. In this case
d2y == (80x3
d2x.
dx2 + (20x4 -14x)
2.6.9.Find differentials of higher orders (x an independent va-

(1) Let

-14)

-14)

riable):
(a)

(b)

(c) y

2
== 4 x ;
== Vln2
== sin2 x;

fi

nd

d2y;

x-4;d

find
3

find

2.6.10.
y=ln II\037::;

d2y;

y.

d2y if: (a) x is an independentvariable,


(b) x is a function of another variable. Consider the particular

casewhen x == tan

2.6.t t.

find

t.

The volume

of a

sphere of radius

nr3. Find the increment and differential


explain their geometrical
meaning.

is equal to
of the volume and

2.6.t 2. The law of the free fa 11 of a material point is s ==gt 2j2.


Find the increment and differential of the distance at a moment t
and

elucidatetheir mechanicalmeaning.)

* 2.7.AdditionalProblems
2.7.t. Given the functions: (a) f (x)=

x and (b) (x) == x3 [.


Do derivatives of these functions exist at the point x ==O?
2.7.2.Show that the curve y==e1x cannot have a tangent line
at the point x ==O. What is the angle between the one-sidedtangents to this curve at the indicated point?
Show that the function
2.7.3.
I

cp

f (x)==

x-a

cp

(x),

where (x) is a continuous function and (a)=1= 0, has no derivative at the point x == Find the one-sided derivatives
(a) and
<p

t:

(a).)))

a.

<p

f\037

\037

2.7.4.Given the

2.7.Additional

Problems

127)

function
'

f (x)==

X2

0,

Si 11 ( 1/ x) a t x =1=

at x==O.

Use this exampleto show that the derivative of a continuous


function is not always a continuous function.

2.7.5.Let)
f(x)= f
\\

x2,

if

x\037xo,

ax+b, if x>xo'
is continuous

Find the coefficients a and b at which the function


and has a derivative at the point XO'
3
By differentiating the formula cos3x ==cos
2
3
deduce the formula sin 3x==3 cos x sin
sin x.

2.7.6.

2.7.7.From

x-3 casxsin x

x-

the formula for the sum of the


1

+ x + x2 + ... -1-xn ==

-_
1

xn + 1
x

geometric
progression
(x

=1=

1)

deduce the formulas for the following sums:

... + nx - 1; ... + n 2x 1.

(a) 1 + 2x+3x2 -t(b) 12+ 22X + 32x2 +

2.7.8.Prove

the identity

2' ,x

cosx+ cos3x+ ... + cos(2n 1)x == sinsIn2nxx


and

=t=

kn

deduce from it the formula for the sum


sin x+ 3 sin 3x+... + (2n-l)sin (2n-l)x.

2.7.9.Find
(a)

(c) y

----..:

y' if:
2

f (si n x)

+ f (c0S2 x);

== log\037 (x) '\" (x)

2.7.10.

(<p

(x)

(b)

> 0;

y
'\"

=:-:f (eX) eJ (X) ;

(x)> 0).

Is it reasonableto assert that the product F (x):=-:;f(x)g(x)


has no derivative at the point x ==Xo if:
(a) the function f (x) has a derivative at the point xo, and the
function
(x) has no derivative at this point?
(b) neither function has a derivative at the point xo?
Considerthe examples:
(1) f(x)==x,g(x)==lxl;
<p

Is it

(2) f(x)==lxl,g(x)==Ixl.
reasonable to assert that the sum F (x)==f (x)+ g (x) has no

derivative at the point x ==X o

if:)))

II. Di(Jerentiation

Ch.

128)

of Functions)

(c) the function f (x) has a derivative at the point xo, and
ion g (x) has no derivative at this point?

the

funct

(d) nei ther function has a derivative at the point xo?

2.7.11.
Prove that

the derivative of a differentiable even function

is an odd function, and the derivative of an odd function is an


even function. Give a geometric
explanationto these facts.

2.7.12.
Prove that the derivative of a periodicfunction
T is a periodicfunction with period T.
2.7.13.
Find F'(x)
2

with

Veriod

if)

x
1 2x 3x2
o 2 6x
x

(x)==

3)

Find
2.7.14.

the derivative of the function y==xlxt.Sketchthe


the
of
given function and its derivative.
graphs
function y==f(u), where
Suppose we have a composite
look
we
for points at which
what
should
u
Among
points
(x).
function may have no derivative?
the composite

2.7.15.

\037

<p

function always have no derivative at these


Does the composite
the
function
Consider
y==u2, u==(xl.
points?
Find
2.7.16.
(a)

==

Is there

x3

y\"

y\"

for the following functions:

(b )

==

(O)?

X2

{o

sin ( 1/x), x =F 0,

x=O.

at

2.7.17.
(a) f (x) =x ; show that
n

+ f' +
\037

(b) f (x) =

I)

f(2

f ( 1)
II)
X
1/
xn-1e ; show that

\037

\037

+ ... +

[f (x))In) = (-I)n
y==x2e-x1a ; show that

fx\037\037

(n =

l) = 2n

fIn
\037

1,2,

2.7.18.

fin)

(0)= (_I)n;\037\037_I)

Show that
2.7.19.
2
==

(n

\037

\037

...).
2).

the function y==arcsinxsatisfies the relation


Find
2) by applying the Leibniz fory<n) (0) (n
) y\" xy'.
mu 1a to both members of this identi ty.

(l-x

Prove
2.7.20.

\037

that the Chebyshev

polynomials

(n= 1,2,...))))
Tn(x)==2n_1Cos(narccosx)
I

2.7.Additional

\037

satisfy the equation

(1_x2)

-xT;z(x) + n 2T

(x)
The derivative of the
T\037

2.7.21.
the

form)

(e-x

C)

\")

II

(x) = O.

order of the function

nth

(1l) ==

12:))

Problenls)

e-x2 has

e- 2Hn (x),
X

where Hn (x) is a polynomial of degree n


Hernzite polynomial.
Prove that the recurrencerelation

called the

(n=-=l,2,
Hn+l(X)-2xHn(x)-t-2nHll-1(x)==O

Chebyshev..

...))

IS valid.)

2.7.22.Sho\\v that there existsa single-valuedfunction y ==y (x)


defined by the equation + 3y ==x, and find its derivative
2.7.23.
Single out the single-valuedcontinuous branches of the
4
inverse function x ==x (y) and find their derivatives if y ==2x2
y\037

U ==
2.7.24.

2\"

In

. du dv == 1.
+ v ; check the relatIon
dv du
1 -v

Y\037.

-X .

2.7.25.Inverse trigonometricfunctions are continuous at all


points of the domain of definition. Do they have a finite derivative at all points of the domain? Indicatethe points at which the
following functions have no finite derivative:
. I
x+ I
(a) y==arccos\037;(b) y==arcslnx '
the function y ==y (x), defined parametrically:
2.7.26.Sho\\v that
2
y=t +tltl, is differentiable at t==O but its derivax==2t-ltl,
the usual formula.
tive cannot be found
by

the parameters a, b, c in the equation of the


2.7.27.Determine
2
parabola y ==ax + bx + c so that it becomestangent to the straight
==

(-1,

the point x = 1 and passes through the point


0).
2.7.28.Prove that the curves Yt ==f (x) (f (x) > 0) and Y2 ==
== f (x) sin ax, where f (x) is a differentiable function, are tangent to each other at the common points.
line

x at

for any point M (xo, Yo) of the equilateral


that
2.7.29.Show
2
2
hyperbola x _y2==a the segment of the normal from the point
M to the point of intersectionwith the abscissais equal to the
radius vector of the point M.

2.7.30.
Show

that for any position of the generatingcirclethe


line and the normal to the cycloid x==a
t),
pass through the highest (at, 2a) and the lowest
(at, 0) points of the circle,respectively.)
tangent

y=a(l-cost)
-3

I 48)))

(t-sin

Ch.

130)

2.7.31.Show

== a

cc;uality

d3y

==f

ion of Functions)

cardioids p ==a (1+ cascp) and p =

two

(1-coscp) intersect at

2.7.32.
Let

<p

that

II. Differentiat

angles.
(x). Prove

right

(u), where

==f'\" (u) du 3

u ==(()

the validity of the

+ 3f\" (u) du d2u + f' (u) d:iu .

2.7.33.
Let y==f{x},where x==cp(i); the functions f(x)
(i) are twice differentiable and dx O. Prove that

and

=1=

\"

Yxx

2
2
== d ydx-3dyd x
dx

,)

where the differentials forming the right member of the relation


are differentials with respect to the variable t.

2.7.34.How will

the expression
(1

dy
x2) dx 2 _ xdx)
-t-Y
d2\037

be transformed (where y is a twice differentiabIe function of x) if


\\ve introduce a new independentvariable i, putting x ==casi?
In determining an electriccurrent by means of a tangent
2.7.35.
ga1 vanometer use is made of the formula

I ==k tan

\\vhere

cp,)

J ==current
k ==factor of proportionality (depending on the instrument)
==angle of pointer deftection.

cp

Determine the relative error of the result \\vhich depends on the


inaccuracy in reading the angle ((). At what position of the pointer
can one obtain the most reliableresults?)))

Chapter

3)

APPLICATION

OF

DIFFERENTIAL

CALCULUS TO INVESTIGATION

OF

\037

FUNCTIONS)

3.1.BasicTh.eoremson Differentiable
Functions

Fermat'sTheorem.Let a function y == f (x) be defined on a certain interval and have a maximum or a minimum value at an interior point Xo of the interval.
If there existsa derivative f' (xo) at the point xo, then f' (xo) ==O.
Rolle'sTheorem.If a function f (x) is continuous in the interval
a finite derivative at all interior points of this interval,
[a, b], has
and f (a) == f (b), then inside [a, b] there exists a point s E (a, b)
such that f' (s)==O.
Lagrange'sTheorem.If a function f (x) is continuous in the interval [a, b] and has a finite derivative at all interior points of
the interval, then there existsa point s E (a, b) such that
f (b) f (a) == (b
f'
Test for the Constancy of a Function. If at all points of a certain interval f' (x) ==0, then the function f (x) preserves a constant
value \\vithin this interval.
Cauchy'sTheorem.Let (x) and '\" (x) be two functions continuous in the interval [a, b] and have finite derivatives at all interior points of the interval. If these derivatives do not vanish simultaneously and (a) =1= (b), then there existss E (a, b) such that

-a)

(\037).

cp

cp

cp

--

'I'(b) 'I'(a) =
<p

(b)'

<p

3.1.1.
Does the

(a)

'1\"

(\037)
\302\267

-1
[1,

<p'

(\037)

function f (x) ==3x2


satisfy the condition of
in the interval
2]?
Solution.The given function does not satisfy the condition of the
Fermat theorem, since it increasesmonotonically on the interval
2],and, consequently,takes on the minirrlum value at x== 1 and
the maximum one at x == 2, e. not at interior points of the interval. Therefore, the Fermat theorem is not applicable;in other
words, we cannot assert that f' (1)== f' (2) == O. Indeed, f' (1)==6,
f' (2) == 12.)
the Fermat theorem

[1,

5*)))

i.

Ch.

)\037j2

3. .2.

III.DifJerenttal

CalcuLus: Investigation

of Funct's)

t
Do the following functions satisfy the conditions of the
Rolle theorem?
(a) f (x) ==
V X2 in

1-

[-1,1];
(c) t(x)==l-ixi [-1,
1].
do not, explain

(b) t (x)== In sin x in [n/6, 5n/6];


in

If they

why.

[-1,

Solution.(a) The function is continuous in the interval


1];
==t (1)== O. Thus, two conditions of the Rolle
furthermore, t

(-1)

theorem are satisfied. The derivative f' (x) == -2/(3Vx) exists at


all points exceptx == O. Since this point is an interior one,the third
conditionof the theorem is not satisfied. Therefore, the Rolle theorem is not applicable to the given function. Indeed, (x) =1= 0 in

I'

[-1,1].

3.1.3.Prove

that the

equation
3x&

has only one real

+ 15x-8==0)

root.

Solution.The existenceof at least one real root follows from the


15x-8is of an odd power.
fact that the polynomial f(x)==3x&+
Let us prove the uniquenessof such a root by reductio ad absurdum. Suppose there exist two roots Xl < x2. Then in the interval
5
[Xl' x2] the function f (x) ==3x + 15x-8satisfies all conditions of
the Rolle theorem:it is continuous, vanishes at the end-pointsand
has a derivative ,at all points.Consequently,at somepoint Xl < < x2,
f' ==O. But f' (x) 15(x4 -j- 1)> O. This contradictionproves that
the equation in question has only one real root.
3.t .4.Does the function f (x) == 3x2 5 satisfy the conditions of
the Lagrange theorem in the interval
OJ? If it does, then
which figures in the Lagrangeformula f (b) f (a) ==
find the point
\302\243,

\037

==

f'

\302\243

-[-2,

(\037)

-a).
\302\243

(\302\243)

(b

Sulution. The function satisfies the conditions of the Lagrange


theorem,since it is continuous in the interval
0] and has a

[-2,

finite derivative at all interior points of the interval. The point


is found from the Lagrange formula:

f'

\\vhence

==
\302\243

-1.

(\302\243)

==

6\037

== f

\302\243

(0)- (-2)= -5-7==-6'


f

0-(-2)

3.1.5.Apply the Lagrange formula to the function t (x) == In x in


[1,e] and find the correspondingvalue of
2
3.1.6.
Ascertain that the functions f{x)==x
-2x+3and g(x) ==
3
==x -7x2+ 20x-5satisfy the conditions of the Cauchy theorem
in the interval [1,4] and find the correspondingvalue of

the interval

\302\243.

\302\243.)))

\037

3.1.BasicTheorems

on Di(Jerentiable

133)

Functions)

Solution.The given functions f (x) and g (x) are continuous everywhere, and hence,in the interval
[1,4] as well; their derivatives
2
-14x+20are finite everywhere; in
f'(x)==2x-2and g'(x)===3x
addition, g'(x) does not vanish at any real value of x.
Consequently, the Cauchy formula is applicable to the given
f unct

ions:)

1.e.)

- - I'
- (1<s<4).

f (4)- t (I) _
g (4) g ( 1) g'

(\037)

,)

(\037)

2
11 2
27-9= 3\0372-14\037+20
2\037

Solving the latter equation,we

==
\0372

4.

Of these two values only

\037l

two values of

find

== 2 is an

functions f (x) = eX and g (x)=


ditions of the Cauchy theorem in the interval

.8.

3.t On the curve y


line is parallel to the chord

B (2, 8).

the point at
the

through

\0371

== 2

and

interior point of the interval.

3.t.7. Do the

==xa find

\037:

.)
I

sat isfy the con-

[-3,3]?
;\"x\037

\\vhich

points

the

1)and
(-1,-tangent

[-1,

2],whose end-points are the absand B, the function y ==x3 is continuousand


has a finite derivative; thprefore the Lagrangetheorem is applicable.
According to this theoren1 there \\vill be, on the arc AB, at least
one point M, at which the tangent is parallel to the chord
Let us write the Lagrange formula for the given function:
Solution.In the interval

cissas of the points

AB\037

f
or)

(2)-f(-1)==f' [2-(-1)],)
8 -1-1 == .3;)
(\037)

3\302\2432

whence)

Sl ==

-1,

==
\0372

1.

The obtained values of are the abscissas of the desired points


in
(as we see,there exist two such points). Substituting and
the equation of the curve, we find the correspondingordinates:
\037

Y1

- - . - -_ .
1:3
\0371

Y2

1:3
\0372

\0371

--

\0372

Thus, the requi red poi n ts are: M 1 ( 1, 1) and M 2 ( 1, 1), 0 f w hi ch


only the former is an interior point on the arc AB.
Note. This problemcan be solved without using the Lagrangetheorem; write the equation of the chord as a straight line passing
t\\\\,O
given points, and then find the point on the curve at
through
the tangent is parallel to the chord.)))
\\vhich

134

Ch.

III.Differential

3.t .9.Taking

tion, deduce the

Calculus: Investigation

of Funci's)

advantage of the test for the constancy of a funcformulas known from elementary mathfollo\\\\'ing

emat ics:
(a) arc sin x+ arc casx == n/2;

1-

(b) sin 2 x==: ( 1 cas2x)/2;


2

x
(c) arccosI+x\037 ==2arctanxat O\037x<oo;

(d) arcsin

l-j\037xX2

Solution.(a) Let

J1-2arc tan x at

2 arc tanx

at
at

1,

-l<x<l,
\037

{ -n-2arctanx x\037-l.
us

considerthe function

f (x) == arc sin x

-1,1].

+ arc casx,

The derivative of the indicated


defined in the interval f
function inside this interval equals zero:

f' (x)= V/_X2 V/_X2= 0

(-1< x< 1).

to the test for the constancy of a funct ion t (x) ==const,


e. arc sin x+ arc casx ==C 1 < x < 1).
To determinethe constant C let us put, for instance, x== 0; then
we have n/2 == C) whence

(-

!\037ccording

i.

The

arcsinx+arccosx==n/2
(-1<x< 1).
validity of this equality at the points x == + 1

directly.
(b) Let us take the function
t (x)= sin2 x+

IS

verified

i-cos2x

-00< x < 00.The derivative


zero:
x-sin2x == O.

defined throughout the number scale:


of this function is everywhere equal to
f' (x) == 2 sin x cos

According to the test for the constancy of a function


.

sin2 x +

2cas2x ==C
1

.)

To determine C put, for instance, x== 0; then we get 1/2==C.


Wherefrom)

'2x+ \"2C'.os2x-_J2')))

SIn

\037

3.1.BasicTheorems

on Differentiable

135)

Functions)

or)

. 2x== 1 cas 2x
sIn
2
(c) Let

\302\267

introduce the function

us

(x)

1-

arc cosI

===

x2

+ x2

2 arc tan x,

::;;:> 0:1.
=
l+x2-2x(l+x2) -

determined along the entire number scale,since

The derivative of the function f (x) is zero for all x

I'(x) =

-4x

/1_(
JI

-'

I-X2 2(1+X\037):a
I

I\037

4x

+ )
x2

+x2

\302\267

According to the test for the constancy of a function


I -x
arccosl_t-x2
-2arctanx===Cat

x>O.

1,

0-

To determineC let us put, say, x


which gives C arc cos
2 arc tan 1 O.
The validity of the proved formula at x 0 is verified directly.
Note. At x = 0 the function arc cos
no derivative. At
=;=;: has
x < 0 its derjvative is
===

===

arc cosI

(
which

===

===

-+ ) = - +
X

x2

x2

enables us to deri ve the forrnula


2
arccos1 +X2 :=: 2 arc tan x (x < 0).

l-x

The latter formula can be obtained on the strength of the fact


is an even function, and 2 arc tan x is an odd one.
that arc cos

:;;:

3.1.

to. As is known, (eX)'= ex for all x. Are there any more


functions that coincidewith their derivatives everywhere?
Solution.Let the function f (x) be such that f' (x) f (x) every\037

where.

Let us introduce the


qJ

runct

(x)=

ion
f
e\037)

= f (x)e-x.

The derivative of this function equals zero everywhere:


X
(()' (x) ==f' (x) e-X-e- f (x) == O.
By the test for the constancy of a function f
f (x) ==CeX
.)))

(x)/ex = C, whence

136

f'

Ch.

III.Di(Jerential

so,we have

And

(x) ==f

the group of functions for which

proved that

X2

> Xl'

the inequality
arc tanx2

-arctan < -XI

Solution.To the function


apply the Lagrange formula:)

X2

Xl

(X 2

=-=-

Xl

Since)

,)

(x) ==arc tan x on the interval [Xl' x 2]

arctanx2 -arctanxl
\\vhere

of Funcl's)

(x) is covered by the formula f (x) -= Cex .

3.1.t 1.Prove

where

Calculus: Investigation

l-I-\0372

< < x 2.

-XI),

\302\243

< 1+;2< 1 and X2-XI > 0,)


arc tan X2 arc tan Xl < X 2 Xl'
1

\302\260

then)

In

particular, putting

Xl

\302\260

arc tan

3.1.12.
Show

and

Xl

x<x

==X,
(X

we get

> 0).

the square roots of two successive natural

that

numbers greater than

N2

differ
f

Solution.To the function

apply the Lagrange formula:

less than Ij(2N).

by

(x)==Vx

on the interval [n,

n =
f(n+l)-t(n)=J/n+l-V\"
2
where n < < n + 1.

+ 1]

,
\037--

\037

\037

If

> N2,

then

\302\243

> N2,
V-n

3.1.13.
Using the

of the

hence

1/(2V1)< Ij(2N), whence

n < Ij(2N).
+ 1-1/\"

Rolle theorem prove that the derivative

ff

(x)

function)
f

(x ) -==

(xsin x

{a

<

\037

at

> 0,

at x ==

\302\260

vanishes on an infinite set of points of the interval


1).
Solution.The function f (x) vanishes at points where
\037O,

sin (nix)==0, nix ==kn,


k

x == Ijk,

= 1, 2, 3, ...)

Since the function f (x) has a derivative at any interior point


the Rolle theorem is applicable to anyone)))
of the interval [0,

1],

\037

3.J. BasicTheorems

of the intervals

...,[1/(k+l), ...o..

on Diflerentiable

Functions)

137)

[1/2,1],[1/3,1/2],

l/k],

Consequently,inside each of the intervals of the sequence,there is


a poi n t
And

so

\037

-i- 1)< k < 1,/k , at w hi ch the der i vat i ve


k) ==
have shown that the derivative vanishes on an infinite

k' 1/ (k

\\\\\"e

{'

(\302\243

set of points (see Fig.38).


3.1.14.
The Legendrepolynomial is

a polynomial defined by the following


formula (Rodrigues'formula):
Pn (x) = 2

llfn.
I

\302\267

dn
d x)II

(x2 -1)n(n == 0,

1,2,

...).

Using the Rolle theorem, prove


that the Legendre polynomial Pn (x)
has n different real roots,all of them
:c)
0
found between 1 and +
Solution. Consider the function
2
f (x) ==(x
1)'1==(x 1)n (x -1-1)'1.
This function and its n 1 successi.
ve derivatives vanish at the points
Fig. 38
x ==+ 1 (use the Leibniz formula for
higher derivatives of the product of two functions).
I t follows from f (1)==f ( 1)== that inside the interval
1]
= 0,
a point
can be found at which f'
x == will be the
root of the first derivative. Now apply the Rolle theorem onceagain
to the function f'(x) on the intervals
We find
1 the function f\" (x) has two more roots
that besides -t- 1 and
on the interval
1, Reasoning as before, we will show that,
fhe (n-I)th derivativ.e has
more
apart from + 1 and
roots on the interval
the function {tn-I)(X) has all in
al1 n -t-- 1 roots on the interval
1, which divide this interval
into n parts. Applying the Rolle theorem once again, we ascertain
that the function fen) (x), and hence,the function P\" (x)= n fir/) (x),
2 'n!
has n different roots on the interval

\302\2431

1.

- -[- 1].

(\302\2431)

[-1,

\037I]'

-1,
i.e.
[-1,1],[1],
[-1,1].

3.1.15.
Check whether

[.-],

i.e.

\302\260

\037I

[\302\2431'

1].

(n-l)

the Lagrange formula is applicable


to the
functions:
following
2
(a) f (x) ==x on [3,4];
(b) f(x):=-:lnxon
2
on [0,
-i(c) f(x)=-= 4xa
4
==
x
on
(d) f (x) V
1/2,1/2].
1)
If it is, find the values of
appearing In this formula.
_

-5x [1,3];
x-2
1];
(x- [\302\243

3.1.16.
Using the

Lagrange theorenl

estimatethe

value

In(l-re

))))

Ch. III.Di(1ere.ntial Calculus: Investigation

i38

3.1.17.
Using the
1

of Funct's)

Lagrange formula prove the inequality

\037

< In ( 1 + x) < x

at x

> 0,)

':

Forms.
* 3.2.Evaluation of Indeterminate
Rule
L'Hospital's

I. Indeterminateforms of

the type

If the functions r (x)


and g (x) are differentiable in a certainnpighbourhood of the point a,
except,may be, at the point a itself, and (x)=1= and if
lim f (x)== lim g (x)=lirn f (x) == lim g (x)== 0 or
\037

g'

x-a

x-a

then)

I 1m

x-a

00,

x-a)

f', (x)

(x) == I 1m

_ a g (x)

0,

_a

(x))

lim t:
exists( L'Hospital'srule).The point a
provided the limit x...ag
X
QU.
may be either finite or improper + 00 or
I I. Indeterminateforms of the type O.00 or 00 00 are reduced

- -

\037x))

to forms of the type

I.

II
Indeterminate
to forms of the type

mation [f (x)]qJ(X)

===

or

by algebraictransformations.
or
are reduced
forms of the type 17J ,
O. 00 by taking logarithms or by the transfor\037

eqJ(x) In

00\302\260

0\302\260

(x).

3.2.t. Applying the L'Hospitalrule, find the lirnits of the


. eax_e- ,.
1m VTT2X+ ;
(a) xItm
(b) I'
(J + x)
0
-I r 2+x+x
.
e-x ---2x
.
In(l+x2) '.
(C)
(d) x - 0 cos3x-ex-sIn. x ;
0
si 3x2
I'
. e /x _ 2
.
(e) x1111
2 x)
(f) hm0 cos(2x
x ... 2 arc tan x

wing functions:

,r--

2aX

\037

1 IIn

In

1 1rn

In

-I-

,)

1t

00

\302\267)

both functions f (x) eax _e-2ax and g(x)==


are infinitesimals in the neighbourhood of zero,since
Ii rn f (x)== 1
Iim g (x) == 1n 1 O.
1 == 0;

(a) Here
:::Solution.
(1 x)
In

eX

-+

-+

follo-

-\037

\037

===

-\037

0)

g'

1,
> - 1).

Furthermore f' (x) and


of the
(x) exist in any neighbourhood
-x
does
not
contain
the
that
x-::=:O
and
point
point

g'(x) =--\037
l+x)))

=1=

(x

3.2.Evaluation

\037

of Indeterminate

139)

Forms)

Finally, there existsa limit of the ratio of the derivatives:)


. (x) = . aeax +2ae-2ax = 3a.

!' !t!?o
L'Hospital
. -- = .
(I+x)
g'(x)

:\037\037no

Therefore the

rule is applicable:)
e 2ax
aeax+2ae-2ax-=3a.)

eax

!l/.l1
o

1/(1 +x)

111

(*))

I/(I+x)

!t\037no

of the ratio is computed according to the


result is usually written directly as shown in (*).
Whether the desired derivatives and limits exist is ascertainedin
In casethe ratio of the derivatives
the courseof calculation.
again represents an indeterminate form, the L 'Hospitalrule should
be applied for a second time, and so on until the indeterminacy is
removed or until it becomesclearthat the required limits do not
exist.Therefore, henceforward we write only the necessary transformations, leaving to the reader the task of checkingwhether the
conditions of their applicability are fu lfilled.

Note. When the

limit

L'Hospitalrule the

\037:

(b )

x--+-l)

(e)

.t

--+

0)

In

3x

sin

tim

V + 2x + =
y'2+x+x
2

Ii In

cos(2x2

2/(3 V(1+2xr\037)

1/(2V2+\"X)+1=9;

-6x
_
-x)- rl!?o
xl\037\0371

cos3x2 CQS (2x2 -x)_

-x)
-x) r

1)sin (2x2
cos3x2 cas (2x2
6r
4x I
t l!?o

i;\037

(4x\037

2
x l\037o sin (2x

-x)

\302\267)

factor is computed directly, the limit of the


second one, which representsan indeterminateform of the type
IS
found with the aid of the L 'Hospitalrule:)
The limit of the

first

\037

x
cos3x2 cos (2X2_X\037 - I
1m
2
I
4x
si
n
(2x
_
0
x 0

6 I 1m
--+

-6. -]

\037

that,

x)

(4x-l)cos(2x2 -x) = 6.--I. =-6


as x + 00, the functions Xk (k > 0);
--+

lim

3.2.2.It is known

--

\302\267

\037

(a > 1) are infinitely

large quaFltities. Applying the L'Hoslogax;


pital rule, compare these quantities.
1
loga e
1oga x
x
SaIu t ton. 1 I un xk = I 1m kxk -1 == I ogae ]1m 1 = 0;
kxk
X
t
+
x
aX

--+

--+

xa
Xffl

00

==

00

I 1 IT)

--+

+ (fJ

-=

--+

mxTll

ax

111

+ 00

\302\267

\302\267

\302\267

\302\267

I 1 rn

\302\267

.
.

--+

I 1m

--+

00

aX

(1n

!
a)m)))

-.
+

00

140

III.DiOerential

Ch.

Calculus: Investigation

of Funct's)

power function Xk (k > 0) increasesrnore rapidly than


the logarithmic function logax (a > 1), and the exponentialfunction
the base exceeding
increasesmore rapidly than the
aX with
unity
ln
power function x

Hence,the

.)

Find the limits:


3.2.3.
-\037 .
li01 \037
(a) x-I
( x x- ) '
\037. .
(c) - ( \037)
0 x
In

cot x-\037 ;
(b) xIim
)
_0(

Iiln

eX

x)

have an indeterminate form of the type 00 00.


to an indeterminate form of the type . and then

Solution.(a)

We

---

Let us reduce it
apply the L 'Hospitalrule:)

I Itn

,,-+ I

\037

. x-I -In.x== 11m


. I-I/x
x
x-I x+ l/x ==
.
x- == .
== 1m

1m
x- ) == x-I
(x-I)

( In x

In

In

--.

In1
x-+lxlnx+x-l x-.11nx+2
I

2)

3.2.4.Find the
(a)
{b)

-m

x n In

Iim
x

x (n

fl n

(1+ sin2 x) cot

0)

11m xn

:X-O

(1+ x) ] .

-==.

We

In x

-.0[In(1+

lim

==

'

sin2

ljx

nx- n - 1

1m
x-1m0 x-n x-O

(b) We have an
.x

n2

:
- 1

have an indeterminate form of the type 0'00.


and then apply the L 'Hospitalrule:
Let us transform it to

Solution.(a)

> 0);

0)

Ii

limits:

--- .
I

I 1m X'Z==

x-O)

. n >O.
' since

indeterminate form of the type O. 00:

x) cot

In 2

(l + x)]=
I

.
=hm

lim

x _ 0 an n ;;\037x
\037n

'.)

sln\037

(\037\037

x\037

SIn 2x

x- 0 2 {I+tan2 [ln 2 (l +x)]}In (I +X)' I


l +x

== I Iln
x\037

3.2.5.Find the

sin x
In

(I -1-x )

===

Hm.its:

(ljx)sin x;
(a)x lim
-+0)

(b)x Ii-.In
-t

Xl
0)))

- .

/ 1 n (e1: 1)

.01

1I
K

0)

COS

l+x)

\037

1.)

3.2.E\"oaluation

\037

Forms

of Indeterminate

141)

Solution. (a) We have an indeterminate form of the type


Let y\037(l/x)sinx; then
In y ==sin x In
1 i IT1

-\037

-1

ny

==
X

\037

(l/x),

sin x 111 ( 1/x) (i n deter m in ate for m 0f the

1 i In

+ 0)

Let us transform

to

it

00
00
x

00\302\260.

and apply the

ty

eO.00).

L'Hospitalrule:

.m n = . -In. == .m -l/x. 2 == .m sin2 x == 0.


x x_OXCOSX
x-o-o -(cOSX)/Sln
.t-o-+O
Hence,xlim
- ==e == 1.
3.2.6.
Find the limits:
I1

I I n1

I1

I1

l/SII1X

x\037+O

-10)

(a)

(sinx)tanx;

IiIn
x

3.2.7.
Compute)

:rj

Solution.Let

(b )

2)

- !im+

Jtj

Ii m XX.

x-O)

(tan x) cot x.
0)

take advantage of the

us

(tan x)

cot x

identity)
x
In
tan
==ecot
x,)

but)
x

cot X In
\\im
+ Jtj 2 0

tan

-0-

x=

\\im

tan

x _ + Jtj 2 0

Whence)
tim

x-+Jtj2 - (tan x)
the existence
of

I\037

an

x)

x_

--===.
0
y=tanx-+oo
I 1m

Iny
y)

cot x ==eO ==

1.

0)

Ascertain
3.2.8.
.
si (1/x) .
.
,
(a ) 1m
I

x_ 0

the following

limits:

X2

-.
SIn

2+2x--j-sin\037x ;
(b) xlim
_ (2x+sin 2x) esm x
00

(C)

.
1m

-0-

rfj2

tan

secx

L'Hospitalrule be applied in computing them?


\037pplication lead to the C0rrect answer?
Solution.(a) The limit existsand equals zero.Indeed,
. si.n (1;x) == I 1m
. .x I ImXStn-==.
. .
1 0 = 0.
I 1m
SIn x
SIn x

Can the

Does its formal

X2

-0-

x_0

x)

- -

But the limit of the ratio of the derivatives does not


1
2xsi11 (1/x) cos(]/x)
0 I 1m cas ,
I In1
cos
x
_
x
0
x 0

x)))

\037

exist.Indeed,

142

Ch.

III.Differential

cos(l/x) does
applicablehere.
but

lirn

not

-+

Calculus: Investigati0n

exist, hence

the

Funct's)

of

L'Hospitalrule

the ratio of the funct ions does not exist:


2
2 + 2x-I-sin 2x _
. x
I 1m
I UTI
1 --L
.
e- x
2
Sill
2x
-1e
(
x x
x
(2x+ sin 2x)
e-sin x does not exist,since the function e- x traverses

(b) The

limi t of

- . 'x

\\

S1I1

\037

but

IS not

oc

-;in

/)

\037

\037ill

li\037n

X -+

the)

oc

values from lie to e infinitely


many times.
Now we will show that the limit of the ratio of derivatives exists:)

I un

\037

\037

2-t-2 cos 2x
cas
2
2x+ (2x-1-sin 2x) cas xl e
[2+

-.

I 1m

\037

S11l

4 COs\"2

.
)
4 COs:'
x+-(2x+sin
=-=

cosx

2x)

I i 111
x -+ X)

e-_ \"in

4 coS x

e
2x+4cosx-+-sin2x

-; i

--,
0

cos
--+ O.
.
stocethe function r'inx IS bounded, and ')....x+ 4 4cos
SIn 2x x
Herecosx,which vanishes for an infinite set of values of x, has
been cancelled
out. It is the presenceof this multiplier that makes
the L'Hospitalrule inapplicable in this case,sinc2 it simultaneously
nullifies the derivatives of the functions being compared.
. tan x . sec2 x
. seex . t n x
(C) I 1m secx = I 1m seex tan x == I 1m tCln x = I un see = ...
X

x_

JTj2

\037
I

-\037

00

- Cl

x-n/2

x-JTj2

t-JT/2

x)

Hereapplication of the L'Hospitalrule gives no usefuJ result,though


there existsa limit:
. sinxcosx== I Inl. Slnx==
. tan x I 1m
. 1.
I 1m
x-JTj2

-=
seex

x-JTj2

eosx

:r./2)

\037

3.2.9.Using the L'Hospitalrule find the limits of the follo\\ving


. (x2-3)
a x -x
Iim
(b)
(a) It m 2 + 3 __ 10;
t_2
.
] 4 sin 2 (nx/6) .
x-x
,
(c) 1m x SI. ;
(d ) Iim
x:!
x_ 0
.
x-acot (x-a); (f) lim (Jt-2arc tan x) In x;
1m arc sin
(e)x-a
a
xtJnx
.
;
(h)
(a Ijx 1)x (a > 0);)
(g) 1m

functions:)

ln

In
X

X\037+O

( )
l

tan

x)

I)

+\037)

lilll

-\"_00)

(j)

lin1
x\037

x)

x\037l)

x)

(i) lim (cosmx)f1/x;


X-O)

-In

a(

_..:.
)

tan (Jtxj(

a)))

2a\302\273;

.\037

(k)

lirn
x

_+

\037-\037;
)
( x
tin

\037

(x

I\037

(1)

In

lirn

(m)

3.3.Taylor's Formula:

-cot2

);

(n)

lirn xl/In

->-

0)

(e''V-1);

x2 In

}n:, [x
\037

x)

_-

\037

(q) xlim
..10)(lncotx)tanx;
\037

+ )] ;

\037

/
.
5
(p) t11m
-..0 ( 2+ V 9 -t- x )
. e /x
.
(r ) ltm 2 arc tan x 2

(0) lim x2 -cosh\037--l ;


J
x-+XJ
[

143)

Calculation\037)

.4pproxinzate

t sin x

00)

j'[

3.3.Taylor'sFormula.Application
to Approximate

Calculations)

continuous and has continuous derivatives


order
on the interval [a, b],and has a finite derivative
of the nth order at every interior point of the interval then at
xE [a, b] the following formula holds true:
If

the function f (x) is

through

n-l

{(x) = {(a) + f' (a) (x

a)2

a) + f\" (a)

(x\037-!

+ f'''(a)(x -;-! + ... + (In-I)(a)

a)\037

(x(\037

where)

and
\037==a+8(x-a)
It
If

\037\037)\037

(s)! --;:Ia)n
X

f<n)

.)

0<8<1.)

is cal1edTaylor's formula of the function f (x).


this formula we put a = 0, we obtain Maclaurin'sforn-lula:

in

{(x) = {(0) +f' (0)x +

f\"

(0)

+ + {(n-l)(0)(n
xn
+
\" where
\302\267

\037;

\302\267

\302\267

\037\037;

f(n>

The last term

in

(\037)

n.)

the Taylor formula

Lagrange'sform and is denoted Rn (x):


R n (X ) = f<m

accordingly,the remainder

in

==8x, 0

< 8 < 1.

is calledthe remainder in

[a+8 (x-a)](x_ a )n.,


n.),

the Maclaurin formula has the form

n
R n (X ) = f<n> n.)(8x)
, x

3.3.1.Expand the

\037

-2x

-x +2x-1

4
+x:J 2
polynomial P(x)==x
in powers of the binomial
using the Taylor formula.
Solution.To solve the problem it is necessary to find the value
The)))
of the polynomial and its derivatives at the point x =

x-I

1.

144

Ch.

III.Difierent

ial Calculus: I nvestigaiion

relevant calculationsare given

of Funct's)

belo\\v.)

P(I):----O, P'(I)===O,

(1)= 18,
P (1)== 120,

0,

(1)

P\"

=-=-

P'\"

P(4) (1) 72,


p<ll) (x) ==0 (n

u\302\273

=--=

at any

x.

6))

\037

Substituting the values thus found into the Taylor formula, we get

3.3.2.

x-

x-

x-

18
72
120
[).
P (x) _ 3!
1)3
(
+4f( 1)4 +51( ),
P (x) ==3(x-l)3+ 3 (x-l)4+ (x-l)5.

Applying the Maclaurin formula,


9
(up to x , inclusive) the function

expand

f(x)==ln(l+x),
[0, 1].Estimate the

defined on the interval


the remainder.
Solution.)

f (0)

===

In

===

in

po\\vers

O.
\037

_I )n-l (n-l)!
,
( + x)n

===

error due to deleting

The derivatives of any order of the given function (see


f (ll) (x )

of

2.3):

...).

(n=l,2,3,
f(ll)(O)===(-l)n-l(n-l)!

Substituting the derivatives into the Maclaurin formula, we get


In

x-2+ x3-

(1+ x) ==

x\037

\302\267

where the remainder R]o(x)


as follows:
R 10 (x) ==1(10)

10==

- 10!

+x

\302\267

\"9

-t- RIo (x),

the Lagrange form

9!

\\vill

be

\\\\Titten

x lO

(0 < < x).


10(1-1(I-ILet us estimatet.he absolute value of the remainder RIo (x);kepping
x 1 and > 0, we have
mind that 0

10!x
(\037)

in

in

\302\267

\037

\037

RIo (x )

xlO :==

\037)lO

\037

\302\243)lu

\302\243

<

10
10
1
How many terms in the Maclaurin formula should be taken
for the function f (x)==eX so as to get a polynomial representing
this function on the interval
accurateto three decimal
place-?
Solution. The function f (x) ==eX has a derivative of any order
I

(\037\037\037)

3.3.3.

[-1,1],

(n)

(x) =

eX.)))

\302\267

\037

3.3.Taylor's Formula:

.\037

145)

Calculations

Approximate

to this function. Let


Therefore, the Maclaurin formula is applicable
derivaus compute the values of the function eX and its first
tives at the point x ==0, and the value of the nth deri vat ive at the
Ox (0 < 8 < 1).We will have
point
- (0) =-= 1;
==- lll lJ
f
f (0)-:=:f' (0) ==f\" (0)==
e;=-= e8x
f(ll>

n-l

===

\037

...

===

.)

(\302\243)

Whence)

f (x) == 1

x
+ 2!-r .
+ 11

x ll

\302\267

-J-(n _ 1 )!+ R n (x),)

x\037

\302\267

where)

==,e .
XU

(x)

Rll

Since,by hypothesis,
I

Hence,if

1 and 0

\037

Ix III Bx
(x) == n., e

Rn

< 8 < 1,then


< n. < n.3

,e -,
I

\302\267

the inequality)

8x

n.)

3
fiI 0.001

(*))

\037

is

ful

filled, then the inequali ty

0.001

(x)
7
will
be fulfilled apriori.To this end it is sufficient to take n
(7!== 5040). Hence,7 terms in the Maclaurin formula wi 11 suffice.
I

Rn

\037

\037

At
3.3.4.

what

values of

x will

cosx

the approximate formula)


x2

\037

x4

1-2!+4T

have an error less than .O.00005?


Solution.The right member of the approximate equation represents the first six terms in the Maclaurin formula for the function
cosx (the second,fourth and sixth terms are equal to zero; checkit i).
Let us estimateR6 (x). Since(COSX)(6)= cosx, then
cos-ex 6 .--- x 6
R 6 (X ) ==
6!
6! x
c'hoosethe values of x that
For the error to be less than 0.00005,
the
inequality)
satisfy
6

< 0.00005.
inequality, we get x < 0.575.
I

61

Solving this

\037

3.3.5.
Compute the

approximate va lues of:


(b) sin
(a)
cos5\302\260;

20\302\260,)))

146

Ch.

accurateto

III.Differential

Calculus: Investigation

decimal places.

five

-2!+ 4f-... +

Solution.(a) Into the Maclaurin

cosx== 1

x2

ot

Funct'\037)

formula
x2n

x4

(-I)1Z(2n)!

-t- R 2n +

2)

substitute x == Jtj36; since

2T= 2.362 = 0.003808,)4! = 6 (\\ )


Jt2

x\037

x\037

\037

\037

== 2

\037

we confine ourselves to the following

cosx

.4 . 10 -6,

terms:

l-xJ2,
2

\037

the error being estimated at


IR 4 (x)I=1
co:!ex
x41\037
And

E'O,

6.
<2.5.10-

I\037r

the required accuracy

within

J1

cos5\302\260==cos

36

==

==0.99619.
1-0.00381

3.3.6.
Compute the approximate value of V 83 accurateto six
decimal places.
3.3.7.Prove the

(a) x-xj2 <


2

(b)

tan

In

inequalities:

(1+x)< x at x> 0;

at
x > x+x3j3
2

0 < x < Jt;2;

V l+x<I+2 x at O<x<oo.
(c) 1+2
Solution.(a) According to the Maclaurin formula with

x-T<
x

inder

we have)

R 2 (x)

In

(1+ x) ==

where 0 < < x.


According to the same formula

the rema-

.)

x-2 (I +
x'\"

,)
\037)2

\037

In

'0 <

\037l

<

X.
x2

Since 2 (l +

(I + x) ==

>0
\037)\037

with

x2

x-T+ 3 (I
x3

and 3
(l +

\0371)3

x-xj2<
2

3.3.8.

the remainder
x3
+\0371)3

(x) we have

, where)

> 0 at x > 0, it
In

R3

follows that

(1+x)< x.

Show that sin (ex + h) differs from sin ex + h


r.oore than h 2j2.

Sollit ion. By Taylor's formula

sin(a+h)=sina+hcosa\037

sinG;)))

cosex

by

not

3.4. Taylor'sFornzula:

\037

whence)
1

\037

Evaluation

h2

2 sin T

(sin a + h cosex) 1==

sin (ex + h)

]47)

of Limits)

h2

\037

\037

\302\267)

3.4.Applicationof Taylor'sFormula to E'valuation

of Limits)

The expression
f' (a)

\"

(a)
(a)
f(x)==f(a)-i-, (x-a)n\037]., (x-a)+\037(x-ap+...-tf

f<lZ)

L.

x-a

+ 0 (I
Ill)
the
in Peano'sform \\vhere
the
with
remainder
is
Taylor formula
==
\037 a, the function
x
a
means
as
that,
(x) has a high(x)
(x)]
= O.
er order of smaJIness than the function 1tJ (x), e. !inl x)
X

['l'

cp

In

cp

i.

particular,
f (x) = f (0)

at a

+ f'

===

,\\0)

\\ve

\037\302\253

x\037a't\"

have

f\"2('0)

x2

+ ... +

x\"
f(\"\037:O)

+ 0 (i x In).

Peano'sform

of the remainder for Taylor's formula shows that,


substituting the Taylor polynomial of degreen for f (x) in the
neighbourhood of the point a, we introduce an errorwhich is an
infinitesimal of a higher order than (x-a)nas x \037 a.
The following five expansions are of greatest importance In solving practical problems:

when

x
x
+ . + nT
+ x + 2T
+ a (x );
.
x
.
1)
+
+
(
slnx-x-3f 4
(2n-I)!+ a (x ),
x
+
cosx===
2T+4!+'\"+(_I)1Z (2n)' +0 (X );
x2 + .. + (1,(a-l) (a-n+J)
X /l + a (x ll) .
,
(1+X ) 1+ax+ a(a-1)
21
n!
2
x
X3
x
In (1+ x)
x-T+ :3+ . . + (- n + a (x ).)
eX

===

1-

1Z

...\302\267

lZ

\302\267

1Z

X 21Z

2lZ

...

X 21Z

X2

C1==

\302\267

2lZ

1Z

===

3.4.1.
Expand the

gral powers of x
wi th

r.espectto x.

I YZ- 1

\302\267

up

function f (x) = sin2 x


to the terrns of the

x%e-X

fourth

lZ

in positive inteorder of smallness

Solution.We have
X3
_X2 l-x+ +0(x2) =
t(x)=[X-T+O(X4)
]
1
[
x4
5 4
3
4
==x2
a
_X2 + X
x + 0(x4
+
+ a (x ) = x3 -lf

-T

2
X2

-T
X4

(xf'\302\273

).)))

148

III.DifJerential

Ch.

3.4.2.Expand the

(a) f (x) == xVrl-x2


(b)

ollowing functions:

-cosxIn (1+x);

positive integral powers of x


smallnesswith respect to x.

in

of

3.4.3.
Applying
.form,

(C)
.(d )
(e)

.
.
1m

It nl

eX

\"

It in

sin x

x (1-I-x)

.\037

ex +e-x
x2

-2

.'

\302\267

Solution.(a) Retaining the terms

respectto x
x

\037

I-

up

fourth

the denominator and the numerator, we get

in

. 1-(I+x2)1/2cosx
I+x2 cosx= 1m
=
x4
x 0
x2
x4
5
x4 +0
1-[ + 2 x2 + 1/2(-1/2)
2
1 [ 1-2 + 24 +0 )]

tan 4 x

\037

\037

I i rn
x
0)

order with

x.)

x--\037o

!im

to the

X\"\"

cosx_e-x2 / 2
X

\037

Peano's

V +3x--:-Vl+2x

x\037o
1

in

\037

the remainder

with

mits:

tan4

lim
x

order

Y 1 + x 2 casx ;

x\037O)

(b)

fifth

to the terms of the

up

the Taylor formula

compute t he li

(a) r1m

of Funct's)

(I + sin x)

(x) == In

Calculus: Investigation

\037

_'
-

hm

(X

(X4)

4\"

x\037o

3.4.4.Expand the

x4

+\"8x

4)

- 1...4+ -_'
24

,t-

0 (x 4)

0 (x4) _
+
4
] -3\"
[

hm

x4

X-40 3

following functions

in

\037

\302\267

X)

positive integral powers

of the variable x up to the terms of the indicated order,inclusive:


2X x2
up to the term containing
(a) f (x) ==e
cos
to
x
the term containing x6;
In
up
(b)
4
up to the term containing x
(c)
I

xC>;

.)

eX \037

\037

3.5.Testinga Functionfor M@notonicity

Let a continuous function f (x) be defined on the interval [a, b)


and have a finite serivative inside this segment.Then:
(non-increasing)on [-a, b] it is
(1) For f (x) to be non-decreasing
all. x in (a,
necessaryand suffi.cient that f' (x) 0 (f' (x) 0)
\037

\037

f\037r

b).)))

3.5.Testing

\037

Function

149)

for Monotonicity)

(2) For f (x) to be increasing (decreasing)on [a, b] it is suf ficient to fulfil the condition f' (x)> 0 (f' (x)< 0) for all x in (a, b).
Determine the intervals of monotonicity for the following

3.5.t.

funct

ions:

(a) f (x)== 2X2

9x\037

e ;
(c) f (x) =-::x:!

.\\-

(d)

f(x)=lnlx\\\037

(e) f (x)==
(f) f (x)

===

4x:\037

eX

-Inx;

(b) f (x) = 2X:1-

24x -r-7;

21x2+ 18x-+ 20;

+ 5x .

Solution.The solution of this problEm is reduced to finding the


intervals in which the derivative preserves its sign. If the function
f (x) has a continuous derivative in the interval (a, b) and has in
it a finite number of stationary points Xl' x2,
X n (a
Xl
then ff (x)
< X2 < < Xn < b), where ff (xk) = 0 (k == 2,
preserves its sign in each of the intervals (a, Xl)' (xJ , X 2),
(xn P Xn) , (xn' b).
(a) The function is defined at X > O.
Let us find the derivative
f (x) 4x 1IX
The function increases if
1Ix > 0,
X > 1/2.
The function decreases
if
< 0, e. X < 1/2.
And so, the function decreases
in the interval 0 < X < 1/2and
increasesin t he interval 1/2< x < -+-00.
(b) Evaluate the derivative
== 6x2
== 6 (x2
ff (x)
It
vanishes at the points x ==
and x== 4. Since ff (x) is a
trinomial
with
a
at
coefficient
its
quadratic
highest-powerterm 6 > 0,
then f' (x) > 0 in the intervals
and ff (x) < 0
00,
!J)
(4,00),
in the interval
4). Consequently, f (x) increasesin the
two intervals, \\vhereas in
first
4) it decreases.
the derivative
(c) In this 2case
x)
x ) e-X vanishes at
f' (x) (2x-==
2)
o)
the poi nts x 0 and x == 2. In
the intervals
00,0)and (2,00)
Fig. 39)
the derivative ff (x) < 0 and the
in (0, 2) the derivative ff (x) > 0 and the funcfunction decreases;
tion increases(see Fig. 39).)
Find the intervals of decreaseand increasefor the follo-

.,

\302\267

\302\267

..
< <
1, ...,n),

\302\267

_.

===

4x-

4x-llx

- i.e..

-18x-24

i.

-1

(- -1),
(-1,
===

(-

3.5.2.

wing

functions:)))

(--1,

-3x-4).

...,

150

Ch.

(a)

(x)

III.Differential

Calculus: Investigation

cos(njx);

\037

on
(b) f(x)\037sinx+cosx

Solution.(a) The function y


throughout

the number

As

[0,2n].
\037

cns (njx) is defined and differentiable

scale,exceptat
Y

of Funct's)

Jt. .

the point

x == 0;)

Jt

== -:)Sin

X)

X\"\"

is obvious, the sign of y' coincideswith that of the multi-

pliersin (njx).

(1) sin (njx)> 0 if


(k\037O, + 1,+ 2,
2kn<njx\302\2532k+l)n
(2) sin (njx)< 0 if
(2k + 1)n < njx < 2 (k + 1)n.
Hence,the function increasesin the intervals
( 2k

and

In
decreases

\037

'

'

2\037

...);

the intervals

( 2k

3.5.3.Investigate the

\037

2k \037

\302\267

behaviour of the function f (x) ==2 sin x +


the interval
nj2).
Solution.The derivative)
x-2cos2 x) =
1
- x) (I +cos
3 = (I-cos
f (x) = 2 cosx + cos.2 X
cos.2 X
4 sin 3 (xj2) sin (3x/2'
cos2
is positive in the intervals
0) and (0, nj2) and vanishes
x==
O.
i
n
at
Hence,
nj2) the function f (x) Increases.
only

+ tan x-3x

(-nj2,

in

'

X)

(-nj2,
(-nj2,
3.5.4.Prove that at 0 < x 1 the iniqualities
x-x3j3< arc tan x < x-x3j6)
\037

are fulfilled.

Solution.We

will prove only the

is proved analogously).

right

inequality (the left one

The derivative of the ftInction


f (x)

is equal

to)

,
f (x)

x3
= arc tanx-x+\"6)

+
I

\037

x2

x2

x2

(x
I)
2= 2 (1-1)

1 -1-

x2

\302\267)))

\037

3.5.Testing

Function

tor Monotonicity)

151)

The function f (x) is continuous throughout the entire number


continuous in the interval [0, and inside this segment f' (x) < O. Therefore, f (x) decreases
on the interval
[0,1]and, ==consequently,for any point x, 0 < x 1,the inequali ty
f (x) f (0) 0 or)
x3
arc
< 0)

1],

scale,in particular, it is

\037

\037

tanx-x+(f

IS

u I fi

lied, whence)

arctanx<x-x6.

Prove the
.3.5.5.

inequalities

x-x/6 < sin x < x


3

3.5.6.

x> O.

at

Prove that for 0 p 1 and for any positive a and b


the ineq uali t y (a -+- b)P a P -t-bp is valid.
Solution.By dividing both sides of the inequality by bp we get
\037

\037

\037

: :

( +IY\037( Y+I)
or)

(1+ x)P

b.

where x =

Let us show

Introduce the

that the
function)

\037

1 -t- x P

(*))

,)

inequality (*) holds true at any positive x.

-+

t (x) == 1 -t- xP (1 x)P;


The derivative of this function

O.

\037

' x- pxP-l-p(l + x)P-l_


--p[Xl-P _ (l+.X)L-P]

f ( )

is positive everywhere, since, by hypothesis,

1-

>

p 0 and x O.
function increasesin the half-open interval
00),
whence 1 xP (1 x)P, which
t (x) == 1 xP (1 x)P f (0) ==
then we obtain
completes the prcof. If \\v e put p

Hence,the

-+

[0,
+ > +

>

3.5.7.

3.5.8.
Determine

functions:
(a) f (x) == x3 + 2x

followit1g

i.e.

0,=
Iln,
a+b\037v/ a + Vb (n I).
the function y ==x5 + 2x3 + x increases
everywhere,
3
==
\037

\037/

Prove that
and the function y

\037

l-x decreaseseverywhere.

the intervals of

--5;

(c) f (x) == cosx -x;)

increaseand decreasefor

(b) f (x) == In ( 1
I

x2);
I

(d) f(x)=3\"x3 --X;)))

the

152

Ch.

III.Differential

Calculus: Investi[!,ation

of Funcl's)

2x

2x

(e) f(x)=TfiX;
(f) f(x)= l+x\037
Prove the following inequalities:
tan
x > x + x3/3, if (0 < x < :rrj2);
(a)
(b) eX 1 + x for all values of x;
(c) eX > ex a t x >
to.
values of the coefficient a does the function
At what
3
increasealong the entire number scale?
f (x) = x
:>
value of b does the function
At what
\302\267

3.5.9.

1.

\037

3.5.
-ax
.5.11.

f (x) == sin

decreasealong
\037

x-bx+c

the entire number scale?)

3.6.Maxima and Minimaof a

Function

a function y == f (x) is defined on the interval X, then an inof


Xo of this interval is called the point of maximum
the function f (x) [the point of minimum of the function f (x)] if
there exists a neighbourhood U E X of the point xu, such that the
inequality f (x) f (xu) [f (x) f (xo)] holds true within
The genericterms for points of maximum and minimum of a function are the points of extremum.
A NecessaryCondition fQr the Existenceof an Extremum. At
points of extremum the derivative
(x) is equal to zero or does
not exist.
The points at which the derivative (x)== or does not existare
If

teriorpoint

\037

it.

\037

f'
f'

calledcriticalpoints.

\302\260

Sufficient Conditionsfor the Existenceof an Extremum.


I. Let the function f (x) be continuous in some neighbourhoodof
the point xo'
If f' (x)> at x < Xo and f' (x) < at x > Xo (i.e.if in moving from left to right through the point Xo the derivative changes
sign from plus to minus), then at the point Xo the function reaches

1.

a maXimum.
If
(x)

2.

\302\260

\302\260

<

<

>

> (i.e.

at x Xo
at x Xo and
if in mo(x)
to
left
the
the
derivative
from
right
ving
changes
point Xo
sign from minus to plus), then at the point Xo the function reaches
a mInimum.
If the derivative does not changesign in moving through the
point xo, then there is no extremum.
I
Let the function f (x) be twice di\037erentiable (that is f' (xo) = 0)
has a
th\037n at Xo the function
at a critical point xo' If f\" (xu)
then at Xu the function has a minimum;
maximum; if f\" (xo)
but if f\" (x() == then the question of the existence
of an extrenlum
at this point remains open.)))

f'

\302\260

f'

through

3.

I.

0,

> 0,

< 0,

\302\260

3.6.Maxima

t\037

I.

I'

and Minima

..

of

153)

Function)

0,

. {<n-l)(xo) ==
O. If
II Let
but f(ll) (xo)
(xo) I\" (xo)
is even, then at f(fl) (xo) a there is a maximum at xo, and at
a minimum.
{<'l) (X(I)
If n is odd, then there is no extremurn at the point xO'
IV. Let a function y == f (x) be representedparametrically:
n

===

===

===

<

> 0,

=1==

x==cp(t), Y=='P(t),
where the functions
and

cp

(t) and 1jJ (t) have derivatives both of the first


a certain interval of changeof the arguFurther, let, at t == to

secondorders within
t, and

ment

(b)

0.

=I=-

'P'(t) == 0.)

Then:
(a)
== Xo

<p' (t)

if

===

cp

if

\",\"

< 0, the function y ==f (x) has a


f (x) has a
(to) > 0, the function y
(t 0)

max I mum a t x ==

(to);
\",\"

===

minimum

== Xo == cp (to);
(c) if \",\" (to) ==

at

x ==

0, the question of the existenceof an extremum


remains open.
The points at which cp'(t) vanishes require a specialstudy.

t. Using the
3.6.
wing

first

functions:

derivative,

find

the

extremaof

the follo-

I (x) 43 X 4 -x3 -9x2 + 7;


4
3
(b) I (x) x -8x-t- 22x2 24x + 12;
(c) f(x)==x(x+l)3(x-3)2;
2
x2-3x+
(d) f (x)
+2x +

(a)

===

===

===

\302\267

x\037

Solution.(a) The function is defined and differentiable over the


entire number scale.Therefore, only the real roots of the derivative

--

-3)

18x 3x(x + 2) (x
are crit ical points. Equating this expression
to zero,we find the
x2 == 0, x3 == 3 (they should always be arcriticalpoints:Xl ==

f' (x) == 3x3

3x2

===

-2,

ranged in an increasingorder). lJet us now investigate the sign of


the derivative in the neighbourhood of each of these points. Since
there are no critical points to the left of the point x==
the
has one and the same sign: it
derivative at all the points x <
is negative. Analogously, in the interval
0) the derivative is
positive, in the interval (0, 3) it is negative, at x > 3 it is positive. Hence,at the points Xl ==
and X 3 == 3 we have minima
and
and at the point x2 =0, maxi-

-2
(-2,
-2

{(-2)o=-9 {(3)=-40+,
=
mum f

(0)

7.)))

-2,

154

Ch.

III.Differential

Calculus: I nvesligation

Funct'

of

s)

(c) Just as in item (a), the criticalpoints are the roots of


derivative f' (x), since the function is defined and differentiablethroughout the number scale.Find f' (x):
th\037

I' (x) = (x +

(x

+ 2x (x + 1)3x
3)2+ 3x (x + 1)2(X-3)2

2
)2(x-3)(2x
-3x-l).
x(x-3):=3(X-i-

l)\037

to zero,we find the criticalpoints:


Equating this expression
x I == I, X2 == (3 V 17)/4, X 3 == (3 -1-J/-17)/ 4, X 4 3.
\037-=

Let us tabulate the sign\037 of the derivative


tween the criticalpoints:)
x

lnterva Is
Sign of

I' (x)

< Xl

Xl

minimum
a minimum at

x 3,

3.6.2.
Using the

first

ions:

funct

x2

< x < X 3 x:j < X < X 4

Xt

X..

<X
-1-)

I)

table,there is

from

wing

<

the
is seen
there is a
-1,
and
point
As
==

Xl

<

the intervals be-

in

no extremum at the point


at the point X2, a maximum at thethe point x4

derivat ive,

the

find

extremaof

the follo-

2
(a) f (x)== 3 V \"X2- x ;

(b) f (x) =

V(x-l)2+V(x+ 1)2.

Solution.(a) The function is defined and continuous throughout


the number scaIe.
Let us fi nd the) derivative:
,
f (x) =

2(Vx

From the equation f' (x) == 0 we

+ 1.

x==
F ur

-x)

the roots of the derivative:

find

- 1,

to
thermore,the derivative goes
==

Thus, the critical points are

Xl

\037
\037O 1'-...-)

\037

\037

-1

x)))

3.6.3.Using the

second derivative,
the following functions:)

o.

1.point
The results

of investigating the sign


of the derivative in the
neighbourhood of these

points are given

40.

The

in

Fig.

investigation
shows that the function

maxima:f(-I)==2; f(I)==2 and

extremaof

x ==

at the
infinity
X 2 == 0, X 3 ==

Fig. 40

has two

\302\267

find

a minimum

out the

f(O)==O.

characterof the

\037

(a)
(b)

y
f

== 2 sin x
3
(x) == 2x

3.6.Maxima

-+

and Minima

of

155)

Function)

cas2x;

15x\037-84x
+ 8.

Solution.(a) Sincethe function

is a periodic one we may confine


2Jt]. Find the first and second deri-

[0,

ourselvesto the interval

vatives:
y' ==2 casx

2 sin 2x 2 cosx ( 1 2 sin x);


-2
sin x-4 cas2x.
2 sin x) = 0 determine the
From the equation 2 cosx (1he
interval
on
[0,2Jt]:
points
y\"

==

=-\037=

critical

== 'JI/6,

-<
> 0;
-<

Xl

X2 == Jt/2,

x 3 == 5Jt/6,

x4 == 3Jt/2.

find the sign of the second derivative at each criticalpoint:


(Jt/6) == 3 0; hence, we have a maximum y (Jt/6) == 3/2 at
the point Xl = Jt/6;
=-=
hence, we have a minimum y (Jtj2) == 1 at the
y\" (J1j2) 2
==
X
point 2 'JI/2;
== 3
0; hence, \\ve have a maximum y (5Jt/6) ==3/2
y\" (5'JI/6)
at the point x:J == 5Jt/6;
==
0; hence,\\ve have a minimum y (3Jt/2) == 3 at
y\" (331/2) 6
the point x4 == 3Jt/2 (see Fig. 41).)
Now

y\"

>

x)

Fig. 41

3.6.4.Investigate the
(a) f (x)

-_

-2x

following functions for


(x

< 0),

extrenla:

;;;::

3x + 5 (x 0);
2X2 + 3 (x=1= 0),
(b) f (x) == 4
{
(x==O).
Solution.(a) Though the derivative
2 (x < 0),
f' (x) = f
3
\\
(x > 0)
existsat all points, exceptthe point x = 0, and changessign from
minus to plus when passing through the point x = 0, there is no
)

minimum

here:)

(0)= 5 > f (x)

at

-1< x <

O.)))

155

Ch.

III.Differential

Calculus: Investigation

of Funct's)

This is explained by the fact tha t the function

is discontinuous

at the point x == O.

(b) Herethe derivative ff (x) == 4x (x=I=- 0) also existsat all points,


sign from minus to plus when passexceptat x == 0, and it changes
==
we have here a maximum
x
O.
the
Nevertheless,
ing through
point
which can readily be checked.
but not a minimum,
It is explainedby the fact that the function is discontinuousat
the point

x ==O.

3.6.5.Find

the

extremaof

(a) f (x) ==3x4 + 8x3

(b) f (x) == J/ex2

50

-.

18x2

the following functions:

+60;

Solution.(a) Hereit is simpler to find


-18x2 + 60.Since

the extrema of the func-

tion f1 (x) == 3x4 + 8x3


f\037(x)==
f\037

the

(x)==

2
12x3 +24x2 -36x==12x(x
+2x-3),

12(3x2 + 4x-3),

criticalpoints are:
Xl

==-3, X2 ==0, X 3 1,
===

and the character of the extremais readi ly determined from the


> 0; hencer at the point Xl ==-3
sign of the secondderivative
and the given function f (x)
the function f1 (x) has a minimum,
== 2/3,
obviously has a maximum f
(0) < 0; hence, at
the point x2 == 0 the function f1(x) has a maximum, and f (x) a
minimum
f (0)==5/6; (1) 0; hence, at the point x 3 =-= 1 the
function f1 (x) has a minimum, and f (x) a maximum f (1)==50/53.
(b) In this case it is easier to find the points of extremum of
the radicand)
== ex2
1,
f 1(x)
f\037

f\037

(-3)
(-3)

l;

>

which coincidewith the points of extremum of the function f (x).


Let us find the criticalpoints of f1 (x):
x2
(x) ==0 at the point x ==O. Determine the sign of
t; (x) ==2xe ;
the second derivative at the point x ==

0:

f\037

f\037

(x) ==2ex\037

(1+

2x2

),

f\037

(0)==2 > O.

Therefore the point x ==0 is a minimum of the function f1 (x); it


of the given function f (x): f (0)==O.
11 also be a minimum
Investigate the characterof the extremum of the function
y==coshx+cosxat the point x==O.
Solution.The function y is an even one and apparently has an
extremum at the point x ==O. To determine the characterof the)))

vvi

3.6.6.

\037

3.6.Maxima

and Minima

of

Function)

157)

extremum let us evaluate the derivatives of this function at the

x==0:)

point

x-

sin x, y' (0)==0;


y' == sinh
==
coshx-cos
x, y\" (0)== 0;
y\"
==
sinh x + sin x, y'\" (0) ==0;
y'\"
==
coshx + cosx; y(41 (0)== 2 > O.
y(4)
Sincethe first non-zero derivative at the point x ==0 is a deri vative
of an even order,which takes on a positive value, we have () minimum y (0)== 2 at this point.

3.6.7.
Investigate the
x ==0:

the point

(a)

;-!2

y==cosx-l+

Solution.(a) y' ==
y\"

y'\"

following functions for an extremun1 at


2

(b)

y==cosx-l+ .
\037

x-

- x-I;+ 1- - *
sin x +

==
===

\037!

cosx

sin

\037

x;

y'\"

y' (0)==0;

y\"

(0)==

(0) ==0;
1 O.

And so,the first non-zero derivative at the point x == 0 is a derivative of the third order, e. of an odd order; this means that
there is no extremum at the point x == O.

i.

3.6.8.Investigate
the
4 - x2

following functions for extrema:


{(x)== sin 3x 3 sin x.
Solution.(a) The function {(x)== x4e-x2 is continuously differentiable everywhere. Equating the derivative
(a)

{(x)==x e

(b)

2
2
2
.
2
{, (x) == 4x.e- -2x5e- =x3e- (4-2x)
the criticalpoints:
J

to zero,

find

Xl

==

-V2;

X2 ==

0; x3 == V 2.

Compute the values of the second derivative at the

-8xe- -10xe-

criticalpoints:

+4x6e-= = 2 2
2x e- (6 9x2
V2) < 0; (V2 ) < o.)
(0) ==0;
Consequently,at the points x] == V2 and X3 = + V2- the function
x2

e{\"(x)==12x
2

{\"

2
X

2
X

.2

[\"

(-

'

2
As far
maximum
f ( -+-V 2 ) = 4e- =
==
x
point 2 0 is concerned,nothing definite can be

reaches a

e\037

have to
first

as the critical
said as yet, \\ve

derivatives of f (x) of higher orders (up to the fourth


process is cumbersome, therefore \\\\'e will turn to
sufficient condition of an extremurn:let us find the signs)))

find

order!).But
the

--7- 2x\\037

{\"

this

158

CIl.

ofthe

fi r

III.Differential

st der i vat i ve

Calculus: Investigation

f'

Hence,at

n e i g h b 0u r hood

the

in

X 2 ==0:)

of Funcl's)

0f the cr i tica I poin t

(-1)< 0; f' (1)> O.

the point x 0 the function has a mInImum f (0) ==O.


The function y ==f (x) is rppresentedparametrically:
===

3.6.9.

'l'

\\

Find the

-- --

x == (p (t) == t j
== 4t 3
y == (i)

extremaof

Solution.We have

5r

3t 2

\037

5t

<I

- -20.
15t2

O.
(-2,2) cp'(i)to=Fzero:
equate
(t) == 12t -6t

In the interval
Find
(t) and

Whence i 1 ==

(-2< < 2).)

this function.

(P' (t)

'1\"

20t -i- 7,
18t -1-3

-18

it

\037'

1 and 12 ==

==O.

3/2.

These roots are interior points of the considered interval of vari ation of the parameter i.
Furthermore:)

'l'''(-1)

== 30 < 0,
6;
(3/2)-= 30 > O.
Consequently, the function y=f(x) has a maximum y==14 at
at x==31)and a minimum
y==-17.25at 1==3/2

'l'''(t) ==24i

'1'''

t=-I (i.e.-

(i.e. at x= 1033/32).
3.6.to. Find the nlaxima) and
-

2
( a) f (x) = x e x;

minima of the following functions:


(b )

4x

(x) == x2

_I-

4;
14

(c) f (x) == x2 V (X-2)2; ( d) f (x) ==x4 _ 8x2 +2


(e) f (x)== V2x3 + 3x2 -36x;
2
(g) f (x) ==x In 2 x.)
(f) f (x) ==x I n x;)

3.6.t t. Investigate the

the point x ==0:


(a) f (x)== sin
\037

following functions for an extremum at

x-x;

(b) f (x) == sin

x4

x\037

(c) f(x)==sInx-x+3f-4f;
(d) f (x)=

el/X

if

{ 0,'if

x
x=

=1=

;)

0
0.')))

x-x+ x /3;
3

\037

\037

3.7.Finding

the Grfatcst

and the Least Values 01

159)

Function

3.7.Findingthe Greatestand the Least

Valuesof a Function

The greatest (least) value of a continuous function f (x) on an


interval [a, b] is attained either at the critical points, or at
end-pointsof the interval. To find the greatest(least) value of the
function
we have to compute its values at all the criticalpoints
on the interval [a, b], the values f (a), f (b) of the function at the
end-points of the interval and choosethe greatest(least) one out
of the numbers obtained.
If a function is defined and continuous in some interval, and if
this interval is not a closedone, then it can have neither the
greatest nor the least value.
th\037

3.7.1.

Find the greatestand the least values of the following


functions on the indicated intervals:
2
3
i2x+1 on
(a) f{x)==2x
5/2];
2
==
(b) f (x) x -I n x 0n
e
];
x 0n 0
, -1-00] ;
(c) f (x) == xe
r
on
(d) f(x)==V(I-x2)(I-t-2x2)
Solution.(a) Find the derivative f' (x):
I' (x)==6x2 6x 12.

-3x

[-2,

[1,

[-1.1].

2.

It vanishes at two points: XI == 1 and X2 == They both lie inside the indicated interval
consequently both of them
must be taken into consideration.To find the extrenlevalues of
the function it is necessary to compute its values at the points
x1 and x2, and also at the end-points of the segment:

[-2,

\037

];

f{-2)=-3,f{-I)=8; f(2)=-19,f( )=-16 .


== 8 and the least f (2)==
19.
Hence,the greatest value is f (-1)
The derivative
critical
the
Find
\037

points:

(b)

\037

f'(x)==x(1+2Inx).

[I,

e].Th\037refore
f' (x) does not vanish inside the gi ven interval
there are no critical points inside the indicated interval. It now
remains to compute the values of the function at the end-points of
the interval
e]
2
f ( 1) == 0; f (e) == e
2
Thus, f(I)=Ois the least value of the function and f(e)==e the
test.
grea
Find the greatest and the teast values of the following
funct ions on the indicated intervals:
00,00);)))
(a) y ==sin x sin 2x on

[1,

3.7.2.

(-

160

Ch.

III.Differential

(b) 11 == arc cosx2 on

Calculus: Investigation

[-

\037/

of Funct's)

2/2,J;r-2/2];

(c) y == x + J/-x' 0n [0, 4].


Solution.(a) Represent the function
y==

cosx-cos
3x

== sin x sin 2x In the form

whence it is seen that the function is an even one and has a period 2Jt. Hence,it is sufficient to seek the greatest and the least
values among the extrema on the interval [0,Jt]. Find the derivative y':)

y' =
In

\037

(3 sin

3x-sinx).

[0,Jt] the derivative vanishes at the points


= 0, X2 = arc cos ' X3 = arc cos
= n.
,
(
)
XI

Compute the values of


y (0)= y

Hence,the

;3
the function

(n)= 0, y

at

cos +
[arc (

x\037

Y\0373

these points:

;3) ] = + 3

\037:f'

least value of the funct ion in the interval


is equal to
V 3 ), and the greatest to 4/(3 V3).
The function

3.7.3.

-4/(3

(-00, 00)

x>o)

f(x)==ax+.!!...
(a, b,
x)

summands: one summand is proportional to the


variable
x, the other inversely proportional to it.
independent
Prove that this function takes on the least value at x == Vb/a.
Solution.Find the roots of the derivative f' (x) in the interval

consi\037ts

of two

(0, 00):

f' (x) = a

-:'

\302\260

at x == V b/a (x > 0). Sincef\" (x) ==2b/x3 > for any x > 0,the function f (x) reaches a minimum at this critical point. This is the
in the interval (0, 00). Hence,at
only extremum (minimum)
x == J/ bla the function f (x) attains the least value.
3.7.4.As a result of n measurements of an unknown quantity
x the numbers Xl' X2,
Xu are obtained.
I t is required to find at what value of x the sum of the squares
of t he errors
\302\260

...,

f (x) == (x
will be the

least.)))

-XI)2 + (X-X2)2 + ... + (X-X )2


n

\037

3.7.Finding

Least Values

the Greatest and the

of

Function

IAI)

Solution.Compute the derivative


f' (x) -= 2 (X-Xl)-t 2 (x-x2) + ... + 2 (x-xn).
The onl y root of the deriva t i ve is
x===

+ X 2-t- ... + xn
n)

Then, for all x \\\\'e have f\" (x) = 2n


f (x) has its minimum at the point)
Xl
x::..:.=

> O. Therefore,

the function

+ 2 + .. .+ n .
X

n)

Being the only minimum, it coincideswith the least value of


the function (cf. Problem t
the best (in the sense of \"the principle of the minimum
And
squares\")approximate value of an unknown quantity x is the arithmetic mean of the values Xl' x2
xn'
Find the largest term in the sequence)

.3.8).

so,

' \"',

3.7.5.

an

-=

n'!.
n3

+ 200

.
.)

f (x) = x 3 :\"200 m the interval

Solution. Consider the function


[1,00). Sincethe derivative

3)
x (400-x
I
f (x) == (x3 200)2

-t

< <

>

is positiye at 0 X
V 400 and negative at x V 400, the
at X V 400
function f (x) increasesat 0 x V 400 and decreases
From the inequality 7 V 400 8 it follows that the largest term
in the sequencecan be either a 7 or as' Sincea 7 ==49/543 a 8 ==
the largest term in the given sequenceis

<

< <
<

>

>

8/89,

49

a 7 == 543

.)

3.7.6.

Find the greatest and the least values of the following


functions on the indicated intervals:

(a)

f(X)=+ X4_\037 X3_ X2+2

(b) f (x) == V 4

on

-x on [-2,2];
2

[-2,4];

[ ;3' V3l
on
(d}f(x}=2sinx+sin2x
[0. nJ;)
(c) f(x)=arctanx-flnxon

\037

-3

I 48)))

162

Ch.

III.Differential

[1,e];

on
(e) f{x)==x-2Inx
(f) f (x) =

2X2

\037

+ :2for

\\

Calculus: Investigation

-2

\037

of Fllnct's)

x < 0; 0 < x

\037

2,

or x ==O.)

3.8.SolvingProblemsin Geometryand Physics


3.8.t. The force of a circularelectriccurrent acting on a
axis perpendicular to

the plane of the


its
centre
is
expressedby the formula
passing through
with

rnagnet

the

Cx

F ==
a == radius of the
x == distance from

circle

the centre of the

(0 < x < 00)

C ==constant.
At

x wi

what

11

+x;!) '
31 2

(d\037

\\vhere

the value of

Solution.The derivative

cireIe

to the magnet

F be the greatest?

F'(x ) ==C (a2a2-2x2


+
==
singlepositive root x a/V2. This solves the
X2)

has a

small

circleand

5/2

problem.
Note. It often happens that reasons of purely physical or geo11letriccharacter make it unnecessary to resort to the differential
methods in investigating a function for the greatest or the least

value at the point under consideration.


Determine the most economicaldimensionsof an open-air
3
swimming pool of volume 32 m with a square bottom so that the
facing of its walls and bottom require the least quantity of ma-

3.8.2.

terial.

Solution. Let

height by

us denote the side of the bottom


y. Then the volume V of the pool will be
V

and the surface

==x2y

(*)

S to be faced

x from

through

--x +
2

-.

the relation (*), we get


V

2
4x---;-==x
+
Xl

128
X)

Investigate the function thus obtained for a

interval

x and the

= 32,

S==x2 +4xy.
Expressingy

by

'-- _

(0, 00):
5

2X

'. 2x-128
x2 ==0;

128
x2

minimum

x=4.)))

In

the

.\037

3.8.Solving

Problems in Geometry and Physics)

163)

The single point thus found \\vill obviously yield the least value
since it has no greatestvalue (it increasesunof the function
boundedly as x \037 0 and x \037 00).
2 m.
And so, the required dimensionsof the pool are:x == 4 m, y

S,

-==

3.8.3.
3.8.4.

Inscribe into a given sphere a cylinder with the greatest


lateral surface.
20 m of wire is available for fencing off a flower-bed
which should have the form of a circularsector.What must the
radius of the circle be if we wish to have
Y
a flower-bed of the greatest possible surface

area?
Solution. Let

us

denote the radius of the

circleby x, and the


(see Fig. 42). Then

length of the arc by y

20 ==2x + y,

whence)

== 2

0)

(IO-x).

Fig. 42

The area of the circularsectorS= xy=x(lO-x)


(O:::;;;x:::;;;lO
has a root x:.=::
The derivative Sf (x) ==
Since the least value S ==0 is reached at the end-points of the
interval [0, 10],the obtained value
x==5 yields the greatest surface area

10-2x

--1-

\037

5.

S.

I
I
I
I

\037x)

3.8.5.It is required to construct


open cylindrical reservoir of capacity o' The thickness of the material
an

is d. What dimensions(the base radius


and height) should the reservoir have
so as to ensure the least possible

expenditureof the material?


Solution.Figure43 represents a longitudinal section of the reservoir,
where the radius of the base of the

Fig. 43

and the
V

==Jt

\\vall

inner cylinder is denoted by x and


the height of the inner cylinder,
by h. The volume of the bottom

of the reservoir

(x+ d)2 d + Jt [(x+d)2_X2) h == Jtd

(x+d)2 + Jth (2xd+d2). (*)

On the other hand, by hypothesis we must have


Vo

= Jtx 2h)

\\\\Thence)

\037\037

3tX2

\302\267)))

164

Ch.

III.DiUerenfial

Calculus: Investigation

of Funct's)

Substituting into (*), we get


V

\037

nd (X

2
+ d )2 + nV.2 (2xd+ d2 ) ==nd (X + d )2 + 2V od + Vxo.d .
II

Jrx

2)

Now we have to investigate the obtained function


extremum at x > O.
We have
V'

(x)== 2nd (x + d) _

od
x2

2\037

2V od2

x3

==

2d

(x+d) (nx a
x3

(x)

V0)

The only positive root of the derivative is the point x =


This solves the prob lem:
h ==

V O V\037

-_ V -_
Vo

V\037

for

an

3/-

V Vo/n.

X.)

3.8.6.

A factory D is to be connected by a highway


with a
straight railway on which a town A is situated.The distance DB
from the
to the rai lway is
]) equal to a,factory
the segment AB of the
R
railway equals t. Freight charges on
//
the highway are m times higher than
/
/
on
the rai lway (m > 1).
a
//
I
DP be
How should the highway
//
I
I
//
connectedwith the railway so as to
/
I
I
ensure the least freight chargesfrom
a/
x)
P
factory to town?
Solution.First,let us make a draw44)
\037

I
I
I
I

--0

0--

A)

B)

Fig.

ing

(see Fig. 44).

clear that

the

It

is

absolutely

also be
straight (a straight line is shorter than any curve connectingtwo
given points!).Furthermore,the point P cannot lie either to the
left of the point A or to the right of the point B. If we denote
the distance AP by X, it will mean that 0 x
Let the freight chargeson the railway (per ton-kilometre) be k,
will be km. The total
then the freight charges on the highway
N
from
D to A amounts to)
for
loads
freight charge
transporting
\037

N ==kx+kmVa2

Hence,we

have to

find

must

highway

\037

1.

+ (l-X)2.

the l east val ue of the

funct

O\037x\037l.
f(x)==x+m\037/a2+(x-lr\037,

Take the derivative)

(x -l)
f'(x)= 1 + 11
a2+(x-I)2
m

\302\267)))

ion

\037

3.8.Solving

Problems in Geometry and Physics)

It vanishes only at one point:)

x==lIf

this point lies

in

a)

Vm2

y';_

\302\267

[0,I], I.e.

the interval

\037

165)

if)

or
I

\037
\037

V m2

- 1,

then it yields the least freight charge (which is easy to check).


If the indicated inequality is not observed, then t (x) increaseson
and therefore the least freight chargeis obtained at x ==
In constructing an a-c transformer it is important to insert
into the coil a cross-shaped iron coreof greatest possible surface
ion
area. Fig. 45 shows the cross-sect
r
of the core with appropriate dimensions. Find the most suitable x and
y if the radius of the coil is equal
to a.)

0.

[0,l]

3.8.7.

3.8.8.If the

source of current is an

electriccel1,then
obtained

the effect

P (watts)

cutting a resistanceR
the circuit is expressed
by

by

(ohms) in
the formula)

ER
P == (R+
Ri)2

I
,)

where E is electromotiveforce

tance in ohms.

Fig. 45
in

volts and R i the internal reSlS-)

Find the greatest effect which can be obtained at given E and R i -

3.8.9.A
What

use

of a given volume V has the form of a


the ratio of its height h to diameter 2R
the least amount of material for its manufacture?)
tin

must

\037e

cylinder.
so as to

3.8.

to. In a given cone inscribe a cylinder having the greatest


1ateral surface so that the planes and centresof the base circles
of the cylinder and cone coincide.

3.8.11.

Given a point (1,2) in the orthographic coordinates.


Through this point draw a straight line so that it forms, together
a triangle of the least area.
with the positive semi-axes,
Given a point M on the axis of the parabola y2 2px
at a distance a from its vertex. Find the abscissa of the point
on the curve nearest to the given point.)))

3.8.12.

=-==

166

Ch.

III.Differential

Calculus: Investigation

of Funct's)

3.8.13.

The expensessustained in one hour's sailingof a ship


are expressedin roubles by an empirical formula of the form
a + bv 3, where a and b are constants for a given ship, and v is the
ship'sspeed in knots (one knot is equal to 1.85km/hr). In this
the constant part of the expensesa refers to depreciation
fonTIula
and crew's upkeep, and the second term (bv 3) to the fuel cost.
At what
speed will the ship cover any required distance at the
lowest cost?
A trough is bui It from three boards of equal width. At
\\vhat
slope should the lateral boards be placed to ensure the largest
area of the trough?
cross-sectional
A tank with a vertical wall of height h is installed on
a horizontal plane. Determine the position of an orifice, at which
the range of a liquid jet will be the greatest if the velocity of
flow (accordingto Torricelli'slaw) is equal to }/2gx, where x is
the depth of the orifice.
Two aircraft are flying in a straight line and in the
0
to each other and with an equal
same plane at an angle of 120
speed of v km/hr. At a certain moment one aircraft reaches the
point of intersectionof their routes. while 1 he second is at a distance of a km from it. When will the distance between the
aircraft be the least and what is that distance?)

3.8.14.

3.8.15.

3.8.16.

* 3.9.Convexityand Concavity of a Curve. Pointsof

Inflection
If
(x) < 0 (> 0) on
on this
is

an interval (a, b), then the curve y ==f (x)


interval
convex (concave), it is situated below (above)
any of its tangent lines.
I f I\" (x\\)) ==0 or does not existbut
(x 0) does existand the second
derivative f\" (x) changes sign when passing through the point xo,
then the point (xo, f (xo is the point of inflection of the curve
I\"

i.e.

l'

\302\273

-= f

(x).

3.9.I.

Find the intervals in which the graphs of the following


functions are concave or convex and locatethe points of inflection:
3
4
+
(a) y == x -1-x -.18x2
4
(b) y 3x 8x 3 + 6x2 + 12;
=-:\037

(c)
(d )

==

24x-12;

+x2 ;
==x+ / ;
y

XO

(e) y==\0374V(x-l)5+20V(x-l)3
(x\037 1);
(f)

In:! x

=-==

-X (x >

0);)))

j.9.Convexity

\037

(g)
(h)

and Concavity.

Points of Inflection

167)

(x > 0);

==X sin (In x)

y==2-lx5-11.

Solution.(a) Find the derivatives:


y' == 4x3 + 3X2 -36x+ 24,
y\"

= 12x2+ 6x

36 =

12( x2 +
== -2,X 2 == 3/2.
Xl

3 ,
)

\037

whence y\" == 0 at
and (3/2, 00); y\" < 0
Hence,y\" > 0 on the intervals
on the interval
The
of
the
second
derivative detersign
3/2).
mines the convexity or concavity of the curve in a given interval.
This enables us to compilethe following table:)

(-00,-2)

(-2,
x

Sign of

Conclusion

y\"

<-2 --2< x < -23 x> 32

Concavity

+
Concavit y)

Convexity

Sincethe second derivative changes its sign when passing through


and
the points x 1==-2 and x 2 ==3/2, the points
are points of inflection.
( ,
)
(d) Find the derivatives:

(-2, -124)

; -8

I\037

y' = 1 + X'I.

,)))

\037

\"

10

Vx'

The second derivative is non-zero everywhere and loses its meaning


at the point x == O. At x < 0 we have y\"
0 and the curve is convex, at x > 0 we have y\" > 0 and the curve is concave.
At the point x== 0 the first derivative y' =
the second derivative changessign when passing through the point x== O. Therefore
the point (0, 0) is a point of inflection.

<

1,

(g) Find the

derivatives:
+ cns (In x),

y' == sin (In x)

- ,-,-,-,....

(lnx)]=
y\"=+[cos(lnx)-Sin

k-

\0372

sin (

\037

-lnx).

The second derivative vanishes at the points


k
0 + 1 +2
X
err / 4 + kJt
The function sin (Jt/4

when

passing through

In

x), and together

each point

Xk

sign
. Consequently thechanges
points x
\\vith

it y\",
I

k)

168

Ch.

Ill.

Calculus: Investigation

Differential

are the abscissasof

the points of inflection. In the intervals


(e2kn

the curve IS concave,and

3n/ 4

+ 3t/ 4

is convex.
(h) The given function can
y ==

Therefore)
Y

At

e2kn + 5JT/

be written

, == 1

5x4,
5x4,

\\

the point x == 1 there

y\"

the point

X
X

the fol1owingway:

> 1,
< 1.

is no derivative. Further,)

== J

-20x3,
3

tervals: (-00,0), (0, 1), (1,00).


Compile a table of signs of y\":)

x
x

> 1,
< 1;)

have to invest igate three in-

x<o O<x<l x>]

Sign of

in

x\037

,)

== 0 at

4))

1), x < 1.)

20x
\\
x==O. Hence,we
Y

))

2-(xl>-1), 1,

{ 2 + (xl>

\"

e2kn + JT/ 4

the intervals)

in

(e2kn
it

ot Funct's)

y\"

ConcI usion

Concavity Convex it y)

Convexity

The point (0, 1) is a point of inflection, the point (1,2) being


a corner point.
What conditions must the coefficients a, b, c satisfy for
the curve y == ax4 + bx3 + cx2 + dx + e to have points of inflection?
Solution.Find the second derivative:
2
y\" == 12ax+ 6bx+ 2c.

3.9.2.

The curve has points of inflection if and only


6ax2 + 3bx+c == 0
has different real

roots,i.e.when
3b2

the discriminant 9b2


8ac >
O.)))

if

the equation

24ac> 0, or

\037

.9.3.At

\\\\'

3.9.COn'l.'exity

and Concavity.

val u es of a

hat

Points of

t he

will

169)

1f117eclion)

cur ve

y=x4+ax\037+ x2 + 1
\037

be concave along the entire nun1ber scale?

Solut ion. Find

y\":)

+ 3.

\"--') :>,.-r 6ax


----1....,X
I

scale if

The curve will be concave along the entire number


when
[or all val ues of x,
2
4x -t- 2ax -1-1 0 for a II x.

i.e.

\037

For this it is necessaryand sufficient that the inequality 4a2


be fulfilled; whence)

y\"

\037

- 16 0
\037

lal\0372.

3.9.4.
Show that

the curve

x-+ I

=-=

x\037

lying in a straight line.


Solution.Find the derivatives:)

tion

--x:!

,-

has three points of inflec-

-1-1

2x

--

,
6x 2
Y
i
(x:>' -t- I r
The second derivative becorneszero at three
roots of the equation)
3x 1 == 0,)
x:!-tY

--=

(x\037

\"

=--

whence)

3x\037

\037

f)x\037

r\037

- ::-\037

JI

<x< -2-- V :3 <r-<


--y:f < -2 V 3
\037-

---00
< 2

_\037

\\'

--L

==

--

\"

1.

Hence,(--2

Concavity

<x<00
+

Concavity)

COllvexity

- /r-,-V3.-]
\037

_L

COil vc xi t y

yT <

<x<

Conclusion

X: 1

y\":)

Sigll of y

points, which are the

-2 I3,

2 JI 3, X2
+
x.
compilethe table of signs of
\037-=

Let us

--

--

2x:i -j

,(

2+ V

-3, + Y3) ' (1,1)


i
.J)

are points of inflection. It is easy to ascertainthat aU of

thenl)))

170

lie

CIl.
in

III.Di(Jerential

a straight

Calculus: Investigation

of Funci's)

line. Indeed,the coordinatesof these points satisfy

.
the relatIOn -2-Y3-J
3 4+1
3-1= ((I-Y3)/4--1
-2+-V1+-V-)/
3.9.5.Investigate the curves represented by
\302\267

the following equations for convexity (concavity) and locate the points of inflection:

(a) y\037x-t/(X-3)2;
(b) y

===

esin

(-n/2 x
\037

3.9.6.
Show that

lie on the curve


\037

\037

n/2).

the points of inflection of the curve


(4 + x2) 4x2
===

y2

\037

x sin x

.)

3.10.Asymptotes

A straight I ine is called an asymptote to the curve y


f (x) if
the distance from the variable point M of the curve to thp straight
line approacheszero as the point M recedesto infinity along some
branch of the curve.
We will distinguish three kinds of asymptotes: vertical, horizontal and inclined.
Vertical asymptotes. If at least one of the limits of the function
f (x) (at the point a on the right or on the left) is equal to infinity, then the straight line x==a is a vertical asymptote.
Horizontal asymptotes. If tim f (x) == A, then the straight line
X-:f::oo
y == A is a horizontal asymptote (the right one as x \037 + 00 and
the left one as x --+ 00).
Inclined asymptotes. If the limits
===

tim

(x) = k
I,

tim [f (x)
x-++oo)

x-+oo X
exist,

then

the

straight line

== ktx

asymptote.

(x) == k

and

ktx] ==bi

+ bi

is an incl ined (right)

If the lim i ts
litn
x--oo

Jim [f (x)
x....-oo

k2X] == b2

exist, then the straight line y = k 2x-t-.b2 is an inclined (left) asymphorizontal asymptote may be consideredas a particular
caseof an inclined asymptote at k == O.

tote.A

3.to.1.Find the

5x
(a) y == x 3 ;

asymptotes of the following curves:

(b) y == x

- -+

3x

3x; (c)

== .)x
x'\"

3x

;
1

(d) y=-x+4X2; (e) y\037xeX; (f) y==21n\037e3x)

;)))

3.10.
Asympto!es

\037

(g) y == VI + x2 -1-2x; (h)


( i)

== 2 V- X2

==

171)

}/l+ x2 sin x ;
\037

+ 4.

Solution.(a) The curve has

I trn

x\0373+0

==

a vertical

asymptote x== 3, Slllce

5x ==

I 1m

3
x_31=Ox-)

+ 00

(the point x==:3is a point of discontinuity of the


Find the horizontal asymptote:

I 1m

x
And

tal one

so,==the

5.

:i:.

second kind).

. 5x ==5.
= 1m
x-3
x-

00

:f::

00)

curve has a vertical asymptote x ==3 and a horizon-

(b) The curve has a vertical asymptote x == I, SInce)

Hn]
x\037l-O

y==

+ 3x) =-00;
( 3\\ +3x)\\=+00.

Hm
(x
x_l-O

!im

!im

x-J+Oy== x\037I+O

3x

\\.A
/
\037

I
I

:
I

x)

1/

inclined asymptotes:)

Find the

'

'

If)

lL=

k== !im

Iim

x-:f:oox
x.-+ :f:

X-:1: (
oo

(y-

b == lim

3x

+ 3x
I

(
x-:f:oo

X)

+ 3 ) =3;

II
I
I

kx) ==

'JJ)

liIll

x)

\037

I
I)

3x ==

x)

3.

Thus, the straight line y == 3x+3 IS an


g . 46
incl ined asymptote (see Fig. 46).
(e) The curve has a vert ical asymptote x = 0, since)
f-

!in]

x-+O

X\037+O)

e
-=+00
t

xe1 /X ==

== liln

Hm

=-1 _ +

t)

00

x)

(see Problem 3.2.2.).

Find the inclined asymptotes:


k ==

(xe1/
x-:i:oo

b ==

lin1

x)

.JL

Ii In

X-:1: x
oo

==

Ii m

e1/x = eO

\037

X\037:f::oo

lim
X\037:l:oo

e1/X _ I
I Ix

l''

e-!im
l/x=z-O
Z

z)))

==

1.

III.Differential

Ch.

72

Calculus: Investigation

Thus, the straight line y ==x


of the curve (see Fig. 47). Note

IS)

will be an inci ined

== linl

<0

!It)

asymptote

that)

xe1 / x = o.
x-+-o)
defined and continuous at
liin

.\\-)0-0

(f) The function

of Funcl's)

e-

> 3e] .

and x

> 0, I.e.at

3x

Since the

function is continuous at
of
the domain of definition,
every point
vertical asymptotes can exist only on
finite boundaries of the domain of definit

ion.

x--+-Owe have

As

- 0 y = Iim- 0 3;In ( e- ) ==

Iim

X-)o

== _\037

In

liln

(e+z)== 0
z

z + 00
-)0

= _J..
3x))

i. e. the straight
(see Problem 3.2.2.),

47

Fig

3\037

X -)0

line x ==0 is not a vertical asymptote.)

As x --+ + 0 we have
3e)
\037

.
3
1m
y=2
.t-+l/(3e)+0
}

i. e. the

line

Now let us

k ==

lirn
.t -+ :i:. 00

b=

lim
X\037i:oo

.JL
X

an

x 1n

I
x\037I/(3e)+0

e--) =-00,
]

3x)

Ij(3e) is

x-==-

find

a vertical asymptote.
the inclined asymptotes:)

3
Iim
2 x :f:
-)0

[y-kx]=

e--)
(
1

In

3x

00

lim x -In
\037

X-+:f:oo

3
2 ;

==

e- ) -1

_ 2 rtrn

_\037

Ine

3x
=2

==

In( l-\037)__\037

- I.

X-+:f::oo

Hence, the straight


(seeFig.48).

3\037

_x
I

- .

_J..
__\037
3e )
2e

. .

2e IS an Inc1 Ined asympt 0t e)

(g) The curve has no vertical asymptotes, since the function is


continuouseverywhere. Let us look for inclined asymptotes. The

limits will be different as x --+ + 00 and x --+ 00, therefore we have

to considertwo

casesseparately.)))

8.10.
Asymptotes)
FInd the right asymptote (as x --++ 00):
. V + xt + 2x . -.V \037+ +2

173)

\037

k t ==

x\037

+ 00

:= I 1m

I trn

x\037

= 3;)

x\"

00

b]= liln+ (VI+x2 +2x-3x)=


x\037

+x2 x2 =0.
2
=!im
( Il+x -x =!im
'2+
+ y' -I
x-

00)

\"1)

\037

00

\037

x)

= 3x.)

Thus, as x \037 + 00 the curve has an asymptote y

x)

:c)

11-1

-1)

1C1C)

48)

Fig.

Find the

left

k2 =

b2 =

asymptote (as x

.
11m

VI+

x2

-.-(0):

.
+ 2x = 11m

00

x\037

00

x\037

00)

x-+

x'
49)

r\"1
V x2 + +2x

Iim IVl+x +2x-xl=lim


-

K-

Fig.

00

+I.)
X'\"

\037-=

==0,

since both summands (VI +x2 and (-x))in the denominator are
positive at x < 0.
00.
And so,the curve has an asymptote y = x as x \037
(h) The curve has no vertical asymptotes, since it is continuous
at x =#= 0, and in the neighbourhoodof the point x == the funet ion
is bounded.
Let us find the inc1inedasympt otes.We have

\302\260

Iim x

X-+ :f:

00

hrn
x-+ :f:

- 1.0=0.

Ixl Vl+\037sin\037
x
=+
x
X

00)

Then

b-;-:lim

X)))

t\037:f:oo

lasx-++oo,

sin-=\\I _1 as X-+-oo.
(y-kx)= litn Ixl V I+ 2
X
X\037:1:OO

Ch.

174

III.Differential

Calculus: Investigation

of Funci's)

Thus, the curve has two horizontal asymptotes:y :=+ 1and y == 1


(see Fig. 49). The same result can be obtained proceedingfrom
symmetry about the origin and keeping in mind that the function
y

is

odd.

3.10.2.
Find the

inclined asymptote of the graph of the function


00
as
X-+
and show that in the interval (100,00)
this
y= I\037X
function may be replaced by the linear function y ==
with an
'J

1.
.
k ==

x-I

error not exceeding0.0


Solution.Find the inclined asymptote:

! \037-x
\037n\037

b == Hrn
X-+OO

so,

the equation of the


Form the difference:

And

8 == I

x2

+x

Hence,assuming)
Y=
for all

x2

+x) == 1;
=
l+x) )

(I

asymptote is

-(x

1)

-1.
y

-==

x-I.

===

+x

\302\267)

x2

I+x\037x-I,

x > 100,we introduce an

error of not rnore than


Find the asymptotes of the following curves:

3.10.3.
x2-6x+3
(a) y= x-3 ;

(b)

0.01.

y==xarctanx;

2
(c) y==x+(sinx)jx; (d) y=ln(4-x
);
I
(e) y==2x-arccos-.
x)

Functionsand
9 3.J J. GeneralPlanfor Investigating
SketchingGraphs
The analysis and graphing of functions by elementary methods
were consideredin Chapter I
and
Using the methods
of di.fferential calculus,we can now carry out a more profound and
comprehensivestudy of various propertiesof a function, and explain
the shape of its graph (rise, fall, convexity, concavity, etc.).
I t is convenient to investigate a function and construct its graph
accordingto the following plan:
Find the domain of definition of the function.
2. Find out whether the function is even, odd or periodic.)))
(\037\037

1.

1.3

1.5).

\037

8.11.
Investigating

Functions

and Sketching Graphs

175)

3.

Test the function for continuity, find out the discontinuities


and their character.
4. Find the asymptotes of the graph of the function.
Find the points of extremum of the function and compute the
values of the function at these points.
6. Find the points of inflection on the graph of the function,
compute the values of the function and of its derivative at these
points. Find the intervals of convexity of the graph of the function.
7. Graph the function using the results of this investigation.If
it is necessaryto specify certain regionsof the curve, calculate
the
coordinates of several additional points (in particular,the x- and

5.

y-intercepts).
This is a very tentative plan, and various alternatives are possible.For instance,we recommend the student to begin sketching
the graph as soon as he finds the asymptotes (if any), but in any
casebefore the points of inflection are found. It should be remembered that in sketching the graph of a function the principal reference points are the points of the curve corresponding
to the extremal
values of the function, points of inflection, asymptotes.

3.11.1.
Investigate and graph
6

(a) y==x
(C)

==

-3x+3x -5;

2x3
x2

_4 ;

(e) y = x+ In
(g)

the following funct ions:

-Vx+l;

(b) y=V x

l-x ;
(d ) y =
3

x2

(x

-1);

(f)

= {-sin 2x+ casx;


. x2
y = arc SIn +x2
y

==x 2e1 /x;

(h)

\302\267

Solution.(a) The function is defined and continuous throughout


the number scale,therefore the curve has no vertical asymptote.
The function is even, since f(-x)==f(x).Consequently, its graph
is symmetrical about the y-axis, and therefore it is sufficient to
investigate the function only on the interval [0,00).
There are no inclined asymptotes, since as x -+ 00 the quantity
large quantity of the sixth order
y turns out to be an infinitely
with respect to x.
Investigate the first derivative:
3
+ 6x ==6x (x4 2 + 1)==6x(x2
y' ==6x5-12x

-2x

the

criticalpoints are:
Xl

==

Since in the interval


increases.)))

- 1,

[0,00)

X 2 ==

0,

Xa

-1)2;

= 1.

the derivative y'

\037

0, the

function

176

Ch.

III.Different

Calculus: Inuest igat ion of Funct's)

ial

Investigate the second derivative:


y\"

== 30x4

36x

:a

+6

===

6 (5x4

6x2 -1-1).

The positive roots of the second derivative:


x J == 1I}/-5, x2 ==

1.

For convenienceand pictorialnesslet us compile the following


all the points of interest are arranged in an ascending
order:)

table,where
0

(0, ;5-)

Y5

\037

\037

. I)

(I.(0)

I
I

II

III

93

LJ

\037

25 Y-5-

1.7

-t-

,
I

I
I
I

I'

()

-t-

--

0
I

\037-\037

I
I

II

11

-4 \037

23

-,.-

-\037

:.)

On the right one more additi.onal value of the function is computed to improve the graph after the point of inflection.
Using the results of the investigation and the above table and
taking into consideratio.nthe symmetry principle,\\ve construct the
graph of the function (see Fig. 50). As is seen from the graph, the
funct ion has roots x + a, where a
1.6.
The
function
is
and
continuous
over the entirenumber
defined
(b)
scaleand is negative everywhere, si.nce V x < V x+ 1.
I'hegraph has neither vertical, nor inclined asymptotes, since the
order of magnitude of y is less than unity as x -?-00.Determine
the horizontal asymptote:
===

liln
X-IOO

y== Jim
X\037:1:00)

\037

(Vx -Vx+l)===
= hm.
x

Hpnce, the straight line y


gra

ph.)))

===

-':1: V- +
00

x2

3
V

-1

/ x (x + 1)+ 3/(x -t- I 1= O.


v

0 is the horizontal asymptote of the

3.11.
Investigating

\037

The

Functions

and Sketching Graphs)

177)

deriva t i ve

first

V (x+1)2--VXi

=3V

x2

- -1,

3VX2(x+I)2

3V(X+l)2

becomeszero at the point x2 =


!f)

and

\037

=
Xl

at

infinity

the points)

x == O.)
b

y)

-1-..L
0

x)

x)

-1)

2)

1)

o)

-1/2)

Y5)

1)

-5)
Fig. 50)

Fig.

The second derivative


y

\"

2.

=3\"( -3\") V x

&

-3\"(

does not vanish and is


Compile a table:)
x

y'

-I

(-I.-)
-00
00

-1

\\

-2,0)
I

16

+9V2
3

'-.:.'(f/

X 3 ==O.

(0, (0)

00
I

1
I

V'X5]

- 1,
1)\302\260

same points Xl =

0
I

-r-

<X+l)&

+ =-2[V'gVlx(x+l)j&
(x

at the

infinite

\037

y\"

-3) V

51)

00

-1

-0.26)

With the aid of this table,and of the asymptotey = 0 construct


the graph of the function (see Fig. 51).)))

Ch.

178

III.DitJerential

Calculus:Investigation

of Funci's)

is defined and continuous over the entire axis


the points x == + 2. The function is odd, its graph is
symmetrical abou t the origin, therefore it is sufficient to investigate
the function on the interval [0, 00).
The straight line x == 2 is a vert ical asymptote:

(c) The function

exceptat

1m \0374=-t-00.
=-00; x-2+0
.)
x....
2 - 0 x\"\"- 4
2X3

I 1m

Detern1inethe inclined asymptote:


k == Urn .!L= Iim
x....+

'The curve has


The

first

an

= 2,)
x....
+ x2-4

00

2X3

2X2

00)

8x
b= x....
Iim (y-2x)=!im x 2 -4) =0.
+00
x-++oo)
inclined asymptote y = 2x, and

8x
>o at x>2,
y-2x=x2-4
{ < 0 at x < 2
derivative
, 6x2 (x2 -4)-4x4 =
(x2 -12)
Y =
2X2

(x2-4):'.

(x2-4)2

In
[0, 00) vanishes at the points
x == 0, x == 2 V3 3.46
and becomesinfinite at the point x == 2.
The second derivative)

the interval

\037

\"

(x2 + 12)
= 16x
2
(x -4)3

: becomeszero at the point x == 0 and


Compile a table:)
x

(0, 2)

y'

-0

00

+0

y\"

\037

(2,

infinite

at x ==

2J!3) 2J!3

00

-;;;:
\\!)))

2.
(2 J!3\",00)

V3

I
I

\037

8.11.
Investigating

\037

and Sketching

Functions

179)

Graphs

Using the results of the investigation, sketch the graph of the

(see Fig. 52).

function

(e) The function is defined and continuous at all values of x for


which x2 I > 0 or x >
on two intervals:
00,
1)
and

(1,+ 00).)

!J

(-

1, i.e.

I
I
I

I
I

//

1/
I

/)

!f)

I
I

/1/

I
I

x)

2)

I
...71)

-1\\
I
I
I
I
I
I

Fig.

Fig. 52)
We

53)

seek the vertical asymptotes:


00;
- - 0 y== x-lim
- - 0 [x+ln(x2-1)]==-

lim
x\037

lim y==

2
-1)]==-00.
[x+ln(x

Iim

x\037I+O

x\0371+0)

Thus, the curve has two vertical asymptotes:


x == 1 and x == +
Find inclined

asymptotes:

k== lim JL== Iim


X
x

\037

b ==

:f:

X-:f:

00

Ii In

1.

[y

x+ln (x2 -1)=


X

00

x] == Iim In (X2

+ In
1) == + 00.
litn
x _ :f:

00

(X

2
x

-1) = 1,
]

x-:f:OC'
x-:f:oo)
Hence,the curve has neither inclined,nor horizontal asymptotes.
Sincethe derivative)
Y

existsand

, == 1

+ x22x

is finite at all points of the domain of definition of the


function, only the zeros of the derivative
==
1 V 2; X2 ==
Xl

can be

--

criticalpoints. At

-1+ J/2

the point x2 ==

- + V2
1

the function

is)))

180

III.DitJerenlial

Ch.

hence,there

not defined;

Calculus:Investigation

criticalpoint

is one

of Funci's)

(-00,-1).In the interval


> 0 and the function increase.

ging to the interval

derivative y'
The second derivative

XI

-(1,00)
1- 2

belonboth the

J;r

\"=_ '2 (x2 + I) < 0 ,


2

- 1-- --

(x

I)

\037

hpnce, the curve is convex everywhere, and at the point


=
the function has a maximum
V2\037-2.41

XI

O.R4.
V-2+ In (2 + 2 V 2)
where there are no characteTo plot the graph in the interval (1,00),
ristic points, we choosethe following additional points:
y

V 2)

\037

\037

x=2;y=2+ln3\0373.10and
x=I.2;y=I.2+lnO.44\0370.38.

.The graph of the function is shown in Fig. 53.


(f) The function is defined and continuous throughout the number scale and has a period 2Jt. Therefore in investigating we may
.confine ourselves to the interval [0,2Jt]. The graph of the function
has no asymptote by virtue of continuity and periodicity.
Find the

first

derivative:
x.
y' = cos2x-sin

On the interval

[0,2n]
xt

it

has three

roots:

=6' x =6' xs=T')


n

5n

3n

Evaluate the second derivative:


= 2 sin 2x-cosx.
y\"

[0,2n] it has four roots:


x2 = n + arc sin (1/4),xa =
3;, X. = 2n-arcsill (1/4).

On the interval
XI

table of the results of investigation of all criand second derivatives (the table also incl udes the end-points of the interval [0,2n]).
Since in the interval
(0, 3;) the roots of the first and second
derivatives alternate,the signs of the second derivative in the intervals between its critical points are indicated only for the last
three intervals.
The results of the investigation enableus to construct the graph
Let us draw

tical points

up a
of the

first

Fig.
positive and continuous on each of
00,0) and (0, 00). The point x = 0 is a disconti-)))

of the function (see


54).
(g) The function is defined,

the intervals

(-

\037

3.J J. I nvesligating

- -

5Jt

31

- e;

31

\037

]81)

Funct{ons and S/?etching Graphs

x2

3n
2

. 3;)

\302\267

4 (x4,2n) 231

i
'I

L/

'

-2

IJ.
8

\037

II

i
I

1'1

- Y3
:3

\\

\"

Jl3
2

:L

II

I
I

r \\ \037
JV3

-'\"\"

\\!\037

16

\302\245!/

cO

16

/\302\245fI

I)

nuity. Since(see Problem 3.2.2.)

x2e 1lx ==

Iim y== Iim

=00

t=

e;
(
1_+00')
the straight line x ==0 is a vertical asymptote. But
x\037+o

Iirn

x\037+O

HIll y

X'\037-o

==

\037

),

x2el I x == O.

lirn
x\037-o)

There are no inclined asymptotes, ince the function y == x2e1 Ix


has the second order of smallness with respect to x as x ---++ 00.)
\037

.in
6

2n)

:..:c

-f)
Fig. 54

Let us find the extremaof the function, for which purpose we


,evaluate the derivative:
== 2e1/ x
y' == 2xellx-el/x
1/2),

(x-

whence we

Sincefor

find

the only

criticalpoint x=

IIX _
Y\"(X)=2e

;e

;.

+ eIIX= eIIX(2x2_2x+
1)>

11x

\0372

\0372

0,)))

CIl.

182

III.Differential

Calculus:Investigation

of Funci's)

on each of the intervals of the domain of definition the graph of


the function is concave,and at the point X==1/
2 the function has
a minimum

1.87.

2 \037

Y(+) =+e

From the information obtainedwe can sketchthe graph as in Fig.55.


To specify the graph in the intervals
00, 0) and (1/2'00) the
additional points are used:
following
!f

(-

y==e-l\037O.37; x=I,
y==e\0372.72.

x==-I,

(h) The function is defined and continuous


throughout the number scale, since at any x

-1 0 1/21
Fig.

:c

\037

\037

\037:

1.

Since the function is even, we may confine


ourselves to the investigation of the function

55)

at x O.
the function is continuous,the graph has no vertical asymptotes,but it has a horizontal asymptote:
\037

As

linl

y == arc sin

x\037+oo

The
,
Y

derivat ive

first

=\"li1-

at

the

function

decreases.

-1,

x)

-2
'fC,

-92(I+x2)2x
-__\037
>
(l +
(l +

(x) u_

4x

(1+X2)2

x == 0 the

point

1.

\"

If)

derivative is equal to
and the left one to +
The second derivative is posi-

right

ti ve:)

)2

> 0, therefore the

\037

that

\037

=-2/x/

(l+x2)2

The derivative is non-existent


at the point x == O. By virtue of
the symmetry of the graph about
the y-axis there will be a maxiNon1um at the point y (0)=

tice

-.

2
-2x(l+x2)-2x(l-x
)

(1-x2r\037
2
X

V
(1
is negative for x

(-1)=

60J

)4

)3

Fig.
0 f or a 11 x

56)

> .
\302\260

the graph of the function is concave.


the interval (0,00)
Also note that the curve intersectswith the x-axisat the points

Hence,in
x == +

1.)))

\037

3.12.
Algebr.

and Transcendent.

Equations

183)

Taking into consideration the results of the investigation, const r u ct the gr a ph ofthe fun ct ion (see Fig. 56).
3.1t .2.Investigate and graph the following functions:
4

x
x
(b) Y= (l+X)3;
y=l+x2 -\"2;
x3
2
; (d) Y= x- ;
(c) y=-+4x
x
(e) == v x2 V x2 4;
== 2
== 3

(a)

(f)

(h)

-- +

In

(x

2); (g)

= f xarctan
\\

at
\037

\037

x e-4X;

x 0,
=1=

at x == O.)

* 3.12.
ApproximateSolutionof Algebraic
and TranscendentalEquations
Approximate determination of isolated real roots of the equation

carried out

the intervals [a, which


1.Separating roots, i.e.determiningstages:
contain one and only one root of the equation.
2. Specifying the roots, i.e.computing them with the required

f (x) ==0 is usually

in

two

\037]

degreeof accuracy.

...,

The process of separation of roots begins with determining the


signs of the function f (x) at a number of points x ==aI' a 2
of the function f (x).
whose choicetakes into account the peculiarities

If it turns out that f (ak) f (ak + l) < 0, then, by virtue of the


property of a continuous function, there is a root of the equation
f (x) ==0 in the interval (ak , ak +l ).
Real roots of an equation can also be determined graphically as
x-interceptsof the graph of the function y ==f (x). If the equation
has no roots closeto each other, then its roots are easi1 y separated
by this method. In practice,it is often advantageous to replace
a given equation by an equivalent one
'l-\037l

(x) =='P2(x),

where the functions 'PI (x) and 'P2(x) are simpler than the function
f (x). Sketch the graph of the functions y == 'PI (x) and y == 'l'2 (x)
and find the desired roots as the abscissasof the points of inter-

section of these graphs.


The Methods of Approximating a Root. 1.Method of Chords. If
the interval [a, b] contains the only real root S of the equation
f (x) ==0 and f (x) is continuous on the interval, then the first approximation Xl is found by the formula
Xl

==a

t (a)
(b
t (b)__ f (a)

a).)))

184

Iff.Differential

Ch.

Calculus:Investigation

of Funct's)

To obtain the second approximation x 2 a similar formula is applied to that of the intervals [a, x 1 ] or [Xl' b],at the end-points
of which the function f (x) attains values having oppositesigns.The
process is continued until the required accuracy is obtained, \\vhirh
is judged of by the length of the last obtained segment.
2. Method of Tangents (Newton's method). If f (a) f (b) < 0, and
f' (x) and f\" (x) are non-zero and retain definite signs for a x b,
then, proceedingfrom the injtial approximationxo(xoE[a, b]) for
which f (xo) f\" (xo) > 0, we obtain all successive approximationsof
the root by the formulas:
\037

\037

(xo)

... -

(xd

\037

n d
X ==
, Xn ==X n -1 f'
f' (X o) , 2 Xl I' (Xl)'
(X n 1)
To estimatethe absolute error in the nth approximationwe can
apply the general formula
Xl

==X o

t
\037

Xn

\\
I

\037
-......;;::::

f (Xn)

\302\267

ml

where)
m l == min

(x

'\" (x)

')

,.

a<x<b)
Under the above conditions the method of chords and the rnethod
of tangents approximate the sought-for root from different sides.
Therefore, it is usual pract iceto take advantage of their combination,
i.e. to apply both methodssimultaneously.In this caseone can obtain
the most preciseapproximation of a root more rapidly and the calculationscan be checked.Generally speaking, the calculationof the
approximationsXl' X 2, ... , Xn should be continueduntil the decimal
digits to be retained in the answer ceaseto change (in accordance
with the predetermineddegreeof accuracy!).
For intermediate transformations we have to take one or two spare digits.
3. Iteration Method. The equation f (x) == is first reducedto the
b. Starform X == (x) where cpl (x)
X
q < 1 (q ==const) for a
from
initial
value
ting
any
Xo E [a, b], successiveapproximationsof
the root are computedby the formulas Xl == (xo), X2 ==Cp (Xl)\" . .,Xn ==
=-=
(xn - l ). The absolute error in the nth approximation can be
estimated by the following formulas:
\302\260

cp

\037

\037

\037

cp

\302\243

<p

Is-xnl\037qq lxn-t-xnl.)
1

if

the approximationsxn -l and

Xn

lie on the sameside of

the root,

and)

Is-Xnl:;: lxn-t-xnl.
I\037q

if

the approximationsxn -l and

Xn

lie on di'fferent sides of the

root.)))

3.12.
Algebr.

\037

3.t 2.t. Locate the

and Transcendent.

185)

Equations)

-x:'-6x+

roots of the equat ion


2 O.
f (x)
\037

Solution.Compile a table

of

-..
- x

signs of f (x) at some chosen points)


x

(x)

-3
-1
00

--

(x)

+
+

+00

+)

From this table \\ve draw the conclusionthat the equation has
three real roots lying in the intervals
(0, 1) and (1 3).
Determine the number of real roots of the equation

(-3,--1),

3.12.2.

+ eX ==O.
Solution.Sincef' (x) == 1 +ex > 0;f (-00) ==
f (x) == x

root.

the given equation has only one real

00; f (+ 00)==+00,

An approximate value of the root of the equation f (x) _


3.12.3.
-x
1 ==0 is x == 1.22.Estimate the absolute error in this
root .

== x4

-1

1he root
== 4x3

\302\243

- -

-1.22 1 = 0.0047.Since at
2.2153
-1.23 ==0.0588,
f (x) ==2.2888
lies in the interval (1.22,
The derivative f' (x) ==
1.23).

Solution.We have

x == 1.23)

f (x) ==

increases monotonically, therefore its least value

given interval is
m1

-1

3
=4 X 1.22
=4 X 1.816-1
==6.264,

wherefrom we get an estimate of the error

x-sI:::;;;f
I

\037\037)

\037
\302\2606..0206\037

3.12.4.
Solve graphically the
x log

Solution.Let

us rewrite the

0.00075
< 0.001.

equation

x-I

== O.

equation in the

log x =-=

-.
I

x)))

form

in

the

186

Ch.

III.Differential

Here'PI (x) == log x,

Calculus:Investigation

'1'2(x) =

of Funcl's)

-.
I

There are tables for the val ues of

these functions, and this simplifiesthe constructionof their graphs.


Constructing the graphs y == log x and y = x (see Fig. 57), we find
the approximate value of the only
/:I
\037

2.5.
3.12.5.
Find the real

root

\037

\302\243

root of the

equation

+ 3x-5==0)
4:
up to 10-

f (x) == x 3 _2X2

1)

wi

(a) by applying the method of


chords,
(b) by applying the method of

x)

o)

than accuracy

tangents.
Solution. Let

us first make sure


the given equation has only
one rea] root. This follows) from the fact that the derivati ve)
Fig. 57

that

f' (x) =3x2 -4x+3> 0.


3 < 0, f (2)== 1 > it follows

that the given


Then, from f (1)==
polynomial has a singlepositive root, which liesin the interval (1,2).
(a) Using the method of chords, we obtain the first approximation:
\302\260

Xl

== 1

Since)

-4-3
-0.5156<0,
\302\267

1 == 1.75.)

==
f (1.75)

and f (2) == 1 > 0, then 1.75


<
The second approximation:

\302\243

x2 ==

< 2.

== 1.8350.)
1.75
+ 0.0850
+ 0.5156
1.51560.25== 1.75
\302\267

= 0.05059
Sincef (1.835)
< < 2.
< 0, then 1.835
The sequenceof the approximations converges very sloVJly. Let
\302\243

us try to narrow down the interval, taking into account that the
is considerably
val ue of the function f (x) at the point x 2 ==
less in absolute value than f (2). We have

1.835

<
Hence,1.835

\302\243

> O.
(1.9)== 0.339

< 1.9.)))

3.12.
Algebr.

\037

Applying

Xa

approximation:
===

1.9),we
(1.835,

-0.05059 0.065
==
1.8350.339+0.05059 1.8434.
\302\267

calculations by the method of chords yield

1.8437,X5 1.8438,
sincef (1.8437)
> 0, then
< 0, and 4 f (1.8438)
X 4 ==

and

187)

Equations)

the method of chords to the interval

will get a new

Further

Transcendent.

and

===

\037

1.8438
with

the required accuracy of 10(b) For the method of tangents we chooseXo=2 as the initial
4 > 0 in the inapproximation, since f (2) 1 > 0 and f\" (x)2==
terval (1,2). The first derivat ive f' (x) == 3x 4x+ 3 alsoretains its
sign in the interval (1,2), therefore the method of tangents is ap-

-6x-

===

plicable.
The

first

approximation:
Xl

== 2

\037

1/7== 1.857.)

The second approximation:

= 1.857 0.0779
=:
x2 == 1.857f'{(1.85?)
5.9275 1.8439.
(l.857)

The third approximation:


xa ==

==
1.8439f' (1.8439)
(1.8439) 1.8438,)
f

already gives the required accuracy. I--Ierethe sequenceof the approximations convergesmuch more rapidly than in the method of
chords, and6 in the third approxinlationwe could obtain an accuracy
up to

10-.

3.12.6.

Find the least


an accuracy up to

positive root of the equation tan x ==x \\vith

0.0001
applying Newton's method.

a by
Find the real root of the equation 2-x-Iogx==
3.12.7.
combining the method of chords with the method of tangents.
Solution.Rewrite the left member of the equation in the following

way:

-x)+ (-

log x),
f (x) == (2
the
function
seen that
f (x) is a sum of two monoto-

whence it is
itself. Connically decreasingfunctions, and therefore it decreases
sequently, the given equation has a single root
Direct verification shows that this root lies in the interval (1,2).
This interval can be narrowed still further:)
\302\243.

1.6<\302\243<1.8,)))

188

Ch.

sInce)

III.Differential

Calculus:Investigation

of Funci's)

> 0; {(1.8)=-0.0553
< O.
f(I.6)==0.1959

Then)

f' (x) ==
and

f' (x) < 0;

-1

\037

log e;

{\"

(x) ==

\037
x)

> 0 over

log e

[1.6; 8].

the whole interval


1
both
the
method
interval
of
chords
and the
Applying
method of tangents with the initial point x() == 1.6we obtain the
{\" (x)
to this

-(1.8f(l.8)-r(J.6) . -1-.
= 1.6= 1.6+ 0.1540
= 1.

approximations:

first

x)==1.6
x;

f,

1.6)f (1.6)== 1 6 0 1559 ==.


1 7559',
7540.

\037\\

..\037)

the same methods to the interval [1.7540,


1.7559J,we
ions:
the
second
approxin1at
get
I .7559)f (I .7559
) == 1 75558
x == 1 7559 (I.
Applying

75\0370

--

.
,=
x2 1.7540
J' (l.7540)
2

(I 7540)
(1 . 7540) =
f

the root
-x;=0.00001,
0.00001.

Since x2
to

up

(I 7559)

1.75557.
\037

is computedwith an accuracy

3.12.8.
Using the combined method find all roots of the equation
5x -t- 1 == 0 accurateto three decimal places.
3.12.9.
Applying the iteration method find the real roots of the
equation x-sinx=0.25accurateto three decimal places.
f (x) == x3

Solution.Representthe given equation in

the form

= sin x.
x-0.25

Using the graphical method, we find that the equation has one
real root which is approximatelyequal
to Xo == 1.2(see Fig. 58).
!y)
Since)
\302\243,

\037

\037\037.

\037)

sin 1.1
==0.8912
> 1.1-0.25,
=
=
sin 1.3 0.9636
< 1..3 0.25,
1 3).
the root lies in the interval (1.1,
Let us rewrite the equation in the

\037t$>

\037-)

!I=sinx)

\037

x)))

form

+ O.25.
interval (1.1,1.3)does.

x == (x) == sin x

Fig. 58

cp

in the
Since the derivative cp'(x)==cosx
exceedcas1.1< 0.46< 1 in absolute value, the itpration rnethad
is applicable.
Let us write successiveapproximations
xn == sinxn _ +0.25 (n == 1, 2, .),)

not

..

\037

taking

Xo

3.12.
Algebr.

1.2for

and Transcendent.

189)

Equations)

the initial approximation:)

1.2 +0.25=0.932
+0.25=1.182;
+0.25=1.173;
xa==sin1.175+0.25==0.923
x1 ==sin

X2

= sin 1.182
+ 0.25 0.925+ 0.25== 1.175;

x4

=sin1.173+0.25=0.9219+0.25=
1.1719;

===

1.1715;
xr.=sin1.1719+0.25=0.9215+0.25=
xA===sin

1.1715+0.2.5===0.9211
+0.25===1.1711.)

q
Sinceq == 0.46and hence, I -q
< 1, we have

===

1.71 wi thin
\037

the

required accuracy.

3.12.10.
Applying

root of 1 he

the iteration

equa t ion)
XB

method, find the greatest positive

+ x = 1000)

accuratp to four decimal places.


Solution.Rough estimation gives us the approximatevalue of the
root

Xu

We

= 10.

can rewrite the given equation


X==

or

In

the

1000-x
a

,)

x----

form)

1000

x2

or

in

the

the lorm)

in

')

form)
X==

1000-xand

\037/

so

on.)

The most advantageous of the indicated methods is the preceding


[9,10]for the main int erval and putting

one, since taking

(P

we

find

that

(x) =

V 1000-x,)

the derivative)
cp'(x) ==

-I
3 V (1000-X)2)

does not exceed1/300


in absolute value:)
1

cp'(x) I

\037

3 V 990
2)))

1
\037

300

= q.

190

Ch.

III.Differential

Calculus: Investigation

Compute successiveapproximationsof

Xn with

the formula)

Xn+l

n
(n == 0, 1,2,
V 1000-x

===

of Funct's)

one spare digit

by

...),

10,
==
==
Xl
V 1000-109.96655,
== 9.96666,
9.96655
X2
V 1000==9.96667.
x3 == V 1000 9.96666
4.
9.9667
with an accuracy of 10We may put
lvTote. Here,the relatively rapid convergence of the processof iteration is due to the smallness of the quantity q. In general,the
Xo

==

\037

===

\302\243

smaller the q, the faster the process of iteration converges.


3.12.11.
Applying the method of chords, find the positive root of
the equation)

with

f(x) == x3 + l.lx2 +0.9x-I.4=0

an accuracy of

0.0005.

3.12.12.

Using the method of chords, find approximate values of


I:
the real roots of the following equationswith an accuracy up to 0.0
X
e
-2(I-x)2==0.
(
b)
(a) (x-I)2-2sinx==0;
Applying Newton's method, find with an accuracy up
1 the positive roots of the following equations:
to 0.0
(b) x:{+x-32==0.
(a) x:}+50x-60=0;
Using the combined method find the values of the root
of the equation)
==a

3.12.13.
3.12.14.

x3-x-l

on the interval

[1,2] with

3.12.15.
Applying
tion

an accuracy

up

to

0.005.

the iteration method, find all roots of the equa-

4x-5 In x= 5 accurateto four

decimal places.)

Problems
* 3.13.Additional
Does the
3.13.1.

function
f (x)

=
{\037/x)

if

x<1

if

x\0371

satisfy the conditions of the Lagrangetheorem on the interval [0,2]?


the number
Prove that for the function y==ax2+\037x+y
in the Lagrange formula, used on an arbitrary interval [a, b], is the
arithmeticmean of the numbers a and b: =:;(a + b)j2.)))

3.13.2.

\302\243

\302\243

\037

3.13.3.
Prove that

8.13.
Additional

ii the

Problems)

191)

equation

+ a -1+ ... + an-lX ==0

QoXn

x ll

has a positive root x o, then the equation


n

naox

- l + (n-l)QlXn - 2 + ... + an -

has a positive root less than xo'


Prove that the equation x4
real roots.

3.13.4.

3.13.5.
Prove

the function

that

-4x-l

== 0

== 0

has two different

f(x)==xn+px+q cannot

have

roots for n even and more than three for n odd.


3.13.6.
Prove that all roots of the derivative of the given poly-

more

two real

than

are real.
f(x)==(x+l)(x-I)(x-2)(x-3)

nomial

3.13.7.
Find
The

mistake in the followinl reasoning.

*0,

function)

f (x) =

x2 sin ( 1/x)

\\

for x
for x = 0

is differentiable for any x. By Lagrange'stheorem


x2 sin x = x 2t sin \037-cos
\037

whence)

casTI =

\037

\037

2t'
. SIn -r- sin. X
1

\302\243')

\302\243

(0

< t < x)
\037

\302\267

As x tends to zero will also tend to zero.Passing to the limit,


we obtain Hm cos(l/\037) ==0, whereas it is known that lim cos(1Ix) is
\037

\037

non-existent.

x\037

3.13.8.

Find a mistake in the following deduction of Cauchy's


formula. Let the functions f (x) and (x) satisfy all the condit ions
of the Cauchy theorem on the interval [a, b].Then each of them
will satisfy the conditions of Lagrange'stheorem as well. Consequently, for each function we can write the Lagrange formula:
a < S < b,
f
(a) = f' (s)
=
a
< < b.
(p(b)-(()(a) (()' (s)
Dividing the first expressionby the second,we obtain:
<p

(b)-f
I
cp

(b-a),
(b-a),

- - --

\037

(b)-I(a) _ I' (b-a)_ t'


(\037)

(b)

q; (a)

cp'

(\037)

a)

(b

cp'

Prove the following inequalities:


3.13.9.
a-b
< In < a-b 0 < b < Q
(a)
a

\037

if

')))

(\037)
\302\267

(\037)

III.DitJerential

Ch.

192

Calculus: I nuestigation

of Fanel's)

if O<y<xand p>l.
(b)pyP-l(x-y)<,xP_yP\037pxP-l(x-y)

Prove that
3.13.10.

all roots of the Chebyshev-Laguerre


polynomial)
Ln (X )

are positive.
Prove that

t.
3.13.1

if

conditions:
(1) it is defined and has

-e

n
x d
dx

ll

n
(x

e)

.\\'

the function f (x) satisfies the following


a continuous derivative of the (n

order f(n-u (x) on the interval [xo, x n ];


(2) it has a derivative of the nth order

'

(x o x n );
==f (xrJ (x 0
(3) f (x 0) ==f (Xl) ==
then inside the interval [Xo, x,J there
==0.
that f(ll)

f(n)

(x)

in

1)th

the interval

< Xl < ...< Xll) ,

...

is at least one point

\302\243

such

(\037)

The limit of the ratio of the functions


3.13.12.
. e-xl+2tanx
.
==
.
1m e-2X(cosx+2sinx)
l+tanx
e-X(cosx+slnx) tm
.
. . .
x
tan
.
2
.
+
IS d Iscont Inuous
th e expreSSIon
t t ,SInce
IS non-eXlSen
I
x\037oo

x\037oo

...),
1

+ tan

at
at the same time the

the points
0, 1,
+
limit of the ratio of the derivatives does exist:
Xn

I 1m
x\037oo

Jt/2 (n ==

==n:rt

but

[e-2X(cosx-+2sinx)]'
.
' = I.

-.

5
-5e-2Xsinx
. =
x-oo -2e-xslnx 2

+slnx)]

[e-X (cosx

1111

I 1m

e_ x

==.
0

x\037:y))

Explain this seemingcontradiction.

3.13.13.
Prove

formula of the

that the number

first

tends to 1/3 as

order

e in

the remainder of the Taylor

(a+h) = f (a) +hf'(a) +

\037;

f\"

(a+8h)

f'\"(x) is continuousat

a and

(a) =1=0.
Prove that the number e is an irrational number.
h

\037

\302\260

3.13.14.
Prove that
3.13.15.

if

for

\302\260

<x

decreases.From this obtain

\037

f'\"

Jt/2 the function f (x) (sinx)jx


the inequality 2x/Jt < sin x < x for
\037

=--=

o < x < 'Jt/2 and give its geometricmeaning.


Show that the function f (x) ==x+cosx-aincreases;
deduce that the equation x + cosx== a has no positive roots
\\\\'hence
for a < 1 and has one positive root for a > I.

3.13.16.

3.13.17.
Show that

root found

in

the

has only one positive


equation xex ::::::2

the interval (0,

1).)))

Additional
3.13.

.\037

3.13.18.
Prove that

193)

the function

2.

( X + x Sin x f or
{\"2
1

f(x)=
is not monotonic
graph f (x).

Problems)

x=l= 0,

for x = 0

l 0

any interval containing the

in

3.t 3.19.Prove the

origin. Sketchthe

theorem if: (1) f (x) and (x) are continuous


the interval [a, b] and differentiable inside it; (2) f (a) = (a);
and (3) f' (x) > cp'(x) (a < x < b), then f (x) > (x) (a < x < b).
cp

in

cp

cp

has neithermaxima,
Show that the function f (x) =
3.13.20.
;;t\037
nor minima at ad-be O.
In the trinomial x2 + px + q choosethe coefficients p and q
3.13.21.
so that the trinomial has a minimum at x = 3 and that the minimum equals 5.
Test the function f (x) = (x-xo)n (x) for extremum at
3.13.22.
the point x x o' where n is a natural number; the function (x) is
and
continuous at x
(xo) O.
3.13.23.
Given a continuous function
=1=

cp

\037-=

cp

-==

Xo

=1=

cp

(2

f (x) = f

sin
\037

\\

xI

at

x 0,

at

x=O.

=1=

Show that f (x) has a minimum at the point x = 0, but is not


monotonic either on the left or on the right of x = O.
Find the greatest and the least values of the following
functions on the indicated intervals:

3.13.24.
(a)

== x

(b) y = E (x)
I

3.13.25.

for
for

--

\037

\037

\037

\037

1,

1.

Do the following functions have the greatest and the


least values on the indicated intervals?
for
(a) f (x) ==cosx
(b) f (x) -== arc sin x for

3.13.26.
3.13.27.
Prove that

-Jt/2 x <
-1
< x < 1.
\037

Jt,

Prove that between two maxima (minirna) of a continuous


function there is a minimum (maximum) of this function.
the function

f (x)

-_ {

X2

sin2 (l/x) for x =1=


for x =

0,
0)))

194

Ch.

III.Differential

Calculus:Investigation

of Funct's)

at the point Xo ==0 (not a strict minimum).


Prove that if at the point of a minimum there existsa
right-side derivative, then it is non-negative, and if there existsa

has a minimum

3.13.28.

left-side derivative, then it is non-positive.


Show that the function
3.13.29.
y ==

Ijx2 (x > 0),


{3x2 (x 0)
\037

has a mInImum
not

0, though

at the point x==.:

its

first

changesign when passing through this point.

derivative does

3.13.30.

Let Xo be the abscissa of the point of i flection on the


curve y = f (x). Will the point Xo be a point of extremum for the
funct ion y == f' (x)?

3.13.31.

- -1-

Sketchthe graph of the function


if
bourhood of the point x ==

3.13.32.

1)==2, f' (

f(

For what

probabilities\

choiceof

- yn

have points of inflection

1,f

1)==

lf

= f (x)

1)==0, f'\"

the neigh-

in

(x)

> O.

the parameter h does the

h2x2

(h

>

\"curve

of

0))))

x == :f:a?

Show that any twice continuously differentiable function


3.13.33.
has at least one abscissa of the point of inflect ion on the graph of
the function between two points of extremum.
2
3.13.34.
+ 8 as an example,
Taking the function y ==x 4 -1-8x3 + 18x
no
be
of
extremum between the
ascertain that there may
points
abscissasof the points of inflection on the graph of a function.
Prove that any polynomial with positive coefficients,
3.13.35.
which is an even function, is concave everywhere and has only one
point of minimum.
3 has
Prove that any polynomial of an odd degreen
at least one point of inflection.
Proceedingdirectly from the definition, ascertain that
the straight line y = 2x + 1 is an asymptote of the curve y ==

3.13.36.
3.13.37.
2x 4+X3

3)

\037

Chapter4)
INDEFINITE

\037

INTEGRALS.
INTEGRATION)

METHODS
OF

Bt\\Slc

4.1.DirectIntegration
and the Methodof Expansion)
Direct integration consistsin using the following table of integrals:
un+1

(1) 5 undu= n+1+C

(n*-l);

=
5 d: In u + C:
(3) 5 aU du = a aU + C;
(4) \037cosudu=sinu+C;
(5) coshu du = sinh u + C;
du
(6) cas2 u = tan u + C;
(2)

ell

du

\037

1\037

5 SI.n

du

t.I

cosu + C;

sinhudu = coshu + C;)

\037

=-

sin u du =
\037

\037

du

= e'l+ C;

'2

cot u -{-C;

(7)

-a-arccota-+C1(a>0);
5 u2+a2 =a-arctana-+C=

(8)

. =arcstn-+C=-arccos-+C
1 (a>0);
a
a
S V a2 -u'l

(9)

du

.du

J V u 2 :f:a2

(10)5

= In(u+Vu 2 + a2) +C;

-a =-2a In

\"du 2

U\"

a
+ +C.
a)

In all these formulas the variable u is either an independent


variable or a differentiable function of some variable. If)
\037

f(u)du=F(u)+C,)

then)

5f(ax+b)dx= F(ax+b)+C.
\037

The method of expansionconsists in expanding the integrand into


a linear combination of simpler functions and using the linearity)))

Ch. IV.

196)

Integrals)

Indefinite

property of the integral:

aJi(x)dx =

i\0371

4.1.1.Find the

aj ) fi (x)dx

i\0371

C\037I

ai I> 0

).

1=S X2+\037;-ldX.

integral

Solution.)
2
1= S +5X-1dx= S (x'/.+5x'/.-r'l.)dx=
X

\302\245-X

\037

x'/.dx+5) x'/.dx-)x-'/.dx=
52

=s+C1 +3X3/2+C2-2xl/2+C3==)5
2x \"/2

\302\267

=2 V-X ( + 3X
\0372

- )+C.
1

Note. There is no need to introduce an arbitrary constant after

calculatingeach integral (as is done in the above example).


By combining all arbitrary constants we get a single arbitrary constant,
denoted by letterC, which is added to the

final

answer.

d
4.1 2 / 5 6X3+X2_2x+
2x_
\302\267

\302\267

413/= 5
\302\267

\302\267

X.)

\302\267

dx
sin 2 x cos2 x

\302\267

Solution.Transform the integrand


I

sin 2 x

cos2 x

in

x+cos
2

sin 2
sin 2 x

2x

the following way:


I

Hence,)
dx
dx
= tan x
1= 5 cos...
x + 5 sIn x
'J

4.1.4.1=

.\037

tan 2 X dx.
Solution.Since tan 2 x = sec2
\037

1=

\\

\037

4.1.5.1=
\037

tan 2

cot x +

\302\267)

c.

x-I,then

xdx=j cosdx2

- dx=tanx-x+C.
\

x)

\037

(x2 + 5)3dx.

Solution. Expanding the integrand

we

cos2 x + sin 2 x

cos x

by the

binomial formula,

find

15xo 75x3
1=5 (x6 + 15x4 +75x2 + 125)dx=T+T+a-+
125x+C.
4.1.6.
1=\037(3x+5)17dx.)))
X7

\037

4.1.Direct

and the Method of Expansion

InteRration

197)

Solution. Here it is not expedientto raise the binomial to the


power, since u ==3x + 5 is a linear function.
Proceedingfrom the tabular integral)

17th

S
we

U 17

du=18 + C
Ul8

')

get)

. (3x+18Sp8-t- C
I --J.3

\302\267

4.1.7.
I = s \037x
x+l-VXx '
4.1.8.
I=

cas(J'tx + 1)dx.
Solution.Proceedingfrom the tabular integral (4)
\037

\037

casudu=sinu+C,)

we obtain)

4.1.9.
I=

==\037

3t

sin (nx+ 1)+ C.

cas4x cas7x dx.


\037

Solution.When calculatingsuch integrals it is advisable to use

the trigonornetric

product formulas. Here

cas4x cas7x ==2\" (cas3x + cosl1x))


1

and therefore

I = + S cas3xdx + S casllxdx =
\037

Note.

\037

solving such problems

sin 3x + 212 sin llx+ C.


is

to

expedient
ities:
sin mx casnx = {-[sin(m-n)x+ sin (m + n) x];
sin mx sin nx = -}[cas(m-n)x-cas(m + n) x];
cosmx cosnx==2'[cos(m-n)x+ cos(m + n) x].

When

it

use the

following trigonometric ident

4.1.10.
I=

cosxcas2x cas5xdx.
\037

Solution.We have
I
(cosx cos2x) cos5x ==2'(cosx -{-cos3x) cos5x=

= 4I [cos4x + C056x] + 41 (cos2x+cos8x).)))

Ch, IV. Indefinite Integrals)

198)

Thus,

1. I. 1. 1.

=
1= [5cos2xdx+
5 cos4xdx+
5 cos6xdx+
5cos8xdxl
sIn 2x+16
s1n4X+ 24 SIn 6x+ 32 SIn 8x+C
=8
4.1.t 1.I = \\ sin2 3x dx.
\037

1-cos

. SInceSIn. 2 3x =
t
Sal Ulan.

6x

.\"

' th en

I=-}5 (1-cos6x)dx=x-/2 sin6x+C.


4.1.12.
1= cosh2 (8x+5)dx.
\037

\037

Solution.Since cosh2 u = cosh ;U+

, then

1=-}5 [1+ cosh(16x+ 10)]dx=-}X+32 sinh (16x+lO)+C.


4.1.13.
1= 5 x2+:+5
=
Solution. 1= 5 X2+\037:+5 = 5 (X+:;2+
1 arc tan (x + 2) + C.
'
4.1.14.1=
5
4.1.15.
1= 5
dx
4.1.16.
I = r 4-9x2
S
1

\302\267

4X2\03725

\302\267

X2+\037+1

\302\267

=
=
4/9-x 3 arc sin 2 + C.
s r 4-9x 3 S r-=.
4.1.17.I=
Sv 5-x2-4x
dx

Solution.I =

3X

dx

dx
. I=
Sal u t ton.
2
J' 5-x
S ..r
4.1.t 8.I = V dx

4.1 19.I =
\302\267

Solution.

x\037+

S4 -x

6x+ 1

4x

\037x

4x

4.1.20.
1= S

lOX\037\037

\302\267)))

\302\267

\302\267

dx

4x

\302\267

\302\267

- =,8-(x+2)2=

I= S 4-xdx2

2
= arc sIn x+
3 +C

dx

S V 9-(x+2)2

\302\245'2

In

2Y2+X+2
Jf2-(x+2)

)2

-t-

\302\267

4.2. Integration

\037

4.1.21.
Eval ua te

dx

3-2

(b) r

\037

x-I dx;

dx'
lX')
X

(d)

cos

\037ot2

2 3x2

5 x2 (l +x 2) dx.)

2x+1-5x-l

r
dx;
I
x4
J VI-\037+VI+X2

(c) 5

(b)

cos2x dx'
sin x '

5 cosx

dx; (d) 5 (sin 5x

lOX

199)

J V X2

4.1.22.
Integrate:
(a)

Substitution)

the following integrals:

(a) JC x 2 6x+ 13 ;
(c)

by

sin 5a)dx.)

4.2.Integration
by Substitution

The method of substitution (or change of variable) consists in sub(t) for x where
(t) is a continuously differentiable
function. On substituting we have:

stituting

<p

cp

\037

f (x)dx =

\037

[cp

(t)] cp'(t) dt I

and after integration we return to the

stitution t ==cp-I (x).


The indicated formula is

old variable by inverse sub-

also used in
f[(p (t)] cp'(t) dt = f (x) dx,

\037

the reverse direction:)


where

\037

4.2.1.
1=

\037

xV

x = (t).
cp

x-5-dx.

Solution.Make the substitution

Vx-5=t.)
Whence)

x-5=t, x=t +5, dx=2tdt.


2

Substituting into the integral we get

1= 5 (t2 + 5) t

\302\267

2t dt

=2

Now return to the initial

5 (t4 + 5t ) dt
variable x:
2

= 2 + lOt + C.
\037

1= 2 (X\0375)'/' + 10(X;5)3/.+ C.

4.2.2.1= 5

\302\267

Solution.Let

\037xex

us

make the substitution 1 + eX = t. Whence

ex=t-l,x=ln(t-l),dx=dtl(t-l).)))

Ch. IV.

\037uo)

Indefinite

Integrals)

Substituting into the integral,we get


J

=
\037:x

But)

-1)=

therefore)

- -T
I

t (t

5 t (t\037 I)

\302\267)

')

dt -- dt
C
1=5 t-I
5 t=lnlt-II-Inltl-t.)

x, we obtain)
eX
C = X -In(1-1-eX) + c.
eX +

Coming back to the variable

1= In +
1

Note. This integral can be calculatedin a simpler way


tiplying both the numerator and denominator bye-x:
e-X
=
5 e- + dx
X

- e--e-X+ 5

dx =

=_ln e

e)

+ dx.
.2.3.I = 5 Y (2x-5)3

4.2.4.I =
\037

=x-In(ex+I)+C.

dx
(x2-1)
'
x2 + l
(x4 + 3x2 + 1)arc tan
x

Solution.Transform the integrand

(I 1/x 2) dx
1 5 [(x+l/x)2+
Ij arc tan (x+I/x)

\302\267

Make the substitution x-t-- x = t; differentiating, we get


\037

\\Vhence)

(1-;2)dx=dt.)
I = 5 (t2+ I)dtarc tan t

Make one more substitution:arc tan

t ==

\302\267

u. Then

dt
t2

+1=du)

and)

1=S u=ln)ul+C.)))
dU

mul-

(e- + I) + C =

In

\037.+

X2

by

4.2. Integration

\037

Returning

by

x, we

to t, and then to

first

Substitution)

201)

have

)/+C.
1=lnlarctantl+C=lnlarctan(x+
\037

4.2.5.
I ==
5

dx.

Y\302\243l2\037

x4

Solution.Make the substitution:


dt

x==T; dX=-fi\".)

Hence,)

I= 5

-1-

Returning to

2t 2

\037\037t2

substitution: Va2t 2

Now make one more

=2zdzand)

=-5 Va

dt
\037\037;\037

a2 S Z

2 dZ

--

3a2 z

- dt.
I

1 = z. Then 2a2t dt

::::

+C

\302\267

x, we obtain
2
1==-(a2_x
23)3/:a +C.
3ax

and then to

4.2.6.I = S a2 sin 2

2 cos2 X

x\037 b

\302\267

Solut ion.

i-

I== a2 sin 2 dx b2 cos2 x ==\037b2 a2 1


x+
tan 2 x
b

\037

Make the substitution

a
b

2)

a
tan x ==t; dt == b

cos x)
d\037

. cosdx2 x

\302\267

. Then

dt
== arc tan t + C.
I ==ab Jr 1+
t2
ab
I

Returning to

x, we obtain
I = arc tan
\037

tan

x) + C.

4.2.7.I=\037 Vl+3sinxcosxdx.

Solution.i\\\\ake the substitution 1 +3sinx==t,


3cosxdx=dt.
Then

..-

-3

-3

I-\037 V tdt-\037 tl/adt


S
S
428 f
\302\267

r sin x dx
J V cosx

\302\267)))

-3

_\037.\037t\037/a

+ C_ (l+3sinX)./a
+C
4

\302\267

...r'

Ch.

202)

1==
4.2.9.
5 (arc cos

\"

x\037x

I ndefinite

IV.

Integrals)

x\037

Solution.Makethe substitution:arc cosx= t;

= dt. Then
Vld\037

x\037

1=-5 tf>=-5 t-f>dt-==Tt-4+C==


4arccos4x+C.
dt

-!- dx.
J V +3x+
4.2.11.I == 5 +sin . 2x.) x dx.
x2

1= r
4.2.10.

x3

SIn'\"

Solution.Make the substitution:


1

+ sin2 x== t;

..,

Then)

2 sin xcosx dx ==sin 2x dx

\037

dt.)

2
1=5 =Int+C=In(1+sin
x)+C.
+ x dx.
4 2 12 1
3
+
5 x x
\037t

In
In

===

Solution.Substitute

and

3 + x In x == t,
get)

1= 5 = In

\037t

(1+.In x) dx ==dt)

+ C = In 3 + x In x + C.
I

Evaluate the following integrals:


4.2.13.
;
(a) Vl:lnxdx; (b) 5
x\037\037x

\037

(c)

x dx
5 V3

d
-x ()
4;

(e) S Si\037X dx;

Fin d
4.2.14.

th

(f)

xll

x21l -!-a dx;


t.\\
\037

S ( In
r
J

X2

\037

sf>

+ dx)

\302\267

dxX

integrals:

follo\\ving

1 x dx; (b)
5 V
(c) CO X Vsin xdx; (d)

(a)

In
X

x dx

\302\245l+lllx

x.

SV

1-. dx.
x\037)

4.3.Integrationby Parts
The formula)
\037

is

known

udv=uv-

vdu
\037

as the formula for integration

are differentiable

functions of

x.)))

by

parts, where [l

and v

\037

4.3. Integra.tion

Parts)

by

203)

To use this formula the integrand should be reduced to the product of two factors: one function and the differential of another
function. If the integrand is the product of a logarithmic or an
inverse trigonometric funct ion and a polynomial, then u is usually
taken to be either the logarithmic or the inverse trigonometric
function. But if the integrand is the product of a trigonometricor an
exponential function and an algebraicone, then u usually denotes
the algebraicfunction.

4.3.1.I ==

arc tan
\037

Solution.Let

us

x.dx.
here)

put

u == arc

tan

dv

x,)

= dx;)

whence)
du

= +dxx2
I

v ==x;)

;)

I ==Sarc tan xdx==x arc tan x-S xdx


+X2)+C
+ x2 =x arc tan x-2'ln(1
4.3.2.1= arc sin xdx.
4.3.3.1= xcosxdx.
I

\037

\037

Solution. Let us

put)
u ==x;)

dv == cosx dx,)

whence)

1=
\037

We will

v == sin x,

du ==dx;

sinxdx==xsinx+cosx+C.
xcosxdx=xsinx-\037
show now what would result from an unsuitable choice

multipliers u and dv.


In the integral xcosxdx let us

of the

put

\037

whence)

du

=\037

=-\037

cosx;

dv ==x dx,)
I

sin xdx'

V==2X

,)

2
\302\267)

In this case)

x2 cosx -t- S X2 sin xdx.


As is obvious, the integral has become Inore complicated.
4.3.4.1= x3 In x
\037=

\037

dx.)))

\037

\037
2)

Ch. IV.

204)

Solution.Let

us

Indefinite

Integrals)

put

u = In x;)

dv ==X 3 dx

,)

- -

whence)

dx
du == x

\302\267

')

-.\037

-I

4)

dx
4
= J..
r 3
1=14 x 1n x- 4 S x
4 x41nx-\037
4 J dx
4.3.5.I = (x2 -2x+5) e- dx.
\037

X4

X4
\037x41nx-J..
4
16 + C

===

\302\267

\037

Solution.Let

us

put

x -2x+5;)

u == 2

dv ==

e- dx
X

whence)
du ==
x

v ==

-e-

x dx.
(x -2x+5)e-dx=-e-(x -2x+5)+2(x-l)e2

I=\037

(2x-2)dx;

,)

\037

We again integrate the last integral by

x-I

== u;)

dv ==

whence)
du == dx'
, v ==

11=2

parts.

e- dx,)
X

-e-.
X

x dx=-2ex +C.
(x-l)+2 e- dx=-.2xe(x-l)ex

\037

Put)

\037

Finally we get

1= e- (x2-2x+5)-2xe-+ c = e- (x2 + 5) + C.
X

Note.As a result of calculationof integralsof the form P (x) eax dx


we obtain a function of the form Q (x) eax, where Q (x) is a polynomial of the same degreeas the polynomial P (x).
This circumstanceallows us to calculatethe integrals of the indicated type using the method of indefinite coefficients, the essence
of which is explainedby the following example.
\037

4.3.6.Applying

\037

Solution. (3x3
\037

the method of indefinite coefficients, evaluate


= (3x3 17)e2x dx.

-17)

e2X

dx = (Ax 3 + BX2 + Dx+ E) e2X + C.

Differentiating the right and the left sides,we obtain


e2X = 2 (Ax 3 + Bx2 + Dx + E) e2X + e2X (3Ax2 + 2Bx+D).
(3x3

-17)

Cancellinge2X , we have

3x3

-17_ 2Ax + (2B+ 3A) x + (2D+2B)x+(2E+


3

D).)))

4.3. Integration

\037

205)

Parts)

by

Equating the coefficients at the equal powers of x in the left and


sides of this identity, we get

right

O=2B+3A;
-17==2E+D.)

3 2A;
o == 2D + 2B;)
\037

Solving the system, we obtain


3

9
B=-4;

A=\"2;
Hence,
3
5 (3x

-17)e

2X

=_ 718

9
77
3
9
dx= ( 2X3-TX2
+TX-s

\302\267)

e2X + C.

(x3 +I)cosxdx.

I=\037

us

4.3.7.Integrate:)
Solution.Let

D=T;)

put
u ==x 3

whence)

+ 1; dv = cosx dx,)

du = 3x2 dx; v = sin x.


3
+ l)sinx-3/
1.
1=(x3 + l)sinx-3 x2 sinxdx=(x
\037

where

11= x2 sin xdx.


\037

- cosx+2/

Integrating by parts again, we get

/1=

12 = xcosxdx.

x2

2,

where
Integrating by parts again, we obtain
\037

/2==xsinx+cosx+C.)
Finally, we have:

I ==

\037

(x

2 cosx-6x
sin x-6cosx + c=
+ 1)cosx dx==(x3+ 1)sinx+3x

== (x3

-6x+1)sinx+(3x
-6)cosx+C.
2

Note. The method of indefinite coefficients may also be applied


to integrals of the form
\037

4.3.8.1=

\037

\037

(x

\037

P (x)casaxdx.

(x2 + 3x+ 5) cas2xdx.

Solution.Let
2

P (x)sin axdx,

us

put

+3x+5)cos2xdx==
== (AOX2 + A1x + A 2) cos2x + (BoX2 + B1x
+ B2) sin 2x+

C.)))

Ch. IV.

206)

Differentiate both

sidesof

Indefinite

Integrals)

the identity:

-2

(x2 +3x+5) cos2x=

(AOX2 + A1x+A 2) sin 2x+


+ (2A ox+ At) cas2x+ 2 (BOX2 + B1x+ B2) cas2x+(2Box+Bt)sin2x=
== [2B + (2B1+ 2Ao) x + (AI + 2B2)] cos2x +
+ [-2AoX2+ (2Bo -2A])x + (Bl -2A2)] sin 2x.
oX2

Equating the coefficients at equal powers of x in the multipliers


and sin 2x, we get a system of equations:
2 (Bl+ Ao) == 3;
2Bo= 1;
At + 2B2 == 5;
B1-2A2 ==O.
2 (Bo-At) ==0;
2A o == 0;

cos2x

Solving the system, we


Ao ==

find

0; Bo= 2\";

Al == 2\"; B1=

Thus,

A 2 ==

3
4\";

B2 == 9
\"\"4

2
2
fX + ;x+ ) sin x+C.
+3X+5)COS2xdx=(+ )cos2x+(
4.3.9.I = (3x2 + 6x+ 5) arctan xdx.

2
(X
\037

3
2;

\037

\037

Solution.Let

us

put

u == arc tan
whence)
dU==

Hence,)

dv == (3x2

x;

+ 6x -1-5) dx,)

dx
3
2
I+x2 ; v=x +3x +5x.)

I == (x3 + 3x2 + 5x) arc tan x-J(' x3+3x2 +5xdx.


1-\037x2

Singleout the integral part under the last integral


the numerator by the denominafor:

by

dividing

X3+3x2+.5X
dx == (x 3) dx--t- 4X-3
/t ==5
x2
5
5 x 2 I dx
2
2
x
2xdx
dx
x
2

1+
=2 -{-3x+ 25

x\037.+

+ =
= + 3x+2In(x
tanx+C.
+ l)-.3arc
S x2 +

-3.

Substituting the value of

== (x3

3x2

11'

\\ve finally

get

2
+ 5x.+3) arc tan x-x2/2-3x-2In
(x + 1)+ C.

4.3.10.
Find the

integral

I=
Solution.Let

2\"

e5x cos,4xdx.
\037

us

put

e5X ==u; cas4xdx =.dv,)))

4.8. Integration

\037

Parts)

by

207)

whence)

5e5 dx = du; v == sin 4x.


.\\

\037
4)

Hence

,)

I=
Integrat ing
(I

5X

Thus,
ebX sin 4x

\037

- -

sin 4x dx;:;;

11 J e
/=

5 e5X sin 4x dx.

parts again, we obtain

by

===

sin 4x -\037
4

5X
4e

\037

\037

1.e.)

e5X cos4x+\"4 e5X cas4x dx.


\"4
1

ebX

\037

cas4x + 5 e cos4x dx )
bX

\037

.
5
25
5x
1==-e
---I
4
4
( sln4x+-cos4x
) 16
1

\302\267)

Whence)

/=
I=
4.3.11.
\037

Solution.Let

ebx

44)

(sin4x+ COS4x)+C.
\037

cos(lnx) dx.
us

put
u ==

whence)

cas(In x); dv ==dx,)

dx
;
du=-sin(1nx)
x)

Hence,)

v==x.

I = cas(In x) dx = x cos(In x) +
\037

I ntegrate

parts once again


II -= sin (In x);)

by

dv

-==

\037

sin (Inx) dx.)

dx,)

\\vh\037ncc)

dx
v==x.
du==cos(lnx)-;
x)

Hence,)

11 =

\037

sin (In x)dx= x sin (In x)

\037

cas(In x) dx.)

Thus)

I ==
\037

cos(In x) dx ==x cos(In x) + x s in (In x)

lienee)
J :=;
\037

[cas (ln x) -t- sin (l n x)] +

c.)))

I.)

,)

Ch. IV.

208)

/ = S x In ( 1 +
4.3.12.
Solution.Let

In

Indefinite

dx.

\037

us transform the integrand

(1+ )=ln

X\037I

\037

Hence)

Integrals)

/ = x In (x+ 1)dx
\037

Let us integrate

11and 12
u == In

whence)

\037

x In xdx = /1-/2.

parts. Put

by

dv == x dx,)

(x-I-1);

dx
du == 1 x ;

V ==

Hence

=ln(x+l)-lnx.)

'2(x2

1).)

-I)dX==

2
/1==S xln(x+l)dx==\"2(x
-1)ln(x+l)-\"2
S I+x

(X

= x22 In(x+l)-\"2(x-l)dx== 2 In(x+I)-Tx2 +-2x + C.


S
I

X2

Analogously,)

/2 = J(' x In x dx =

x2

2 x -\"4x + C.
I

ln

Finally we have

-1)

I = S x In ( 1 + + ) dx = + (x2 In (x+ 1)x; In t + +C.


2
4.3.13.
I = V x + [In + 1)-2In xl dx.
I

\037

<::

Solution.First apply the substitution

1+2 =t.
I

x)

Then)

dt ==

Hence,)

/+

I == 5 -.V

-:r
2dx
x

x2

In

x2

x)

x2

- dt.
- v-

3=
dx

or

\302\267

dx
==
x3

_2
I

\"2

The obtained integral is easily evaluated


u == In

t;

= dtt

.'

Then)

du

dv ==
v

==-

Vidt

\037
3)))

by

5 t In t dt.
parts. Let us

.)

i.

put

4.3. Integration

\037

by

209)

Parts)

Whence

-f

-f [; Vi -; Vidt]
=--}[; tVtlnt-:tVi] +c.)
1=-+ l ; (I + :2
+ :2 -f + :2 + c
5 ViIn t dt

In t

x, we obtain

to

Returning

3/2

I=
4.3.14.

In

= (x2+ l\037tX2+

) 3/2]

sin x I n tan xdx.

[2-3 ( I + :2) ] + c.
In

\037

4.3.15.1=\037ln(VI-x+VI+x)dx.
us
Solution.Let

u=ln(Vl--x+Vl+x);dv=dx,

whence

du=

put

2 V\037 2 V\037)dX=

=.!.y - + y\037+ - =.!. YI-x--1


v=x.)

Vf=X\037YT+X

Hence,
/ =xln( V

dx

\037\0371

l-x+V 1 +x)

VI

-I

YI

x Yx I

x2

VI

x2

dx;

--- dx=
x2

VI

x2

=
=x In (Vl-x+V I +x)--} dX +-}
5
5
=x In(VI-x+VI +x)-fx+f arcsinx+C.
V\037\037X2

Note. In calculatinga number of integrals we had to use the


method of integration by parts several times in succession.
The
result could be obtained more rapidly and in a more conciseform
generalizedformula for integration by parts
by using the so-called
(or the formula for multiple integration by parts):
u
\037

-.
...; -

= u (x) VI (x)-u'(x) V2 (x) + u\" (x)v3 (x) . .


... + (_I)n-1u(n-u(x) Vn (x) (_l)n-I u(n) (x)Vn (x)dx,

(x) v (x) dx

\037

where

vn (x)= v n I (x)dx.
(x)= V (x)dx; v2 (x)= VI (x) dx;
Here,of course,we assume that all derivatives and integrals appeaVI

\037

ring

in

this formula

\037

exist.)))

\037

Ch. IV. I nde.finite

210)

Integrals)

The use of the generalizedformula for integration by parts is


especiallyadvantageous when calculatingthe integral Pn (x) (x) dx,
where Pn (x) is a polynomial of degree n, and the factor (x) is
such that it can be integratedsuccessivelyn + 1 times.For example,)
cp

\037

cp

ekx

ekx

.-iS Pn(x)ekxdx==Pn(x)T-P\037(x)-f2+\"
kx
e
(x) kn + 1 +c==
+(-l)np:zn>
= ekx Pnk(x) kI2 Pn' ( ) + + (_1)n
P(n) (X) + C
kn +
I
J

x..

\302\267

\302\267

It

4.3.16.
Applying

the generalizedformula for integration


the following integrals:
2
3
2xdx,
(a) (x

find

\037

(b)

parts,

-2x +3x-l)cos
(2x +3x -8x+1)V2x+6dx.
2

\037

by

Solution.
3

(a) S (x

-2x+ 3x-l)cos2x dx= (x -2x+ 3x--1)si 2

-(3x2-4x+3)
(_ co: ) +(6x-4)( _
2X

(b) (2x3 + 3x2


\037

Si\0372X

n 2x

Si\0372X

(2x3 -4x2 + 3x)+

)_ 6

2x
co\037

COt62X

+c=

(6x2-8x+ 3) + C;

8x+ 1)V- 2x+ 6 dx =

= (2x3 + 3X2-8x
+ 1)(2X\0376)3/2_ (6x2 + 6x-8)(2xt\037)./2 +
(2x+6)9/2
+ (12x+6) (2x+6)7/2
3.5.7 -123.5.7.9+C=
2
=
396x+ 897)+ C.
(2x+ 6) (70x3 -45x
\037\037;\0376

Evaluate the following i ntegrals:

4.3.17.
\037ln(x+Vl+x2)dx.
4.3.18.V x(In X)2 dx.
arc sin x dx
.
4 3 19
Y l+x

...
\037

x cosx dx
4.3.20.
S sin3 x

4.3.21.3x cos

\037

\302\267

dx.)))

\037

4.4.Reduction

211)

Formulas)

4.3.22.(x3 -2x2 +5)e3X dx.


4.3.23.(1+ X2)2 casxdx.
4.3.24.(x2 + 2x I) sin 3xdx.
4.3.25.(x2 -2x+3)Inxdx.
4.3.26.x3 arc tan xdx.
\037

\037

\037

\037

\037

4.3.27.x2 arccasxdx.
\037

4.3.28.
Applying

the formula for multiple integration by

calculatethe following integrals:


(a)

(3X

parts.

2
2
dx.
+x-2)Sin
(3x+l)dX;(b) -7X+l
S 2x+
X

1)

4.4.ReductionFormulas

\037

Reduction formulas make it possible to reduce an integral dependon the index n > 0, calledthe order of the integral\037 to an
integral of the same type with a smaller index.
ing

4.4.t. Integrating
t he

lat ing

(a)

==

III

by

parts, derive reduction formulas for calcu..

integrals:

following
dx

x
a2)1l ;
5 (2+

(b) In

'cos
-

==

sinn x
m x

dx;

(C)

In=\037(a2-x2)ndx.
Solution.(a) We integrate
u == 2
(x

by

parts. Let us put

+a2)n '
I

dv == dx,)

whence)
du==)

lIenee,
III

== 2 x 2
(x + a )'Z

-_

2n x dx
(x2

a2)lZ+

,)

v=x.)

X2

+ 2n 5 (x2 + a2)'l+1dx ==
x

(x2 + a2

-t-\"\"n
C)

)1Z

whence)

In + J

===

(x2+a2)-a2 _ x
5 (x2 +a2)rl+ dx (x2 +a2)1l + 2nln
1

2n
. a2 I .
. x
2na:!. (x2 i-a 2 + 2n

\037

\037

n)))

)\"

2na2In+t.

Ch. IV.

212)

Integrals)

Indefinite

The obtained formula reduces the calculationof the integral 1n+1


calculationof the integral In and, consequently,allows us
to calculatecompletely an integral with a natural index,since

to the

dx
C.
11 ==S x 2+
a2 =-arctan--ta
X

a)

n ==

For instance, putting

1,we obtain

==/2== \\ (0)+
a 2rl 2a x2+ a 2
x'\"
== 2, we get
putt ing n
dx

2'

_\037.
4a2

to

J--S
3

dx

+-211=2\037
+
I

2a

2a

a\"2+2a3

=-2'(2+ +-

3
x
8a 4- x 2+ a 2

4a

a)

2-

__\037/(x2+a2)2t 4a2

(x2+a2)3

arctan-+C;

a2)2

x
+ 83a arctan-+C.
&

(b) Let us apply the method of integration by parts,


sin x
U = sinn 1 x; dv ==
cosm X dX

a)

putting

\302\267

')

whence

n -2
xcosxdx;V==( m-l)cosm-1x
du==(n-l)sin
(m
I

Hence,

1n,

- xdx =
n-I
1
m-I S cosm(m-l)cos
sinn-Ix
n-I
(m-l)cos -1 m-l n-2,2-m)

-m ==

sin n

Sin n 2

2X

X)

(c) Integrateby parts,


\\\\'hence)

---I

_x2)n;

(nl

=1=

1).)

dv ==dx,)

du=-2nx(a2-x2)n- 1 dx;

Hence

1).

putting

u == (a2

In = X (a

=1=

v==x.)

_x2)n+ 2n x2 (a2 _x2)n-ldx =


=x(a2-x2t+2n(x2-a2+a2)(a2-x2)n-ldx=
==x (a2 _x2)n_ 2n/n + 2na2f
n-l'
\037

\037

Wherefrom, reducingthe similar terms, we obtain


(1+ 2n) J n = X (a2 _x2)n+ 2na2f n-l')

Hence,)
Jn

= x (a2n2-x2)n+ 2n2na 21 1n

-1')))

4.4. Reduction

\037

F or

213)

Formulas)

instance, not ing that

J 1/2 =

we can

fin

dx

5 V a2_ x2

= arc sin 3a + C,

successively

11/2=SVa2-x2dx=(a2 -x2)1/2+ 1-1/2=

a2
x
=-2 V a -x +-arcsin-+C
2
X

13/2 = 5 (a _x )3/2dx=
2

\037

4.4.2.Applying

t ion

formulas:

a'

\037

\037

(a

_x )3/2+

/ 1/2. and

a2

\037

so on.

integration by parts, derive the following reduc-

n1n_l;
In= (Inxrdx=x(lnx)nx +1 (In x)n
n
(b) In== S X (lnx)ndx== a+1
a+l/n -1 (a=#=-l);
(c) In= xnexdx=xnex-nln_l;
n
(d ) 1n = ea.xsin X dx
. n -1X (aSlnx-ncosx
.
n (n
1)
_ -:x- 2 SIn
) + a 2+ ' 1n -)2.
a +n

(a)

\037

\037

\037

,==

efJ.X

n'--

4.4.3.Derive
= C sIn
J

.d\037

the reduction formula for the integration of In==


and use it for calculatingthe integral 13== SIn' x
5
.d\037

4.4.4.Derive the
(a)
(c)

In=
1n ==

\037

reduction formulas for the following integrals:


tannxdx; (b) In= cotnxdx;
\037

xndx

5 yx2 +

a)))

Chapter5)
BASIC CLASSES

OF

\037

FUNCTIONS)

INTEGRABLE

5.1.Integrationof RationalFunctions
If the denominator Q (x) of the

be represented in

proper rational fraction


the following way:

...

can
\037

\037;\037

...,

2
Q (x) = (x-a)k(X-b)l
(x
P)' (x2 +1'x+f.1)S
where the binomials and trinomials are different and, furthermore,
the trinomials have no real roots,the.n

+ax+

P (x) =

Q (x)

Al

A2

Ak

x-a + (x-a)2+ + (x-a)k+


81
8
8,
+ xb+(x-b)2+\" '+(X-b)l+\".
\302\267

...
...,l'

\302\267

\302\267

M 1x+Nt
+ (x2+ax+\037)2 +
+ x2+ax+\037
R2X+L2
+ x2Rtx+Ll
+yx+Jl+ (x2 +YX+Jl)2

where

l'

M2\037+N2

\"', l' l'

..
+..
...,l'
\302\267

M,x+N, +
+ (x2+ax+\037)r
+ (x2Rsx+Ls
+yx+l-t)s
\302\267

+....,
..

B B2'
M N M 2' N 2'
A 2'
R Ll' R 2J L2'
A
are some real constants to be determined.They are determined by
reducing both sides of the above identity to integral form and
then equating the coefficients at equal powers of x, which gives
a system of linear equations with respect to the coefficients. (This
A system
method is calledthe method of comparison of coefficients.)
of equations for the coefficients can also be obtained by substitut ing suitably chosen numerical values of x into both sides of the
identity. (This method is calledthe method of particular values.)
A successfulcombination of the indicated methods, prompted by
often allows us to simplify the processof finding the
experience,
coefficients.
is imp-roper,the integral part should
If the rational fract ion
\037

Mrst

be singled out.)))

\037;\037

\302\267

5.J. Integratioli

\037

5.1.1.)

of Rational

215)

Functions)

I5x2-4x-BI dx.
I ==5 (x-3)
(x+4)(x-I)

Solution.The integrand is a proper rational fraction. Since all


roots of the denominator are real and simple,the integral will
appear in the form of the sum of three simple fractions of the form
D
A
B
15x2 -4x-BI
(x-3)(x-r-4)(x-I)= x-3+ x+4+ x-I'

where A, B, D are the coefficients to be determined.Reducing the


fractions to a common denominator and then reject
ing it, we obtain
the identity

-4x-81=A (x+4)(x-I)+B(x-3)(x-I)+

15x2

+D(x-3)(x+4).(*)

Comparing the coefficients at equal powers of x in both sides of


the identity, we get a system of equations for determining the
coefficients)

15; 3A-4B+D==-4; -4A+3B-12D==-81.


A+B+D:-=:
Solving the system of equations we find A ==3, B ==5, D 7.
=-=

Hence,)

\037=
1==3S \037+5
x-3 .)r \037+
x+4 7 S x-I
== 3 In x
I

-31+

+ 7 In x I + C ==
== In l (X-3)3
C.
(x..t-.
4)a (x--l)71-+-

5 In x -{-4

Note. Let us use the same exampleto demonstrate the application of the method of particular values.
The identity (*) is true for any value of x. Therefore, setting
three arbitrary particular values, we obtain three equations for
determining the three undetermined coefficients. It is most conveto chooset he roots of the denominator as the values of x,
nient
since they nullify some factors. Putting x== 3 in the identity (*),
we obta in B 5; and putt ing x -=
we get A ==3; put t ing x ==
==
D
7
we get
x4 dx
== C
J (2+x)
2
J ==
x
x dx.
5 XL;;-3X2\0373\037Solution.Since the power of the numerator is higher than that
of the denominator,
the fract ion is improper, we have to single
out the integral part. Dividing the numerator by the denominator,)))

-4,

I
5.1.2.
5.1.3.

'
(x2-1)

-x'-

i.e.

===

1,

Ch.

216)

V.

BasicClassesof

we obtain)

Integrable

Functions)

- 1-

2
-3x
x+2
-3x-2__X +
X3-x2-2x
x(x2-x-2)
Hence,
2-3X-2
(x+2)dx '
1=S X4-3X
3 -x2-2x dx= S (x+l)dxS x(x-2)(x+I)

x4

')

Expand the remaining proper fraction into simple ones:


B
D
A
x+2
=-X+x-2+ x+I
x(x-2)(x+I)

')

Hence)

x+ 2 = A (x-2)(x+1)+ Bx (x+1)+Dx (x-2).


Substituting in turn the values Xl ==0, x 2 == 2, x 3 =
(the
sidesof

of the denominator) into both

A=-l;
And

-I

roots

the equality, we obtain

B=\"3; D==a')

so

\037-\037\037.=
1==S (x+l )dx+S dxx -\037
3 S x-2 3 S x+
2
x2
1n x+ 11
+C.
=T+ x + In X 3 1n x-21-3
I

1-

.1.4.I=
dx.
-3:+3
-f-x
S x2:-2x

Solution.Herethe integrand is a proper rational fraction,


denominator roots are real but some of them are multiple:
x 3 _2X2 .t- X

= x (x --1)2.

Hence,the expansioninto partial


A
2X2-3x+3
3
2
x

\\\\Thence

\\\\rhose

fractions has the form


D

-2x+x=7-1-(X-l)2+ x-l'

we get the identity:

2X2-3x
+ 3 == A (x-1)2+ Bx-t-Dx2(x-I)==

==-=(A+D)x+(-2A-D+B)x+A. (*)
Equat ing the coefficients at equal powers of x we get a system
of equat ions for determining the coefficients A, B, D:
A ==3.
A + D ==2; -2A D + B =
Whence A = 3; B == 2; D ==
Thus,

--1.

-3;

dx
J=3S dXx +2 S (X-l)2
xxJ\\\037=3Inlxl-\037-lnlx-ll+C.
1

1)))

5.1.Integration

\037

Functions)

ot Rational

217)

Note. The coefficients can be determined in a somewhat simpler

the identity (*) we put Xl 0; X 2 == 1 (the denominator


X 3 equal to any arbitrary value.
At x
0 we get 3 ==A; at X 1 we will have 2 == B; at X == 2
we obtain 5 A + 2B + 2D; 5 ==3 + 4 + 2D; whence D =-=
way

in

if

===

roots), and

-1.

=--=

===

===

X3+ dx.
5.1.5.1==
S x(x-l):J
5.t 6 I ==S x3xdx
+ .
J

\302\267

\302\267

-x+

Solution.Since x 3 + 1 == (x+ 1)(x2


1) (the second factor is
not expanded into real multipliersof the first power), the expan-

sion of the given fraction will have the form


x3

Hence,
x == A (x2

A
x
Bx+ D
+1= x+1+ x2-x+l

')

+ (Bx+ D) (x -t- I) == 2
== (A + B) x + (-A + B + D) x + (A + D).

x -t- 1)

Equating the coefficients at equal powers of x, we get


I

A==-3'; 8==3'; D==3')


Thus,

+
1==-3S x+J +TS x2_x+l
dx:=:-Tlnlx+l
.+3'/1'
dx

To calculatet he integral

11==S x2X+
-xI,T-

let us

dx

take the perfect square out of the denominator:

-x+1 = (x --}r + ;
and make the substitution x- = t. Then
x2

\037

/1 r
\037

t::;;

= -}In
Returning to

dt

(t +
2

=r

t2t:\037
4

) + V3\" arc

x, we obtain

+;

tan

=
\\

0.1

t2:\037
4

;3+ C.

11==\"2 In (x2 -x+1)+ V 3 arctan2x-l


V3 +C.)))
I

Ch. V. BasicClac;ses
of

218)

Functions)

Integrable

Thus,

I=S

X3\037

dx =
I

=-T1nlx+ 11+6
1

5.1.7.I = S (x2 +

In

(x

- +C.
Y3
-x+I)+\037arctan
yT
2x

\302\267

1\037\037X2+4)

Solution.The denominator has


complex

pairs of different conjugate

two

roots,therefore

Ax+ B Dx+E
+ 1)(x2 +4) = x2 + + x2+4
1

(x2

')

hence)
1 == (Ax

+ B)(x2 + 4) + (Dx+ E) (x2 + 1).

Hereit is convenient to apply the method of part icular values for


determining the coefficients, since the complexroots of the deno-

minator (x == + i and x == + 2i) are sufficiently


Putt ing x == we obta in

i,

whence

=0, B= .

whence D ==0, E ==

38+3Ai == 1,

-3'
\037

x=2i,we obtain -3E-6Di=


I,

Putting

Thus,

dx

= 3S x2dX+

3'S x2 +4 =
dX

S (x2 + 1)(x2+4)

= 3arc tan x-6 arctan x + c.


I

simple.

\"2

(x + 1)dx
(x2

'
/
5.1.8.
S (x2+x+2) +4x+5)
3 + Ilx2 + 12x+8
5.1.9.I _ J x 4(x+4x
2 2x-t-3)2
+
(x+ l) dx.
\\

Solution. Herewe already have multiple


the fraction into partial fractions:

complexroots.Expand

+llx2 +12x+8 2Ax+B + Dx+E + F '


+2x+3)2(x+l)(x +2x+3)2 x2+2x+3 x+l

x 4 +4x3
(x2

- 1;

Find the coefficients:


A = 1; B ==

Hence,

+24x + 11 + 12x+8 dx =
/ = 5 X4 (x
+2x+3)2(x+I)
3

D == 0; E = 0; F == 1.)

x2

x-I

dx

5 (x2+2x+3)2dx+5 x+l =In x+ 11-f-/t.)))


\\

5.2.Integration

\037

Calculate/1=

S (X2+x;:\0373)2

1 ==

-\037-

t. Then we obtain
t

\037

/1

(t2

=..\037

-+

The integra
is

dt

2)2

r
J

(t

+2)2 dt

I)

219)

Expressions

dx.

Since x +2x+3=(x+
1)2+2,let
2

of Certain Irrational

substitution

us make the
dt

S (/2 +2)2

2/2,

2 (12 2)

tIt
tit
dt

tie

/2 = S (t2 + 2)2

calculated by the reduction formula (see Problem 4.4.1):


J2 =

Thus

T /2 +2 + T J /2 +2 = 4\" (2+2 + 4 Y2 arctan y 2 + c.


1

/1=

x, we obtain
I

2 (x2

+ 2x+3)

x+l

2Y2arctanY2 +C.
I

2 (x2 +2x+3) 2 Y2

arc tan

x+1-+C.
y2

obtain

finally

J= S

1
/1=-2(/2+2)
2(/2+2)

Returning to

We

dt

4 +4X3 +IIX2
2

(x

+12X+8

+2x+3)2 (x+1) dx =
x+2
=In x+l 2(X2+2X+3)
I

Find the following

-2Y2

arctanx+ I-+ C
Y2

\302\267

integrals:

+9x32 -22x-8dx

5.1.10 S x 4x
5.1.11.
(X+3)3
S(x+I)
5.1.12.
S (X2-4X+4:\037X2_4X+5)
5.1.13.
S (I +x)(I
(I +x3)
dx.)
5.1 14. S (x +x3+3
1) (x2 + 1)
5X3

\302\267

\302\267

\302\267

(X\037\037)2

\302\267

\302\267

\037:2)

\302\267

\037

5.2.Integrationof CertainIrrationalExpressions

Certain types of integrals of algebraicirrational expressionscan


be reduced to integralsof rational functions by an appropriatechange
of the variable.Such transformation of an integral is calledits rationalization.)))

Ch. V. BasicClassesof

220)

I. If

Functions)

Integrable

the integrand is a rational function of fractional powersof

...,;,:

i.

an independent variable x,
e. the function R (x, ;;\"
),
then the integral can be rationalized by the substitution x== t m ,
where m is the least common multiple of the numbers qt, q2'
qk'
II. If the integrand is a rational function of x and fractionaJ
powers of a linear fractional function of the form ;:t\037 then rationalization of the integral is effected by the substitution
= t m , where m has the same senseas above.
\302\267

\302\267

\"

\037;t\037

5.2.1.1=r x+V x2 +

dx.
J x(I+V\037)
V\037

Solution.The least common multiple of the numbers 3 and 6 is


therefore we make the substitution:
x ==t 6,

whence

===

t 4 + t)
6 (16t+
6
t

tl>

(l -t- 2)

dt

dx 6t dt
I>

==.;

,)

+/3+2 dt =
+t
3
= 6 5 t dt + 6 5

=6

tl>

t2\037 I

Returning to

6,

t4
\037

+ 6 arctant + C.

x, we obtain

1=; x3+6arctanVx+C.
2

5.2.2.1= yx+

i v--v-dx.
V\037

x7

xl>

5.2.3.J ==

(2x-3) dx .
(2x-3)3 +
\037

\037

Solution.The integrand is a rational function


6

refore we

put

2x-3= t , whence)

dx = 3tl> dt;)

Hence,
J

=5

t2\037

1dt

=3

5 (t6

(2x-3)2= t 3 (2x
;)

-+t4

t2

1)dt -I-3 5 I
7

of

V2x-3,the-

3) 3 ==t 2

t2

.)

t
t3
-35+33-3t+3
=37
arc tan t +C.)))
tl>

\037

\037

5.2.Integration

Returning to

of Certain Irrational

221)

Expressions

x, we get

-5(2x-3) 3(2x-3) -

2.

1==3I\"7I (2x-3) 6

\037

\037I
6

Ii-+ arc tan


+ C.
-(2x-3)
(2x-3f\037]

5.2.4.
I=

\302\267

Sx ( 2+ V\037)

-;,

2
2-x
1= S (2 -X)2
5.2.5.
V 2+x dx.
\037

Solution.The integrand is
pression V ;+

a rational function of

x= 2-2t
l+t3
3

Hence)

\302\267

ex-

--x

;)

2-x= t 3

2+x

4(3
x == I +
(3

')

-+

2
dx == (1 12t
dt.)
3
t )2

;)

3 (.12t 2
3
3 dt
)2
1==-S 2 (I16t+6 t(I+t
+ C.
3
)2 df=-2S 'j3=4t 2

Returning to

5.26. I

the

therefore let us introduce the substitution)

V 2+x==t;)
whence)

x and

x, we

-S V 4

(x

get
2

==\037

dx

1)3(x

2+X
V( 2-x) +c.

+2)5

\302\267

Solution.Since

=(x-l)(x+2)V
V (x-l)3(x+2)\037
the integrand is a rational function of x and
us introduce the substitution:

:\037\037

V-:+

\037;

x- = t 4
V X+2
x- -_ t., x+2
I

whence)

+2 ;
x= tt 44_1

3
x-l=t4_1
;

dx =

12t:i

(/4-1)2

dt.)))

')

x+2=t 43t_1;

therefore let

Ch.

222)

V.

BasicClassesof

Hence,)

12t dt
I) (/4-1)
=
1== 5 (144t (t 4 1)2
3.3t

Returning to

dx
5.2.7.
_____
j (I-x)VI-x2 '
/1

\037

\037

5 t2

= 3t + c
\037

\302\267

x, we obtain
X- l

5.2.8.

Functions)

Integrable

1=3V x+2+C.

dx

V (x+ 1)2(x -1)4

\\
\037

\302\267

-./1+xdx
5.2.9.
S (x-2) JI l_x

.)

* 5.3.Euler'sSubstitutions
caIculatedwith
Integralsof the form \037R(x.Vax2+bx+c)dxare
the aid of one of t he three Euler substitutions:
(1)V ax2 -1--bx + c == t + x V a
(2) V ax + bx + c= tx + V c
2

if
if

> 0;
c > 0;
a

Vax +bx+c=(x-a)tif
ax2 + bx + c = a (x-a)(xi.e.if a is a real root of the trinomial ax2 + bx + c.
(3)

\037),

5.3.1.
I = + V xdx2 +2x +2
s

\302\267

Solution.Herea =

> 0, therefore we make the


Vx2 + 2x-t-2 =

t-x.

substitution

Squaringboth sides of this equality and reducingthe similar terms,

we

get)

2
-2,)
2x+2tx=t

whence)

t +2/+2
x = 2t(l -2
+t) ; dx = 2 (1+ t) 2 dt;
2

+ Vx2 + 9_x

_'._

2 ._ 1 + t

_ 2t(I2-+2t) _ t 22+(I4/++t)4

Substitutinginto the integral,we obtain

2t
dt
2 (I + t) (t2 +2/+ 2)
dt = (t2 + + 2)
1= 5 (t2+
4t +4) 2 (1+t)2
5 (1+1)(t +2)2

.)))

\302\267

5.3.Euler'sSubstitutions)

\037

obtained proper rational fraction into par-

Now let us expand the


tial fractions:)

+2t+2
+ I) (t +2)2
t2

(t

Applying

B ==0, D ==

Hence,
t2

-2.

223)

B
D
A
+ + I +2 + (1+2)2

\302\267

the method of undetermined coefficients we find:

+2t -f-2

dt

dt

2
5 (t+I)(t-i-2)2dt 5 t+i 5 (t+2)2 In
Returning to x, we get
x2+2x+2)+x +2
1=ln(x+l+V
==

dx
/ ==
5.3.2.
.
5 x+ y x 2 --x-r-

lt+l/+

A ==

I,

x 2 + 2x+ 2

+2 +C.

+C.

Solution.Since here c == 1 > 0, we can apply the second Euler

substitution)

V x2
whence)

-x+1

==

tx-I,)

2t-l) x==( -I) x .,

dx= - (t2

t2

[2

t _L

-I'r\037

dt;

/, we obtain

Substituting into

x= 2tt 2

--I.'
i

t
x+ Vx2 -x+1 == 1-

an integral of a rational

-1

By the method of undetermined coefficients we

Hence
dt
dt
/=2 5 7-\"2
t_I
5

find

133

=
5 (t+I)2-'25 t+l
1-2 - + t+ -2
3

dt

== 2 In J t

where

\302\267

3
B=-2; D==-3; E=-2-)
1

A:==2;

Yx 2

fraction:

-2t2+2t-2

--x + = s t (t I) (t + I)2 dt ,
5
-2t +2t-2= A + B + D + E
t (t
I) (t + 1)2 T t
(t + 1)2 t +
dx

x -1-Yx 2
2

\302\267

1n t
j

-x+
+ .
x
I

dx
5.3.3.
/=
.
S (l+x) Yl+x-x
2)))

dt

1J

In

+ I1-t--C,

Ch.

224)

V.

BasicClastesof I ntegrable

Functions)

x dx
.
5.3 4 I ==S ( ..r
10_X2)3
7xr
\302\267

\302\267

Solution.In this casea < 0 and c < 0 therefore neither the

first,

But the quadratic


second Euler substitution is app1icable.
2 has real roots a==
trinomial
2, == 5, therefore we
use the third Euler substit ution:

nor the

7x-lO-x

\037

V7x-lO-x2 == V(x-2)(5-x)==(x-2)t.)
Whence)

5-x==(x-2)t 2;
5+2t22 ; dx = 6t dt 2 ;
X=
+t
(1-r-t 2)

2t
(x-2)t=( 5+
l+t2

Hence

I=
where

2(2
dt

-_ Y7x-IO-x
x- '
2\037

5\0372

2 t==

3t
')

:-

l+f\037

s + 2 ) dt = C\037

\037

+ 2t ) + c,

Calculate the following integrals with the aid of one of the Euler
substit ut ions:

5.3.5.
r X\037:2X+4
S x-..r

\302\267

5.3.6.
r 1\037:2-1
S ..r
5.3.7.
r
S ..r
\302\267

\302\267

(2X\037X2)3

5.3.8.(x+YY ++X2)lD
dx.
x
I

t-

\037

\\

2)

5.4.Other Methodsof Integrating


IrrationalExpressions
The Euler substitutions often lead to rather cumbersomecalcu-

lations, therefore they should be applied only when it is difficult


to find another method for calculatinga given integral. For (alculating many integrals of the form
R(x, V'ax2 +bx+c)dx,
\037

simpler methods are used.

I. Integralsof

the form

I=

M x -t- N

S Yax2 +bx+c)))

dx

5.4.Methods

\037

I rrational

of Integrating

are reduced by the substitution x +

I ---M 1

t dt
Yat 2+K

:a= t

225)

Expressions

to the form

+ N 1S Vat dt2+K'

where Ml' N 1, K are new coefficients.


The first integral is reduced to the integral of a power function,
while the second, being a tabular one, is reduced to a logarithm
0, K 0).
(for a > 0) or to an arc sine (for a
I I. Integralsof the form

<

p m (x)

>

dX,

Vax2+bx+c

where Pm (x) is a polynomial of degree m, are calculatedby the


reduction formula:
Pm

=P _ 1 (x)Vax2 +bx+c+KrVax 2 dx
J
+bx+c ,(1)

(x)dx

S JI ax2+bx+c

m-l,

and K is some
where Pm -1(x) is a polynomial of degree
constant number.
The coefficients of the polynomial Pm -1(x) and the constant
number K are determined by the method of undetermined coefficient s.
III. Integralsof the form)

+bx +c

S(X-al)m

v::\037

are reduced to the precedingtype

by

the substitution

1
x-a1=T')
IV. For trigonometric and

hyperbolic substitutions see

\037

5.7.

(x+3)

s y 4x2 + 4x-3.

5.4.t. I ==

dx

Solution.Make the substitution 2x+1 = t, whence


t

---I

x=\037,
Hence,)

I= 4

(t

\037

SY

Returning t o

+5) dt =

-+

4 V [2

\037

r\037---4

dX=2dt.)

\037

In

-+

+ V[2

41

C.

x, we get

I =\037 V4x2 +4x-3+ Inj2x+1 + V4x2 +4x-31+c.)))


\037

Ch,

226)

BasicClassesot

V.

F uncttons)

Integrable

4
5.4.2.1= 5 V x25x+
dx.
2x
+ +5
5.4.3./ = 5 Vx3-x-1
dx.
x 2 +2x+
2

-x-l.
Hence,
=
D.

3
Solution.HerePm(x)==x

Pm
VVe

-1(x)

AX2

+ Bx+

seek the integral in th e form


I == (Ax 2 -1-Bx+ D)Vx2 -t- 2x + 2 + K

dx

5 V x2+2x+2

Differentiat ing this equality, we obtain

-x-1 =
/'= Yx2+2x+2
x3

= (2Ax+B) V x2+2x+2+(Ax2
+Bx+D)

X+ I
V x 2 2x 2

+ + +
K
+ Yx 2 +2x+
2

Reduce to a common denominator and equate the numerators


1 == (2Ax + B) (x2 -t- 2x + 2) + (Ax 2 + Bx-t- D) (x -1-1)+ K.
x3
Equating the coefficients at equal powers of x, we get the following
system of equations:
2A +A == I,
B +4A +B+ A ==0;

-x-

2B+4A+D+B==-I; 2B+D+K==-I.

Solving the system, we obtain


A

Thus,)

/=

(f

1
5
=3\"; B = --6-; D=6';

x2

\037

x+

\037

x2 + 2x+ 2 +

)V

K =2')

;5V

X2

+d;x+2

where)

=
/1= Jr V x2+2x+
2
dx

dx

= In (x + I + V x2 + 2x+ 2) + C.

5 V (x+1)2+1
5.4.4./ = S V4X2-4x+ 3 dx.

Solution.Transform the integral to the

for m

2
1=5 y .-4X2_4x+3
-4x+3+K
4x2-4x+3dx=(Ax+B)V-4x

dx

5 Y4x2-4x+3

\302\267)))

5.4. Methods

\037

the method of

- -{ V -

Applying

I= ( 2 x
1

= +
(

227)

Expressions

u ndetermined coefficients\037
dx

4x + 3 +

4x2

Irrational

of Integrating

S y.(2x-- + 2

we get

l)\037

x-{) V 4x -4x+3++
2

(2x-1-I-

In

4x2

-4x+3)-I-C.

5.4.5.S y9X3_3x2+2
dx.
3x2 -2x+
5.4.6.Vx2 -I-x-l1 dx.
I

\037

5.4.7.I==

(X+\037dX
S (x-I)(X+2)2
x2+x+1

Solution.Represent the given integral as follows:

x+4

(x+4)dx

S (x-I)(x+2r\037V x\037+x+l
Expand the fraction

(x-

x+4

dx

==,f (x-l)(X+2)2 V x2+x+l

\037t\037

\302\267

\302\267

into partial fractions

2)2

(x-I)(x+2)2x-I +(X+2)2 + x+2


Find the coefficients)
A

Hence,

-5 .
-9')

B=

-3; D= -g')
2

.
I--S [ (x-I)-3(x+2)2-9(x+2)
-Vx2+x+l5

'
=9
5

dx

dx

j (x-I)V X2+X+ -\"3


I

\\

tJ

dx

(x+2)2Yx2+X--1-1
5

dx

--9S (x+2)Y
the substitution x --I= +, the

\302\267

x\037+x+

The

first

integral is calculatedby

second and the third by the substitution x-l-2 = +.


We leave the solution to the reader.

Ilx-6 dx.
x3-6x2+
5.4.8.
S Yx 2 .+4x+3
dx.
5.4.9.S 3X;tSX2_7X+9
2x2 + 5x-r-7
5.4.10.
S (x+1)6..r +2x)))
\302\267

\037XX2

Ch.

228)

V.

BasicClassesof

5.4.t 1.5(X2-3X+2\037 ...r2 4x+3


r
.
5.4.12.
::+3X+2
J (x+1)3)1
I) dx
.
5.4.13.
5 x VI +3x2 +X
:XX

(X2

Functions)

Integrable

\302\267

4)

\037

5.5.Integrationof a BinomialDifferential

xm (a + bxn)p dx, wh ere m, n, p are


bers, is expressed
through elementary functions only
ing three cases:
The integral

\037

rational numthe follow-

in

Case I. p is an integer.Then,

if p > 0,the integrand is expanded


the
Newton
the
of
formula
binomial; but if p < 0, then we
by
k
put x==t , where k is the common denominator of the fractions m
and n.
r
is an integer. We put a+bxn==t
Case II. m+1
\\ where ex is
n
the denominator of the fraction p.
I
Case III. m +
+ p is an integer.We put a + bxn == ta.x n , where (1,
n

is the denominator of the fraction

p.

x
5.5.1.
I=\037 V {2+V)2dx.
X

Solution. 1= x+ (2+X-}rdx.Herep = 2,

i.e.an integer; hence,

\037

we have Case I.)

4
5
1=S x3( x+4x2+4) dx= S ( x3+4x6+4x3) dx=
3
= 3 x2x + 3x 3 + c.)
+ 24!.!.
l

\037

()

IT

7\"

5.5.2.1=S x-+(l+X\037f1dx.
5.5.3.I = r-v 1+Vx dx.
J

VXi

Solution. 1= S x
2

\037

(1+ x+ )+dx.

m=-3;n=3\";P=2;

Here

- - ;+ ) =1, i.e.

m+ 1 _ (
n

3)

integer.)))

an

5.5.Integration

\037

We

of

Binomial

229)

Differential)

have Case II. Let us make the substitution)

-2

+x3 = t 2
1

3'x 3 dx = 2t dt.)
I

;)

Hence,)

1= 6 S 12 dt = 2t 3 + C = 2 (1+ x-}) + C.

5.5.4.
1=SX+(2+X

\037

)+dX.

5.5.5.1= S (l+x2 ) dx.


5.5.6.1= Sx-u (l+x4 )-+dx.
\037

x&

. Here p ==
SaIu t ton.

a fraction,

but

Case III. We

-2I.IS

t.

--} -

a f rac lon,

m+ 1 == -11+ 1 =
4

5
\"2

a ]so

= 3 is an integer,i.e.we have
+p=
4
2
4
== x t . Hence
put 1 + x
t dt
x=
; dx==
5
2(t2-1)4
(t2-1)4
I

m\037-

\037

\302\267

Substituting these expressionsinto the integral,we obtain


C2\0371 )-+
1=-+S(t2-1)\302\245

(t 2

ldly\"'

-1)
4

2
= -\037
2 S (t

-1)2dt =

Returning to x, we get

\037+\037_1.+C.
10 3
2)

1=- 10:
V(1+X4)3_ 1/1+x4 +C.
10 V(1+ x4)&+
f'
+ Vx
3\0376

5.5.7. }/y_x
I

dx.

\\
t.I

5.5.8.
Sx

(I+dVx )'
5.5.9.S x3 (1+ x2 dx.
\302\267

f2\"

v-+ .

r
5.5.10.,
J
x4

dx
1

x\037)))

2\0372

Ch.

230)

BasicClassesof

V.

Functions)

Integrable

5.5.11.Vx V I + V x'dx.)
\037

dx

5.5.12.)
\\

')

Jx3VI+
\037

\037

and Hyperbolic
5.6.Integration
of Trigonometric

Functions
I. Integralsof

the form

I=
\037

n
sinm xcos
xdx,

where m and n are rational numbers, are reduced to the integral


of the binomial differential)

n-l

I=

tm
\037

(l-t)z-dt,
2

= sin x

and are, therefore, integrated in elementary functions only


following three cases:

(l)

(2) m

is odd
is odd

an

( mt

the

integer).

an
m

in

integer),

-t- I

fl-

(3) m + n is even ( \037+ \037 an integer )


If n is an odd number, the substitution sin x = t is applied.
If m is an odd number, the substitution cosx = t is applied.
the sum m + n is an even number, use the substitution
If
tanx=t(or cotx=t).
In particular,this kind of substitution is convenient for integrals
of the form)
tan n xdx (or cot n xdx) ,
\037

\037

where n is a positive integer.But the last substitution is inconvenient if both m and n are positive numbers. If m and n are nonnegative even numbers, then it appears more convenient to use the
method of reducing the power with the aid of trigonometric transformations:

cos2 x = + (I + cas2x), sin2 x =

or

.
.
SJnxcosx
=2'sln2x.
1

5.6.1.1=S

sin3 x
cos.2

dx.
X)))

\037

(I-cas2x)

\037

5.6.Integration

Solution. Here m==3

sin xdx==

217

of Trigonom. and Hyperbolic

odd number.

is an

dt, which gives

Funct's

We

231)

cosx==t,

put

1=-S(l-t)t-T dt=-3{3+ ; t T +c:::::;


2

= 3 V casx + cos2
(
dx.

x-1) + C.

5.6.2.
I= 5
5.6.3.
I = sin4 xcas6 xdx.
:\037\037:\037

\037

Solution.Hereboth

m and n are positive even numbers. Let


use the method of reducing the power:

UIS

I = /6 5 (2 sin xcasX)4 cas2 xdx = 32 S sin 4 2x(1 + cas2x) dx = 11+ 12,


1

calculatedby

The secondof the obtained integrals is

tion:

I2
We

sin 2x= t, cas2xdx=


I

32 S

. 4 2x cos2xdx
sin

again apply to the

-5 \037

dt,

5
\037
320sin 2x +
integral the method of reducingthe

64

first

\037

the substitu-

t 4 dt

320+ C
\037

\302\267

power:

dx=
11= 3'2 5 sin4 2x dx = 5 (l-cas4x)2
= (x-+sin4X) + S (1+cas8x)dx=
= x-2;6sin 4x+ 2d48sin 8x+ C.)
And so,finally,
. 5 2x+ 0
3
I == 256
x 256sin 4x + 2048sin 8x + 320Sin
1\0378

2\0376

1\0378

I.
2\0376

- I.

\302\267)

5 .6.4.I == cos6 dx.


5
Solution.Hereboth m and n
is negative.Therefore, we put
sin2

x)

tanx=t.
, cos2 x
I

are even numbers, but

one of them

.
- + 'COS2
=dt.
1

dx

t2

X)

Hence,

1= t2(I+t2)dt=+ +C=
\037

t\037

t\037

ta\0373x

ta\0375x

+C.)))

Ch.

232)

565
\302\267

\302\267

BasicClassesof

V.

Functions)

Integrable

I == 5 C?S4
x.
SIn 2 x d
X

\302\267

Solution.Herewe can put cot x== t'l


by expansion:

dx=
J==5 (l-.si\0372X)2
SIn x

5( .Ie

SIn x

2 +Sin2 x

is simplerto integrate

it

but

) dx===

= -cotx-2x+
dx=
-}S (l-cos2x)
=_
(cotx+

+ 3;)+ C.)

Si\0372X

5.6.6.1=5
5 6.7
e

/=

co\037:

'
dx

Sv

sin ll x

cosx

Solution.Hereboth exponents(
num b

\342\202\254rs

fore we

an d

th

put)

S cos4 x V tan!!x

=S

(t + t

\037

\037

cos?

+ t 2 dt

===

S V t ll

13

5.6.8.Find the

\037
=dt.
x

tanx== t;

J=

Ii and
) are negative
4 IS an even num ber, th ere-

-3-3=11

etr sum

dx

) dt =

- - - ; - + c=
t

\037

\037

\037

__ 83(l+4ta\037+C.

integrals of tan x and cot x.

Solution.)

tan 2 x Vtan2

sin x
xdx 5 cos
dx == In cosx 1-1-c;
x
x =
cot x dx = C?S
dx In sin x + C.
SIn
x
S
5
5.6.9.I == ,.tan? xdx.
Solution.We put tanx==t,x==arctant;dX= dt 2 '
1+t
=
=
t
J = t' I
t3+ t
dt
[2 5 (
I
[2)
5
= \037-\037+\037-J-.ln
=
6
4
2
2 ( 1 + t2 ) +c

tan

===

x)

\037

\037

==

We

get

\037

6\"

tan 6

x-Ttan

I
2
x+\"2tan x+ In cosx1+
I

c.)))

\037

5.6.Integration

of Trigonom. and Hyperbolic

5.6.to. (a) I = cot6 dx;


5.6.11.
. x dx.
/ = S COS4
sin 3 x

\037

raised to an odd power. Let us


sin x dx == dt.
casx = t,

We

is

obtain an integral of a rational

I == S

COS

4 x sin
sin 4 x

233)

(b) I = tan 3 xdx.)

\037

Solution.Heresin x

Funct's

dx ==

put

function.)
t4

S ( I _ t 2)2

dt.

Here,it is simpler to integrate by parts than to use the general


methods of integration of rational functions (cf. Problem 4.4.1
Let us put)

(b).

t dt
dv ==
2
)2

u == t 3

(l-t

;)

\302\267)

Then)

du=3t2 dt;

V=

')

2(1\037t3)

Hence,)

1=-2(I-t +2 S l-t =
t3

t2 dt

=-2(I-t2)+2\"S - + dt=
2

==

t3

t2

l-t\037

t3

3
3
+ +c=
-2t+Tln
I-t

-dx.
5.6.12./=
S

- -- + - -+
I

==
sin4 x
cosx

2(l_t2)

cos3 x

2 sin 2 x

3
2

3
4 In

cosx

cosx
cosx I

+C

\302\267)

I I. Integrals of the form


R (sin x, !.:Os
x) dx where R is a rational
function of sin x and casx are transformed into integrals of a rational
function by the substitution:
\037

tan(
This is

; )=t (-n<x<n).

so-calleduniversal substitution. In this

. x = 2t 2
sIn
I
t
+

casx = II-t
+

2
2
t

;)))

x = 2 arc tan

t;)

dX=

2dt

1+t2

')

case)

Ch.

234)

V.

BasicClassesof

Integrable

Functions)

=t

Sometimesinstead of the substitution tan

it

is more advan-

\037

tageous to make the substitution cot = t (0 < x < 2Jt).


Universalsubstitution often leadsto very cumbersomecalculations.
Indicatedbelow are the caseswhen the aim can be achieved with
the aid of simpler substitutions:
(a) if the equality
R(-sinx,cosx)==-R(sinx,cosx))
\037

or)

== -R(sinx,
R(sinx,-cosx)
cosx)

is satisfied, then

casx = t
(b)

if

it is more advantageous to apply the substitution


to the former equality, and sin x = t to the latter;
the equality

== R(sinx,cosx)
R(-sinx,-cosx)

is fulfilled, then a better effect is gained by substituting tan x ==t


or cot x = t.
The latter
form
\037

caseis encountered,for example,in integrals of


R (tan x)dx.

5.6.t 3.1= 5 sin x (2+C::x-2sin x)


Solution.Let

us

tan

put

\037

= t;

the

\302\267

then we have

t 2) dt
+t 2
= JC t(t(I2+
'
4t
-4t+3)
\037_ 2
2 2+ + t2
l+t (
l+t )

1=

2t

\037

Expand into simple fractions

l+t2

A
=
t-3 t-l
t(t-3)(t-l) t +\037+\037.

Find the coefficients


5

A=T; 8==3\"; D==-l.)


I

Hence
1=\037
3

dt

5t

+\037
3

\037-3 5 \037=
t-l

5t

=3'ln t 1+3 In
I

t-31-ln
It-II+ c =

={-lnltan 1+ Inltan -31-1nltan-ll+C.)))


\037

\037

\037

.. \037

5.6.Integration

of Trigonom. and Hyperbolic

1 5 5 + sin xdx+3 cosx dx


5.6.15.I = S sin. x (2 cos
2 x-I) '

Funct's

235)

5 6 14

\302\267

Solution. If

x-

the expression sin x cos2


we substitute
(2
1)
its
sin
fraction
then
will
x for x,
the
change sign. Hence,we
take advantage of the substitution t = cosx; dt = sin xdx. This
in

-sin

gi yes)

I = j (l-f\037)dt(2t2
Since)

\037

I)

\302\267)

2)
2
_ (2-2/2)-(1-2t
2
2
(I-t ) (I-2t) (l-t) (1-2t) 1-2t'2 1-/2'
I

then

dt
dt
r I-t
r I-2t
2
2=

1=2J

-J

+ +c=
Tln
I-t
l_tY2
1

In

Y2

+ t V2

= \037 In + V!cosx + 2 In +cosxx + c=


V2
V 2 cosx
= \037 In + cosx + In tan
I

\037

CQS

V\037

I-V2cosx

V2

+ C.

\037
2)

x cosx
5.6.16.
1= 5 sin x+
cosx dx.

\037in2

Solution.Sincethe integrand does not change sign when sin x


cosx do changetheir signs, we take advantage of the substi-

and

tution)
t

= tan x;)

dx
2 X)

\302\267

COS

Hence,)

dt

/2 dt
x.costSx dx
I 5 tan2
(tan x+ I) cos2 x
5 (/+ 1) (/2+ 1)2

\302\267

Expand into part ial fractions


12
(t -1-I) (t

\037

A
Bt+D Et+F
+ I)2 = / + + t 2+ + (t 2 + 1)2
I

\302\267

Find the coefficients


1

A=T;

8=-4; D=T;
I

Hence,)

1=45 t+I-45
I

dt

-+1

t
t2

dt

\302\243=2';

F=-2I

t
2

+25 (t +1)2dt
1

;)))

Ch.

236)

BasicClassesof

V.

Functions)

Integrable

1+t _
+t
I = ..!..
4 In V + t 2 4
+t 2 + C =
= {-Insin x+ cosxI-{-cosx
C.
(sinx+ cosx)+
56171= 5 sin 2tanx+3
2 x+2 cos2 x dx.
1

\037.

...

Solution.Dividing the numerator and denominator

substituting tan x = t;

cos x
d\037

3
1=5 sin 2 x+x+
2 cos2 x dx= j
2 tan

\037

==dt,

(2 tan

sin x
sIn x

+.

and

x+3) dx x
=
tan 2 x+2
cos\037

5.6.18.I = 5

cos2 x

we obtain

+3 dt = In (t 2 + 2) +
= (' 2t[2+2
J

by

arc tan

+c=

Y2
Y2
3
2
I n (t an x+ 2) +
arctantan x + C
Y2
Y2

\302\267)

dx.

Solution.This integral,of course,can be evaluated

with

the aid

; = t,

but
it is easierto get the
substitution tan
desired result by resorting to the following transformation of the

of the universal

integrand:
sin x

+ sin x

_ sinx(l-sinx)
-_ +sinx(l-sinx)
(1- =
(I

sin x)

sin x)
sin x
sin 2 x
cos2 X cos2 X

cos2 X

sin x

cos2 X

an 2 x.)

Whence
sin x
2
I == j cos
2 x dx-5 sec xdx+5 dx= cosx -tanx+x+C.
r)

5.6.19.1=
5 cos4 x sIn. 2 x dx.
1

Solution.Here the substitution tan x == t can be applied, but it


is simpler to transform the integrand.Replacing,in the numerator,
unity
by the trigonometric identity raised to the secondpower, we
get)

cos
. 2x
I == 5 (sin2cosx+
4 x sIn
2

==

sin2

S cos4 x

dx -1-2

X)2

dX ==

dx

sin4

5
dx

5 cos2 x + 5 sin. 2 x
I

=\"3tan

3X

2
cos4 x
x+2 sin 42 x cos
. .) x+
CLS

==

sln\037

dX

dx
2
5 tan x cos-.)x + 2 tan x

+ 2 tan x-cc>tx+

C.)))

===

cot x ==

\037

5.7.Integrating

Irrational

Functions

Substitutions

by

237)

I. Integration

of hyperbolicfunctions.Funct ions rationally depending on hyperbolic functions are integrated in the same way as
I

trigonometric f unct ions.


Keep in mind the following basic formulas:
2 x=
cosh2 x-sinh

1; sinh 2 x=

\037

-1);

(cosh2x

cosh2 x = -}(cosh2x+1); sinh xcoshx = + sinh 2x.)


If

tanh

===

\"2

t, then sinh

x = _2tt'l
1

x=2Artanht=ln(:+\037)
5.6.20.I = cosh2 X dx.

+t 22 .'
.' coshx = 1t
I

(-I<t<1); dX=

1\037tt2

'

\037

Solution.

I = 5 -}(cosh
2x+1) dx=

5.6.21.
I=

sinh
\037

2x+-}x+C.)

cosh3 X dx.
Solution.Sincecoshx is raised to an odd power, we
\037

coshxdx===dt. We

obtain

put

sinhx==t;

I = 5 cosh2 x coshx dx = 5 (1+ t 2) dt = t + t:+ C =


= sinh x+ + sinh 3 x+ C.

5.6.22.
Find the
(a)

5 sinh2 x

cosh2

integrals:

x dx; (b) 5 sIn.

h;;
x

cosh x)

\302\267

with the
* 5.7.Integrationof CertainIrrationalFunctions
or Hyperbolic
Aid of Trigonometric
Substitutions
Integrationof functions rat ionally dependingon x and Vax2 + bx + c
can be reduced to

finding

integrals of one of the following

I. R (t,Vp2t 2 + q2) dt;


II. R (t, Vp2t 2 _q2) dt;
III. J R (t, V q2_p2t 2)
\037

\037

dt,)))

forms:)

Ch.

238)

where

BasicClassesof

V.

Functions)

Integrable

= x+ :a; ax2 +bx+c=+ p2t 2 ::f:q2 (singlingout

a perfect

square).
Integrals of the forms I to I II can be reduced to integrals of
expressionsrational with respect to sine or cosine(ordinary or hyperbolic)by means of the following substitutions:

I. t = !L
tan z
p
I I. t = !Lsec2
p
I I I . t = !Lsin z
p

= !Lsinh z.

or

or

t = .!Lcoshz.
p

t == .!Ltanh

or

z.

5.7.1.
1=..'Y (5+dx2x+ 2)3 .
\\

2
Solution. 5+2x+x
==4+(x+ 1)2. Let

IS

us

dx

= 2 tan Z',

dt

x+ 1 ==t. Then

JI(5+2x+X2)3 S (4+t 2)3'

We have obtained an integral of the form


the substitution:
t

put

dt

I. Let

us introduce

cos2 Z

' V4 + t 2 ==2 VI + tan 2 z == cos2 z .

tanz

2dz

\302\267

We get
J

=
\037

S
==

cosz dz =)

-.
I

sIn z +c

== 1

__
+C_
I

+ tan 2 z

4)

+ C-

4
Y 1+t2

x+

4 Jl5+2x+x\037)

5.7.2.I==S(x+1)2V::+2x+2

+ C.

\302\267

Solution.x2 + 2x+2 = (x+1)2+ 1.


Let us

put

x+ 1 =t; then

I--S

dt
t2

J1t2

+1'

Again we have an integral of the form


t = sinh z. Then
dt

= coshz dz;

Vt

I. Make the

substitution)))

+ I = VI +sinh2 z = coshz.)

5.7.Integrating

\037

Irrational

Functions

by

Substitutions

239)

Hence,)

I==

sinh\037

z coshz

s sinh 2 z ==-cothz+)C==
sinh 2 z
:C
+C=- Yx2+2x+2
+ c= yT+t2
t
sinh
x+ 1 T')

CCJshzdz

dz

z)

5.7.3.I

x2 Vx2

=\037.

-1dx.

5.7.4.I=S Y\0372+1

5.7.5.I =

\037

dx.

V(x2 -1)3dx.

Solution.Perform the substitution:


x ==cosht;

dx ==sinh

t dt.)

Hence)
sinh t dt = sinh4 t dt =
I=
t-l)3
2
=S
dt =
(
)
= 5 cosh2 2t dt -\037 cosh2t dt + {-S dt =
S
= S (cosh4t+l)dt-+sinh2t++t=
\037V(COSh2
COSh

\037

\037t

\037

\037

32 sinh 4t

\037

_J..
4 sinh 2t +

8')

\037

+C

Let us ret urn to x:)

t ==Arcosh == In

(x + Vx2

- -1;
- -1).)
1

);

cosht = 2x V-x 2
sinh 4t ==2 sinh 2t cosh2t ==4x x2 1 (2x2
sinh 2t ==2 sinh

\037/

Hence

1=+x (2x2 -1)Vx2 -I-\037xJ/x2


5.7.6.I = Jr ( + x x . '
I

v-\037xV

-1+ ;

In

(x + V' x2

-x\037

Solution.We make the substitution:


x ==sin2 t;

dx -== 2 sin t cast

dt)))

-1)+C.

Ch.

240)

V.

BasicClassesof

Function\037)

Integrable

and get

/=

2 sin t cost dt
(1 sin t) JI sin 2 t

-sin

=2

1-

4t

sin t

S cos2 t

dt

2 dt

+ sin t

= 2 tan

_ cos2 t + c =
2(VX-l)+C.

I-x+C=
y\037

I-x
\037

y'

l-x

5.7.7.I = V3-2x-x2 dx.


dx
5.7 .8. 1= r
.
\037

J (x2-2x+5)+)

\037

5.8.Integrationof Other TranscendentalFunctions)


5.8.1.1=S X2x dx.
in

Solution.We integrateby parts, putting)


u = In x;)

dx
dv=-'
x2

du=-;
x

v=--;

,)

dx

/ =_

5.8.2.I = S \037
5.8.3.I ==S (l +eXdxe2X)2

In x

+ S dxx2 = _ InXx _\037x + c.

In

\302\267

\302\245l-x

\302\267

Solution.l.etus put: eX = t;

eX

dx =

dt. We

get:)

1= S ( +t2)2 .
dt

App

ly

the reduction formula (see Problem 4.4.


t):)
t
r
1=/2=2(t2+1)
+2'J
1

dt
l+t\037

eX
t
I = 2 (t2+
arctant + C = 2 (1+e2X)
+ 2arctanex + C.)
I) +2
I

5.8.4.I = e- In (\037+ 1)
X

\037

dx.)))

5.8.Integration

\037

Solution.We integrate

of Other Transcendental

241)

Functions

by parts:)

u=ln(eX+l);dv=e-xdx;
eX
du =
+ex dx; V =- e- ;
X

dx
e +l -eX
dx=
=-e-Xln(l+eX)+
1=-e-Xln(l+eX)+
l+ex
5
S l+e
x

-e-

==

aarctan

5.8.5.I = r e

..-

J (1+

586
\302\267

3
2
X2

In

(I + eX) + x-In(1+ eX) + C.)

dx.

r X arc tan x dx
J VI X2

\302\267

Solution.Integratingby parts, we
u

= arctan x;) dv=

du
J

get)

dx

x2)

xdx
;
1 +X2

v= VI +X 2

;)

= VI +X2arctanx- VI +x2
=
5
= +x2 arc tanx-ln(x + V x 2 + 1)+
1

V\037l

\037X2

C.)))

Ch.

242)

V.

BasicClassesof

Functions)

Integrable

5.9.Methodsof integration
(Listof BasicFormsof Integrals))
\037

No.)

Method of integration)

Integral)

[<p

(X)] <p'

Substitution

(x) dx

cp

(x) =

t)

\037

2)

(x) <p' (x) dx)

Integration

\037

parts

by

(x) dx = f (x) It' (x)

f (x) <p'
\037

This

<p

(x)

\037

t' (x)dx.

is applied, for example, to


form
p (x) f (x) dx, where
and
f (x) is one of the
pol ynomial,

method

\037

Is
is
p (x) a

of the

in tegra

\037

functions:

following

cosax; sin ax; In x;


arc tan x; arc sin x, etc.

e'Xx;

and also to integrals


nential function by
3)

Reduced

(x) q,(n) (x) dx)

\037

f<n)

to

integration
formula
by parts

(x) cp (x)

tegration

of products of an
sine.)

cosineor

by the

f (x) q;<n) (x) dx = f (x) <p(n


\037

expo-

of the product
for multiple
in-

-1)(x)-

- +
-+ ...
- ...(-1)n-lf<n-l)
f' (x)

2) (x)

cp(n

f\"

-I-

(x) cp<n -:J)(x)


(x) cp (x)

+(-I)n

f(n)

(x)

<p

(x) dx)

\037

e'xPn (x) dx,

4)
\037

where Pn (x) is a polynomial of degreen.)

Applying the formula for multiple


tion by parts (see above), we get

integra-

e'XPn (x) dx=


\037

e7.X

pn (x)

p\037

X)
ex2

+(_
5)

Mx+N dx,

5 x 2+px+q

p2-4q<

0)

P\037

x+E.=t

...+

(X)
a/Z + 1)

p\037n)

1)1l

Substitution)
2)))

(x)

a3

] +C

\037

5.9.Methods
J nt

No.)

P(x)

Method

1)

formula

Reduction

In =

is

used

2n

In +
(2n _ 2) (x2 + l)n -1 2n __ 2

1)

+
+
)2+'..+
8 +
. + (x-x +
+ + +
...)
+
+x 2+px+q+ (x2+pX+q)2+
kX + N k
+ .+(xM2+px+q)k)
=

...

..

(x-

+...

:....

f R (x.x .
.where
R is a rational

) dx.

function of its arguInents.)

..

+
+

ax b
dx,
ex
d)
[ (
\\vhere
R is a rational
r
J

x,

function

\037

+c

- ...,m)

Reduced to the integral


tion by the substitution

ax+b
cx+d)

of its arguments.)

Mx+N
r
J Yax 2+bx

+...

Reduced to the integral of a rational fraction by the substitution


x t k where k is a
common denominator of the fractions
n

10)

integration)

where

x-\302\243-

9)

of

243)

P(x)
is expressedin the form of a
Integrand
Q(x) sum of partial fractions
is a proper rational frac- P(x)
A2
Al
At
tion
Q(x) (X-Xl) (X-Xt
(x-x1)l
Q (x) = (X-X1)l
1
B2
8m
x 2)m
(x2 px
(X-X2) (X-X2)2+
2)m
q)k
M 1x+N 1
M 2x+N 2

S Q(x) dx,

8)

(List of BasicIntegrals)

Integration

In=S) (x2dx
+ I)n)

6)

7)

egr a

of

r)

s)

of

rational

frac-

tn

By the substitution

x)

is

reduced

to

the integral
x+2\037=t
sum of two integrals:
tdt

Mx+N dx=M1 r
r
J Vax 2 bx+c
J

Vat\037+m

+ N 1Jr

dt

at\037

-1-m

The first integral is reduced to the integral


of a po\\ver funct ion and the secondone is a
tabular integral.)))

244)

Ch.

No.)

integral)

II

V.

BasicClassesof I ntegrable
Method

(x.Vax 2 +bx +c)dx.

\037

\\vhere

function

R is a rational
of x and

Vax\037+bx+c

Functions)

of

integration)

to an integral of rational
substitutions:

Reduced

+bx+c=t va

:f: x
(a > 0),
Yax 2
Yax 2 +bx+c=tX:f:
(c > 0),
2
2<
Yax
0).
(X-Xl) (4ac-b
\\vhere Xl is the root of the trinomia I ax2-t\302\245c

+bx+c=t

+bx+c.
The indicated

integral
ted by the trigonometric
/
2

can also be eva luasubstitutions:

-4acsIn. t
Vb 2 -4accost
()
Yb

x+2a=<

2a

...a

(a < 0,

2<
4ac-b

-4acsect
x+2a= Yb 2 -4ac
cosect
(

Yb

2a

, Y4ac-b2
2a

b
2a)

(a > 0,

4ac-b2 < 0))


tan t

x+-=< Y4ac-b2 cott)


2a

Pn

(x)

dx,

Write

pol yno-

r
S .r

S Y ax2 + bx+c
where

Pn (x) is a

mial of degree n.)

0))

2a

12)

fraction

by the Euler

(a > 0,

4ac-b2 >

0))

the equality
Pn

ax\"!.

-l.

(x)dx

+bx+c =Qn-dx)Vax +bx+c+


dx
+k
S Yax2+bx+c,
2

of degree
where Qn-l (x) is a polynomial
Differentia ting both parts of this equan
2
bx+c, we
by Yax
lity and multiplying
get the identity
,
2
==
Pn

(x)

Qn-l (x) (ax +bx+c)+


1
Qn-l (x) (2ax+b)+k,

+2'

gives a system of n 1 linear equathe coefficients of the


tions for determining
Qn-l (x) and factor
pol ynomial

which

k.)))

\037

No.)

5.9.Methods

(List

of Integration

BasicIntegrals))

of

Met hod of

Integral)

n t\037grati

245)

on)

And the integral


dx

is

S y ax2 + bx +c

taken

by the method

consideredin No. 10

(M==O; N==I).)

13 r
J (X-xl)m

dx
2
\302\245ax

This integral

+bx+c sidered integral

is reduced to

above-con-

the

by the substitution
I

x-x1=Y)
xm (a

14)
\037

+bxn)P dx,

where m, n, p are rational


numbers (an integral of a
binomial differential).)

This integral is expressedthrough elementary functions only if one of the following


conditions is fulfilled:
(1) if p is an integer,

. m+

(2)

If

(3)

if

nl

1.

IS an In t eger,

+ + p is an
1

n)

integer.

1st case
if p is a positive
integer, remove the
brackets (a bxn)P according to the Newton
(a)

+calculatethe

binomial and

irHegrals of powers;
negative integer, then the
substitution
t k , where k is the common
denominator of the fractions m and n, leads
to the integral of a rational fraction;
2nd case

(b)

if

p is a

if

x=

m+ 1 is
n

a+

bx n

tion
denominator

an integer,

= tk

then

the substitu-

is applied, where

of the fraction

p;

is

the

3rd case

+!+pis an integer, then


n
tu tion a + bx ll ==xnt k is a pplied,
the denominator of the fraction
if

R (sin x,

15)

cosx) dx)

Universal

\037

If

then

substitution

tan

the substiwhere k

p.

is

===t.
\037

R(-sinx,cosx)=-R(sinx,
cosx),
the substitution
cosx = t is

- x=-

applied.

If
cosx) == R (sin x, cosx),
R (sin x,
then the substitution
sin
t is applied.
'
If
R
x,
x) ==R (sin x, cosx),
'then the substitution
tan x ==t is applied.)))

(-sin -cos

Ch.

246)

I ntegr

No.)

16)

R (sinh

V.

BasicClas5esof

Integrable

Funct ions)

Method

al)

x, coshx) dx)

The substitution

of

Integration)

2:= t

tanh

\037

case

this

sIn h

17)

sin

ax sin

sin

ax cosbx dx

bx dx

\037

\037

cosax cosbx dx)

is used.

I +t
cash x=l_t
2; d

2t

x=l_t2 ;

In

2dt

x=1_t2

')

Transform
the product of trigonometric
functions into a sum or difference, using one
of the following formulas:
sin ax sin bx

=2[cos(a-b)x-cos(a-t-b)
1

\037

x])

cosax cosbx =
I

=2\"[cos(a-b)x+cos(a+b)x]

sin

ax cosbx =

=i-[sin (a-b)x+sin(a+b)

xl)

sin m x cosn

18)
\037

where m and n
gers.)

x dx,
are inte-

If m is an odd positive number, then apply


cosx t.
the substitution
If n is an odd positive number,
apply the
sin
substitution
If m
n is an even negative number, apply
the substitution
tan x t.
If m and n are even non-negative
numbers,
use the formulas

x=t.

sin 2 x

l-cos2x ,

.)x = 1 +cos2x

cos'\"

2)

2)

sin P x

19)

cosqx dx

\037

< 1[/2),
q-rational num-

(0 < x
and
bers.)

Reduce
differentia!
sinP

by the substitution

of the binomial
sin x t

xcosq xdx=

(l-t)q-l

to

the

integral

\037

\037

tP

dt

(see No. 14)


.)

20)

R
\037

(eax ) dx)

into an integral of
Transform
function by the substitution eax

a
t)))

rational

Chapter

6
THE DEPINITE INTEGRAL)

* 6.1.Statement of the Problem.


The

Lowerand Upper IntegralSums

Let a funct ion f (x) be defined in t he closed i nterva 1


following is calledthe integral sum:)

[a,b].The

n-l
In==

where a ==Xo

The sum Sn ==

n-l

Sn == \037 mi

i=o

(\302\243i)

\037Xi'

0)

< Xt < X 2 < ..< Xn -1 < Xn ==b,)

-n-]

i ==Xi +1

\037X

\037

i=

\037Xi

Xi;

\037

i=o

Mi

\302\267

\302\243

E [Xi' Xi +1] (i ==

\037Xi

is

0, 1, .
\302\267

\302\267

,n

1).)

calledthe upper (integral)sum,

is called the lower (integral) sum, where M i =

= sup f (x) [mi == inf

f (x)] for xE [Xi' xi+l]'


The definite integral of the funct ion f (x) on the interva I
the limit of the integral sums

n-l

\037

f (x) dx= Hm.\037 f (s;)


L=O

If this limit
val [a,
Any

b].

and

\037Xi

when max

\037Xi

[a, b] is

1-+o.

function is calledintegrable on the intercontinuous function is integrable.

exists,the

6.1.1.
For the

integral)
1t
\037

sin x dx

o)

find

the upper and lower integral sums corresponding


to the division
closedinterval [0,n] into 3 and 6 equal subintervals.)))

of the

Ch.

248)

/. The Definite

Integral)

Solution.Divide the closed interval


the points:)
===

Xo

3
1t

0,)

Xl

===

[0,1t]

3'

21t

X 2 ===

')

into 3 equal

by

X 3 == n.)

sinx increasesmonotonically on the

The function

parts

interval

[0, ] ,
=

and therefore for this interval we have mo ==sin ==0, Mo


\302\260

= sin

.
IS
3'\"3
l
]
J1

\0373

\037

2n

The least value of the function on the interval

m 1 == SIn

= sin = 1. On

Y3 an
3= 2'
11

monotonically

S3=k=o M k 1Xk =

+ 1+

\037

subdividing the closedinterval

- \"6'
51t

')

V3

\037

.
pOints
X6

= 1t,

xo===O, x t

we

find

\0373

[0,1t]

=2'

2'

6 =2'
1

m 5 == sin 1t ==0,)

\037

2Jt

1t

6= 2 '
1t

M 1=Sln

2.86.

into 6 equal intervals

1t

\302\267

1t

= 1t(Y\037 + I)

M o == SIn
1t

V3 \0370.907,

by analogy:)

.
Sln(f=2'
.
Y3
m 2 ==
slnT
. 2n = Y3
m 3 ==slf1
T
.
51t
m 4 S10
===

Jt

==6' x2 ==a' x3 ==2' x4 ==3'


11

mo ==0,)
m 1 ==

decreases

Y3

2+0 =

Jt

i: !

by the

function sin x

M t ==

1t

\"3

s3=\037nlkdxk==3
(0+
k=o)
\037

IS

===SlOa=2')

are equal to

dX k

great est va I ue

the interval

===

When

th

[2;, n 1 the
and therefore)
. 21t
m 2 == sin n
0,) M 2

\037

Sinceall

Y3
3=2')
1t

== 1
M 2 == sin \037
2

')

M3

=sin

M4

. 21t V3
=sln

==
\037

1,

3=2'

. 51t
M 5 ==sln

6=\"2')))

Xs

===

\037

6.1.The Lower and

249)

Sums)

Upper Integral

For this division we obtain

+...+mJ==lf(l+V--

j'[

j'[

s6=-6(tn+m1
O

3)\037

+MJ=6(3+V3)
S6==(f(Mo+Mt+...
j'[

j'[

As

would be

expected,the

1.43,
2.48.

\037

inequalit ies)
n

S3 \037 S6 \037 .J\\

sin x dx

\037

Sf)

S3

\037

o)

hold true (the


At
6.1.2.

exactvalue

what

>a

\037

:rt

\037

does the relation)

n-l

sin xdx-

.\037
t=o

sin\037k

Solution.Since sn

differ

by

it

Llx k

< 0.001

< O.
for the required inequality to
then
In < Sn'
is sufficient that the upper and the lower integral sums

follow from the inequality max

hold true

equal to 2).

of the integral is

\037Xi

0.001:
a < Sn-sn< 0.001.)

less than

But)

Sn

<

-5

n-l

n-l

.-m.) < 8 i= (J\\1.-m.


I')

\037
== \037
(M I

i= 0

\037X.

\037
\037

0)

..n-

where M i and m; are the greatest and the least values of the function sin x on the interval [xi' Xi+l] (i ==0,
1).Assuming
for simplicity that the point
is chosenas one of the points of
division and taking advantage of monotonicity of the function sin x
n
and
we obtain)
on the intervals 0,

1,

\"

\037

n-}

\037

[ ' ]'
\037

(Mj-mj)=2 (sin -sinO)=2.


L.
t=
\037

0)

I,
Consequently,the required inequality is satisfied if 20 < 0.00

i.e.8 < 0.0005.


Show that
6.1.3.

the Dirichlet function [seeProblem 1.14.4


(b)]
integrable in the interval [0, 1].
Solution. In dividing the closed interval [0,1] into a fixed number of parts we must take into consideration, in part icular) t\\VO
possiblecases:(1) all points Si are rational\037 (2) all points;iare)))
is

not

Ch.

250)

irrational.In

/. The Definite

/ ntegral)

first case the integral sum is equal to unity,


in
zero.Hence,no matter how we reduce the maximum
length of subintervals, we always get integral sums equal to uni ty
and integral sums equal to zero.Therefore, the limit of integra I

the

the second to

sums is non-existent,which means that the Dirichletfunct ion is


not integrable on the interval [0, 1].
6.1.4.Find the distance covered by a body in a free fall withi n
the time interval from

= a secto

===

sec.

Solution.A body moves in a free fall with constant acceleration


g
and initial velocity Vo = O. Consequently,the velocity at the instant t
is equal to the velocity increment within
the time interval from
to t,
e. v (t) == For a short time period the velocity increat the instant t
ment is approximately equal to the acceleration
is constant, thereBut in our case acceleration
multiplied by
fore \037v==g\037t, and hence,v(t)==gt, since \037t==t-O==t.
Let us subdivide the time interval from t ==a to t ==b into n
of each subinterval will be equal
equal parts; then the duration
==
to
We
that
assume
during each subinterval of time the
n
body moves uniformly with a velocity equal to its velocity at the
beginning of this interval, i.e.)

i.

\302\260

\037v.

\037t

\037t.

b-a.

\037t

\037t

Vo ==

ga,
==g a + 1 b-a '
)
(

...........
VI

V2 ==

g (a + 2

fl
b-a

-n)

')

.)

b-a
vn-1==g[a+(n-l)-n_
l
\302\267

Whence we

distance covered by
ith
body during
b-athe
) . The entire distance covered
by the body is
the

find

v.l (

the

-- - - -

subinterval: n
approximately equal to)
s

\037

b-a(vo + + + v -1) =
n
b-a
b-a
= n g na + 1 b-a
2 n + ... + (n-1)b-a =
n +
[
]
b-an{n-I)
(b -a) g [a + fi2 2 ]

Sn ==

VI

\302\267

\302\267

\302\267

With

\302\267)

increasingthe distancecovered can be evaluated Illore accu-)))

6.1.The Lower

\037

Upper I ntegral

and

Sums)

251)

rately. The exact value of s is found as the limit Sn as


=
1-\037
s== Iim Sn== HIn g(b-a)
n

(
[a+-}(b-a)

n-oo

n-(\302\243

-+ 00:

)]

= (b2-a2).)
=g(b-a)[a+-}(b-a)]

SinceSn

'n')

IS an integral sum)

n-]

\037
Sn == \037
v.\037t.
I
I
i=

the distance S is an integral:)


b

; -a).

s= S vdt=Sgtdt=
a

a)

6.t .5.Proceedingfrom

b-a
==)

( \037t.==\037t

0)

\037

(b

the definition, compute the integral)


1
\037

xdx.

o)

Solution.By

definition,)

1
\037

xdx= Hm

n-l
.\037

\302\243i

l=O)

where)

O==Xo

< Xl

/).Xj

as max /).xi \037 0,

<...<

xn ==

1,

[Xi'

\302\243iE

Xi+l]'

\037Xi==Xi+l-Xi')

1.Subdivide the
poi n ts

==
xi ==\037
n (i

closed interval

0, 1,2, \"',n).

[0,1] into

The length of each subinterval is eq ual to

as n

-+ 00.

L\\x.l

equal parts by the


==

-n' 1

and

\037

Let us take the right-hand end-points of the subintervals as the


==Xi + 1== i + I (i == 0,
,n
1).
i
poin ts
n
Form an integral sum:)
\302\243

i:

1,...

\302\243

n-l
.\037=\0372
In ==Sn ==\037 i+l
(1+ 2 +
n
n
n
\037

\302\267

\302\267

\302\267

+ n) == n (n + 1)-.)

i=o)

As

n ---+00 the limit of this sum

is equal to

. n+l = _
1

hin
n

00

n)))

\302\267

2n:a

Ck.

252)

I. The Definite

Hence,)

Integral)

SXdX=-}.
o

2. Using this example,we


the limit of the

will show that for any other

integral sum will be the sanle.


points ;i
Take, for instance, the mid-points of the subintervals as

of

\"

+2
n

(i=O, 1,

;i: =

...,n-l).)

Form an integral sum

n-l

In = L 2i2n+ n = 2n2 [1 + 3 + 5 +
I

choice

\302\267

\302\267

\302\267

\302\267

i=O)

\302\243i

+ (2n 1)J1 = 2n4n 2 = 2\"


I

\302\267

Hence)

lim In

6.1.6.Proceedingfrom

00)

= -}

\302\267

the definition, compute the integral:

(m=l=-I,O<a<b).

\037xmdx
a)

Solution.In this examplethe following points can be conveniently


chosenas points of division:)

- ...,=

:r, ..., :
:
1

Xo

= a; XI = a

They form a

Xi

=a

( ) n,

geometricprogressionwith

Xn

:r -.
fL

b.

the common ratio

1)

q=( )n>1.

The length of the

-1).

subinterval is equal to

ith

== aqi (q
I1xi= aqi+l_aqi

Therefore the maximum length of the subintervalsequalsmax dXj


rl-l
1

= aqn-l(q_1)= a

: -1]
n

and tends to zero

( ) [( )
lirn q = 1.
increasingn, since n-+oo
Now let us choosethe right-hand end-points of the subintervals
+

as

\302\243

i:

\302\243

i = Xi +J = aqi

(i

= 0, 1,2,

...,n

1).)))

with

6.J. The Lower

\037

integral sum:)
n-l

Form an

n-l

In=i= 0
\037

== am+1

i=0

(q-l)

qm

=am+1(q

m u'+

\"'22a

\037r\037xi=

_l)q

um

I ntegral

and Upper

..

Sums)

253)

aqi(q_I)==

[I+qm+l+. +q<n-l) 1)] ==


q
q <m+l)n_1 = bm+l_am+l
)q qm+l_1.
qm+l-l (
<m+

-l

Let us calculatethe

of the integral sum

limit

i.e. as q ---+1:

as max AX i ---+0,

+1
limln ==(bm+1_a + 1) qli m q qmq-I
+ 1 = (bm+1_a ) m + 1 .
m

\037

Thus,)
b

S xmdx=m\037 I (b

+1_a +1).
m

a)

6.1.7.
Proceedingfrom

the definition, compute the integral:)


2

S d:
1)

Solution.Subdivide the interval


points of division Xi (i ==0, 1,2,
gression:)
Xo == 1; X 1

==;
q X2

whence q == V 2
The length of the

2.
q ,

X3

2] into

parts so that the


the geometricpro-

,...- -\-

n) form

3.

Xn

subinterval is equal to

ith
\037Xi

[1,
...,
- .

= qi+l_qi== qi (q-1),

---+0 as n -+ 00, i.e. as q ---+I.


so max \037Xi== qn-l
let us choose the right-hand end-points of the subintprvals
as the points Si' i.e.,Si=Xi+1=qi+l.

(q-I)

and

Now

Form an integral sum:)

-1).

=E..
n (2+
In = \037
(q-l)=\037
q
..!.._ 0 \037qj(q-l)
._ 0 \037\037x.= \037
ql +
1.,n
2
' n 2+ -1)= n 2,
.
1m J n == 1m (
-.
n
2
I

\037i

-I

since 2 n

1 ,.....,

n)))

-+

In

00

t1

00

2 as n --+ 00.

n)

Ch.

254)
And

I. The Definite

so,)

2
dX

SX

Integral)

= In 2.

I)

6.1.8.Evaluate

the integral
5

I = V25-x2 dx.
\037

proceedingfrom its geometric meaning.


2 is the
Solution.The curve y== V25-x
upper half
x\037

+ y2 = 25.The

of the circle
to
the
variation
curve
of
the
corresponding
portion

of x from 0 to 5 lies in the first quadrant. Hence,we conclude


that the curvilinear tra pezoid bounded by the lines x == x==5;
2
2 is a
quarter of the circlex y2 == 25;and
y == and y = V25
25n
I t S area IS equa I t 0

-x

. 0, .

4'

Hence,)

0;

1=5 V25-x2 dx=

2\0371t

6.t .9. Evaluate

the integral, proceedingfrom its geometric

meaning:)

I = (4x-l)dx.
\037

I)

6.1.10.
Prove that)
x
2 -x2
1=S Va2-x2 dx={xVa
+

arcsin
\037

\0372

(0 < x\037a).

o)

!J

Solution.The

integral)

I = Va2 -x2 dx

A)

\037

a)

Fig.

59)))

:\302\243)

the area SOAMx of the porexpresses


tion of a circleof radius a lying in
the first quadrant (see Fig. 59).
This area equals the sum of the
areas of the triangle OMx and the
sectorOAM.
x
xy
SOiUx=2=2
V a2

-x.)
2

6.1.The Lower and

\037

The area of the sector)

SDAM
where sin t =

\037
a)

Hence,)
SOAM

and consequently

Upper

2\"

Integral

a 2t

Sums)

255)

,)

. x
= a2 arc SIn
a

')

t)

Va
1=\"2

6.1.11.
Proceedingfrom

show

that)

a2
x
-x +2
arc sin(i.)
2

the geometricmeaning of the integral,

211:

(a)

sin3
\037

xdx=0;

(b)

o)

Solution.(a) The graph

e- dx= 2 e-

X2

X2

-1
\037

dx.

\037

0)

of the function

==sin3 x

is shown

in

Fig. 60. Let us show that the area situated above the x-axisis
equal to that I ying below this axis. Indeed, let 1t X 2Jt, then
x==Jt+x where O\037Xl\037Jt and sin3x==sin3(1t+xt)==-sin3xl
\037

\037

Therefore, the second half of the


graph is obtained from the first one
it to the right
by shifting
by Jt
and using the symmetry about the

x-axis.Hence,

If)

x)

o)

231

\037

sin3 x dx= O.
Fig.

o)

6.1.12.
Given

60)

[-2,3],

the function f(x)==x on the interval


the lower (sn) and the upper (Sn) integral sums for the given
interval by subdividing it into n equal parts.

find

6.1.13.Proceedingfrom

integral, prove that:)


11:

(a) sin 2x dx= 0;


\037

o
2

the geometricmeaning of the defini te


211:

(b)

\037

o
3

cos3 X dx = 0;

(c) S (2x+1)dx=6;(d) S V9-x2 dx= 9;.


1)

-3)))

Ch.

256)

6.t .14.Passing to

/. The Definite

Integral)

the limit from the integral sums, compute the

integral)
4

I = x3 dx,
\037

1)

by

subdividing the interval [1,4]:


(a) into equal parts;
(b) by points forming a geometricprogression.In both cases

choose as:
(1) left-hand end-points of the subintervals;
(2) right-hand end-points of the subintervals;
(3) mid-points of the subintervals [xi' xi + 1].)
\037i

\037

6.2.EvaluatingDefiniteIntegralsby

Newton-LeibnizFormula

the

The following is known as the Newton-Leibnizformula:


b

\037

f (x) dx = F (x) Ib = F

(b)-F(a),

Q)

where P (x) is one of the antiderivatives of the function f (x),


P' (x) f (x) (a \037x\037b).

6.2.1.Evaluate

i.e.

the integral)
V3

I = S +dxXl

\302\267

1)

Solution.Since the function P (x) ==arc tan


derivatives of the function
formula we get

f (x) =

\037X2

is one of the anti-

using the Newton-Leibniz

vr

n
Jt
n
dx
V3
==arctanV 3-arctanl=3\"-T=
J= S l+x
2 =arctanx
12.
/
1

1)

6.2.2.
Compute the

integrals:)
:rt

:rt

(a) S
o)

sin 2xdx;

(b)

cosX
.

(\\ sIn 3 X
1t

6')

dx; (c) S
YI:\037X2
o)))

\302\267

.\037

6.2.3.
Given the

6.2.Newton-Leibniz

function
\\

\\

Eval uate
\037

x2

1
f (x)==

for 0
x for 1

257)

Formula)

\037

\037

\037

\037

1,

2.

f (x)dx.

Solution.By the additivity property of the integral


2
\037

f (x)dx=

f(x)dx+ f (x) dx= 0 x dx+ Vxdx=


2

\037

\037

\037

\037

;3 1

r-

+ x21= + 4r32_
\"I

\037

o)

6.2.4.Eval uate

3 2

\037

;=+(4V2-1).

the integral

1=\037ll-xldx.
o)

Solution.Since

1--x for
II_ x = ,I x-1 for
I

we

\037

\037

\037

x
x

\037
\037

1,
2,

obtain, taking advantage of the additivity property of the integral,)

2
o

(l-x)dx+(x-I)dx=
11-xldx=
0
\037

\037

1)

==_ (1-X)2
+ (X-l)2
2
2
1

Evaluate
6.2.5.

the

=\037-j-\037=l
2
2
\302\267

I)

integral)
b

J == \037 dx

where a

< b.)

Solution.If 0

=b-a.

'

a)

\037

<

b, then f (x)

= I; = 1,therefore

\037

f (x)dx=)

a)

If

a<b\037O,then

f(x)=-I

and

\037f(x)dx=-b-(-a)=

a)

==

a--b.Finally,

if

< 0 < b,

then divide the integral

('
\037
a)))

(x)dx

I. The Definite

(x)dx+) f (x) dx=

bOb
Ch.

258)

into two

Integral)

integrals:
\037

(x)dx=

The above three

\037

b-(-a).

0)

casesmay be represented by

single formula:)

dx = b I-I a I.
I

\037

a)

Note. When evaluating integrals with the aid of the Newton-Leibshould be paid to the conditions of its legitimate use. This formula may be applied to compute the definite

niz formula attention

integral of a function continuous on the interval [a, b] onl y when


the equality F'(x) == f (x) is fulfilled in the whole interval [a, b]
[F(x) is an antiderivative of the function f (x)]. In particular,the
antiderivative must be a function continuous on the whole interval
[a, A discontinuousfunction used as an antiderivative will lead
to the wrong result.
Find a mistake in the following evaluation:

b].
6.2.6.

V3
dx

5.
o

l+x2=2

where

arc tan

(-}arc tan

V 3)-arctanO]
l-x2 =2 [arctan(=-6'
J/3

2x

I-=-XX2

3t

)'=

(x

=#=

\037X2

1).

Solution.The result is a prioriwrong: the integral of a function


positive everywhere turns out to be negative.The mistake is due
arc tan
to the fact that the function
=
of the first kind at the point x 1:)

\037

.m

The

2x

arc tan
x-I-O)2
11

correctvalue
V3

\037X2

_ x2 = 4 ;
j'[

has a discontinuity

. _ arc tan
2

hm

x -+ I +

- =-2x

of the integral under

= arc tan x

\037X2

j'[

-)

x...)

considerationis equal to

= arc tan V 3 -arctan =


\302\260

1\0373

\302\267

\302\267

\037

o)

Herethe Newton-Leibnizformula is applicable,since the function


F (x)= arctan x is continuouson the interval [0, J and the equality
F'(x)= f (x) is fulfilled on the whole interval.)))
\037

6.2.Newton-Leibniz

\037

6.2.7.Find
1t

259)

Formula)

mistake in the following evaluation of the integral:)

a
JT

+
S
o
1

dx
2 sin

===

2X

r
J

dx

cas:!x-j-3 sin<a

==

0)

=r

cos2 x

'+3d\037an2X

,..arctan(V :1tan x) --0.


V3

o)

(The integral of a function positive everywhere turns out to be

zero!
)

Solution.The Newton-Leibniz formula


arc tan
since the antiderivative F (x)=
tinuity

!im

at the point
F (x) =

1(
x\037--o
2

x=
\037

lim

3
n
V'x\037--o
2

!im F (x) = !im


IT

X\037-+O
2

arc tan

S
o

cot2 x+3

0)

It

can also be found

;3arctan(1/3tan x).
[0,nJ

n
.
arctan(+oo)=
2 V3

.r-arc

J(

dx

discon-

y=
1

v'-3 arctan(V\"3 tan x)

The correct result can be obtained

=r
cos2 x+3 sin 2 x J

(V3tan x) has a

(V3tan x) =

=r
n

applicablehere,

. Indeed,)

=
1(
x\037-+O
2

is not

dx
sin 2 x

in

tan

(-00)=

- v- .
n

the following way:)

=-

V3

(J/3 cot x)]:=

arc tan

\302\267

;\037r

the aid of the function F (x)==


For this purpose divide the interval of

with

[ '

and
into two subintervals,
and
[0,
n]
take into considerationthe above-indicatedlimit values of the function F (x) as x ---+ + O. Then the antideriva tive becomesa continuous function on each of the subintervals, and the Newton-Leibniz)
integration

\037

\037

9*)))

\037

Ch.

2:0)

/. The Definite

/ ntegral)

becomesapplicable:)

formula

2
.1\"J

= + =
sin 2 x
50 5n

5o (OS2

x\037-\\

2)

arctan(V3 tanx) =
+ ,;._
f3

a rctan(V'-3tanx)

\037

\037

V3

6.2.8.
Compute the

;3 [(;-0 ) + ( 0 -( - ;) )] = ;3
l

2)

\302\267)

integral)
Jt

2x

j'o V +c;s
Solution. J/ I +c;s2x =

dx.

x
2
V cot =

cosx 1=

::;;,
x::;;'

cosx,

==

COSx,

\037

\037

\037

n.)

Therefore)
n'
_a

J'[

2
I

-t- cas 2x

Jt
\"

dx== cosxdx-t- (-cosx)dx==


j
J
0

J'[
2)

== sin X

\037

(-sin x)

Jt

n ==

-1-

- -(-

(1 0)+ (0

\302\273

= 2.

\"\"2)

Note. If we ignore the fact that casx is negative


and

put)

2x _
... + cos
-cosx,)
2

we get the wrong result:)


n
\037
o)))

cosxdx=sinxl\037=O.

in

\037

1t

6.2.Newton-Leibniz

\037

6.2.9.Evaluate

261)

Formula)

the integral)
IOOJt

J=
Solution.We have
V1

Since sin x has

Vl-cos2xdx.

o)

cas2x == V2 s n x I.
I

period Jt, then


lOOn

lOOn

\037

Vl-cos2xdx=V2 Isinxldx=
\037

0)

= 100V2

Jt

sin xdx= 200V2

\037

o)

Evaluate
6.2.10.

the integrals:

-I dx
==
I
(a)
S (11+5x)a ;
-2

-2 dx
==
1
(b)
S x2
-3

Jt

' x,
Sin2x d'

(c) 1== S

(d)

2\"

-Jt

x2

J=S0 x2 +

dx;

2
J't

dx

(e) I=S x In
e

.
x '

(f)

I=l

sinx
I

x2

dx;

Jt
I

eX

(g) 1 S I -te dx;


0
==

(h)

2X

(i)

1=J Yx+l+xdxY5x+l
0)

Jt

(j) 1=

3
xdx;
V COSX-COS

\037

J1

Y3

(k) 1=

dx

l (I +
1)))

3
x2 2

x3 dx
1=S l+x
S;

\302\267

/. The Definite Integral)


an Integral.
* 6.3.Estimating
The DefiniteIntegralas a Functionof Its Limits
Ch.

262)

1.If

f (x)

\037

cp

(x) for a

\037

x b,

then

\037

\037

(x)dx\037

\037

cp

(x)dx.

a)

In particular,)
b

\037

(x)dx

\037

\037

f (x) dx.
I

m(b-a)\037 f(x)dx\037M(b-a).

2.)

\037

a)

where m is the least value, and M the greatest value of the function f (x) on the interval [a, b] (estimation of an integral).
If the function f (x) is continuous on the interval [a,b], then

3.

\037f(x)dx=f(s)(b-a),a<s<b

a)

(mean-value theorem).
4. If the functions f (x) and
cP

(x), in addition, retains

its

(x) are continuous on

cp

\037

f (x)

cp

(x) dx = f (s)

\037

cp

(x) dx, a

<s <b

a)

(generalized mean-value theorem).)


a

5. :x5 f (t) dt = f (x); :x5 f (t)dt =

f (x) at

each point

of

x)

a)

cont inu

[a, b], and

sign on this interval, then

of the function f (x).

ity

6.3.t. Estimate

the following integrals:)


;ry)

(a) I = 5 V 3

-\037

x3

dx; (b) I = 5

I)

si: dx;
J(
4)

(c)

5
I == 5 x 2+
+ 2 dx.
X2

Solution.(a) Sincethe

function f (x) V 3 x2 increases monothen m==2, M==V30, b-a==2.)))


tonically on the interval
===

[1,3j,

\037

6.3.Estimating

an

Integral.

as a

Integral

Funct. of

l-Ience,the estimationof the integral has the


3

2 2

V 3 + x3 dx

\302\267

\037

\037

\037

Its

Limits

263)

form

30.2,

1)

1.e.)

\037

f/3+ x3 dx 2 V 30 10.95.
sin x
on the interval
f (x) = x decreases
\037

\037

\037

(b ) The integ rand


since its derivative

sin x
' = x cosx= (x-tanx2x) cosx < 0
.)

f (X )

Hence,the

\302\267

x\037

least value of the

ion:

funct

m=f( )= 3\0373- ,

its greatest value being

M=f(

Therefore)

\037

)=

.)

2\037i

Jt

_!:) :::::::: dx
3

\037

Y3
2n

\037

r
J

\037

sin x
x

2
\037

Vf

\037_\037

Jt

i.e.)

4)

(3

')

4)

Jt

3-

-S

0.22 yf
\037

\037

u\\
Jt

f)
Y2

sin x

-Xdx

\037

\037

0....4.
C)

6.3.2.Estimate the

absolute value of the integral


19
sin x

5 + x8
1

d X.

10)

Solution.Since sin x
I

1, for x 10the inequality


< 10- is fulfilled.)
\037

\037

Therefore)

t\037;8

19

510

dx
t\037\037

10- <
< (19-10)

(the true value of the integral

\037

-10t4).)))

1O-\037

\037

,
\037

\302\267

Ch. VI. The Definite I nfegral)

264)

6.3.3.Which

of the two

integrals
1

\037Vxdx,
o

is the greater?

Solution.As

IS kno\\vn,

\037x3dx
0)

Vx>x3 for 0 < x < 1.Therefore


Vxdx > x3 dx.
o
1

\037

\037

0)

6.3.4.Prove the

inequalities:)

(a) 0 <

'

..
\\

o)

X7

dx

< \"8;

V ) +x8

(b) 1 < ex2 dx < e.


\\

o)

Solution.(a) Since 0 <


then)

X'

V I +x

8)

:'

o<

\037

S
o

< x < 1,

for 0

dx

<x

<

r x7 dx
J

V]+x\037

I
=-\037
\037\037

i.

(b) Since for 0 < x < 1 there existsthe inequality


then)

\037

6.3.5.Prove

< ex2 < e,

000)
I

Hencethe inequality

dx < ex2 dx < edx.


\037

\037

considerationholds true.

under

the inequality)
1t

S
o

e-R

sin

x dx

<

(l-e-

Solution.Since the function

(R > 0).

R)

21!R

t (x) = sinxx

;=
( ;)

decreases
on

Problem 6.3.1
(b)], then for 0 < x <
x
t (x)= si:

Hence,on

this interval

>t

\037

sin x > 11' x, therefore


\037

e-R

sin

<e

--x
2R
Jt)))

(0,

\037

) [see

\037

6.3.Estimating

an

Integral. Integral as a

and)
n

\037

Funct.

:::dX=-2 [e-

\037

e-R sinxdx < { e-

\037

0)

of

Its Limits

265)

J\037

= (l-e-R).
2\037

6.3.6.

Prove that for any funct ions f (x) and g (x), integrab Ie on
the interval (a, b), the Schwarz-Bun
yakovsky inequality takes place:
.-

./.

\037

t (x) g (x) dx \037'v

Solut ion. Consider


where

is

g2 (x) dx.

t (x) dx

\037

the f unct ion


F (x) == [f (x)

\037

a)

2
(x) ] ,
real number. SinceF (x)
then

any

Ag

0,

\037

\037

[t(x)-Ag(x)Pdx\037O,

a)

or)
b

\037

1.2 g2 (x) dx-2A t (x) g (x)dx+


\037

f2

(x) dx

\037

O.

\037

a)

The expressionin the left side of the latter inequality is a quadwith respect to A. It follows from the inequality that
at any 'A this trinomial is non-negative.Hence,its discriminant is

ratic trinomial

non-positive, i. e.

t (x) g

Hence)

{f

(X)dX)

-f

(x)g (x) dx

\037

\037

which

g2 (x) dx

(x)dx

/.

f2

11

t (x) dx

from

above)

\037

\037

the
completes

O.

\037

g2 (x)dx,

a)

proof.

6.3.7.Estimate the integral

===

x
+ dx.

(' sin
J 1

x\037

o)

Solution.By the generalizedmean-value theorem we


1

5o

sin x
l+x\037

.
dX==Sln\037t

dx

. t

5 l+x2=Sln\037arct an x
0)

have)

1)

o)))

T sin

\037

(0 <

\037

< 1).

Ch. VI. The Definite

266)

Integral)

Since the function sin x increaseson the interval


sin < sin 1.Whence we get an upper estimate of the
\302\243

sin

sin 1
Tj'[.

5 I +x2 dx <

\037

[0,1] then
integral:)

0.64.

o)

It is

theorem
I

possible to get a better estimation if we apply the same


the

in

form)

sin

x
2

5o l+x

dx==

I-cosl\037O.46.

S slnxdx==l+\037 \302\243(l-cosl)<

1+\0372

0)

6.3.8.Proceedingfrom

geometricreasoning, prove that:


(a) if the function f (x) increasesand has a concave graph

interval

[a, b],

(b-a)f (a) < 5 f (x)dx < (b-a)

f fa)

a)

(b)

if

interval

the

(b)

the

in

the

ion f (x) increasesand has a convex graph


then

(b-a)
f

funet

[a, b],

in

then

(a)

f (b)

< 5 f (x)dx < (b-a)f (b).


a)

Solution.(a) \\\\Tithout limitation


(x)> O. Concavity of the graph of

of generality we may assume


a function means, in particular, that the curve lies below the chord
through the points A (a,f (a)) and
B (b, f (b)) (see
Therefore the
area of trapezoid aABb is greaterthan
that of the curvilinear trapezoidbounded above by the graph of the func-

!J)
f(b))

--,-----,.. \".\"/:.
-- \"------J
.\".,

I(a))

Fig.61).

AI
I

o)

I
I)

:c

a)

b)

Fig.

61)

t ion,
b

i. e.

S f (x)dx < SaABb

f (a)
= (b-a).

a)

The inequality)

(b-a)f(a)<

f(x)dx
\037

a)

is obvious.)

6.3.9.Estimate

the integral
\037
o)))

VI + x4 dx using

f (b)

\037

6.3.Estimating

an

Integral. Integral as a

Funct. of

Its Limits

267)

(a) the rnean-value theorem for a definite integral,


(b) the result of the prece
ding problem,

(c) the inequality VI + x4 < 1 +

,
\0374

(d) the Schwarz-Bunyakovskyinequality (see Problem


Solution.(a) By the mean-value theorem)

6.3.6).

I = VI +x4 dx VI + 1;4,where
'--=

\037

\037

1.

1;

\037

o)

But)

1
whence)

< VI + 54 < V2,)

1<I<V2\0371.414.

(b) The function f (x) == VI + x 4 is concave on the interval

since)

f\"

4
2X2
(x)== o +(xx 4+3)
3 2 > 0' 0
) /

\037

[0,1],

x -.;;:::1.
\037

On the basis of the precedingproblem we get


1

< 5 V 1 + x4 dx < +[2 1.207.


I

\037

(c) < I = S VI + x4 dx < S ( 1 + x;) dx = I + -io= 1.1.


1

0)

(d) Put f (x) == VI x4, g (x)== 1 and take ad vantage of the SchwarzBunyakovsky inequality
1
1
1
1
....
4

=JVI+x dx=l<V

\037V-l+x4dx

(J

\037(l+x4)dx.\037
()

== V

Find the
6.3.10.

derivative

with

functions:)
x3

(a)

F (x) =

In t
\037

dt

(x > 0),

2)

v-x)

(b) F (x) =

\037

1)

x)))

COS(t2) dt

(x > 0).

12dx=

0)

1.2 1.095.
\037

respect to x of the following

Ch.

268)

/. The Definite

/ ntegral)

Solution.(a) Write the given integral

F (x)=

In t

dt

\037

In t

dt

dt

\037

\037

x2

In t

x2

the following way:)

in

x3

x3

In t
\037

dt,

c)

c > 0 is an arbitrary constant.


let us find the derivat ive F'(x) using the rule for differenfunction and the theorem on the derivative of
fiat ing a composite
an integral with respect to the upper limit:
\\vhere

Now

'

\037

(x) ==

In t dt

\037

'

[I

(X3)\037_

x3

Int dt

= In x3

(X2)\037

-lnx2 2x=

3x\037

x2

== (9x\037

v-x

4x) In x.)

2
(b) F(x)=\037cos(t
)dt-l-\037cosW)dt=
I

X)

1
-:;

v-x

cos(t2) dt -I-

\037

+
F'(x) [
[
]
= - 2 -2 + cosx.
y=

cos(t2)dt

\037

\037

\037

cosx

6.3.t. Find the

derivative

==

x)\037=

V x

2 COSx2 + 2 y- cosx.
1

x)

2;X

(a)

] __(V

respect to x of the following

\\vith

funct ions:)

F (x) = S

'
2

)dt
V?cOS(t
c

)\037

cos(t 2)dt;

\037

c)

t
Si\037

dt;

(b) F(x)=SV1-l-t4dt.
x)

o)

.I

6.3.12.
Find the points of extremum of the function F (x) =
S
.in

the domain x

> O.

Solution.Find the derivative)

F'(x) =

Si\037t
[\037

The crit ical points are:


x = nn

(n

==-

1,2,

dt

I=sin

x)

x)))

...),

where sin x = O.)

Si\037t

dt

6.3.Estimating

\037

Integral. Integral as a

an

Funct.

of

Its

Limits

269)

Find the second derivative at these points:


P\"

x cas
(x ) ==

x- .

- -(sin x

x2

')

= nn cos(nn) ==
I

P\"

(nn)

nn)

)n=l= O.

Since the second derivative

...),

is non-zero at the points x ==n:rt


these points are points of extremum of the funct ion,
namely: maxima if n is odd, and minima if n is even.
6.3.13.
Find the derivative of y, with respect to x, of the funct ion representedparametrically:)

(n

===

1,2,

13

\037

x=

Vzlnzdz;y=

\037

Solutio/l.As

is known,

y\037

== Yt,

z2
\037

lnzdz.

Vi

Xt)

Find x; and

y;:

'

xi ==

(]{Vz

(
y; ==
\\

z dz\\ (t3)i =

In

'

Z2

z dZ

In

\037

Vi

tint 3 .3t 2 = 9t 3 1n t;

13

) (Vi);=

In

1/\",

VI 2

=
\037_t)

- }It
-{-

In

t;

whence)

y\037

9t 3 In t

-- IrI

4 r

6.3.t 4. Find

==

t In t

_.36t 2 Vi
x

sin

(arctan

\302\245xdx

(c)

x-o)

.
11m
x\037

+ 00

X)2

dx

\037

\037

Iim

> 0).

the limits:)

.{2

(a))

(t

(b) Iim
+

3)

x\037

ex'dx
(\037

o)

00)

Yx 2

1)

\302\267

(' e2x2dx
J
o)

Xl

Solution.(a)

0 the integral sin Vx dx equals zero; it is


)
o
easy to check the fulfilment of the remaining conditions that ensure)))
At

x=--=

Ch. VI. The Definite

270)

the legitimacy of using the

Jim

Vxdx
l'sin
0

(c)

We

L'Hospi tal

x-+

\\0

00

'

sin VXdX
3X2

x--+O

]x'

(X2);

= lim 2Xt\037 x =
X)

x--+O

have an indeterminate form of the type

00
00

2
3

Use the

rule:)

( f ex2 dX

]1m

L'Hospitalrule. Therefore

=!im[0

x3

x--+O

Integral)

x2
2 ex2 dx.e

\037

o)

Jim)

X_

(' e2x2 dx

2X2)

00)

J
o)

2 ex2 dx
\037

o)

Iim)
X--+

6.3.t 5. Find the


y

(a)
\037

e-t2 dt +

et dt
\037

o
x

e)

Ii tn
X--++OO

x2
2 xe2 == O.
e 2

x)

of the follow ing imp licit fund ions:)

\037\037

sin2 t dt = 0;

\037

0)

(b)

x2

derivat ive

+ 00)

==

x2

sin t dt = 0;

\037

0)

(c) \037V3-2sin2zdz+\037costdt=0.
0

\037

2)

Solution. (a) Differentiate the

respectto x, putting y==y(x):


y

e-

t2

\037

dt

. +

\037\037

y2 dy

Hence.solving

X2

sin2 t dt
\037

dx

dy

\037

(X2)\037

= 0;

x2

+ sin2 x2.2x

the equation with


dx)))

side of the equation

e-

left

-==

O.

respect to

-2XR+y2sin2 x 2

\037\037,

we get

with

\037

6.3.Estimating

an

tn/tgral.

(c) Differentiate the


y ==y (x):

Integral

sideof

left

putt ing

Funct.

5 V3-2

of

the equation with

Its

Limits

271)

respect to x,

'

'

= O.
+ s cost dt
[0
]y

sin2 Z dZ

as a

\037\037

n:

x)

\"2

Whence

V3-2sin2x+cosy=O;)
Find: (a) the points of
6.3.16.

cosy)
extremum and the points of

dx)

inflect ion on the graph of the

funct ion)

I=

x
\037

Y3-2sin 2 x

dy

\037\037

(t-l)(t-2)2dt;

o)

(b) curvature of the line defined by the parametric equations:


(

t
X = a Vn
cos dt.
1t\0372

- ,..

\037

\037

nt
y = a V n sin
\037

dt)

(the Cornu spiral).


Solution. (a) The function is defined and continuously differenthe entire number scale.Its derivative
tiable throughout

1\037==(x-l)(x-2)2
equals zero at the points Xl == 1,x2 ==2, and when passing through
the point Xl it changes sign from minus to plus, whereas in the
neighbourhood of the point x2 the sign remains unchanged.Consequently, there is a minimum at the point
extremum at the point X 2 ==2.
The second der.i'vat i ve)
==3x2
lOx+ 8

I;

vanisHes at the points Xl -=

:.

Xl

==

1,and

there is no

=2

and changessign when passing through these points. Hence,these points are t he abscissasof
the points of inflection.
(b) We have
, ==

Xt

X2

T'
nf2

a V 1t COS

-.

nt 2
, ==
a V 1t Sin T')))

Yt

Ch.

272)

/. The Definite

hence,)

-,-

, =.:::Yt ==
tan

Yx

Xt)

2'

-,-

== (

\"

'Jtt\"!.

Yxx

6.3.t 7.

(0, 00)

by

y\"

II

); ==

Y.\037

+ (y') 2

Vn
a

Vi(

,.

'lIt!.

a cos:3

2)

Xt

whence the curvature)


K==

/ ntegral)

'1
I

Prove that the function L (x), defined


the integral)
x

In

the interva

t'

dt

L (x)

\037

5
1)

is an inverse of the function eX.


Solution.Let us take the derivative

L' (x) =

(x

\037

> 0).

Since the derivative is positive, the function


and, hence,has an inverse function
X==L-l(y).

---

\037=

L (x) Increases

The derivative of this inverse function is equal to


dx

-x

,
L' (x)
it follows (see Problem 3.t.to) that
x== CeY.
C, substitute x -== 1.Since
L(I)==O,i.e.ylx=l==O,)
dy

\\vhence

To

find

then)

1 ==Ceo= C

,)

\\vhich

proves our

assertion:
x == 1.J

6.3.t 8. Given the gra ph


the

of the

(y)

==

funct

eY

io;}

shape of the graph of the antiderivative

==

f (x)

I ==

(Fig. 62), find

f (t)

di.

\037

the given function


a].increases.
On the

Solution. On the interval [0,


tive; consequently, the antiderivative

is posiinterval)))

6.3.Estimating

\037

;I = I

l0, ] the

an

Integral.

Integral

as a

Funct.

of

Its Limits

273)

derivative of the given function is positive; hence,the

curve
(x) is concave.On the interval r , a] the derivative
of the given function is negative; consequently, the curve I == I (x)
is convex, the point x == being a point of inflect ion. The interval [a, 2a] is considered in a similar way. The point Xl == 0 is a
since the derivative (x) ==f (x) changesits
point of minimum,
sign from rninus to plus; the point x2 ==a is a point of maximum,
since the sign of the derivative changes from plus to minus.)
\037

/'

!J)

x)

o
Fig.

\037

3a 2a.

x)

62)

Fig.

63)

The antiderivative I (x) is a periodicf.unction with period 2a,


since the areas lying above and below the x-axisare mutually
cancelledover intervals of length 2a.Taking all this into account,
we can sketch the graph of the antiderivative (see Fig. 63).
6.3.t 9. Find the polynomial P (x) of the least degreethat has
a maximum
equal to 6 at x == I, and a minimum equal to 2 at
x ==

3.

Solution.The polynomial is an everywhere-differentiable function.


Therefore, the points of extremum can only be roots of the derivative. Furthermore, the derivative of a polynomial is a polynomial.
The polynomial of the least degree with roots Xl == 1 and X 2 == 3
has the form
Hence,

a(x-I)(x-3).

Sinceat

t he
x

- -+

P'(x)==a (x I) (x 3) ==a (x2 -4x+ 3).


point x == 1 there must be P ( I) == 6, we
x

P (x) = P' (x) dx -1-6 = a (x2 4x


\037

\037

ha ve)

3) dx + 6 =

- + - -}

1)

=a

C;

2x2

:3x I

) -1-6.

The coefficient a is determinedfrom the condition P (3)= 2, whence


a ==3. Hence,)
p (x) ==x a 6x2 + 9x-I

2.)))

Ch.

274)

/. The Definite

/ ntegral)

Find the polynomial P (x) of the least degreewhose graph


6.3.20.
has three points of inflection:
(I, 1) and a point with
abscissa 0 at which the curve is inclined to the axisof abscissas

(-I,-I),

at an angle of

60\302\260.

Solution.Since the required function is a polynomial, the abscissas of the points of inflection can only be among the roots of the
second derivative. The polynomial of the least degree with roots
0, 1 has the form ax (x2 1).Consequently,

-1,

P\"

x == 0

Since at the point

(x) ==a (x

-x).

P'(0)==tan

the derivative

have)

get)

P (x) = S P'(x)dx+I = a (

\037\037

- + :0 +
)

\037

\037

C\037

o)

Then, since P (1)== I, we

==V3, we

- )+V .

P'(x)= S P\"(x)dx+V3 =a

60\302\260

V3

(x-I)+l.

The coefficient a is determined from the last remaining condition


60(Y3 I)

P(-I)==-l,whence a=

P(x)='V37

Hence,

- (3x -IOx +x V-3.


7

\302\267

6.3.21.

Taking advantage of the mean-value theorem for the


definite integral,prove that
1

(a) 3 < V q +x2 dx < 10,


\037

2;, I.

3t

/-3
xdx<2 2'
(b)2<So V 1+2\"sln
j'[

2n:
dx

2n

'II

<T'
23t

(c) 'i3<S 10+3cosx


o

6.3.22.
Using
3 dx
5 V I +x <

the Schwarz-Bunyakovskyinequality, prove

\0375

. Make sure that

that

the application of the mean-

value theorem y.ieldsa rougher estimate.)))

6.4.Clzan[!.in[!.the

\037

6.3.23.
Find

Variable

in

Definite

275)

Integral

the derivatives of the following functions:)


x2

(a) F (x)= S In t dt (x

> 0);

(b)

F (x) ==

dt

ST

\302\267

2)

1)

x)

6.3.24.
Find

t he

\037\037

rically:)
t

(a)

111

o In z
J

( eZdz;

Jit

sin t

(b)

S
c

arc sin z dz,

== C sin

Z2

dz.

n)

6.3.25.
Find the
X

(a)

representedparamet-

x= zdz, y= J'
2
5
X ==

of functions

derivative

\037

(2

-2(1-t)dt;

F (x)= e
1

points of extremum of the following functions:)


2

x2

(b)

2
dt.
F(x)=S t -5t+4
2 +e
t)

\037

6.4.Changingthe

Variable in a DefiniteIntegral

If a function x == (t) satisfies the following conditions:


(t) is a continuous single-valuedfunction defined in
and has in this interval a continuous derivative cp' (t);
the values of the function x ==
(2) with t varying on
do not leave the limits of [a, b];
cp

(1)

[a,

cp

[a,

\037]

(3) cp(a)==aand

\037]

cp

(t)

cp(\037)=b,

for changing the variable (or substitution) in the


definite integral is valid for any function f (x) which is continuous
on the interva I [Q,
then the formula

b]:

\037

f3

f (x)dx = f
\037

[qJ

(t)] qJ' (t) dt.

ex)

Instead of the substitution x == (t) the inverse substitution


(x) is frequently used. In this casethe limits of integrat ion
a and are determined directly from the equalities a = '\" (a) and
=='\" (b). In practice,the substitution is usually performed with
the aid of monotonic, continuously differentiable functions. The
change in the limits of integration is conveniently expressed
cp

t =='\"

\037

\037

in)))

/. The

Ch. V

276)

Definite

Integral)

the tabular form:)


x

t)
I

ex.

b p

.)

V3-

6.4.1.
Compute the

integral

-Va
\037

4-x dx.
2

SoLution. Make the substitution x=2sint, assuming that


t
The function x == (t) ==2 sin t on the interval

iable,monotonic

\037

\037

\037

the

\037

cp

' ] satisfies all the


variable in a definite
\037

\037

cp

so,

And

x ==2 sin t;

since cost

>0

conditions of the theorem on changing

integral,since it

- -V3,

\037

dx = 2 cost dt;

V\037

-J/3\"

cp

V4

4-x dx=4
2

\037

= V3

x 2 ==

.)

21cost

::=2 cost,

' ].

\037

n
3

\037

[-

on the interval

Thus,)
V3

is continuously differen-

and)

\037

1t

eos2 tdt=2

_\037

3
\037

(l+C0s2t)dt=

_\037

3)

=2 [t+-}sin2t] = 4;+V3 .
4

6.4.2.Compute the

integral S

\037n

3)

y x 2 4 dx.
x4

2)

Solution.Make the

substitution)
x ==2 seet;

dx =

sin t
2 cos2 t

xl

di''

2 0
4

n
3)

the function 2
On the interval
[0,
is
ion
valid.)))
fore the substitut
\037

see t is monotonic, there-

6.4.Changing

\037

the Variable

in

Definite

277)

Integral

Hence,)

- .2

3't)

.S
2

y x2---4
dx==
4

V 4 sec2 t 4
16sec4 t

sin t
==
cos2 dt
t)

0)
:rT)

11)

= 4 JC sin2 t cost dt ==12sin3 t


I

o)

6.4.3.Compute the
a

(a) S

x2 V a 2

-x dx;
2

Jf-;
(b))

dx

V (1

1)

' 6-5

(a) .J
\\

\302\267

X2)=1

11

cosx dx
sin

integrals:)

:'T

\302\267

integrals:)

o)

6.4.4.Compute the

V3
= 32

x+sin 2 x ; (b) S

x'
2/C:5

o)

o)

Solution.(a)

Apply

the substitution)

sinx=t;

x It

cosxdx=dt;)

I)

The inverse fund ion x= arc sin t ( 0

\037

for 0 t
1) sat isfies
.all conditions of the theorem on changing the variable.Hence,)
\037

\037

\037

\037

\037

j(

t-3 1=ln\037
dt
2 ==ln t-2o
2 x = 5 6-5t+t
3'
5o 6-5sinx+sin
2

1==

cosxdx

0)

(b) Make the substitution


x ==2 arc tan t ,

t=tan'2
dx == 1 2dtt 2

,)

.11

2)

which is valid due to monotonicity of the function tan

2' on

the)))

Cil. V

278

interval

lO,

\037

/. The Definite

Integral)

\"

:rt

dx

=
x

jo 2+cos

50 2-1-

Y3

dt
2dt
--/2 . 1+/2 = 2 5 3+/2 =
-t--

\037

arctan

Y3

6.4.5.
Compute the

0= Y3 (arctany3 -arctanO
) 3Y3

j'[

\"

integral)

1t

4
dx

a:!.cos2 x+b2

sin\037

(a > 0, b > 0).)

o)

Solution.Make the substitution

tanx=t,

dx
cos2 x

It

j'[

411
= dt

')

4)

Hence,)

j1.

dx

a?cos2 x+b2 sin 2 x


S
o

= S a\"2+bdt 2t 2 = b2 S a2dt =
2
0
0 b2 + t
I

=-.-arctana
a ==-arctan-.
a
b

bt

b2

\\

=b = I, then :barc tan\037 =arc tan 1 =

If a

cides\\\\'ith

the result of the substitution a

===

b == 1 into

.:rT)

= dx
S a2 cos2 x+b2 sin 2 x S
dx

6.4.6.Compute the
'/3-

(a))

5
1)

V1

+2 x2 dx;

0)

integrals:)
e

dx

(b) S x V I + In x ;

V(X-2)l dx.
(c) 5 3+
3
V (X-2)2)))
3

1)

=--:

ab

,whichexactlycoin-

\037

integral)
:rt)

1t

4\"'

\302\267

the initial

\037

6.4. Changing

the Variable

in

Definite

279)

Integral)

:'1

6.4.7.Comput

e the

nt

x sin x
I == S +
cost x dx.

egra 1

o)

Solution.Reduce this integral

to the sum of two integrals:)

Jt

x sin x
- I2'
dx = I1-1
I = S l+cos2xdx -f-S 1+cos2x
x

sin

Jt
2)

To the integra 1

x sin x
dx
I2 = S 1+cos2x

Jt

apply

the substit ut ion

x==='Jt-t,
dx

===

dt ,

Jt

3t

'Jt

Then)

J 2 ==

1[
2

(3t-t)sin (3t-t) dt
Sn l+c082 (3t-t)

===

S
0

(3t-/) sin

dt ==

2t
l+cos

2)

1[

222
2

1C

=n S + cos2 t dt _ S +t sincost2 t dt
sin t

o)

Hence)

1[

1C

3t

t sin t dt
sin t dt _
x dx
n
I I1+ I2 S x+sincos
x + S + cos t
+cos t
S
0
o
===

=-=

\037

\302\267

o)

\037

\302\267

\037

0)

Sincethe first and the third integrals differ only


of the variable of integrat ion, we have)
31
2)

I=n S +sin t dt t
1

o)

cos\037

\302\267)))

In

the notation

Ch.

280)

I. The Definite

Integral)

To this integral apply the substitution

u ==

du ==

cast ,

sin t dt,

0)
j[)

o)

\037)

==

du

Jt \\

du

==Jt r
u2

j0

(1

J1\037

I -t-- u 2 ==\"\"4

\302\267

S1I1

Note. The indefinite integral J I eos x dx is not expressed in


elementary functions. But the given definite integral,as \\ve have
shown, can be computed with the aid of an artificial method.
Evaluate the integral
\\

\037

\037

6.4.8.

1 5 In(l+x)
dx.
+ x2
o
===

Solution.Make the substitut

ion

x == tan t,
dx

== dt2
eos) t

'

t
I)

o)

j'[)

1)

4)

Hence,)

n
4

I == JC

In

+tan2 t)

(1

see t

Transform the sum 1 + tan


1

see2 t

tan t ==tan

4*

d t == l
J

t:
n

In

(1+ tan

t) d t

+ tan

V2-sin

t =

\037

(t -/- 4 ) .

cost

Substitutinginto the integral, we obtain

\"tI
n
4

)dt-Sincostdt

\037

()

n
4

\037

In 2

\037

c: + 5o

(t +

Insin

In
\037

2+

=cc5\037ln2dt+Slnsin(t+

) dt-51ncos
0
t

\037

n
4

51nsin
(t + ) dt 5 In cast dt
o
\037

dt

0)))

-=
\037

-/2'

In2 + 11

\037

Now

6.4.Changing

let us show

the Variable

dz

.11

0)

,)

Integral

281)

the substitution)

ap\037ly

Jt

\037=

Definite

1 ==120 To this end

that

t==--z
'
4
dt

in

3t

3't

/2 =

to the integra I

SIncast dt

\302\267

())

Then)

/2 =

Jt)
()

S cas(
In

\037

-z)

.t

dz =

\037

In sin
S
0

\037

\037

-z)] dz =

4)

..

J1)

S In

sin

\037

.+

z)

dz = /t.

o)

Therefore)
Jt
2.)
I ==BIn

Note that in this problem, as well as in the precedingone, the


In
dx is not expressed
in element ary funindefinite integra I
S l
et ions.
\037 \0372X)

6.4.9.Prove that for any given integral with finite limits a and b
one can always choose the linear substitution x pt + q (p, q constants) so as to transform this integral into a new one with limits
o and
Solution. We notice that the substitution x===pt+q satisfies
explicitly the conditions of the theorem on changing the variable.
Sincet must equal zero at x a and t must equal unity at x ==b
\\ve have for p and q the following system of equations
\037

1.

\037

a==p.O+q,

b ==

p.1 --1-q

,)

b-a, q ==a. Hence,)

whence p ==

\037

f(x)dx=(b-a)f [(b-a)t+a]dt.
\037

0)))

282

Ch,

6.4.10.
Compute the

/. The Definite

sum of two

integrals

2
3

-5

e ( x-3\")

\037e(X+5)2dx+3\037

-4

/ ntegral)

dx.

us transform each of the given integrals into an


0 and 1 (see the precedingproblem).
with
limits
integral
To this end apply the substitution x == t 4 to the first integra Then dx == dt and

Solution.Let

--

1.

11=
Apply

e<x+5)2dx =

\037

the substitution x=

dx--3) and

e(-t+

1)2dt

\037

\037

e<t-I)2dt.
\037

to the second integral. Then

\037

dt

2
3

/(

12=3\037
1

2 2
X-3\" )

dx=\037e(t-1)2dt.
0

3)

Hence)

11+ 12 =

\037

e(t-1)2dt + e<t-I)2dt=0.
\037

0)

2
9

Note that neither of the integrals (' e(x+5)2dx and J e (


evaluated separately in elementary functions.
Prove that the integral)

X_\037

\037

6.4.11.

3T

() sin 2kx
\\

cJ

SlIlX

dx

equals zero if k is an integer.


Solution. Make the substitut ion)
dx

-- ,

x ==j(

dt

S
o

2kx
sIn x

si\037

dx =

S
\037

o)

Jt

Jt)

o)

,)

Then at k an integral number we


IT

t)

2k
sin.
sIn

get:

(1[-t) dt ==_

(It-i)

\037

S
0)))

2kt
sIn t

si\037

dt.

3 ) dx IS

\037

6.4. Changing

the Variable

in

Definite

283)

Integral

Sincethe definite integral does not depend on notation


able of integration, we

of the vari-

have)

I == I

6.4.t 2. Compute the

,)

whence

O.

===

integral)

V3
2

dx

r
J

JI'I

\302\267

x\037

2)

Solution.Apply the substitution x

sin t (the given function is


==
The
new
dx
cas
limits
of integration t 1 and
t
dt.
monotonic),
are found from the equax)
. I ==SIn,
. t . Y3 ==sIn
. t
t Ions
2
\"2
===

not
t2

\302\267

We may put

3'

3t
===

but

t1

3t

(3 and

===

t 2 ==

other values may

also be chosen, for instance,


5n
23t
11
==6 and 12=-=3\"'

Jl
3)

1C

In both casesthe variable


x ==sin 1 runs throughout the

entireinterval

[ .

\0373

\037

monotonic both on

t)
k.&\302\243

Fig.

5)

64)

(see Fig. 64), the function sin t being

5
[ . ] and [2;. 6 ] .
1t

\037

\037

Let us show that the results of the two integrations will


Indeed,)
V3

rr

r
J

dx
x\302\245l-x

=
2

rr

cost

3
df

rr

\037

\037
sint

- sin/cost.,
-

\\

I 3=
= In tan2)

Jt

rr

tan\"::=
''::-In
6
12)

On the other hand, taking into considerationthat


21t
on the interval 3 ;
we obtain)))
\0371t

:n:

6)

= In tan

coincide.

In
2\037\0373

cos1 is negative

Ch,

284)
.1\"3

2)

-.f5n
6

-J 6

3
X V\037\037X2

Jntegral)

5n

2\037

2)

.\\'
1

/. The Definite

dt

sin \037O(\037::s

t)

sin

t)

2\037

3)

51t

= In

tan\"2

51t
Note. Do not take t l = 6

the interval

51t
, 6

(0, (0)

that

= In

t an
tan

123t = In
1t

t2

\037

'

3
[\037,

\037

].

the function L (x) defined on the interval)


x

by the integral L (x) = 5

possessesthe following properties:

\037t

L (X IX2) = L (Xl) + L (X 2),


L

:\037

= L (X I

Solution.By the additivity

C dt

===

J
1

Let us change the variable

)-L

(x2).

property)

X 1X 2

L (X 1X2)

in

() 2dt

X'I

X'1

= dtT +
5

T'

X,)

the second integral)


z

t
I

t ==XIZ,

1
x

Xl
X 1X 2

dt ==x1dz,

2)

Then)
Xt

L (xt x2) ==

ere X IX2 = X 3'

\302\267

X2

L (X3) = L (Xl) + L

dt

dz

5T+5z
1

P U t t Ing

2 + Y3
V-3

since, with t varying on


' the values of the function x = sin t lie beyond

the limits of the interval


Prove
6.4.13.

6\"
2n

===

L (Xl) + L (x2).

--,
1)

X3

we 0bt aln

Xl)

(:;), i.e. e:) =


L

L (x3)

L (Xl)')))

6.4.Changing

\037

It is

in

I nte[!,ral

Definite

also easy to obtain the other corollary L(x ) =


\037

integral

any

the Variable

and n.

positive m and n this follows

I ndeed, for

L(x

\037l

and for a negative

from

285)

L (x) for

the relations)

)=mL(x+), L(x)=nL(/n).)
exponent,from

L (1)= 0, L

(r1) = L (-1-) =L (1)-L(x)=- L (x).

Now, taking advantage of the continuity of the integral as a function of the upper limit,we get the general property L (xa) ==aL (x).
Note. As is known, L (x)== In x. Herewe have obtained the principal properties of the logarithm proceedingonly from its determination with the aid of the integral.
3

6.4.14.
Transform the
== t.
(X-2)2
Solution.A

integral

\037

dx
(X-2)2

the substitution

by

the
application of the substitution throughout
would lead to the wrong result, since the inverse
the function x
function x == (t) is double-valued:x = 2+ V
The former branch canX 2 ==2 +
has two branches: Xl ==2
not attain values x > 2, the latter values x < 2. To obtain a COfrect result we have to break up the given integral in the following

interval

formal

[0,3]

-Vi;

cp

way:)

\037

put

i, i.e.

002)
200)
3

and to

Vi.

(X-2)2dx=(X-2)2dx+(X-2)2dx.
\037

\037

x ==

2-Vi in the

integral,and

first

second.Then we get
2

/1=

S
o

(x-

2)2dx =

J4

2 \037T

S
0

\037

dt

==2 -t- J/- t in

the

Vidt =

\302\267

\037

t
tdt
/2=j (X-2)2dx=
j 2VT =\"2S V =3'
\037

which
Hence,1==3+3=3,

is a

correctresult.It can be easily)))

Ch. V

286)

/. The Definite

Integral)

verified by directly computing the initial integral:

(xS
o

= + + = 3.

2)2dx = (X--;2)'1
I

0)

6.4.15.
Compute the integrals:)
5
dx
.
r
r
' (b) I
(a ) I

- +
I - 5 I-sin
1

y-x

dx

2X-1- Y3x+ I

J
o)

o)

31

dx

(c)

2
dx;
(d) 1=5 V-2x-x

\302\267

31

o)

4)

31

e) I - 5
4

sin

x+cas x
+ sin 2x dx,
\302\267

o)

a
(f)

I =.fx2 -V:+;dX, a> 0;)


o

2a

2
dx;
(g) I = 5 V2ax-x

I = Jr (1+dx

(h)

-1)

o)

\302\267

x\037r\037

6.4.16,

Applying a suitable change of the variable,


lowing definite integrals:)
a
2
dx
r
;
(a) .1 vX+T VlX+ 1)3 (b) f
2 ;
a
o)

V(a 2+b 2 )j2

2
dx

+x 4) ;

(d))

xdx

Y (x:!-

.f)

a\037)

V(3a 2 +b 2 )j2)

I)

6.4.17.
Consider the

integral

-2)
that

it

is equal to

\037

4\037\\2

-x )

'

It

,)

is easy to conclude

4-( -4) ] =4'

5 4+x2 =2arctan2-2=2[
]

-2)))

(bt

. Indeed,)

2
dx

the fol-

,x+ 1/:-x

o)

(C).f x (l

find

j'[

j'[

j'[

6.4.Changing

\037

the Variable

in

1ntegral

Definite

On the other hand, making the substitution x


x

dx=

\037=

\037b7)

+. we have)

-(i' -2
dt

2
I

2)

r dx 2 =
4+x
-J2

r
J

dt

--

t2

+ t2 )

\037

r
J

dt
4t 2

2\"

+1=2\"arctan2t

--

_\037

=-4.)
3t

2)

This result is obviously wrong, sincethe integrand


4+x2 > 0, and\037
this
be equal
the
function
cannot
of
definite
integral
consequently,
Find the mistake.
to a negative number
I

-.
\037

23'1

6.4.18.
Consider the

dx
integral 1==JC 5-2cosx ' Making the substio)

tution tan

2:= t
x

\\ve

23'1

\\

Jo

have
0

dx

2 dt

5-2COSX=\\

J (I+t )(5_21-t
1+t 2)
2

=O.

The result is obviously wrong, since the integrand is positi ve,


and, consequently,the integral of this function cannot be equal to
zero.Find the mistake.
Make sure that
6.4.19.

a formal change of the variable

=:x

2
f)

leads to the wrong result in the integral


V x2 dx. Find the
-2
mistake and explain it.
Is it1 possible to make the substitution x= sect 111 the
6.4.20.
\037

integral

I=

\037

V x2

+ 1 dx?

o)

6.4.21.

Given the integral

x = sin
for

t?)))

t. Is it

\037

l-x dx. Make the


2

possible to take the numbers It and

3t
\"2

substitution
as the limits

Ch, VI. The Definite Integral)

288)

Prove
6.4.22.

the equality)
Q

-Q
\037

for any cont inuous

f(x)dx=

dx
[f(x)+f(-x)]

ion f (x).)

funct

6.4.23.
Transform

\037

0)

2Jt

the definite integral

\037

f (x) cosxdx

by

the sub-

o)

stitution
\037

sinx===t.)

6.5.Simplification
of IntegralsBasedon

Properties
of Symmetry of Integrands

1.If

then)

f (x) dx ==2 \\ f (x) dx.

\\

-Q
\037

2.

[-a,a],

the function f (x) is even on


a

\037

[-Q,a],

the function f (x) is odd on

If

the

then)

f (x) dx = O.)

\037

-Q)

3. If

periodicwith period T, then)

the function f (x) is

b+nT

\037

f (x)dx =

\037

a+ nT)

where

11

f (x) dx,

is an integer.)
1

6.5.1.
Compute the

integral

Solution.Sincethe integrand

-1
\037

x dx.
I

f (x) ==

IS

an even function, we

have)
1

-1
\037

x dx = 2
I

6.5.2.
Compute the

\037

x dx = 2 xdx= x2 10
I

\037

--=

1.

0)

integral
7

4
r x sin x dx
.) x6 2

-7)

\302\267

Solution.Since the integrand is odd, we concludeat once


the integral equals zero.)))

that

6.5.Simplification

\037

6.5.3.Evaluate

289)

of Integrals)

the integrals)

Jt

(a)

f (x) cosnx dx;

\037

-3T
:IT

(b)

f (x) sin nxdx,

\037

-J[)

if: (1) f (x) is an even

funct

ion; (2) f (x) is an odd

funct ion.)

6.5.4.
Calculate the

integral

x
dx.
x 4 +2x2 +
5
-5
xl>

sin\037

Tn

6.5.5.Compute

the integral

.
xdx
5 cos4
periodicfunction with period Jt, since
S\037\0372:in4

3T)

Solution.The integrand

is a

(x +11)
= cos4 x + sin4
(x+ Jt) = cos4(x +n) +
sin 4 (x+n)

sin 2

Therefore it is
and lower limits:)

possible to subtract the number

Tn

sin 2x

:rt

sin 2x dx
cos4 x + sin 4 x

3T

50

1t

= f (x).
from the

upper

\"

sin 2x dx
cos4 x + sin 4 x

=2

4
tan

x dx

5 cos2 x (1 + tan 4 x)

\302\267

0)

Make the substitution

= tan x,
dx
dt = cos2 x '
t

:rr

tan x dx

Prove the
6.5.6.

4)

-_ 5

2 S cos2x(l+tan4x)
n

0
n

\"\"4

2 _
_
-arctant
o-T.
l+t4
2t dt

1t

0)

equality)

cosxf (x )dx= 2 cosxf (x2) dx.


2

-0
\037

Solution.I t

IS

\037

0)

sufficient to show that the integrand IS even:


cos
t [( _X)2J = cosxt(x2

(-x)

).)))

Ch,

290)

6.5.7.
Compute the
V\"2

/. The Definite

Integral)

integral
3 -12x
2 +x+I
+3x6-IOx-7x
dx.
2

2x'1

-V2\

+2

Solution.
V\"2

+3x6

2x'1

-V2')

- -+210x() 7x3
x2

12x2

+x+ dx =
1

V2

\037

V2'

x-

2x'1-10xD-7x3+x
3X2(X4_4)+I_
dx +
d _
x2 +2
x2 +2
S
S
-V2
-V2
V2

=0+2 S [3(x4 -2x2) +


o

X2\0371

2
x
=-56 x -4x + ,m
arc an ..rr 2
r 2
0

V\"2

] dx=
=

-- -2+ v- .

In calculatingwe expandedthe given integral

two

integrals so as to obtain an odd integrand

and an even integrand in the


Compute the integral

second.

in

16
5V

j'[

'2

into the sum of

the

first

integral

6.5.8.

\"2

S cosxIn
1

:+

\037dx.

2)

Solution.The function f (x)= cosx is even. Let


funct

+x

us prove that the

-x is odd:
I-x=n I+X -1=-lnl+x=-cp(x).
cp(-x)=lnl+x
l_x
( I-x)

ion

cp

(x) -==

In

Thus, the integrand is the product of an even function


one, i.e. an odd funct ion, therefore
I

T
SI

cosxIn:+;dx=O.

.'2)

6.5.9.Prove

the validity of the following equalities:)))

by

an odd

6.5.Simplification

\037

(a)

Jt

sinD

XU
\037

xdx= 0; (b)

ecosx dx = 2

\037

3t

8)

2)

291)

of Integrals)

\037

eCOSX dx;

(c)

sinmxcosnxdx=O
(m

\037

and n natural numbers);

-JT)
Q

(d)

sin xf (cosx) dx ==O.

\\

-a)

6.5.10.
Prove the

equality)
h

\037

f (x) dx =

\037

f (a

+ b-x)dx.

a)

Solution.In the right-hand integral make the

-==

a+b

t,

dx ==

dt

x
,)

Then we obtain)

\037

t)

b)

a)

dx =
f (a+b-x)

\037

substitution)

f (t) dt =

\037

f (t) dt =

\037

f (x) dx.

a)

Note. The relationestablishedbetweenthe integrals can be explained geometrica


11 y
The graph of the function f(x), consideredon the interval [a, b],
is symmetrical to that of the function f (a +
consideredon
b
the same interval, about the straight line x =
Indeed,if the
point A lieson the x-axisand has the abscissax, then the point A',
which is symmetrical to it about the indicated straight line, has
the abscissa x'-==a-tb-x.
Therefore,
==f (x). But symmetrical figures have equal areas which are
expressedby definite integrals.And so,the proved equality is an

b-x),

at .

f(a+b-x')==f[a+b-(a+

+b-x)]

equality of areas of two symmetrical curvilineartrapezoids.


6.5.t I. Prove the equality
t
\037

f (x) g (t -x)dx = g (x)f (t -x)dx.


\037

0)))

Ch. V

292)

/. The

/ntegral)

Definite

Solution.Apply the substitution t -x==z in the right-hand integral;

then we have)

o
\037

g (t-z)f (z)dz=
1[

6.5.12.
Prove

the equality

31

sinm x dx =

cosm x dx and apply the


\037

\037

obtained result

in

(t-z)dz.

f (z) g

\037

0)

computing the following integrals:)


31

31

sin2 xdx.

cos2 X dx and
\037

\037

0)

we
Solution.On the basis of Problem6.5.10
1[

1[

sinm x dx =

\037

\037

Hence,in

sinm

\037

\037

1[

:rt

1[

x) dx = cosm x dx.

part.icular,)

2
=

have)

cos2 xdx;

sin2 xdx=
\037

\037

0)

add these integrals:)


31

:rt

2/=
hence,

(sin x+cosx)dx=S dx=


2

S
o

\037

.;

0)

1=4'
1t

Prove
6.5.13.

the equality)
31
31

(sinx) dx = 2

\037

\"'2

f (sin x) dx.
\037

0)

Solution.Since)

2'
:rt

1[
f
o
\037

(sinx) dx =

\037

f (sinx)dx +

:rt

f
\037

1[

T)))

(sinx) dx,

6.5.Silnpli

\037

it

is sufficient to prove

/ ntegral

fication of

293)

s)

that)

1[

1[

f (sinx) dx=
\037

f (sinx) dx.
\037

Jt

2')

In the left integral make the substitution)

dx=

dt

- -

x= 'Jt-t,

j(

j'[

,)

'Jt

0)

Then)
0

n
\037

Jt

f[sin(n-t)]dt=)
f(sinx)dx=-\037
1[
\"2)

2)

1[

31

(sint) dt =

f
\037

6.5.14.
Prove the

f
\037

(sinx) dx.

0)

equality)
:t

:rt

S
o
Solution.In the

xf (sinx) dx =

left integral

\037

f (sinx) dx.
S
0

make the substitution

X='Jt-t,
dx= -dt,)

t)

Jt
o)

'Jt)

Then we obtain)

- (n-t)
0

:rt

\037

xt (SIl1x)dx=

\037

[sin(n-t)]dt=

1[)

\037

nf (sint) dt

Whence)
31

;rr

xf
\037

(sinx)dx= n f (sinx) dx,

which is equivalent to the gIven

1[

3t

\037

equality.)))

\037

0)

tf

(sint) dt.

294

Ch,

6.5.15.
Using the
sin

/. The Defini.fe

Integral)

equality

+ )x _
= 2 + cosX + cos2x + ... + cosnx,
. x
I

\037

2 sIn

prove

2)

that)
sin

31

('
J
that

+ ;) x

Sin

Prove
6.5.16.

dx == n.

2)

2 cos2x+
cp(x)=+ao+atcosx+btsinx+a

if

then
+b2 sin2x+...
+ancosnx+bnsinnx,
2n

231

(a)

cp
\037

(x)dx = 3ta o; (b)

cp
\037

(x)coskx dx = na k;

231

(c)

cp
\037

(x)sin kx dx = nb k

(k

= 1,2,

o)

\037

...,
n).

6.6.Integrationby Parts.ReductionFormulas
If

and v are functions of x and have continuous derivatives,

then)
b

U
\037

or, more

(x)v' (x) dx = u (x)v (x)\\:

\037

v (x)u' (x)dx

a)

briefly,)

b
\037

udv=uv\\:- vdu.
\037

a)

6.6.1.Compute the

integral

xe dx.
X

\037

o)

Solution.Let

us

put)

x = u,
du ==dx;

eX

dx == dv;
v ==eX ,

since the funct ions u =- x and v ==eX are


which is quite legitimate,
continuous and have continuous derivatives on the interval [0,
Using the formula for integration by parts, we obtain
)

xe dx = xe
X

I\037
\037

1].

\037

eX

dx =

e-

eX
I\037

1.)))

6.6.Integration

\037

by
.)

Parts.

Formulas

Reduction

295)

bn

6.6.2.
Compute the

I = eax sin bx dx.

integral

\037

o)

Solution.Let

us

put

= eaxdx',
I
v = (!lx.

u == sin bx,

du

dv

= b cosbx dx,)

a)

v = \037

a eax togetherwith their deri.


the formula for invatives are continuous on the interval
tegration by parts is applicable:)

Sincethe functions

u == sin bx,

[0,It],

b --j
I=-eaxsinbx

j{

'\\

eaxcosbxdx=

o)

b
:rt

-\037
a

5 eaxcosbxdx=

\037

11.

o)

= cosbx,

du ==
Then)

11'Put)

parts the integral

Now let us integrate by

dv ==eax dx,

b sin bx dx,)

v ==

a)

eax

,
n
b
( J...
_.!!..
=
1 a a eaxcosbx 0 + a S eaxsin bx dx ) =
I
I

\\

\\

\037

a:t

= _ a (_
\037

eb
a

an

_ a )_ a2 I =
\037

b\037

b+

(e

2)

Hence)
a2

+ = (ea;2+ I )
a2 I
a
b2

1_

In part icular, at a ==b

,)

b+
aT[

(e

= 1 we get)

1);

SexSinxdx=(e +1).
1l

\037

o)))

a2 +b 2

)
\302\267)

-b2

a)2

\302\267

Ch, VI. The Definite Integral)

296)

6.6.3.
Compute the

xdx.

In 3

integral

\037
1)

1[2)

6.6.4.Compute the

integral

\037

sin V:Xdx.

o)

Solution.First make the

substitution)

vx- = t,
x = t 2,
dx == 2t dt

j[

1[2

,)

2)

Whence)
n2
4

3in
\037

Vi dx = 2

t
\037

sin t dt.

0)

Integrateby parts the latter integral.


Put)
t

sin t dt = dv;

= u',

du =

v=

dt;)

cast

.)

Then)

2?
o

tSintdt=2-tcostl0 + ? coStdt =2Sint\\ =2.

6.6.5.

Compute the integral

I= r

arc sin

, VI

\037

+x

a)

dx.

o)

Jt

6.6.6.
Compute the

integral

\037

x2 sin xdx.

o)

6.6.7.Compute the
tural

number.

integral

2
2
In = (a _x )n dx,

where

is a na-

\037

Solution.The integral can be computedby expandingthe integrand


2
(a _x )n accordingto the formula of the Newton binomial, but it
2

It is simpler to deducea formula


involves cumbersomecalculations.
reducing the integral Into the integral In-I.To this end

for

let)))

6.6.Integration

\037

us

expand the integral


fn =

\037

2
(a _X 2)n-l (a2

Formulas)

297)

in the following way:)


a
2 dx a2f
X (a2 _X 2)n-l X dx
)

In

Parts. Reduction

by

n-l-

-X

\037

0)

and integrate the latter integral by parts:)


(a2 _x2)n-l X dx == dv,

u=x;)

v=

du ==dx;)
We obtain)

-x a -x

In ==a2jn -1+ 2n
I

2n

(a2 _x2)n

2n

(n

=1=

0).)

--

--I.

2n

)'ldx==a21
n -1
5 (a2-x2

2n

o)

Whence)

jn = a2 2n2n+ In -1
\302\267

This formula is valid at any real n other than 0 and


In particular, at natural n, taking into account that)

-i-.

I0 =

\037

dx = a,

o)

we get

In ==a

2n +

2n (2n

- - . .6....5.3
4) .

2) (2n

(2n+I) (2n-l)
(2n-3)

where)

6.6.8.
Using the
lowing

== a 2n + 1

...
I)!! .3.5. ..

(2n)!!==2.4.6
(2n +

4.2

(2n),

(2n + 1).

== 1

1-3+ 5-7 +...+(-1)


2n+l
c\037

C\037

c\037

==

are binomial coefficients.


Solution.Consider the integral

\\vhere

')

result of the precedingproblem obtain the fol-

formula:)
c\037

(2n)!!

(2n+I)!!

C\037\037

J n =- ( 1
5
o)))

x 2) n dx ==

(2n)!!

+I)!!
\302\267

(2n

(2n)!!

'
(2n+l)!!

Ch,

298)

J. The Definite

J ntegral)

Expanding the integrand by the formula of the Newton binomial


and integrating within the limits from 0 to 1, we get:)
I

In=

(l-x)ndx=
2

\037

o)

= (1
o
\037

= [x-

C\037X2

CAX3

C\037X4

C\037xD

C\037X7

the
completes

+...+ (-I)n

dx =

x2n + 1
1

_
J 01

(_l)n
+ 2n+l
-3\"+5-7+'\"
Ch

c\037

C;\037

')

proof.

6.6.9.
Compute the

integral)

1[

Hm =

dx =

sinmx

\037

\037

a natural number).
Solution.The substitut

2'
31

(m

C\037x2n)

2n+

=1
which

+ ... + (-1)n

C\037X6

cosmxdx

0)

ion)

sinx=t,

casx dx = dt,

Jt

2)

reducesthe second integral to the


:rt

\"2

Hm=

m-}

\037

m-l

(l-sinx)-;-cosxdx=(l-t)-;-dt,
2

integral)

\037

0)

consideredin Problem 6.6.7with a

\037

1 and

n ==

m-l .

the reduction formula)

li = m-l
m. Hm-2
m

is valid here, since)

2.m-l
2
Hm==/m-l= m2. 2 +
\037

1)

Therefore,

(m=i=O, m=l= 1)

m-lI - = m-I
- = ---1

Jm 1

2)))

Hm -2

\302\267

\037

If

6.6.Integration

by

Parts.

Reduction

Formulas

299)

is an odd number, the obtained reduction formula reduces

Hm to)
:rt

HI= cosx dx = I,
\037

o)

therefore)

Hm = (m-I)f!
mil')
reduction formula transforms

If m is an even number then the

Hm

into)

j[
2

Ho=Sdx= ,
\037

o)

therefore)

(m-I) ..::

H =

II

Tn

6.6.to.Compute the

2.)

mIl

integral)

1=

1t

xsinmxdx

\037

o)

(m a natural

number).

t4 and
results of Problems6.5.

000)

Solution.Taking advantage

we get)
6.5.13,

of the

1t

1t

1=S xsinmxdx=

S sinmxdX=JtS sinmxdx,

\037

which, taking into considerationt he result of Problem


n2

\302\267

\"2

(m

- I)!!. .
m!!
If

I=\037xsinmxdx=
Jt (m-l)fI.
If m
o

mil

m IS
IS

6.6.9,gives

even,

odd.

6.6.t 1.Compute
na tural

the integral

1n =

\037

xm (In x)n dx; m

> 0, n

is a

number.
the integrand f (x)-== xm (In x)n
can be made continuouson the interval)))

Solution.First of all note that,

has no meaning at

x= 0

it

though

[0,1]for

any

by virtue of
m

/. The Definite / ntegral)


m > 0 and n > 0, by putting f (0)==O. Indeed,
!irnxm(lnx)n=!irn
(x lnxr=O
x -+0)
x-++O
Ch. V

300)

Problem 3.2.4.

follows that the integral In existsat


we integrate by parts, putting
dv==xmdx,
u=(lnx)n,
xm + 1
n (In x)n-l
du=
dx, V=

Hence,in particular, it

> 0, n > O. To compute it

m+l

Hence,
1
Xm

')

+ 1(In x)n

n
xm(1nx)n-ldx=ln_l'
m+,
m+'
S
o-m+'o
In=\037xm(1nx)ndx=
The formula obtained reduces In to In-I' In particular, with a na\\

tural n,

taking into account that


1

I
/ 0 = S xm dx = m+1
,
o)

we

get)

In=(_I)n(m+ ;)n+l'
n

6.6.12,
Compute the
where m and

Solution.Let

integral

Im.n=o xm(l--x)ndx.
\037

are non-negative integers.


us

put

(l-x)n=u; x dx =dv;xm+l
v= m+I
du=-n(1-x)n-1dx;
m

')

Then)
1

1m.n =

:n\037J,

(1--x)n]+
\037

m +1

m\037 I

Sx

(l-x)n-ldx =

o)

m\037

+1.\"-I'

- 1.

If n
The obtained formula is valid for all n > 0, and m::---is a positive integer, then, applying this formula successivelyn
times,we get

(n-I)

/m,n=m+ 1/ m+ 1, n -1 = (m+l)(m+2)Im+2,n-2=='\"
n

..- (n-I)...[n-(n-I)]
n

\302\267

.(fn-t- n )
(m+ 1)(m+2)..

m +n

O')))

\037

But)

6.7.Approximating

301)

Integrals)

-m+n+1 -m+n+1

-5
lm+n,o1

xm +n + 1

xm+ndx

o)

Hence,)

Definite

\302\267

.3.2.} .
(n-I)(n-2)..
Im, n = (m+ I)n(m-t--2)..
.(m+n)(m +n+I)

The obtained result,


in the form)

non-negative integer,can be written

m a

with

mln!
'
1 , n == (m+n+
1)1
m

6.6.13.
Compute the integrals:
(b) (x-l)e-xdx;
(a) arctanV'xdx;
o
1

\037

\037

0)

Jt

(c) 5

xdx
sin2

(d) xarctanxdx;

x;

\037

3t

o)

4)
:rt

(e)

x n (1+
1

\037

x2

) dx;

(f)

In (1-\\- tan

\037

x)dx;

o)

o)

:rt

16

(g) sin 2x arc tan (sinx)dx;

(h)

\037

arc tan
\037
1)

Prove that
6.6.14.
1

-{Vx-ldx.

=
5 (arccasx)n dx n(
Prove
6.6.15.

lowing formula

that

r-l- (n-l)S (arccasx)n-2

if

is valid

xf\"

(n

> 1).)

0)

f\"

(x)

is continuous on [a, b],then the fol-

\037

dx

\037

(x)dx = [bf' (b)

- - f (b)]

[af'(a)

f (a)].

a)

\037

6.7.ApproximatingDefiniteIntegrals
1.Trapezoidal
interval [a, b]
. formula. Divide the b-a

...,

into n equal
k==O, 1,
n, and
parts by pOints xk=a+k h, where h==---,
n
apply the formula
b

b-a

Jf\037)dx\037l1
[ 2f(xo)+f(xt)+...+f(xn-l)+\"2f(xn)
]
\302\267

t1)))

Ch,

302)

error R

The

J. The Definite

Integral)

this formula is estimated as follows:


M,
. where M, = sup I f\" (x)
IR I
l(\037;a)a
in

\037

a\037x\037b

(assumingthat the second derivative is bounded).


2. Simpson'sformula. Divide the interval [a, b] into 2n equal
b
parts by points Xk = a+kh. where h = 2n a, and apply the formula
h

dx
S f (x)
a

{f (xo) + f (x,n) + 4 [f (XI) + f (xa ) +

6n

\302\267

\302\267

)+...
+f(X - )]}.

+f(X 2n -1)] +2 [f(x2)+f(x4

that

Assuming

...

b
\037

is estiInated

(x) existsand is bounded, the error


the following way:

in

flv

in

h
IRI M4(b-a)O
180(2n)4 ' were
\037

M4

2n

If

IV

this formula

X.
)1

a\037\037\037b

6.7.1.

Approximate the integral

formula at

n ==

10.

I = S I':X using the

trapezoidal

...,

Solution. Let us tabulate the values of the integrand,


= 0, 1,
10) being calculatedwithin
Yi = f (Xi) (i

nates

cimal places.)

I +x&

Xi

0.0000
O.]000
0.2000
0.3000
o 4000
0.5000

I
1
1

.0000
.1000
.2000
.3000
.4000
.5000

Yi=r+x

1 +Xi

Xi
I,

0.6000
0.7000
0.8000
0.9000
1 0000

I 0000
0.9091
0.8333
0.7692

0.7143
0.6667

I
I
I

.6000
.7000
.8000
.9000

2.0000

ordi-

the

de-

four

Yi

=I

+ Xt

0.6250
0.5882
0.5556
0.5263
0.5000

-----)

Using the trapezoidal formula, we obtain


1

/=5
o)

I \037x

\037
l\037

1.000\302\2601\302\260.5000

+0.9091
+0.8333+

+ O.5S56+
+ 0.6667
+ 0.5882
+ 0.6250
+ 0.7692
+ 0.7143
;::::;
0.6938.)))
+ 0.5263= /0 6.9377= 0.69377
)

\302\267

6.7.Approximating

\037

Definite

303)

Integrals)

have f\" (x)=


(I \037x>3
t.hen f\" (x)
2. Consequently,we may take the
Sincea x
number 2 as M 2 and estimatethe error:

Estimate the error


\037

\037

the result

in

1,

\037

obtained.We

\302\267

\037

2 = 1
12X 102 600

< O.00 17

\302\267

We calculated the ordinates accurateto four decimal places,and


0.00005
= 0.00005more
the round-off error doesnot exceed 10 (1+ 9

. 9 = 0.000045,
since the

precisely,

O.O\037\03705

xl)

ordinatesYo and

YIO

are

exact numbers).Thus,

the total error due to using the trapezoidal


18.
and
formula
rounding off the ordinates does not exceed0.00
Note that when computing the given integral by the NewtonLeibniz formula we obtain)
1

dx
1=S I+x
=In(l+x)0=ln2\0370.69315.
1

o)

i.

Thus, the error in the result obtained does not exceed0.0007,e.


we have obtained a result accurateto three decimal places.
1.5
eO.IX
the
dx
6.7.2.Evaluate by Simpson'sformula
integral S
0.5
accurate to four decimal places.
accuSolution.To give a value of 2n which ensures the required
e 1x
=
we
we
find
flv (x). Successivelydifferentiating f (x)
get
racy,
x

-,
O

flv

3
2 -2.4x
lx'-O.004x
+ 24)= p ;:>e .IX ,
+ 0.12x
(0.000

(x) = eO;:x

P (x) is

the polynomial in parentheses.On the interval


1X increasesatld therefore reaches
the function (x) =eO.
= eO.IO
=
The upper estimate
its greatest value at x
(1.5)
can be
of the absolute value of the polynomial P (x) divided by
obtained as the sum of moduli of its separate terms.The greatest
therefore
value of each summand is attained at x=
0.000-1 0.004 0.12 2.4 24
P (x)
where

[0.5,1.5]

< 1.2.

1.5:
cp

cp

xC>

0.5,

xO

<

+ x4 +
0.0002
+ 0.016+ 0.96+ 38.4+ 768 < 808.
X 808
< 1 000.Hence,the number 1 000may
(x) < 1.2
x

x2

\037

And

so,

flv

be taken as M..)))

x3

XO

\037
\037

Ch. V

304)

/. The Definite

/ ntegral)

We have to compute the integral accurateto four decimal places.


To ensure such accuracy it is necessary that the sum of errors of
the method, operations and final rounding off should not exceed
For this purpose we choosea value of 2n (which will de0.0001.
termine the step of integration h) so that the inequality
I

= 5.1O1<+ 0.0001
1i)

\302\267

is sat isfied.

Solving the inequality

lox 000 < 5 X


1

180(2n)4

we obtain)

10

0
,)

> 19.
20;then the step of integration h will
b-a= ] = 0.05.
h
2n

Let us take

2n ==

==2n

be equal to

20

more accuratecalculationshows that at 2n == 20

/R/<3.5xlO-o.

i. e.

with an error
If we calculateYi within five decimal places,
then the error of the final rounding off will
not exceeding
also be not greater than
Thus, the total error will be less
0
than
X
eO. IX
Now compilea table of values of the function y==-Xfor the va10-\302\260,

1.
4.5 10-< 0.000

lues of x from
are carriedout

0.5to 1.5with
within

0
1

2
3
4
5
6
7

8
9
10
11

10-\302\260.

Xi

0.50
0.55

0.60
0.65
0.70
0.75

0.80
0.85
0.90
0.95
.00
1 .05
I

five

The ralculat ions


the step h == 0.05.
decimal places.)

O.1 Xl

0.050
0.055
0.060
0.065
0.070
0.075
0.080
0.085
0.090
0.095
O.100
o 105

\037o.

IX,

Yi

.05127
.05654

.00184
] .06716
1.0725]
I .07788
I .08329
I .08872
I 09417
] 09966
7
I 1 051

.
.
.
l.1107

2.lO254
I
I

.92098
.76973

. 178

.53216

6\037

.4:3717
l.35411

.28085

1.21574 !
I

.15751
.10517

.05782)))

6.7.Approximating

\037

17

18
19
20

.10
.15

I
I

0.110
0.115

1.20
1.25
] .30
1.35
1.40

o 120

O.125
O.130
O.135
O.140
O.145
O.150

1.45
1

.50

305)

Integrals)

eO'1xi

O.lx,

Xi

12
13
14
15
16

Definite

.11628
.12187
1.12750
.13315
.13883
.14454
I.15027
.15604
.16]83
1

Yi

1.01480

0.97554
0.93958
0.90652
0.87602
0.84781

0.79727

0.77455)

I
I

0.82162

For pictorialness sake we use the tabular data to compile the


following calculationchart:)
Yi

0
I

2
3
4
5
6
7

8
9
10
II
12
13
14
15
16
17

18
19
20

Xi

0.50
0.55
0.60
0.65

at

l=O and

i=20

2.10254

.50

Sums

.28085

1.15754
I

.05782

at an even t

.76973

.53216

.35411

1.21574

.10517
l.0 480

I
I

1.45

0.80
0.85
0.90
0.95
1.00
1.05
..15
10
1.20
1.25
1.30
.35
.40

.92098
.64178
.43717

0.70
0.75

I
I

at an odd l

0.97554
0.90652
0.84781
0.77455
2.87709

0.79727
12.02328

0.93958
0.87602

0.82162
10.62893)

Using Simpson'sformula, we get

1.5
I
eO.IX
+
+ 4 x 12.02328
(2.87709
xdx\037
60
5
0.5)

.
=
+ 2 X 10.62893)::;::
60 72.22807 1.2038.)))
1

Ch,

306)

I. The Definite

Integral)

6.7.3.

The river is 26 m wide. The table below showsthe successive depths of the river measuredacross its sectionat steps of 2 m:)
0

12 14 16 18 20 22 24 26

10

0.3 0.9 .7 2. 2.8 3.43.33.03.52.9 .7 .20.80.6)


I

Herex denotes the distance from

one bank and y, the correspondthe mean rate of flow is


ing depth (in metres). Knowing that
m/sec,determine the flowrate per second Q of the water in the
river.
Solution.By the trapezoidal formula the area S of the cross-section

1.3

26

S=

ydx
S
o

\037

+ 0.9+ 1.7+2.1+ 2.8+3.4+


2[ (0.3+0.6)
+ 0.8] = 55.5(m 2).)
+ 3.3+ 3.0+ 3.5+ 2.9+ 1.7 + 1.2
\037

Hence,)

1.3

72 (m 3/sec).
Q = 55.5X
It is impossibleto estimate the error accurately in this case.Some
indirect methods of estimationenable us to indicate approximately
the order of the error. The error in S is about 3 m 2, hence, the
error in Q is about 4 m 3/sec.
\037

6.7.4.Compute the

following integrals:)

J1

x
(a) S Si; dx accurate to three decimal places,using Simpson's
IT

formula;)
1

(b)

\037

rx2dx accurateto

three decimalplaces,by the trapezoidal

formula.

6.7.5.
By Simpson'sformula,
1

J=

approximate the

.36
\037

f(x)dx,

1.05)))

integral)

6.8.Additional

\037

if

\037

307)

Problems)

the integrand is defined by the following table:)

.10

.15

1.20 1.25 1.30 1.35

) .05

t (x)

2.36

2.50 2.74

3.46

3.04

3.98

4.6)

6.8.AdditionalProblems
6.8.t. Given the funct ion)

I-x

f (x) =

\302\260

{ (2-X)2

Check directly that the

at
at
at

O\037x\037

1,

< x< 2,
2 < x\037 3.)
I

funct ion)
x

F (x) =

\037

f (t) dt

[0,

is continuous on the interval


3] and that its derivative at
interior point of this interval existsand is equal to f (x).
Show that the funct ion

6.8.2.

;<
::
-I
<

f(X)={
\037

\037;:

(\037an

on the interval

6.8.4.A
point x

angle

\037

=a

[a, b],

then

I,

\037

at

is integrable on the interval


6.8.3. one assert that

each

x==l

[0, 1].

if
it

a function is absolutely integrable


is integrable on this interval?

line tangent to the graph of the function iJ = f (x) at the


the axis of abscissasand an
with
forms an angle
\037

at the point x = b.
b

Evaluate

f\"

\037

(x)dx,

if

f\"

(x)

IS

a cant inuous fund ion.

a)

6.8.5.Prove that)
x

S E (x)
o)))

dx = E (x) (E2(X)-I)+ E (x) [x-E(x)].

Ch

308)

Given
6.8.6.

ct ions
FI(x).=

J.

The Definite

Integral)

Jt

5 1+cos2 x . Make sure


dx

the integral

that

the

fun-

o)

cas
arc cas V2
and F (x) = .r--arc tan .r
.r2
2
r
r 2
V l+cos
x
2x

tan x

\"

are antiderivatives for the integrand. Is it possible to use both antiderivatives for computing the definite integral by the NewtonLeibniz formula? If not, which of the ant iderivat ives can be used?

6.8.7.For f (x) find such an antiderivative which attains the given


magnitude y = Yo at x = Xo (Cauchy's problem).)

6.8.8.At

what

value of

I)

; is the equality

e2X dx=e2\037
\037

(b-a)ful-

a)

filled? Show that)


t

10

6.8.9.
Investigate the
x
F (x)=

\037

> a+b
2')

function defined by the definite integral

VI-t'dt.

6.8.to.Show that

the inequalities
1

I
o 692::::;;; XX dx::::;;;
\037

o)

are valid.

6.8.1
t. With

the aid of the inequality

x sin x
\037

\037
\037

Jt

2
\037

2') show that < 5o dx <


6.8.12.
Using the inequality sin x x- x;
.n \\

sin

2\"

\302\267

\037

Bunyakovskyinequality, show

x ( 0::::;;;
x::::;;;

(x\037O)

and the Schwarz.

that)

:rr

1.096<
Vxsinxdx< 1.111.
\\
o)

6.8.13.

Assume that integra ble funct ions PI (x),P2 (x),P3 (x), P4(x)
are given on the interval [a, b], the function PI (x) is non-negative,)))

\037

6.8.Additional

309)

Problems)

and the functions P2 (x), P3 (x), P4 (x) satisfy the


P3 (x)

Prove

\037

P2 (x)

\037nequality

P4 (x).)

\037

that)

\037

/J

Pa (x) PI (x) dx

\037
\037

P2(x) PI (x) dx

P4

\037
\037

(x) PI (x) dx.

a)

Let
6.8.14.

Prove

that

the function f(x) be positive on the interval


the expression
h

5f

(x)

reachesthe least value

[a, b].

dx.5

f\037:)

a)

only

f (x) is constant on

if

this interval.

Prove that)
6.8.15.
:rt

arc tan x

5o

dx == 2
\037

-!-

5 sIn t dt .
0)

6.8.16.

Prove that one of the ant iderivat ives of an even funct ion
is an odd function, and any antiderivative of an odd function is
an even funct ion.
Prove that
6.8.17.

f (x) is a continuous

if

period T, then the integral

I=

a+T
\037

f (x) dx

periodicfunction

with

does not depend on

a.

a)

Prove
6.8.18.
through
b

uv\\fl}
\037

a)

that

if

u===u(x),

v=v(x) and their derivatives

order n are continuous on the interval [a, b], then)

dx = [uv<n-u -u'v<n-2)+ ... +

(-1)n-lu(n-II v]

I\037

+(

1)

U (11) V
\037
a)))

dx.

Chapter7
APPLICATIONS

OF THE DEFINITE

INTEGRAL)

7.1.Computingthe Limitsof SUlnswith


the Aid of DefiniteIntegrals

\037

It is often necessary to compute the limit of a sum when the


number of summands increasesunlimitedly. In some casessuch limits can be found with the aid of the definite integral if it is possible to transform the given sum into an integral sum.
For instance, considering the points \037,
, n as points of
n
n

2,

....!!:..

division of the interval [0.1]into n equal parts of length


for each continuous fun,ction f (x), we have

\037x

= -!z.

[f ( )
COlnpute

}\037\037

7.1.t.

-k-

\037

Ii rn
f1.

-+

\037

00

+f ( ) +

\302\267

\302\267

\302\267

\037

2n
n

\037

.....,

in'::+ sin
n

-+-f
( ) ] = So

+ . .. + sin (n

1)n

(x) dx.

1.

Solution. The numbers in brackets represent the values of the


function f (x) = sin x at the points

=-; =-,
n

2n

X2

xn

- = (n-I)
1

11

n)

==.::.
subdividing the interval [0,Jt] into n equal parts of length
n
nn
Therefore, if we add the summand sin n = 0 to our sum, the latter will be the integral sum for the funct ion f (x) = sin x on the
\037x

interval [0,Jt].
By definition, the limit of such an integral sum as n -+ 00
definite integral of the funct ion f (x) = sin x from 0 to n:
n

lim
n

-+

(1J)

..n

n ( sIn n + SIn n +
\037Jl

\302\267

\302\267

\302\267

. (n
+ sIn

i\037

the

. nn =
+ sIn
n)

1)n

:11

= S sinxdx=-

cosxl:= 2.

o)))

\037

7.1.Computing

7.1.2.
Compute the
.
1m

limit

...

Solution.l'ransform the
I

1
_+I

Y4n2-1

+
4n 2 -22
( y 4n 2-1+ y__
I

n-+oo

Y4n 2

-2 +
2

sum

311)

the Limits of Sums)

+ y 4n 2-n2) .
1

\302\267

\302\267

\302\267

parenthesesin the following way:

in

+_Y4n 2 -n2 _
I

+ -.f
+\"'+-of4- '
/
4( V 4-fi2
( y)
(n2Y

=n

\037

The obtained sum is the integral sum for the function f (x) =
=
on the interval [0,I] subdivided into n equal parts.
2

4-x
V\037

The limit of this sum as n \037 00 is equal to the definite integral


of this funct ion from 0 to 1:)

I 1m

n- (
00

Y4n 2

-1+

___ + . .. + Y4n 2-n2 ) =


Y4n 2 -2
I

:\"

/)

=../
S
1

\037

\"

\037

II +

. _x I =
=arc SIn
2
0

_n6

\302\267

o)

7.1.3.
Compute
}\037moo

x2

n\0373

+Y

n\0376

+Y

n\0379

]'

+'\" + V n+3\037-1)

Solution.Transform the given expression in t he following way:


\037

l l-t===

-./+ + -./+ +
n

-.

\037

r-

+ 0 +..
V
I

I
1

.. -./

2.+
n

\302\267

\037

- -

+3 (n 1)
+ ' + -.

+\037
n

.. /
1

I
1

--

+3 (n n I)J

\302\267

The obtained sum is the integral sum for the funct ion f (x) =
= V 1 on the interval [0, therefore, by defin it ion,
\037x

}\037noo

\037

33

3];

(1+Y
=

n\0373

j'o Y

+Y

l\037x

dX=

n\0376

+'

.. Y
+

n+3\037n-lJ

J(l+X)-2dx=2VI+x :=4-2=2.
0)))

Ch.

312)

II. Applications

7.1.4.
Using the

Integral)

definite integral, compute the following

.. . + n +n ) ;

lin1
(a) n-+oo
( n + I + n +2 +
-. 1
-.
lim J.1

(V +n +

(b)fl-+OOn

of the Definite

\037

limits:

/+

- ..

\037

+ .. . + -.V 1 + n ) ;
\037

. + V + V 3+ . + V .,
V 4
.
21t
1 cas -1-cas + . + cas(n-I)1t'.
(d )
2n
2n ( +
2n
2n
)
.
n
(e)
[(n + 1)2+ (n+2)2+. (2n)2 J
ni,
7.1.5.
Compute the limit A = Iim
(c)

I Hn

-+

\"2

-\037

00

1t

11

..

\302\267

n l\037oo

fl

\302\267

-t--

\037\037

Solution.Let

n-+

us

\302\267

V n.)
n

00

take logarithms
nj

InA=Iimln v n n! =Iim

l.

n
n
n
[ ln\037+ln\037+...+ln\037
the
The expressionin brackets is the integral sum for
integral
\037

n-+oo

fl-+OO

In
\037

Consequently, In
\037

xdx= (xIn X-X)I\037=-1.

=-1and !irn

fl-+

V;;J =e-1.
n)

00

7.2.FindingAverage Values of a Function


The average value of

(x) over the interval

[a, b]

is the number

f.t

b-aJCf (x) dx.


a

The square root

f
\\

\037

()

b-aj

[f

(X)j2 dX

\\

of the average value of the

square ot t he function is calledthe root mean square (rms) of the


function f (x) over [a, b].
Find the average value
the interval [0,

7.2.1.

Solution.In

1].

this

case

-5
1

f.l

c)))

f.t

of the function f (x)== V x over

\037

3x 3
3 r
V x dx == 4

= \"43

\302\267

7.2.Finding

\037

Average Values of

313)

Function)

7.2.2.Find

the average values of the functions:


(a) f (x)
[0,2Jt];
1
==
(b) f (x) eX l over [0,
==

sin2 x over

2].

the
7.2.3.Determine
x2

average length of all vertical chords of the


lover the 1nterval a x 2a.

y2

==
b2

hyperbola a2
Solution.The problem consists
f (x) ==2y -==

the function

-2a

f.t

== 2

() b

2
a J a Vx
\\

[; V x -a
2

:\037

7.2.4.Find

the interval
Solution:)

a)

\037

the average
in finding
2 over the interval

-a

[a, 2a]:

a; In (x + Vx

-a ) l:a=b [2 V3-ln(2 + V3)].


2

the average ordinate of the sinusoid

[0,jt].

n
n
5o sinxdx==-.!.cosx

=\037

==\037

= sin x over

rr
f.t

value of

-a dx=

a)

2!!.V

x2

\037

\0370.637.

0)

Rewrite the obtained result


!.t

\302\267

Jt

in

the following way:)


;r

\302\267

\037

Jt

= sin xdx.
5
o)

Using the geometricmeaning of the definite integral,we can say


that the area of the rectanglewith the altitude f.t ==
and the base
n
Jt equals the area of a figure bounded by a half-wave of the sinusoid y sin x, 0 x Jt, and by the x-axis.
7.2.5.Find the average length of all positive ordinates of the
\037

===

\037

\037

circlex2 + y2 == 1.

7.2.6.

Show that the average value of the function f (x), contiis the limit of the arithmeticmean
nuous on the interval [a,
of the values of this function taken over equal intervals of the
argument x.

b],

...,

Solution.Subdivide the interval [a, b] into


b a
(i=O,
n).
points xi=a+i
n)))

1,2,

equal parts

by

the

Ch. V

314)

II. Appl icat ions

of the Definite

...
..

Integral)

Form the arithmeticmean of the values of the function f (x) at

points of division Xo, Xl'

=f
I1n

(X o

-:

xn 1

+ f (Xl)

\302\267

(Xn-l)

\037

n-l
= L.f (Xj).
;=0
\037

This mean may be representedin the following form:


\037n

==

n-l

b-a L.f (Xi) dXit


i=O)

where)

b-a.
[\\X.=I

n)

The latter sum is the integral sum for the function f (x), the-

refore)

tim I1n

n-oo

which

=b

n-l

f (Xj) /1xj = b
a n-oo
i=O

I_ Hm

L.

r
aJf
a

(x)dx = 11,

the solution.
completes

7.2.7.Find the average value of pressure (Pm) varying from 2


10atm if the pressureP and the volUllle v are relatedas follows:
3
pv = 160.
Solution.As p varies from 2 to 10atm, v traverses the interval
[4V4, 4V IOO]; hence

to

2\"

4
Pm

t;/ 00
1

-V-) 4V

,f

4 (V\037
100

3
-2'
dv =

160v

-- (V-100-r-) -+ Vt./.- -_ V-(V-10+V-)


320

V 4

too

40

20

___

\"\"

4.32atm.

7.2.8.In hydraulics there is Bazin'sformula expressingthe velocity v of water flowing in a wide rectangularchannel as a function
of the depth h at which the point under considerationis situated
below the open surface,
v=v o

20 VHL

( )2,
\037

where 00 is the velocity on the open surface, H is the depth of


the channel, L its slope.
of the
Find the average velocity Vm of flow in the cross-section
channel.)))

7.2.Finding

\037

of a

Average Values

Solution.We have
H
Vm

=
l\037

o
S
lV
o

-20VHL (

YJ

\037

315)

Function)

dh = v o

VHL.

230

7.2.9.Determine the

average value of the electromotiveforce


over the time from t = 0 to t = T, if
over one period,
electromotive force is computed by the formula

i.e.

Em

T'

E = E0 sIn 2nt

where T is the duration of the period in seconds,Eo the amplitude


(the maximum value) of the electromotiveforce correspondingto
the value t = O.25T.
The fraction 2;t is calledthe phase.
Solution.)
T

Em

. 21tt EoT
..2 T =0.
=y So sIn ydt = T.21t
-COSy
[
]0
1tt

Eo

Thus, the average value of the electromotiveforce over one period equals zero.
Each of the two vertical poles OA and CD is equipped
with an electriclamp of luminous intensity i fixed at a height h.
The distance between the poles is d. Find the average illumination
of the straight line OC connectingthe bases of the poles.
7.2.1 Find the average value of the square of the electromotive
over the interval from t = 0 to t = (seeProbforce

7.2.10.
t.

(\302\2432)m

\037

lem

7.2.9).

Solut ion. Since)


we ha ve)
2
(\302\243

2.

)m == T

21tt
E = Eostny,)

\302\243

2\"

2
0

S
o

. 2 21tt dt
sIn
T

T
2

-TO'
2r

\037

41tt

cos T
2

\302\243

dt

[tdefined on
\037

7.2.12.
If

[0,00), then

0)

a function f (x) is
its average value will be
b

\037

=
l\037\037

-}Sf (x)dx,
o)))

\037

an

sin 4;t o; =
J
infinite

\037\037

interval

316)

Ch,

this

if

II. Applications

Integral)

exist.s.Find

limit

the average power consumption of an


the current intensity I and voltage u
the following formulas, respectively:

circuit
alternating-current

are

of the Definite

expressedby

if

I = I cos + a);
= Uo cos + a + ),
(ffit

(ffit

cp

where
is the constant phase shift of the voltage as compared
with the current intensity (the parameters
and a will not enter
into the expressionfor the average power).
Solution. The average power consumpt ion
cp

ffi

wm=!im
locos(cot+a)uocos(CJ)t+a+cp)dt.
T-+oo S
0
Taking into considerationthat

cosa cos = 2\" [cas(a + P) + cos(a


1

\037

we will

\037

)]

,)

get)
T

Wm

= !im

5o [cos(2(1)t + 2a+ cp) + coscp] dt

T-+oo

I\037\037o

.
_ 1m

lou o
T-+oo 4w

. sin (2wT +2a+


T

sin

(2a+

lou o

_ lou o COS

+ 2 COS I 2
{
Hence,it is clear why so much import ance is at tached
quantity cos in electricalengineering.
I

<p)

<p)

cp

cpo

to the

cp

Find
7.2.13.

the average value

indicated intervals:

f.t

of the function f (x) over the

(x) = 2X2

+ lover [0, 1];


(b) t (x)=+ over [l, 2];
x
-2x+3over [0,2].
(c) f{x)=3

(a) f

A body falling
7.2.14.

to the ground from a state of rest acquis =SI' Show


res a velocity
1 on covering a vertical path
the average velocity Vm over this path is equal to
that
v t = V2gs

2\0371

The cross-section
7.2.15.
of the trough has the form of a parabolicsegment with a base a and depth h. Find the average depth

of the trough.

Find the
7.2.16.

average value Im of alternating current intensity


over time interval from
to
(see Problem 7.2.12).
\037

\302\260

w)))

\037

7.3.Computing

7.2.17.Prove

Areas

Coordinates

in Rectangular

317)

the average value of the focal radius of an


b2
and 8 is
ellipsep = 1 ccos , where p = a ; a, b are the semi-axes
eccentricity,is equal to b.
On the segment AB of length a a point P is taken at a
distance x from the end-point A. Show that the average value of
the areas of the rectanglesconstructed on the segments AP and
a2
PB is equal to \"6

that

<p

7.2.18.

\302\267

7.2.19.
Find

the average value of the function


cos2 x
f (x) == sin2 x+4 cos2 x

over the interval

\037

to
\037

the

within

\037

that

this average,equal

certainx =

\302\243

lymg

indicated interval.)

7.3.Computing Areas in Rectangular Coordinates


Y2

(x) (a

\037

by the straight lines x= a, x ==


==Y2 (x), provided Yl (x)<
=
Y
Yl (x), Y

is bounded

If a

figure
= b (a <plane
b) and the
\037

]. Check directly

lO,
value
of the function f (x) for a
, is the

curves

x < b), then

its area

is computed by the formula

S=
\037

[Y2

(x)-

Yt

(x)]dx.

a)

In

certain casesthe

x = b)

left boundary x ==a (or the right boundary


can degenerate into a point of intersectionof the curves)
c)

!J)

!I)

J])

: 11::111
(x)
x)

I)

o)

x)
a)

(a))

b)

o)

a)

b)

(b))

Fig.

65)

and Y = Y2 (x). Then a and b are found as the abscissCls


points of intersectionof the indicated curves (Fig.65,a, b).
7.3.1.Compute the area of the figure bounded by the straight
lines x==O,x=2 and the curves y=2x , y=2x-x2 (Fig. 66).)))

= Yl (x)

of the

Ch. V I J.

318)

Jntegral)

of the Definite

Applications

tained
Y

= 2x

2x-x

2 is atmaximum of the function Y =


=
at the point x 1 and is equal to
and the function
1 on the interval
we have

Solution.Since the

1,

[0,2],

\037

S = [2X
S
o

-(2x-x )] dx =
2

7.3.2.
Compute the

l\037x2

area of the

-(x - ) I:= -

1:

figure

X=_2y2, x= 1-3y2 (Fig. 67).)

X;

I\037

\302\267

\037

bounded by the parabolas

II)

4)

I:;
<v
II
\037)

o)

z)

-2)

2)

!J=2x-:x

1)

1)

x)

66)

Fig.

Fig.

67)

Solution.Solving the system of equations


J
\\

X=2y2;

x== 1

3y2 ,

find the ordinates of the points of intersectionof the curves Yl =


for
\037y\037 1,then we have)
Y2= 1.Since 1-3y2\037-2y2

-1

s= S [(1-3y2)_(-2y2)]
dy=2(y_
7.3.3.Find

-1

) I\037=

:.

the area of the

contained between the


parabola x2 ==4y and the witch

figure

of Agnesi

\0373

-1,

2 4
XI
\037

x2=4!J)

(see Fig.

!J-x
_

68).
Solution. Find the abscis-

sas of the points A and C of


intersection of t he curves. For
this purpose eliminatey from

+4

-2

Fig.

68)

X)))

7.3.Computing

\037

Coordinates

Areas in Rectangular

319)

the system of equations)

Y=X2\0374'
x2

w h ence

8
x2

- --.

{ Y=T'
x2
4

' or x + 4x2
<I

32

-2

The real roots of this equation are the points Xl =


and
x2 =2. As is seen from the figure. X2\0374
on the interval
(It is also pDssibleto ascertainthis by directly computing
the values of these functions at any point inside the interval, for
\037

[-2,2].

\0372

instance, at X = 0.)
Consequently,
2

s= S C2\0374
-2)

7.3.4.Find

= x2 + 1 and

\0372

dx = 4 arctan

\037

\037\037

1\0372

=2n-

\302\267

\037

the area of the figure bounded by the parabola


the straight line x + Y =

3.

7.3.5.

Compute the area of the figure which lies in the first quadrant inside the circlex2 y2 = 3a2 and is bounded by the parabolas x2 = 2ay and y2 = 2ax(a 0) (Fig.

>

If)

69).
Solution.Find the abscissa of the po-

int

of

= 2ax

avJ)

intersection of the parabola


2
2

and the circle x


y2 ==3a
the
from
Eliminating y
system of equa-

y2

t ions)
2
( x

+ y2 = 3a2,
y2

\\

= 2ax

:x)
,)

+2ax-3a

2
we obtain x2
=0,=whence we
Fig. 69)
xA
Analoget the only positiveroot:
gously, we find the abscissa of the point D of intersectionof the
circle x2 + y2 = 3a2 and the parabola x2 = 2ay; xD = a V 2.
Thus, the sought-for area is equa I to)

S=

a.

aV7
[Y2
\037

(x)

Yl

(x)]dx,

o)

= X22a '
were
h
Yl (X )

==
Y2 (X)
')

Vr 2ax

t V

for

O\037x<a,
2
< x a V-.)))

3a2 x 2 f or a

\037

Ch. VII.

320)

of the Definite

Applications

Integral)

By the additivity property of the integral


aV2
u
S = S ( V2ax ) dx S (V 3a2 x2
) dx =
a
o
3
,.
X
X3 a
2
2
==
V 3a2
'2
3
6a
0
]
a
[
[2

--

- -

\037:

\037:

V2a.-x2---+

-x +-arcsin J!_3
3d\037

--]

2
x3 a V=
6a a

.
J!2
V-2
a2 3a2
2
= 2 V-2
arcsinV3 -arCSin
3 a -6'+T(
J!3) -\037a2+6a2=
I

.1

2
_ V2 + 3
(\037 2\"arCSlng) a.

Herewe make use of the trigononletricf ormula:


2
arcsina-arcsin\037
=arcsin(a
Jlrl_\0372_\037 VI-( )

(a\037

> 0)

for transforming)

. -. /2 -./2
.
/23-arcsln
=arcsln
( V\"3 3-V3 Y3. ) =
va

. -.
arcsIn
V

. ]
= arcsin
g

7.3.6.Compute the

.)

area of the

figure lying in the first quadrant


and bounded by the curves y2 = 4x,
x2 == 4y and x2 y2 = 5.)

!J

7.3.7.Compute the area of the


bounded by the lines y ==
x+ 1, y=cosx and the x-axis

figure

-1)

o)

(Fig. 70).)

:&

1C/2

Solution.The

Fig. 70

x+ 1
= f (x) = ( cosx if

if

-1

\037x\037O,

< x<

of the curvili-

\037

rr

rr

-2

s= S f(x)dx= S (x+ l)dx+S cosXdx=


-I
-1
7.3.8.Find the area

the line

\037

[-1,] . The area

is continuous on the interval


near trapezoid is equal to)
2

funct ion)

(X\037I)2

of the segment of the

x = 2 is the chord determining

+sinxlo; = ;
curve y2 == x3 --x2 if
\302\267

1\0371

the segment.)))

7.3.Computing

\037

A reas in Rectangular

Solution. From the equality y2 ==x2


x (x-I) 0, therefore either x ==0 or x
2

Coordinates

321)

follows that
(x1.In1) other words, the
it

\037

\037

domain of definition of the implicit function y2 == X 3 _X 2 consists


of the point x == 0 and the interval
00). In computing the area
the isolated point (0, 0) does not play any role, therefore, the
interval of integration is
2] (see Fig. 71).
we see
to
over
explicitrepresentation y == -I- xV
Passing
and
that the segment is bounded above by the curve y == x V
below by the curve y=
Hence,)

[1,

[1,

x-1,
x-I

-xVx-I.

x-l-(-xVx-I)]dx=

S = [xV
\037

\037

xV

x-Idx.

Make the substitution)

!I)
x)

t)

x-l==t2,

\037'

dx= 2t dt,)

2)

I)

I.\037
\037)

Then)
1

S= 4

2
2
=4
[ +
5o (t + I) t dt
\037

\037

o)

] = 7;

7.3.9.

31)
7)

\302\267

Determine the area of the figure


bounded by two branches of the curve
(y- X)2 ==x 3 and the straight line x =
Solution.Note first of all that y, as an
Fig. 71
implicit funct ion of x, is defined only for
x 0; the left side of the equat ion is always non-negative.Now
we find the equations of two branches of the curve y==
x,
and
we
have
Since
x+xVx\037x-xVx,
x\037 0,
y ==x+xV-X.
therefore)

1.

115

\037

x-xV

4
4 2
x
S= 5 (x+xV x-x+xV x)dx=25 x V xdx=-=5
0=5'
o

y2

7.3.10.
Compute
(x 1)2.
-=_

0)

the area enclosedby the loop of the curve

Solution.The domain of definition of the implicit function y is


the interval 0 \037x < + 00.Sincethe equation of the curve contains y to the second power, the curve is symmetrical about the)))

Ch VII. Applications

322)

x-aXiS.The pOSitive branch


Y

= Yl (x)= Vx

Yl

of the Cefinite

(x) is given

by

Integral)

the equat ion

l-x),0
x-l = { VV x(x(x-l),

\037x\037

1,

x> 1.

The common points of the symmptrical branches Yl (x) and Y2 (x) =


= Yl (x) must lie on the x-axis.But Y1 (x) = VX\"
0 only
11-=
at Xl = 0 and at X2 =
Consequently, the loop is formed by the curves
and y = IIX'(1 x), 0 x 1 (see Fig. 72), the area enclosed
bei ng)

1.

- -

<

S= 2

7.3.t t.

Find

f\"-

5o V

x(

t he

= (x-1)(x-2)2.)

x-

y=Vx(l-x)

\037

1-

x) dx = 2

5 (x

-:

1-

) dx

8
= 15
.

0)

area enclosed by

the loop of the curve

x)

x)

D)

Fig.

- - is.+ 3,

Find
7.3.t 2.
2

=
y

Fig.73)

72)

2x
x
and the LJ-ax

the area of the figure bounded by the parabola


the line tangent to it at the point 1\\1 (2,

-5)

-5)

Solution. The equation of the tangent at the point M (2,


or y=7-6x. Since the branches
has the form
of the parabola are directeddownward, the parabola lies below the
3 on the interval [0,2] (Fig.73).
\037-x2-2x+
tangent, e.7
Hence,)

y+5=-6(x-2)
i. -6x

s= 5[7-6x-<-x
-2x+3)]dx=
5<x-4x+4)dx= .
Z

\037

0)))

7.3.Computing

\037

7.3.13.
na t es.
We
7.3.14.

A reas in Rectan{!ular

Coordinates

323)

Find the area bounded by the parabola y===x2-2x+2,


the line tangent to it at the point M (3, 5) and the axis of ordi-

take on the ellipse)


x2

y2

\302\24322+&2\"=

(a > b)

lying in the first quadrant.


the sectorof the ellipsebounded by its semi-major
axisand the focal radius drawn to the point M has an area

a point M (x, y)

Show

that

ab
x
.
S = \"2
arccosa

With the aid of this result deduce a formula for computing the
area of the entire ellipse.)

x)

a)

-1 0
Fig.

Fig.

74)

Solution.We have (Fig. 74):


SOMAO:=: S
+ SMABA1; S
\037OA1B

\037O\0371B

MABA1

(}

==
t.\\y

-xy

\037

2a

Va 2

b
b
2
dx= 5 -J/a2-t2dt==a
2a ( tVa

75)

-x2.
,)

-t +a arcsin-)
2

2
2
l-xVa-x2+a (
.
x
arc COS-,we 0bt aln

= :a

===

SMABM

a)

Hence)

=S

\037OMB

+S

MABM

+ ab arc COSax

- ;
arc sin

\037

[-xVa -x +a arc cos ].)

= :a

SOMAO

X)

.
. _Jt -arcsIn
SInce
a
2

x)

\302\267)))

===

)J

\302\267)

Ch.

324)

II. Applications

of the Definite

Integral)

= 0, the sectorbecomesa quarter of the ellipse,i.e.


ab
j[ ab
arccos0 = ab 2=4
It,
\"4 SeJlipse=-2\"
and consequently,Se1Jipse= nab. At a = b we get the area of a circle
2
Scircle= na .
x2
Find the area bounded by the parabolas y=4x2,
7.3.15.
Y=g
and the straight line y = 2.
At

2'

Solution. In this case it is advisable to integrate with respect


to y and take advantage of the symmetry of the figure (seeFig.75).
Therefore, solving the equations of the parabolas for x, we have:

+ 3 V y.
x=:f:yy
2 ,x=

By symmetry of the

about
figure
SOABO:)

equal to the doubled area

the y-axis the area sought is

\037

S=2S0ABo =2S (3Vy--}V y ) dy=5 S Vy dy= 20


o

\0372

7.3.16.
From an arbitrary point M(x, y) of the curve y=x
(m > 0) perpendiculars MN and ML(x > 0) are dropped onto the

coordinateaxes.What part of the area of the rectangleONML does


the area ONMO (Fig. 76) constitute?)
If)

1)

!!

L)

x)

o)

x)
N)

Fig.

--

-1

--(;i,j;
........\037:-e\037)

76)

Prove that
7.3.17.

Fig.

the

x-axisand half-waves of
geometricprogressionwith

77)

areasSo, SI' S2' S3'. .., bounded by the


the curve y==e-axsinpx,
x\037O, form a
an
the common ratio q = e B.
Fig. 77 intersects the positive semi-axis

Solution.The curve of
Ox at the points where sin pX= 0, whence
xn = n; n = O. t, 2,

...
.)))

7.3.Computing

\037

Areas

Coordinates

in Rectangular

325)

sin px is positive in the intervals (X2k' X 2k+1)


The function y e-a.x
the sign of the function in the
and negative in (X2k+l' X2k+2)'
with that of the number
interval (xn , xn +1) coincides
)n. Therefore

i.e.

(-1

(n+ 1)3t

(n+

Sn=

Iyldx=(-l)nn3t
\037

\037

n3t
B

But

I):rt

e-axsin\037xdx.

\037

-+

the indefinite integral is equal to

e-a.xsin dx =
\037x

e-(J.X

a2

B2

(a sin px + cospx)+ C.
\037

Consequently,
Sn = (_I)n+1

(n + 1 ) 3t

(a sin

= <;;;

[e-a (n+

a\037:\0372

\037;21

\037x

+ cas

\037x)

\037

(-I

] Im/
If

q-s;;--) Sn+l e

ex

which

the
completes

7.3.18.
Find

(_l)n]=

)n+l_eannl/l\037

l)n/fi\037

-a

Hence)

(n + 1 ) :rtJ (3
cxn:rt/(3

proof.

==

e-ann/f:J
\037

e-

\0372

(I+ e-=//I).)

cxrr /(3
,)

the areas enclosedbetween the circleX 2 +y2_2x+


V3.
+4y- 11 0 and the parabola y = x-2 + 2x +
of
the
we
have:
Solution.Rewriting the equations
curves,
===

(x-I)2+(y+2)2=16,

y==-(x-I)2-2V3+2.

1-2
!I

Consequently, the centre of the

circle lies at the point C ( 1,-2)


and the radius of the circleequals
4. The axis of the parabolacoincideswith the straight line x == 1
and its vertex lies at the point
B (1, 2,
V3) (Fig.78).

:JJ)

-2

The area SABDFA of the smaller


figure is found by the formula
X

SABDFA =

\037

XA)

(Ypar-Ycircle)dx,
Fig.

78)))

Ch.

326)

where

/1.Applications

of the Definite

Integral)

and x D are determined from the system of equations

XA

J
\\

(X_I)2+(y+2)2==16,

+4,
y+2==-(x-I)2_2Y'\"3

xA=-l,xD==3.

whence
Hence,)

3
SARDFA =

[(-x2+2x+1-2V3)+(2+V16-(x-l)2)]
-1
r
/ 3)x+x 2 t - 16-(x-I)2+
= -3+ +(3-2}
l
16
X-I 3 32 8 J;j-4] =3- 3 +2 Y --12+3216arcsin2= 8
+2\"arcsin
X3

dx=

\037

X2

4 V3 +3 n.
=\"3--

area of the second figure is easy to determine.


Note.The computation of the integral can be simplified by using
= z and taking advantage of the evenness of the
the shift

The

x-I

integrand.

7.3.19.
Compute the
2

==

16-x

and the

area bounded by the curves

x-axis.

7.3.20.
Compute the

y= (X-4)2,

area enclosedbetween the parabolas

x ==y2;

7.3.22.

= 3 y2 + I.
4\"

7.3.21.
Compute the

area of the portions cut off by the hyperthe ellipsex2 + 4y2 == 8.


Compute the area enclosedby the curve y2 ==(I_X2)3.
the area enclosedby the loop of the curve
Compute
3
2
4 (y2
) + x == O.
7.3.24.Compute the area of the figure bounded by the curve
VX- +V.ry ==1 and the straight line
Compute the area of the figure enclosedby the curve

bola x2

3y2 == 1 from

7.3.23.

-x

x+y=l.

7.3.25.
2
y2 == x (l-x2).
x3

7.3.26.
Compute the
2
+ x y2 ==O.
7.3.27.Compute the

the curve x ==y2

area enclosedby the loop of the curve


area bounded by the

(1-y). area

7.3.28.
Compute the
2

bounded by the curve y ==x4 2x3+


two ordinates correspondingto

+ x + 3, the axis of abs(issas and

the points of minimum

axis of ordinates and

of the function

(x).)))

\037

7.4. A reas with

Parametrically

Boundaries

Represented

327)

7.4.ComputingAreas with Parametrically


Represented
Boundaries

\037

1fthe

boundary of a

is

figure

representedby parametric
equat ions

y=y(t),

x==x(t),
then the area of the

is evaluated by one of the

figure

three for

mulas:)

S=

Ii

Ii

x'
S = x (t)y' (t) dt;
S
S Y (t) (t) dt;

u..)

s=

B
\037

ex.)

S (xy' -yx')dt,
ex.)

are the values of the parameter t corresponding


(X and
respectivelyto the beginning and the end of the traversal of the
contour in the positive direction (the figure remains on the left).
where

\037

7.4.t. Compute the

area enclosedby the

x==acost, y=bsint
Solution.Here it
xy'

is convenient

yx' ==a cost

ellipse

(0\037t\0372Jt).

to compute
+ b sin t X a sin t ==ab.)

first

b cost

Hence)

s=

2n

2\037

\037

S abdt=nab.
S (xY'-yx')dt={

7.4.2.Find

0)

the area enclosedby the astroid

Solution.Let

3\"
= 1.
( ) +( )
3\"

\037

us write the equat ion of the astroid in


0 t
Here it is
form: x ==a cos3 t, Y ==a sin3

nient

to evaluate

xy'

==a 2

yx'

t,

\037

\037

2n.

parametric
also conve-

first

(cos 3 sin2 t cost + sin3 t 3 cos2 t sin t) ==


3t

\302\267

\302\267

2t
==3a2 cos

Hence,)

2n

S = + S (xy'-yx')dt

2n

a2
\037

S
0

sin2 2t dt

= a2n.

7.4.3.Find the area of the region bounded by an arc


y=a(l-cost)and the x-axis.
cycloid x==a(t-sint),
(0

Solution. Here the contour consists of an arc


t
2n) and a segment of the x-axis(0 x
\037

\037

\037

f,

apply the formula S =

\037

a)))

yx' dt.

sin2 t.)

of the
\037

of the

cycloid

2na). Let

us

Ch. V

328)

II. Applications

of the Definite Integral)

Since on the segment of the x-axiswe have y ==0, it only reto compute the integral (taking into account the direction
ains
rv
of a boundary

traversal):

2n

(l-cost)2dt=
a(1-cost)a(1-cost)dt=a
S=-\037
0
2n
2

2n

= a2

7.4.4.Compute the

S
o

[1

\037

2 cost +

\037

(l + cos2t) ] dt = 3:rr.a2.

area of the region enclosedby the curve

x==asint,y==bsin2t.

Solution.When constructingthe curve one should bear in mind


it is symmetrical about the axesof coordinates.
Indeed,if we

that

substitute

'It-t for t, the

variable

x remains unchanged, while

changesits sign; consequently the curve is symmetrical about

only

x-axis.When substituting 'It + t for t the variable y


unchanged,and x only changes its sign, which means

the

remains
the

that

curve is symmetrical about the y-axis.)


If)

t= 1t

2)

x)

t=1rc
4)
Fig.

79)

Furthermore,since the functions x = a sin t; y = b sin 2t have a


common period 2'It , it is sufficient to confine ourselvesto the fol2Jt.
lowing interval of variat ion of the parameter: t
From the equationsof the curve it readily follows that the variablesx and y simultaneously retain non-negative values only
when the parameter t varies on the interval
, therefore at
\302\260

\037

\037

[0, ]
\037

we obtain the port ion of the curve situated in the first


quadrant. The curve is shown in Fig.79.
As is seen from the figure, it is sufficient to evaluate the area
enclosedby one loop of the curve correspondingto the variation)))
o

\037

\037

\037

000
\037

7.4.Areas

of the parameter
n

with

Parametrically

t from
n

0 to

1t

Represented

Boundaries

329)

and then to double the result)


n

S = 2 yx' dt =2 bsin 2t xacostdt =4ab cos2 t sin t dt =


\037

\037

7.4.5.Find the
curve)

\037

= -4ab CO;3t
(

= fab.

) I:

area of the region enclosedby the loop of the


t2

x=3(6-t);y=s(6-t).
Solution.Locatethe points of self-intersection
of the curve. Both
x (t) and y (t) are defined throughout the entirenumber

functions

scale-00< t

<00.

the point of self-intersectionthe values of the abscissa (and


ordinate) coincideat different values of the parameter.Sincex=
=
+ A.. For the func(t-3)2,the abscissascoincideat
tion y (t) to take on one and the same value at the same values
= (3-A)2
must
of the parameter t, the equality (3+A)2
8 (3 -A,)
8 (3 + A,)
==
A,
be fulfilled for =1= 0, whence A,
At

t=3

3-+

:i::3.

B)

A)

J(
L

-4)
x)

Fig. 80)

Fig.

81)

t 1 ==0 and at t 2 ==6 we have x (t 1) ==X (t 2) ==


and
==
==
the poin t (0, 0) is the 0n I y poi n t of self y (t 1) y (t 2)
the points of the curve
intersection.When t changes from 0 to
the
are found in the first quadrant. As t varies from 0 to
point M (x, y) describes the lower part of the loop, since in the

Thus, at

0,

0, i.e.

6,

indicated interval x (t) and y (t) =


tion x (t) begins to decrease,
while

increase,and

3,

then the funcy (t) still keeps increasing.Figure 80 shows the traversal of the curve correspondingto increasing t (the figure remains on the left).)))
3\037X

Ch,

330)

II. Applications

of the Definite Integral)

enclosedby the loop sought

In computing the area


nient to use the formula
6

S=

(xy'
S
o

\037

7.4.6.
Find the
3

/2
yx') dt = S
0

(62\037t)2

is conve-

it

= 2;.

dt

area enclosedby the loop of the curve: x = t 9 ;

y=t- 3
t

7.4.7.Compute the area enclosed by


(I + cast); y = a sin t (I + cost).

== a cost

cardioid:x =

the

Solution.Since x (t) and y (t) are periodic functions, it is suffiThe curve is symmetrical
cient to considerthe interval
for t the value of the
about the x-axis,since on substituting
variable x remains unchanged, while y only changes its sign, and
0 as t varies from 0 to n.
y
As t changesfrom 0 to n the function u = cast decreasesfrom
and the abscissa x=au(l+u)=a
1 to

[-Jt,n].-t

\037

-I,

first

[-{+(u+r]
:_+
\037

decreases
from
xla=!=2a to xlu

and then increases

\037

to x :u= = O. We can show that the ordinate y increaseson the


t
on the interva I
t
:n:
interval
and decreases
)
(
)
The curve is shown in Fig. 81,the arrow indicating the direction of its traversal as t increases.

-1
:::;;;
(0:::;;;

\037

Consequent I y

,)

rr

S=

{-nS

(xy'

\037

yx') dt

= a2 (I + cost)2 dt =
S
0)

:n:a2

:::;;;.

7.4.8.Compute the area of the region enclosedby the curve


7.4.9.Computethe areasenclosedby the loopsof the curves:
2
y=t 3 -t;
(a) x=t

x =- cost, Y = b sin3 t.

-1,

-t

(b) x=2t-t2; y=2t2


t
2
).
(c) x=t2;

y=3(3-t

7.4.10.
Compute the area of the region
by the curve
y = b sin t ('OS2t .
7.4.11.
Compute the area enclosedby the evolute of the ellIpse)
.
= 2 CGs:it. y.=
sIn t C2 --If b2
\037nclosed

x ==a cost;

-b
C

\037

')

\037

-\037

;)

.)))

7.5.The Area

\037

7.5.The

Polar Coordinates)

in

331)

Area of a CurvilinearSectorin

Coordinates

\037

In polar coordinatesthe area of a


=
p ==p (cp) and the rays CPt ==(X and
CP2

Polar

sector bounded by the curve


is expressed
by the integral

\037

r\037

S=+S p2(qJ)dcp.
a)

7.5.t. Find

the area of the regionsituated in the first quadrant


and bounded by the parabola y2 =4ax and the straight lines
y==x-a and x==a.
Solution. Let us introduce a polar system of coordinatesby
placing the pole at the focus F of the parabola and directingthe
polar axis in the positive direction
along the x-axis.Then the equat ion of
the parabola will be p = I P
cos , where p is the parameter of the parabola.
!J2=4ax)
In this case p == 2a, and the focus F has
the coordinates (a, 0). Hence,the equation of the parabola will acquire the
20
and those of the
form p =
I
cos

stra ight
cP

If)

<p

'

<p

lines will become =

and

cp

x)

\037

= (Fig.82). Therefore,)

Fig.

JT

-_ _ --I

SFABF

_ 4a2
(1 ens

dq>
<p

r\037

-_

2a2

\037

j[

d<p

4 sin 4

\037

2=z,)
<p

we obtain)

cot
2

d<p

2 sin 2 (<pf2)

= dz

cp

into

'!t/4Icot('!t/8),

,)

'!t/21

1)

(\037/8)

-l-

2
(1+z2)dz=a
)
(cot + ; cof3
-cos (1(/4)= 1
1t
account that cot = +
+ J/ 2,

SFABF=a S
t

or, taking

\302\267

4)

Changing the variable:)


cot

82)

1t

\037

8\"

SFABF=2a(1+
2

sin (1t/4)

V2\.)))

\037

\037

Ch.

332)

II. Applications

of the Definite

Integral)

7.5.2.Compute the

area of the regionenclosedby


==
cardioid
the
p 1 + coscP;
(a)
==
a cos
the
curve
p
(b)
cpo

7.5.3.Find the area of the regions bounded by the curve


p::n:2a cos3cp and the arcsof the circle p ==a and situated outside
the

circle.

,
p = 2acos3cp has a period T
three equal loops of the curve as vathe radius vector describes
are those at
Permissiblevalues for
and
ries between
whence
which cos3cp

Solution.Since the function

-jt

0,

\037

\0371t

n.

cp

cp

(k==O,+ 1,+ 2, ...).)


-6+3\037CP\0376+3
n

-i

2kn

2kn

Consequently,one of the loops is describedas varies between


and
and ' and the other two loops as varies between
7n
5n
3n
and between(3 and
respectCutt
the
out
ively (Fig. 83).
ing
parts,
belonging to the circlep ==a, we get
the figure whose area is sought. Clearly, it is equal to the triple area
cp

cp

2'

\037

(3'

\037

SMLNM'

Let us find the polar coordinatesof


the points of intersection M and N.
For this purpose solve the equat ion
2a cos3cp= a, e.cos3cp = -} Between

i.

-- -6.
n

Fig.

6 and

83)

\037

is

specifiedby the polar angleCPl


As is

SMLNA1

seen from t he

only

the roots

\037

==

\037

,and the point

rr/9

rr/9

r 4a2 cos2 3cp dcp-\037 r a 2 dcp


2

,J

cp

through the

\037.

a2

t)
-\0379)

7.5.4.Compute the area of


V2a cos and p = 3a sin

Solution.The

by

CP2

figure,

-\0379

p ==3

9 and

(k=O) are found. Thus, the point

==SOMLNO-SOMNO
=

=2

--

the
cpo

circle lies in

figure

bounded

.::
( +
9

by

the

==

;.

__

V3
6 )

circle

the right half-plane and passes


pole p = 0, touching the vertical line.The secondcircle)))
first

\037

7.5.The Area

in

Polar Coordinates

333)

.)

is situated in the upper half-plane and passes through the pole


p
touching the horizontal line.Consequently,the pole is a point
of intersection of the circles.The other point of intersection of the
circlesB is found from the equation 3
a casq> = 3a sin cp, whence
B (arc tan
As is seen from
a
84, the sought-for area
===

0,

V2

V2, V6).

Fig.

If)

:l]
x)

3\037a)

Fig.

2)

Fig.

84)

85)

is equal to the sum cf the areasof the circularsegments OABO


= arc tan
The
and GCBGadjoining each other along the ray
arc BAO is described by the end-point of the polar radius p of
the first circlefor arc tan
, and the arc OCB by the
end-point of the polar radius p of the second circlefor

V2.

cp

V2

\037

cp

\037

\037

\302\260

\037

arc tan

V2. Therefore)

:rr)

2
SaARa

= 9a2

arc tan Y2\"

arc tan

= a2
SOCHa

cos2 qJ dqJ = a2 (
YT

sin2 qJ dqJ =

\037

o)

Hence,)
SOARO

\037

-arctan V2

\037

Jl 2
3 )

cp

\037

y_
--i-).

a2 (arctan V2

2
+ SOCRO= 2.25a
(1t-arctan V2 V2).

7.5.5.Find the area of the figure

V3

cut out by the circlep


sin
from the cardioid p == 1 + cas (Fig. 85).
Solution.Let us first find the points of intersection of these curves. To this end solve the system
===

cp

p=
{P 1

V3sin cp,

===

whence CPt

3t
= 3\"'

CP2

==

It.)))

+ cas

cp,)

\302\260

\037

cp

\037

1t,

cp

Ch.

334)

II. A pplications

of the Defi nile Integral)

The sought-for area is the sum of tWJ areas:one is a circular


segment,the other a segment of the cardioid;the segments ad join
The arc BAa is describedby the
each other along the ray =
end-point of the polar radius p of the cardioid as the polar angle (()
to 1t, and the arc OCB by the end-point of the
changesfrom
<p

\037

\037

polar radius p of the circlefor 0

j'[
\037

\037

cp

\302\267

\"3

Therefore)
:'1

S =-}
S
o

n
.'3sin2 cp dcp

+-}Sn (1+ cascp)2dcp =


\037\037)

3
sin
_-4
( cp- 2

2CP

.
+ sin4 )
+ 2 ( (()+ 2 Sltlcp,-.
I

\037

cP

_I

IT

2<r

\037=

\037

\037)

3 ).
=\037(J(-V
4

7.5.6.Find the area of the =figure bounded by the cardIoid


coscp) and the c rcIe p a.
7.5.7.Find the area3 of ==the regionenclosedby the loop of the

p = a (1
folium

+ y3 3axy.
pass over tJ polar coordinatesusing the usual
formulas X==PCOscp,
y=psincp.Then
the equation of the curve is:
3
3
p3 (cos + sin (p) ==3ap2SIn CuS

of Dt scartesx

Solution.Let

us

A)

cp

or
X

Fig.

3a sin

cp

cos =)

P ==cosa cp+sin3
<p

CV,)

<p

cp

3a sin

2<p

+ cos<p) (2 sin 2(f')


It follows from this equation that,
j'[
p=O at cp=O and at CP==2 '
firstly,
331
and secondly, p -+ 00 as --+4 and
Jt
(P --+
T' The latter means that

86)))

(sin<p

\302\267

cp

the folium of Descarteshas an asymptote, whose equation y==


can be found in the usual way in rectangularcoordinates.
Consequently, the loop of the folium of Descartes is described
and is situated in the first quadrant
as changes from 0 to

=- x-a
<p

\037

(seeFig.,86).)

7.5.The Area

\037

Polar

in

Thus, the sought-for area is equal

Coordinales)

335)

to)

;n)

2
1

9a2cos2

<p

sin.\037

<p

SOAO=\"25 (cos:J4 -t- sin:J (p)2 dcp.


o)

Taking advantage of the curve's symmetry about the bisector


==x, i.e.about the ray
=
we can compute the area of half
cp

'

\037

= 0 to (p =
the loop
(from
les us to apply the substitution)

of

<p

tan

cp

d(p

cos2
which gives

SOAO5t ill

new

Q)

\037

= z,
=dz,

) and
fp

11
1

-)4
:rT

9a2

COS2
(cos3

<p

5o

double it. This enab-

then

sin\037

q;

<p+sin:Jtp)2

cp-

9a 2

z'!.dz

5 (l+z3)2'
0)

substitut ion
z
1

+ Z3=V,
=

3Z2

leads to the integra

dz

v
I

do,

2)

I)

\037

SOAD = 3a2

dv
v2

3 2
= 2'
a

\302\267

1)

7.5.8.
Compute

t he

the area of the region enclosedby one loop of

curves:)

(b) p = a

(a) p ==a cos2cp;

-cos

7.5.9.
Compute the

p = a (I

cp) lying

sin 2cp.

area enclosedby the port ion of the cardioid

inside the circlep = a cos

<po

7.5.10.
Compute
2

the area of
O.

t. Compute
7.5.1

the area of the region

=Qsin(pcos(P,

fJ

a
==a cos
q

(a

>

a>

0).)))

t he

region enclosedby the curve


enclos\037d by

the curve

Ch. VII.

336)

of the Definite

Applications

Integral)

7.5.t 2. Compute the


p=

0)

area of the portion


inside the circle
(lYing
of the figure bounded by the Bernoulli's
lemniscate p=

=a Vcos2cp.

7.5.13.

Passing over
the regionenclosedby
Passing over
the region enclosedby

7.5.14.

\037

to polar coordinates,compute the area of


the curve (x2 + y2)3 = 4a2x2y 2.
to polar coordinates,
evaluate the area of
the curve x4 + y4 = a 2 (x2 + y2).)

7.6.Computingthe

Solid

Volume of a

The volume of a solid is

by
expressed

the

integral)

= S (x) dx
\037

a)

where S (x) is the area of the section of the solid by a pJane perpendicular to the x-axisat the point with abscissa x; a and bare
the left and right boundaries of variat ion of x. The funct ion S (x)
is supposed to be known and cont inuously changing as x varies
between a and b.
The volume V x of a solid generated by revolution about the
x-axisof the curvilinear trapezoid bounded by the curve y ==f (x)
(f (x)
0), the x-axisand the straight lines x ==a and x = b (a < b)
is expressed
by the integral
b
\037

Vx=n\037!/dx.
a)

The volume V x of a solid obtained by revolving about the x-axis


the figure bounded by the curves Y = Yl (x) and Y ==Y2 (x) [0 Yl (x)
by the
\037Y2(X)] and the straight lines x==a, x==b is expressed
\037

\037

integral)
b

Vx = n

\037

(y\037

-yi}dx.

a)

If the curve is representedparametrically or in polar coordinates,


the appropriate changeof the variable should be made in the above
formuJas.

7.6.1.Find the

volume of the
x2
a2

ellipsoid
Z2

y2
+ C2 ==
+ b2

1.)))

7.6.Computing

\037

of a

the Volume

Solid)

337)

Solution.The section of the ellipsoidby the plane x= const

ellipse(Fig.87))

an

b2

with

b ...
semi-axes
V

(
y

\0372

x2

-\302\24322;

section(see Problem 7.4.1)


x2
-. 1
1
S (x) == nb ...
X C
V
V

-\302\24322

Therefore the volume


a
V==

S '!tbe(
-a)

c2
1

x2
-\302\24322

\0372)

x2

Hence the area

-\302\24322.

= '!tbe(

V sphere

the)

(-a\037x\037a).)

ellipsoid

X2

X2
1-\302\24322

of

is)

l- ) dX='!tbeXa2

=1

\037

11

of the

X3

3a2] _a=\"3nabe.

In the part icu1ar casea = b

and we have

is

= c the ellipsoidturns

into a

sphere,

:rta 3

.)

z)

x)

Fig.

87)

Fig.

H8)

7.6.2.Compute the volume of the solid spherical segment of two


bases cut out by the planes x = 2 and x ==3 from the sphere
2

+ y2

--1-

Z2

==

16.

7.6.3.

The axesof two identical cylinders with bases of radius a


intersect at right angles. Find the volume of the solidconstituting
the common portion of the two cylinders.
Solution.Take the axesof the cylinders to be the y- and z-axis
(Fig. 88). The solid OABCDconstitutesone-eighthof the soughtfor

solid.

Let us cut this solid by a plane \037erpendicular to the x-axisat


a distance x from O. In the section we get a square EFKL with)))

338)

Ch. V I J. Application\037

side EF=Va2

-x , therefore S(x)=a-x and

of

the Definite

Integral)

=85(a -x )dx =
2

o)

l6
3\302\260::1.

7.6.4.On

all chords (parallel to one and the same direct ion) of


a circleof radius R symmetrical parabolic segITlentsof the same
alt itude h are constructed.The planes of the segments are perpend icular to the p lane of the circle.
Find the volume of the solid thus obtained (Fig.89).
\"
\"<

1/

\037

\"

h
\"

x2+h
!J=-4h
a

I)

2)

x)
x)

Fig.

Fig.

89)

90)

Solution. First compute the area of the parabolic segment \\Nith


base a and altitude h. If we arrange the axesof coordinatesas
indicated in Fig. 90, then the equat ion of the parabola will be

y = ax

+ h.

Determine the parameter (x. Substituting the coordinatesof the


411
a2
.
a
0 , we get 0 =(X4 + h, w h enCt' a = a'l hence the
pOint B

2'

equation of the parabola is

S= 2

5o y dx

=2

50 (

x\037

4/\037

a)

\037\037

x2

+ h)

\037

desired area

and the

+ h ) dx =

ah.

If the axesof coordinatesare


No\\v
find the volume of the solid.
arra nged as indicated in Fig. 89) then in the section of t he solid
by a plane perpendicular to the x-axisat the point with abscissax
we obta in a p arabolicsegment of area S = ah, where a = 2y =

==

211 - .
R2

S (x) =

\037

x2

V R2

\037

I-Ience,

x2 h and V = 5 S (X) dx =
-R

h
\037

dx =

5 VR2_X2
0)))

nhR2.
\037

7.6.Computing

\037

the Volume of

7.6.5.

339)

Solid)

The plane of a moving triangle remains perpendicular to


the fixed diameter of a circleof radius
the base of the triangle
is a chord of the circle,and its vertex lies on a straight line parallel to the fixed diameter at a distance h from the plane of the
circle. Find the volume of the so1id generatedby the movement
of this triangle from one end of the diameter to the other.
Compute the volume of the solid generatedby revolving
about the x-axisthe area bounded by the axesof coordinatesand

a:

7.6.6.

2 + y2 = a2'.
1

the

parabola x

Solution.Let us find the points of intersection of the curve and


the axes of coordinates:
at x = 0 y = a, at y = 0 x ==a. Thus, \\ve
have the interval of integration [0,
From the

fore
a

= Jt

S
o

all
-

equation of the

y2 dx = Jt

a 2\"

S
(
0

a].

parabolawegety=( a 2
4

dx = Jt

\"\"2)

S
0)

-x

) ; there-

31
+ 6ax(
a2

4a

\"\"2

xT

-4a x2 +X2) dx= IS Jto


I

7.6.7.The
the
the

figure

and the straight line

y==-sin

1l

== 1

revolves about

x)

x)

o)

1\302\243

o)

2)

Fig.

.)

bounded by an arc of the sinusoid y = sin x,

axis of ordinates
y-axis (Fig.91).)

-2)

Fig.

91)

2)

x)

92)

Compute the volume V of the solid of revolution thus generated.


Solution. The inverse function x = arc sin y is consideredon the
interval [0,
Therefore)

1].

Y2

V =

11

= Jt (arcsin y)2 dy.


S dy
S
x2

Yl

0)))

Ch. V

340)

II. A pplications

of the Definite

Integral)

the substitution arc siny=t.Hence)

Apply

t,
y=sin
=

dy

cost dt

o
,)

12)

:rt

:rt

And

so,V = n S t 2 cost dt.Integratingby parts, we get V = n (n:-8).


o)

7.6.8.Compute the volume of the solid generatedby revolving


2 2
about the x-axisthe figure bounded by the parabola Y ==0.25x
+
and the straight line 5x-8y+14=0.
Solution. The solid is obtained by revolving the area ABCA
(Fig. 92) about the x-axis.To find the abscissasof the points A
and B solve the system of equat ions:
1

Y=TX2+2,
{ 5x-8y+ 14== O.
Whence xA=T; x B=2. In our caseYl(x)=4x2+2and
= (5/8)x+ 7/4. Hence,)
1

I 5
x 7
V=1t S I16
( \"2 + )
1/2)

-4 +
1

X2

Y2(X)

891
) ] dx= I 280 n.

\\

7.6.9.Compute the volume of the solid generated by revolving


about the y-axis the figure bounded by the parabolas Y = x2 and

8x= y2.

.)

on the in(y) =
coordinatesto the point of intersecterval from
tion of the parabolas (Fig. 93). Let us find the ordinates of the
points of intersect ion of the parabolasby excludingx from the system of equat ions:)
2
il = x

Solution. It

is obvious that
the origin of the

X2

(y)

= Vy

\037

Xl

{ = 8;.)
y\"a

We

obtain Yl = 0, Ya = 4. Hence,V = n
S

(Y

y\037

\037:

) dy

= 254 n.

o)

7.6.10.
Compute

the volume of the solid torus. The torus is a


solid generatedby revolving a circle of radius a about an axis
lying in its plane at a distance b from the centre (b a). (A tire,
for example,has the form of the torus.))))
\037

\037

7.6.Computing

the Volume

of

341)

Solid)

I. Compute
.6.1

the volume of the solid obtained by revolving


figure bounded by two branches of the curve
(y- X)2 ==x3 and the straight line x ==
Find the volume of the solid generated by revolving
about the line y == 2a the figure boundedby the parabola y2 == 4ax
and the straight line x ==a (Fig.94).
7

about the

x-axisthe

1.

7.6.12.

If)

x)

x')

x)
2)

Fig. 93

Fig. 94
Solution.If we transfer the origin of coordinatesinto the point
(0, 2a) retaining the direction of the axes,then in the new
system of coordinatesthe equation
of the parabola will be

0'

(y'

2a)2= 4ax.

Hence Y2 == 2a + V 4ax

(for the cur-

2a-

V4ax (for
OAB), and Yl =
the curve OCD). The sought-for volume is equal to)
-a)
ve

a
V

=n

S
o

(y\037

yi) dx = n

A)
:\302\243)

a)

[(2a+
S
0

-a

+2Vax)2-(2a-2V ax)2] dx = 332na3.

.
Fig. 95

7.6.13..

Find the volume of the solid generatedby revolving about


the x-aXISthe figure enclosedby the astroid:
x = a cos3 t; Y ==a sin3

t.

Solution.The sought-for volume

obtained by revolving the

is equal to double the volume

(Fig.95).Therefore,
V=2n S y 2 dx.
figure

OAB
a

o)))

Ch. V

342)

II. A pplicat

ions of the Definite Integral)

Change the variable

-3acos
a cos3 t ,

===

2t

dx==

y==asin3 t,)

x ,

sjn t dt,

o
a

Jt/2.
0)

Hence,)
o
V

= 2n

Sj[

a 2 sinG

-3acos

2t

sin t) dt =

2)

Using the formula from

integrals,we get
6
V == 6na3
(7

5 3 98
X

-J

sin7 t dt

6na{f
Problem 6.6.9for
6

54

sin 9

t dt

].

computing the above


2
3\"

32 3
= 105
na .

7.6.14.
Compute the volume of the solid generated by revolving
one arc of the cycloid
y===a(l-cost)about the
x-axis)
Compute the volume of the solid obtained by revolving
shown in Fig.
about the polar axis the cardioid p=a(l+coscp)
Solution.The sought-for volume representsthe differencebetween
the volumes generatedby revolving the figures MNKLO and OKLO
about the x-axis(which is th2 polar axis at the same tIme).
As in the precedingproblem, let us pass over to the parametric
representationof the curve with the polar angle as the parameter:
x == p cos ==a cos ( 1 + coscp),

x=a(t-sint),

7.6.15.

81.

({)

cp

==p sin

<p

cp

=a sin (1+ coscp).


cp

It is obvious that the abscissa of the point M equals 2a (the value


==0), the abscissa of the point K being the minimum of
of x at
the funct ion x == a (1 cascp) cos
Let us find this mininlum:
cp

cpo

= -Qsin (1+ 2 coscp) =0,


2
= 0; = 3
Jt.)
{j)

\037;

(f)1

At

CPl

=0 we obtain

XM

CP2

= 2a)

at

CP2

2
=3
Jt,

XK

T')))

\037

Hence,the

7.6.Computing

=n

-5a

y\037

Changing the variable

- -0

2a

dx n

5a
4)

+ cas<p)2sin2 <p,
-a sin (I + 2 cos

<p)

<p

dx.

y\037

x=acos<p(1
+cos<p),we get

== a 2 (I

dx -==

343)

Solid)

sought-for volume is equal to


V

y2

the Volume of

dcp,)

<p

<p
I

-aj4 2J1j3

aj4 2nj3

2a

0)

n)

Thus,)
o
V

=n

5
2

a 2 (1+ cos

q\302\2732

sin2 q> [

-asin

q>

(1+ 2 cos

q\302\273]

dq>-

31)

:T1

-II 5

a 2 (1+ cas

q\302\2732

sin2 q> [

-asin

q>

(1+ 2 cos

dq>

q\302\273]

J1)

31

na 3

3
2
5o sin (1+ cos (1+ cas
q>

q\302\2732

q\302\273

dq>

=na3 5 (l-u2) (1+U)2(l+2u)du=


-I)

\037

.6.16.
Compute the

na 3

(u=cosq\302\273.

:-

solid bounded by:

volunle of the

Y
= 1 and the plaz:
(a) the hyperboloid of one sheet x:
a +b
c
nes z -== 1 and z = 1;
y 2, the planes of coordinates
(b) the parabolic cylinder z
and t he plane x ==a;
.)
.)
and the plane z ==k (k > 0).
(c) the ellipticparaboloid z = +

=4-

:;

\037;

7.6.t 7. A wedge is cut off from a right circularcylinder of radi us


a by a plane passing through the diameter of the cylinder base and
inclined at an angle a to the base.Find the volume of the wedge.
Compute the volume of t he solid generated by revolving
the figure bounded by the following lines:
1,x=4, y=O about the x-axIs;
(a) xy=4, x=
(b) y=2x-x2, y=O ebout the

7.6.18.

\037-ax!s;)))

II. Applicat ions of the Definite Integral)


(c) y ==x3, Y == 0, X == 2 about the y-axis;
(d) y2==siil x (one wave), y == about the x-axis;
(e) x _y2 = 4, Y == + 2 about the y-axis;
(f) (y_a)2 ==ax, x == 0, y == 2a about the x-axis.
Ch.

344)

\302\260

Find the
7.6.19.
ax3 x 4

curve

=
y2

a)

volume of the solid obtained by revolving the


about the x-axis.

7.6.20.
Compute the

about the
y

= 2 X.

x-axisthe

volume of the solid generated by revolving


bounded by the lines y == sin x and

figure

n)

.6.21.

(\037ompute

the volume of the solid generatedby revolving


catenary

x-axisthe curvilinear trapezoid bounded by the


y = (e + e
) = a cosh and the straight lines Xl =
== c (c > 0).
about the
\037

\037

\037

\037

C, X2 =

7.6.22.
Compute the volume of the solid generated by revolving
about the x-axisthe figure bounded by the cosineline y == cosx and
9
the parabola y = 2n 2 x2

7.6.23.
Compute the volume of the solid generated by revolving
about the x-axisthe figure bounded by the circlex2 + y2 == 1 and
3
the parabola y2 =\"2x.
7.6.24.On

the curve y ==x3 take two points A and B, whose


abscissas are a == 1 and b == 2, respectively.
Find the volume of the solid generatedby revolving the curvilinear trapezoid aABb about the x-axis.
7.6.25.
An arc of the evolute of the ellipsex ==a cost; y ==b sin t
situated in the first quadrant revolves about the x-axis
Find the volume of the solid thus generated.

7.6.26.
Compute the

volume of the solid generated by revolving


ta
the regionenclosedby the loop of the curve x==at2, y==a
(
)
about the x-axis.
7.6.27.Compute the volumes of the solids generated
by revolv2
==a 2 (x2 y2)
ing the region enclosedby the lemniscate (x + y2)2
about the x- and y-axes.
7.6.28.
Compute the volume of the solid generated by revolving
the region enclosedby the curve p ==a cos2 about the polar axis.)))

t-3

cp

7.7.Arc

\037

Length of

a Plane Curve

in Rectang.

Coord's

345)

* 7.7. The Arc Length of a PlaneCurve in Rectangular


Coordinates
If a plane curve is given by the equation y ==y (x) and the
derivative y' (x) is continuous, then the length of an arc of this
curve is expressed
by the integral
b

l=
\037

VI+ y'2 dx

a)

where a and b are the abscissas of the end-points of the given arc.)
Compute the length of the arc of the
M(4,8))
semicubical parabolay2 ==x3 between the points
(0, 0) and (4, 8) (Fig. 96).
Solution. The function y (x) is defined for
x O. Sincethe given points lie in the first qu-

7.7.1.

\037

adrant,

==X

\037)

D)

2. Hence,

434
::

y' =

;V x

and

V 1 + y'2 =

-V\037

+ ; x.

Consequently,)
l=

=
5o Y 1 + x dx
\037

\302\267

Fig.

8
(1+ ; x)21 = 27 (10V 10 1).
0)

7.7.2.Compute

y2

96)

the length of the arc cut


= x3 by the straight line x= -}

off

from

the curve

.)

7.7.3.Compute the
the

arc length of the curve y==lnco\037x between


the abscissasx== 0, x= T

points with

Solution.Sincey' ==

tan

x, then VI + y'2 = VI + tan 2 x ==secx.

Hence,)
n

= seexdx= In tan
5o
( + )
\037

7.7.4.Compute
Xl

==a to

x2

===b

\037

T
I

3n
= In tan 8

\302\267

0)

the arc length of the curve y = In ex + 1 from


eX

(b >

-1

a).)))

Ch. VII.

346)

of the Definite Integral)

Applications

7.7.5.Find the

arc length of the curve X=+y2_{Jny between


the points with the ordinates y = 1 and y = 2.
Solution.Here it is convenient to adopt y as the independent
variab Ie; then

Vl+x'2=V({y+ Y=+y+ '

x'=-\037-y- and
2\037

[=

21y

2\037

Hence,

S V 1 + X'2 dy = S (
1

\037

+ ;y ) dy = +

2.

-222
- -.
+
\037

1)

In

\037

3 = a3
length of the astroid x 3
y
Solution.As is known, the astroid is symmetrical about the axes
of the coordinateangles.Therefore,
of coordinatesand the bisectors
it is sufficient to compute the arc length of the astroid between
the bisectory = x and the x-axisand multiply the result by

7.7.6.Find the

8.

In

at

\037

the

first

-s-.
a

\037

y= (a

quadrant

\037

-x )2and y=O at x=a, y=x


\037

'})

- --3=-X--x
Y'=7[
(-3)X

-) -

Further,)

(a 3

and)

../1+x-3
2

+ y'2 =

Consequent ly,)

[=8

a
\037

(a

32 -x32 )

_2 2
a 3 -x

3)

=( )3.)

a3 x-3 dx=6a.
I

23/t)

all
3

Note. If we compute the arc length of an astroid situated


first quadrant, we get the integral)

the

\037

x-3 dx,

whose integrand increasesinfinitely

as x --+

O.)))

In

\037

7.7.A rc Length

of

a Plane Curve

7.7.7.Compute

in Rectang.

Coord's

347)

the length of the path OABCO consistingof porof t he curves y2 ==2x3 and x2 y2 ==20 (F
97).
SoLution. It is sufficient to computethe arc lengths LOA and lAB
since by symmetry of the figure about the

t ions

ig.

x-axIs)

!f)

l==2(LaA+LAB)'
Solving the system of equations

X2

+ y2 = 20,

y2

==2x3,

we

find t he point A
Find lOft. Here

)/2 2

x)

(2, 4).

==

= \"23

x ,

V 2x,)

y' ==)

VI + y'2 =

}/I+

x.)

Fig.

\037

Hence,)

2
lOA

t\\

97)

-V1 + xdx = (IoVTO 1).


\037

2\037

Sinceon the circleof radius V 20LAB is the

2,

length of an arc

cor-

respondingto the central angle arc tan


== V 20 arc tan 2.)
LAB
Finally we have
I=

:7(10VTO-1)+ 4 V s arctan2.

7.7.8.Compute
==

(a)

the arc length of the curve:


1 cut off by the x-axis;

2-cas

x\037

x) betweenthe adjacent points of intersection\\vith


(2
(b) y
the x-axis.
(c) 3y'!.==x(x-l)2between the adjacent points of intersection
with t he x-axis(half the loop length).
== In

7.7.9.Compute the
y=

arc length of the curve

-}l x Vx2 I-In(x + Vx2

bet\\veen)

x == 1

and x ==a +

1.)))

1)])

Ch, VII. Applications

348)

of the Definite

7.7.10.
Find the
2

the curves
\037

arc length of the


x ==(y + 1)3 and y ==4.)

7.8.The Arc

Integral)

path consistingof portions of

Length of a Curve Represented

Parametrically

If a curve is given by the equations in the parametric form x ==x(t),


Y (t) and the derivatives x' (t), y' (t) are continuous on the interval [t 1, t 2 then the arc length of the curve is expressed
by the

===

],

integral)

t2

1=
\037

V X'2 (t)

+ y'2 (t)dt,

tt)

where t 1 and t 2 are the values of the parameter t correspondingto


the end-points of the arc (tt < t 2).
Compute the arc length of the involute of a circlex =
from t=O to t=2Jt.
y==a(sint-tcost)
Solution.Differentiating with respect to t, we obtain
xi ===at cost, y; ==at sin t,

7.8.1.
=a(cost+tsint),

whence

Vxi 2 + y;2 = at. Hence,


211:

1=

S at dt

=
a\0372

= 2an 2.

1;n

7.8.2.Find the

length of one arc of the

x=a(t-sint),

7.8.3.
Compute the

length of the

Solution.Differentiating

== a

cycloid:

(I-cost).

astroid:x ==a cos3 t, Y ==a sin3 t.

respect to t, we obtain

with

2
xi = -3acos t sin t;
2
=
3a sin t cost.)
Yt

Hence

Vx?+y;2 = V9a2 sin2 t cos2 t =3a sin t cost 1= I sin 2t I.


I

Since the function

\037a

sIn 2t has a period

')

\037

3't

1= 4 X

3a
sin 2t dt
2S

= 6a.

Note. If we forget that we have to take the arithmetic value of


the root and put V X;2 + y;2 ==3a sin t cost, we shall obtain the wrong)))

\037

7.8.Arc

a Curve Repres'd Parametrically

Length of

349)

result, since)
231

3aS sin t cost dt = 3:sin2 t 1:= o.


31

7.8.4.Compute

y==t-t3.

the length of the loop of the curve

x== V 3t 2,

Solution.Let us find the limits of integration.Both functions


x (t) and y (t) are defined for all values of
Since the function
x == 1/--3t2
the curve lies in the right half-plane. Sincewith a
change in sign of the parameter t, x(t) remains unchanged, while
y (t) changessign, the curve is symmetrical about the x-axis.Furthermore, the function x (t) takes
on one and the same value not
more than twice. Hence,it follows
that the points of self-intersection
of the curve lie on the x-axis.
0
at y == (Fig. 98).
The direction in which t he moving point M (x, y) runs along the
Fig. 98
\037

t.

0,

i.e.,

\037

\302\260

curve as
is

00 to 00
t changesfrom
indicated by the arrows.

1.
:i::

Sincex (t 2) == X (t 3) ==V3 , the point


at t 1== 0, t 2, 3 ==
y =
3
of
self-intersectionof the curve. Conse(V , 0) is the only point
the limits t 2 ==
and ==
must
within
we
integrate
quently,
Differentiating the parametric equations of the curve with respect
3t 2, whence
to t, we get x; == 2 V3\"t, Yt ==
But

\302\260

-1

1-

'=\037

1.

2
Vx;2+y ;2=1+3t
.)

Consequently,)
1

l = (l + 3t 2) dt = 4.
-1)
\037

6' 2-

7.8.5.Compute the

t6

{4
arc length of the curve x==
Y =
4
between the points of intersectionwith the axesof coordinates.
7.8.6.Compute the arc length of the ellipsex2a2 + y2b2 ==
Solution.Let us pass over to the parametric representation of the
ell ipse)

1.

x=acost, y=bsint, 0\037t\03721t.

Differentiating

with

respect to t, we obtain

x;== asint;

y; == b cas

t,)))

Ch. VII.

350)

of the Definite

Applications

Integral)

whence)
V X;2

where

+ y? = Va2 sin2 t + b2 cos2 t == a )/-I-E2 cos2 t

is the eccentricity of the ellipse,


c
y a2

E---a- -

b'l.

Thus)

\302\267)

J/l-8cos

2
=
dt 4a Vl-8 cos

Vl-8cos

dt

11:

2Jt

=a

\037

2t

2t

2t

\037

dt.

is not taken in elementary funeis called the ellipticintegral of the second kind. Putting
we reducethe integral to the standard form:)

The integral

\037

tions;
t

it

;-'I,

11:

Vl-8 cos tdt= Vl-8 sin Tdt=E(8).


2

\037

\037

0)

where E (E) is the notation for the so-calledcomplete ellipticintegral of the second kind.
Consequently, for the arc length ot an ellipsethe formula

good.

I ==4aE (E) holds


It is usual practiceto put
lues for the funct ion
E1 (a) ==

For instance,if a =
E

E=sinaand

to use the tables of va-

(arc sin E) = E (E).


10and b = 6, then

== Y

\302\2431

2
.
I02-6
==sIn 53 .
==0.8
\302\260

10

Using the table of values of ellipticintegrals of the second kind,


= 40 X 1.2776 51.1.
find I == 40E1

we

\037

(53\302\260)

7.8.7.Compute the

arc length of the curve

x==t2,

Y=T(t -3)
t

between the points of intersection with the x-axis.


7.8.8.Find the arc length of the cardioid:

x ==a (2 cost --cos2t ),

==a (2 sin t

-sin

2t).)))

\037

7.9.Arc

7.8.9.Find

the

Length

of

Polar

Curve in

351)

Coordinates

closedcurve
= a sin 2t.
x = 4 1/2a sin t;
of the

length

LJ

.8.to. Fin d t he a rc

eva

engt h of the
C2

C2

ofthe e

ute

3
t, Y=-7i sJn 3 t
x=(icos

7.8.11.
Compute the

,)

c2 =a2

11

-b

ipse)

.)

arc length of the curve

x= (t 2 -2)sin t + 2t cast,
= (2 t 2) cast -1-2t sin
y
t1

bet\\\\!\037en

=0

and

t2

t)

= n.

7.8.12.

-cost)find
x=a(t-sin y=a(l
arc of the

On the cycloid
t);
point which divides the length of the first
the ratio 1 :3.)

\037

the

cycloid in

7.9.The Arc Length of a Curve in PolarCoordinates

a smooth curve is given by the equation P = P (cp) in polar


the arc length of the curve is expressed
by the
integral:)
If

coordinates,then

l=

'I),
\037

{p2+

p\0372

dcp,

<P.)

at
are the values of the polar angle
endwhere CPt and
points of the arc (CP1 < (()2)'
Find the length of the first turn of the spiral at Archimedes p = acp.
Solution.The first turn of the spiral IS tormed as the (.>01al angle
changes from 0 to 2n.Therefore)
th\037

CD

CP2

7.9.1.

cp

2n

=
\037

Va 2cp2+ a 2 d<:p

2n

= a Vcp2+ 1 dcp =
\037

0)

=a

[:rt

V 4:rt2

1.

+ 1 + {-In(2:rt + V4:rt2 + J)
spiraI p =

7.9.2.Find the length of the logarithmic


a certain POiLt (PO) CPo) and a moving point (p,

cp).)))

aemr,p

between

Ch. VII.Applicat

352)

ions of the Definite

Solution.In this case(no matter

Po, is greater!)

=
\037

a2e2m\037

+ a2m 2e

which of the

magnitudes, P or

(f)

Integral)

2m ,,!

dqJ

(f)o)

m2

{JJ

= a VI + m2

\\

em\037

dqJ

=a

JI\\;

I-tm 2 P _ Po
m

t,.;

(f)o)

1=
= Y +m 2 ,

em'!'-em'\037o

nz

\037

I ,)

i.e. the

length of the logarithmic spiral is proportional to the


increment of the polar radius of the arc.
Find the arc length of the cardioid P ==a (1+ coscp)
0
>
0,
2n).
(a

7.9.3.

\037

cp

\037

Solution.Here p' ==
cp

Vp'2 + p2

==-

(f)

(1+

2a2

-asin

cp,

coscp) == V4a2 cos2 (cp/2)

===

2a cos(cp/2),
= 2a cos( /2) =
{ -2acos(cp/2),
cp

Hence,by

\037

cp

\037

Jt

\037

cp

\037

Jt

2n.

virtue of symmetry
n

2n

l = 2a

cas
SI

\037

7.9.4.Find the

dqJ = 4a cas
S

\037

dqJ

= 8a.

lemniscatep2 2a2 cos2cp bet\\veen


to == 0 and any point with a
the right-hand vertex corresponding
polar angle < 4 .
Solution.If 0
< ' then cos2cp > O. Therefore
length of the

===

cp

j'[

cp

\037

cp

\037

p=a

Vp2 +
Hence,)

p\"

(f)

V2 cos2cp;

' = _ a V 2 sin

Pcp

= 1/\"2a
, 2 ( cas2cp +

cp

2m
'Y

Vcos2cp

sin 2

cos

2cp

2cp

a Y2
V cos

cp

l=aV 2 s Ycos2<p=a V 2 S Yl-2sin2


o

dcp

2<p

dcp

\302\267

cp

The latter integral is calledthe ellipticintegralof the first kind.


be reduced to a form convenient for computing with the

It can
aid of

specialtables.)))

7.9.Arc

\037

Find the
7.9.5.

7.9.6.Compute

p = a see( qJ

\037

Length of

a Curve

between qJ =

and

\302\260

= a see qJ(

\037

since on the interval


is positive.)
:rr)

sec2 ( qJ

a.f
o

7.9.7.Find the
curve p ==a sin4

j.

\037

\037

(qJ--T);

V 1 + tan ( qJ2

\037

[0, ]

= a sec2

see qJ- )
(

qJ

IS

\037

\037

).

omitted,

this function

\037

qJ

(The sign of the modulus in the function

l=

353)

the length of the segment of the straight line

\037

p\037

Polar Coordinates

arc length of the curve p = a sin3

Solution. p\037=asec qJ- ) tan


(

Vp2 +

in

\037

) dqJ

= 4 \0373 a.
of the

length

Solution.Since the function

closed

p ==a sin4

a)

x)

is even, the given curve is symmetrical about


the polar axis.Sincethe function sin4
has

Fig.

99)

period 4Jt, during half the period from 0 to 2Jt the polar radius
increasesfrom 0 to a, and will describehalf the curve by virtue

of its symmetry (Fig. 99).


Fur ther, p ==a sin3 (cpj4)cos(cp/4) and
\037

V p2

+ p'2 == V a2 sin

(f)

+ a2 sin6 (cp/4)cos2 (cp/4)==a sin3 (cpJ4),

(cp/4)

if O\037cp\0372Jt.

Hence,)
:2

1= 2a

7.9.8.Find
and p

===

4.

S
o

sin3 (qJ/4) dqJ = 8a

/2

3
= 6
50 sin t dt 13 a

the length of the curve

Solution.The
dl

'JT

.\"'1

diff

= V p2 +

p\037

erential of the arc


dqJ

(qJ

= 4t).

qJ=f(p+lip) betweenp=2
dl is

= V p2 dqJ2 + dp2 =

equal to

Vip2 (

2
\037:

-f 1

dp.)))

Ch. V

354)

II. Applications

of the Definite

From the equationof the curve we


4

1=

V p2 -}( 1
S
2
\302\267

::=-{ - ).

find

4
2

p12

+ 1dp = S
2

Integral)

JI-}(p2

2+

;2

p12

Hence,

+ 4 ) dp =

JI(p + r dp={(

s=
\037j

\037

2)

7.9.9.Find the
3

<PI

==\"4

and

CP2

length of the
4

\0372

+ ln p ) I:=3+

hyperbolic spiral

pcp

1\0372

= 1 between

\302\267

7.9.10.
Compute the length of the closedcurve p == 2a (sin + coscp).
7.9.11.
Compute the arc length of the curve p= l+P
cos from
cP

<PI

\037

-2
j'[

to

=2

<p)

j'[

CP2

\302\267)

7.10.Area of Surfaceof Revolution

The area of the surface generatedby revolving about the x-axis


the arc L of the curve y ==y (x) (a x b) is expressed
by the
\037

\037

integral)

P = 2n

\037

VI + y'2 dx.

a)

It

is more convenient to write this integral in the

form

P = 2n ydl,
\037

where dl is
length.
If a curve is representedparametrically or in polar coordinates,
then it is sufficient to change the variable in the above formula,
expressingappropriately the different ial of the arc length (see 7.8
and 7.9).
Find the area of the surface formed by revolving the
the differential of the arc

\037\037

7.10.1.
2

astroid x3\" -t- y3 ==a 3\" about the x-axis.


Solution.Differentiating the equation of the astroid we get
2

whence)

-...!...2

3 +3
x

1
3

y' == 0,)

' _ L13
x

3)))

\302\267

7.10.A rea of Surface of

\037

V I + y'2 = -.
V

Then,

+ y: = x3

a\037

1
x

355)

Revolution)

. Since the

astroid is sym-

3
l

metrical about the y-axis, in comput ing the area of the surface we
may first assume x 0, and then double the result.In other words,
the desired area P is equal to)
\037

p= 2 x2nS yVl + y'2 dx= 4n S ( a3-x3) 2a3x-3dx.


o

0)

Make the substitution)

--x-= ,
I
2 -- =
x dx 2t
2

2
3

a3

-3

I al/3

Then

P = 12na3

t 4 dt

t2

na 2
\0372

t)
I

1
\302\260

dt,)

a3
\302\260)

o)

7.10.2.
Find

the area of the surface generatedby revolving about)


!f
by
Solution. It is easy to check that the 1
given parabolas intersect at the points 0
(0,=0) and B (1, 1).The sought-for area
P PI+ P2, where the area PI is formed
by revolving the arc OCB, and P2 by revolthe

x-axisa closed contour OABCOformed


the curves y = x2 and x = y2 (Fig.100).

arc OAB.

ving the

Compute the area PI' From the equat ion


I
o.
x = y2 we get y = Vx and y' = 2

JIX

Hence,

Fig.

100)

VI +

dx = 2n
4\037

V +
.f 4;
6)

dx =

-.

= 6 (4x + 1)2 = _6 ( 5)r-5


j'[

Now

:c)

PI= 2n S V-X

1)

\302\267

compute the area

P2. We

have
1

\037

j'[

0)

y=:.x2, y'=2x and)

P2 = 2n x2 VI+ 4x2 dx.


o)))

1)

Ch,

356)

The substitution
Arsinh 2

P2 =

II. Applications

Integral)

x = -}sinh t, dx = cosht dt
\037

slnh 2 t

of the Definite

== Jt

cosh2 t dt

32 (

4 sinh 4t

gives)

o)

==

9 V5n
16

Arsfnh

32

==

1t In

(2

+ V 5.)
)

Thus,

P=P1+P2 = (5\037-1)1t
+ 9\03751t

nln(2+V'5)=

3\037

67
\0371t

7.10.3.
Compute the

In

)-

(2+V s

\037

area of the surface generated by revolving:


(a) the portion of the curve y= , cut off by the straight line
\0372

y=

, about the y-axis;


(b) the portion of the curve y2==4+x,rut off by the straight
line x== 2, about the x-axis.
Find the surface area of the ellipsoidformed by revolving
x2
y2 =
the ellipse + b2
1 about the x-axis (a > b).
Solution.Solving the equation of the ellipsewith respect to y
for y
0, we get

7.10.4.

\302\243l2

\037

==

a Va -x
b

2
;)

, ==
.

V 1

Hence)

P - S -V
a

2Jt

-a)

a2

-x

-{i ..r -x
b

x
2
J' a

\302\267

--b
- ( _(a(2a-x)
x--X
2

a4

+ y '2 _

a2

\302\267

2)

a4_(a2-b2)X2d
a2 2 2
(a

2 2
)X

4
==d _2 b
I -E +
a SVa-EX x-Jta (V
1tb

2 2

\037

...I 2 -b2 a

arcSinE

a
== c is the eccentricity of
where the quant ity E == V
a'!.
e IIip se
a the eccentricityE tends to zero and
When b

-).

In1
0

E -+

since the ellipseturns


area of the sphere:)

into a

arc sin
E

the)

= 1,

circle,in
p = 4Jta2
.)))

the limit we get the surface)

\037

7.10.Area

of

Surface of

357)

Revolution)

7.10.5.
the area of the surface obtained
Compute
2
==
4x
ellipse + y2 4 about the y-axis.
7.10.6.
An arc of the catenary
-a
x
y = '2( e:+ e ) = a cosh a '

the

by revolving

whose end-pointshave abscissas0 and x, respectively, revolves about


the x-axis.
Show that the surface area P and the volume V of the solid thJ..ls
2V
generated are related by the formula P = a

000:

Solution.Sincey' ==sinh

y'\037

dx = 2a'Jt cosh2

dx =

\037

hence, P

2V

===

:rr

5o

Therefore

a)

P = 2'Jt 5 y VI +
but)

VI + y'2 ==cosh\037.

we have

\037,

'Jt
\037

2
2
5 a cosh dx,
\037

a2 cosh2

dx = 'Jt

5 y2 dx

= v,

0)

a
Find the area of the surface

7.10.7.

of the curve 9ax2 ==y

obtained by revolving a loop


the y-axis.
by a moving point as y changes
respect to y both sides of the

(3ay)2 about
is described

Solution.The loop
0 to 3a. Differentiate with
equation of the curve:
== (3a18axx'
y)2_ 2y (3a -y) = 3 (3a
from

- (a-

y),

y)

whence xx'= (3a-\037a(a-y) Using the formula for computing the


area of the surface of a solid of revolution about the y-axis, we have)
!J2

Y2

2
P=2'Jt5 xVl+x'2dy=2'Jt
5 Vx + (xx')2dy=
ljl

= 2'Jt

3a

r
J
o

ljl

.. (3a-y)2 +
9a
V
y

3a

(a-y)2 d = (3a2+ 2a _
36a2
3aS

(3a-y)\037

\037

7.10.8.
Compute
== 2

2
y2) dy == 3Jta

the area of the surface generatedby revolving


4 about the x-axis.
the curve 8y2 x
7.10..9.
Compute the area of a surface generated by revolving
about the x-axisan arc of the curve x= (2; y = ((2-3) between
the points of intersection of the curve and the x-axis.)))

-x

\037

Ch. V

358)

II. Applicati

'Jns

cf

the Definite Integral)

== 0, find t 1 == and t 2, 3 == + V 3, and, hence,


y
Whence it follows that the curve intersects
and x 2, 3 =
with the x-axisat two points: (0, 0) and (3, 0). When the parametert changes sign, the sign of the function (x)t remains unchanged, and the funct ion y (t) changes
!J
its sign,which meansthat the curve
1
is symmetrical about the x-axis

Solution.Putting

3.

=0
Xl

\302\260

:c

(Fig. 101).

To find the area of the surface it


is sufficient to confine ourselves to
the lower port ion of the curve OnB
that corresponds to the variat ion
of the parameter between
and

,=

Xt

+ V Differentiating
to t, we find

'-- -)

2t ,Yt

and the linear element


dl = Vxi 2 + Y; i dt
Hence,)

P = 2n

/2
\037

t1

y (t)

Y3

=2n S

\037

\302\260

3.

Fig. 101

t2

with

respect

+ t 2) dt.)

== (1

Vx? + y? dt =
(t

-3)(1+t)dt=-

Va

n
\037

7.10.10.
Compute2the

S
0

(t

-2t -3t)dt=3n.
3

surface area of the torus generated by reabout the x-axis.


Solution.Let us representthe equat ion of the circlein parametric
form: x == r cast; Y ==b + r Si 11 t.
Hence)
,
volving the

circlex + (y-b)2==r 2 (0 < r < b)

The desired area

Xt

==

r SIn t ;

Yt

==r

cost

.)

is)

23t

P = 2n (b+r sin t) V(-r sin t)2+ (r cost)2dt =


\037

o)

2Jt

= 2nr

7.10.11.
Compute t he

\037

(b

+ r sin t) dt = 4n2br.

area of the surface fornled by revolving


the
about the polar axis.
Solution.Real values for p are obtained for cas2cp 0, e. for
(the right-hand branch of the lemniscate),or for

lemniscatep ==a Vcos2cp

\037

\037

cp

\037

\037

\037

n:::;;
cp:::;;n
\037

(the left-hand branch of the lemniscate).)))

i.

7.10.Area

\037

The

lin

ear element

r
dl ==JI p2

of

of t he

Surface of

359)

Revolution)

lemniscateis equal to

a sin
+ p'2dcp == Y a 2 cas2cp + ( \037cos2<p
)
2<p

adcp

dcp ==

\037cos2\037

==a sin Vcos2cp.


The sought-for surface area P is equal to double the area of the
surface generated by revolving the right-hand branch. Therefore)

Besides,y ==p sin

cp

cp

:rt

===

2X

2'Jt

5L Y dl

==4na 2

\302\245

50

cas2<p sin
V cos2<p

<p

d<p

= 2na2 (2 V2 ).

7.10.12.

Compute the area of the surface formed by revolving


about the stra ight line x + y =:.:a the quarter of the circlex 2 + y2 ==a 2
between A (a, 0) and B (0, a).
Solution.Find the distance MN from the moving point M (x, y).
2
2
==
lying on the circlex + y2 ==a , to the straight li ne x + y

a:

MN = x+ \037a2-x2-a= x+
I

2
'V a

\302\2452

since for the points

\302\2452

circle that

of the

x+ y a. Fur ther,

-x -a
2

lie in the

first

quadrant

\037

x
dl=Vl+y/2dx=-./l+
r ( \037a2-x2)

Hence,
a
-

P == 2'Jt j x +
o

\037

a2

-=x

\0372

a dx

adx

dx ==

\037a2-x2

==

\037a2-x2

=V2na[-Va2-X2+x-aarcsin
J:=
\037

\037;

(4-n).

7.10.13.

Compute the area of


the surface formed by revolving
one br anch of the lemniscate p ==
==aVcas2cp about the straight line

:c)

3t

B)

CP==T'

Solution.From the triangleOMN


(Fig.102)we find the distanceMN

of an arbitrary point M
axisof revolut ion
cp

\037

MN

of

= :

= P sin

cp

\037

Fig.

the right-hand

= a Vcas2<p sin

102)))

branch from

<p

\037

;)

the

360)

Ch. VII.

Applications

then)

dl ==

of the Definite

a dcp
JI cos

Integral)

2cp)

1(/4

Therefore P==2Jt

-S3t/

aVcos2cpsin(

\037

=2na2.
-cp) Vad<P
cos
2q:>

4)

7.10.14.
Compute the

area of the surface formed by revolving


3
between x==-2 and
about the x-axisthe arc of the curve y=
x===2.
Compute the area of the surface generatedby revolving
one half-wave of the curve y ==sin x about the x-axis.
Compute the area of the surface generated by revolving
about the y-axis the arc of the parabola x 2 = 4ay betweenthe points
of intersectionof the curve and the straight line y==3a.
t 7. Find the area (f the surface formed by revolving about
the x-axis the arc of the curve x = et sin t; y ==et cost between
j'[
t = 0 and t
==\"2
Compute the area of the surface 3obtained by 2 revolving
t
t
about the x-axisthe arc of the curve x==T; y==4- between
the points of its intersectionwith the axesof coordinates.
19.Compute the area of the surface generatedby revolving
the curve p ==2a sin about the polar axis.
7.10.20.
Compute the area of the surface formed by revolving
about the x-axisthe cardioid
x ==a (2 cos
2t),
==a (2 sin
2t).)
Y
\037

7.10.15.
7.10.16.
7.10.

7.10.18.

7.10.

cp

t-cos

t-sin

\037

7.11.Geometrical
Applications
of the DefiniteIntegral
Given: the cycloid (Fig. 103)
7.11.1.
x==a(t-sint);y=-=a(l-cost);O\037t\0372Jt.
Compute:
(a) the areasof

surfaces formed by revolving the arc OBA


y-axes;
(b) the volumes of the solidsgeneratedby revolving the figure OBAO
about the y-axis and the axis BC;
the

abowt the x- and

(c) the area of the surface generated by revolving the arc BA


about the axis BC;)))

\037

Geometrical
7.11.

Applic's

of the Definite

361)

Integral

(d) the volume of the solid generated by revolving the figure


ODBEABOabout the tangent line DE touching the figure at the
vertex B;
!J
(e) the area of the surface formed by JJ
revolving the arc of the cycloid [see

\302\243)

(d)].
Solution.(a)

item

When
revolving about
the x-axisthe arc OBA generatesa sur- 0
face of area)

Fig.

:!n

Px==2nj'YdL=2nJ
L

::c)

c)

A)

103)

a(l-cost)2asindt=
\037

0)

2Jt

== 8a2n

sin3 t2 dt
S

===

64J[a2
3

o)

When revolving about the y-axIs the arc OBA generatesa surface of area)
:ry

2
=
Py 2n .fxdl = 4na S (t-sint) sin +dt +
0

2Jt

+ 4na2 .f (t -sint) sin

2Jt

dt

= 4na2

\037

S (t -')int) sin

dt
\037

= I6n2a2.

0)

(b) When revolving about the y-axis the


solid of volume
2a

Vy = n

\037

(x\037-xi) dy = n

2a
\037

x\037

OBAO generates

figure
2a

dy-n0

\037

x\037

dy.

where X==X 1(y) is the equation of the curve BA, and x=x2 (y) is
the equation of the curve OB.
Making the substitution y ==a
t), take into consideration
that for the first integral t varies between 2n and n, and for the
second integral between 0 and n. Consequently,

(I-cos

IT

Vy=n

\037

a2
\037

2n

(t-sint)2asintdt-n0 a2 (t-sint)2asmtdt=
\037

= na 3

(t

\037

-sint)2 sin t dt =

2.'1:

Jta\037

f
r_2.

t 2 sin t

dt

2\037

(I-cos2t) dt + f

2Jt

sin3 t dt

== 6n 3a 3

For computing the volume of the solid obtained by revolving


figure OBAO about the axis BC it is convenient first to trans-)))

the

Ch,

362)

II. Applications

of the Definite

Integral)

fer the origin into the point C, which yields the following equations in the new system of coordinates
x' ==- a
t); y' ==a
t).
Taking into account only the arc BA, we get

(t-1t-sin
j[

2a

=n

X'2
\037

dy' = na3 (t
\037

Putting

(I-cas

-n-sin

t)2

sin t dt.

2Jt

t-n= z, we obtain
o

\037

na 3 (z + sin Z)2 sin z dz = na 3 (z + sin Z)2 sin z dz =


\037

\037

Jt

(9n2

Jt\037

(c) Making the above-indicatedshift

_ 16).

origin, we get

of the

Idtl=-2asin dt.)

dL=2asin
\037

\037

Therefore)
n

2a

2nXdL=-4na2 S (t-n-sint)sindt=
p=.f
o
\037

231)

Jt

= 4na2

.f (z +

sin z) cas dz = 4
\037

2n- )
\037

(d) Transferring the origin into the point B and changing


directionof the y-axis, we get
x' ==a (t
t), y' ==a (1+ cost).
Putting

-1t==

-1t to

= 1t

-n
\037

1t

a3 ( I

5
-n

Find
7..11..2.

cosZ)2 ( I + cosz) dz ==n 2a S.

31

1t

y dL

y' ==a (I-cos


z),
for the arc OBA. Hence

+ sin z),

31

(e) P = 2n

the

we have

Z,

changingfrom
V

-n-sin

x' ==a (z
z

na2

= 4na2

z) cas
5 (I-cas
-Jt)

the volume of the

and
z2==8(2-x)

X2 -t-y

2==2x.

\037

dz = 33 :rra2

solid bounded

by

.
the

surfaces

Solution.The first surface is a parabolic cy linder with generatfices parallelto the y-axis and the directrixZ2 ==8 (2-x)in the
plane xOz, and the second is a circular cylinder with generatrices)))

7.11.
Geometrical

\037

of the Definite

Applic's

363)

Integral

x2 + y 2 ==2x in the planexOy.


parallel to the z-axisand the directrix
2
The volume

is computed by the formula

presents the area of a triangle whose base is


tude to 2z:)

= S (x)dx. S (x)reo
equal to 2y and alti-

\037

S (x) ==2y x 2z ==4 V2x-x2 8 (2 -x).)


Hence,
2
2
= 4 VB (2-x)VX'dx =
4 Vx (2-x)8 (2-x)dx
V =
J/\037

\037

\037

= 4 VB

. 1.3.

; Vx 2

\037

256
Vx& ) = 5 .
I\037

7 t
Prove tha t if the figure S is bounded by a simpIe convex contour and is situated betweenthe ordinates YI and Y2 (Fig.104),

solid ge-

then the volunle of the

!J

nerated by revolving this figure


about the x-axiscan be expressed
by

the

!Jz

formula)
Y2

= 2:rt
\037

yh

dy,

!f)

Yl)

where)
h ==X 2 (y)

-Xl(y),

being the equation of the !if


portion of the contour llnd
x ==x2 (y) that of the right portion. 0
Solution.Let the generatingfig-

==Xl (y)

left

Fig. 104

S be bounded by a simple
convex contour and contained between the ordinates
ure

YI

and

Y2'

Subdivide the interval [Yl' Y2] into parts and pass through the
points of division straight lines parallel to the axis of revolution,
thus

cutting the

figure
by

strip and replaceit

the

strips. Single out one


rectangle ABCD, whose lower base is

into horizontal

equal to the chord AD = h specified by the ordinate y, its altitude


AB being equal to
The solid generatedby revolving the rectangle
ABCD about the x-axis is a hollow cylinder whose volume may
be approximately taken for the elementof volume
\037y.

\037V

\037

n (y

\037y)2

h-

2
ny h

= 2ny\037yh + nh

(\037y)2.

Rejecting the infinitesimal of the secondorder with respect t) \037y,


we get the principal part or the differential of volume
dV = 2n yh dYe
Knowing the differential of the volume, we get the volume proper)))

Ch. VII.

364)

of the Definite

Applications

Integral)

integration:)

through

Y2

=2n

\037

yhdy.

Yl)

Thus, we obtain one more formula for computing the volume of


the solid of revolution.
The planar region bounded by the parabola y==2x2 +3,
7.11.4.
and the verticals x== 0 and x == 1 revolves about the
x-axis
the
y-axis.Compute the volume of the solid of revolution thus generated.
Solution.Divide the area of the figure into elementary strips by
of the elemenstraight lines parallel to the y-axis. The volume
is
one
revolving
strip
by
tary cylinder generated
\037V

== n (x-\\- \037X)2

\037V

where

\037x

y-n x2

is the width of the

= 2Jt xy \037x+

ny

(\037X)2,

strip.

Neglectingthe infinitesimal of the secondorder with respectto

we get the different ial of the

\037x,

desired volume

dV ==2n xy dx.)

Hence)

1
V

=
\037

2n xy dx = 2n

\037

x (2x2 + 3) dx = 4n.

0)

7.11.5.
Compute the
z

area of the portion of the cylinder surface


x2 + y2 :::::tax sit uated inside the
sphere) 2
x -t-y2 + Z2 = a2
of the
Solution.The generatrices
cy linder are parallel to2 the z-axis,
2
a
the circle x
+ y2 == a
.)

a)

-2)

serving as directrix(Fig. 105


:c shows a quarter of the soughtfor

surface).

Subdivide the portion of the

circle shown

in

small arcs
passing through

Fig. 105 into

The generatrices
the points of di!I)
cut the cylinder surface
vi\037ion
Fig. 1 05)
into strips. I.f infinitesimals of
of the strip ABCD is equal to)
area
the
are
order
neglected,
higher
\037l.

CD.

\037l.

e polar coordinates of the point D, then


2
and CD==Va
-pz==asincp,and \037l===a.L\\<p, whence
p==acoscp
If p and

cp

are

t\037

we)))

\037

find

7.11.
Geometrical

of the Definite

Applic's

365)

Integral

the element of area:)

dP ==a2 sin

cp

dcp.)

Hence,)
.IT

P = 4 a2 sin

cp

\037

dcp

= 4a2 .

II)

7.t 1.6.Find

the area of the surface cut off from a right


a
cylinder by plane passing through the diameter of the
inclined at an angle of
to
z
the base.

circular

base and

45\302\260

Solution.Let the cylinderaxis


be the z-axis,and the given diameter the x-axis.Then the equation of the cylindrical surface
2
2

be x + y2 ==a , and that of


the plane forming an angle of
with
the coordinate plane xOy
will

45\302\260

will

be y==z.

The area of the infinitely nar- x


row strip ABCD (seeFig.106)will
Fig. 106
be dP ==zdl (accurateto infinitesimals of a higher order), where dl is the length of the
tary arc of the circumference of the base.
Introducing polar coordinates,we get

Hence dP==a 2 sin

elemen-

z==y=asincp; dl=adfP.
cp

dcp

and

:rt

P=a2 sincpdcp=a2[-coscp]\037=2a2.
\037

o)

7.11-.7.
The axesof

sect at

two circularcylinders with equal bases interangles.Compute the surface area of the solidconstitu-

right

ting the part common to both

7.t 1.8.Compute

cylinders.

the volume of the solid generatedby revolving


about the y-axis the figure bounded by the parabola x2 ==
the
axisof abscissasand the straight lines x=.Oand x=
Find the area S of the ellipsegiven by the equation

1. y-

7.11.9.

Ax 2

+2Bxy+Cy2== 1 (B=AC-B2> 0; C>0).


\\vith
respectt o y, w e get
-Bx+VC-bx2
== -Bx-VC-6x\037 .
Y
2

Solution.Solving the equati on


1

,Y

=-.=

where the values of x must satisfy the inequality

C-ox2

\037O.)))

\302\267

1,

Ch. V

366)

II. Applications

of the Definite Integral)

-{

Solving this inequality, we obtain the limits of integration:


'
\037x\037V'
Consequently,the sought-for area is equal to
\037

\037

\037

S=

\037

(Y2-Yl)dx=

.f

_(

in

f2;

t2

Vb

'

the figures bounded by the curves rep-

l+t

resentedparametrically:

x== 2f

_('jx2dx =

S
0

\037

\037

7.11.10.
Find the areasof
(a)

{ Vc

\037

== 2t 2
t

f3;
t 2)
2

(1

;
(b) x== 1+t2
y==
7.11.11.
Find the areasof the figures boundedby the curves given
polar coordinates:
(a) p ==a sin 3cp (a three-leaved rose);
(b ) p

--

\302\267

.
]'

\037
\037
2
[ 4 -.;::::-.;::::
==
==
3
3
sin
and
p V cos
(c) P
4
7.t
Find the arc length of the curve y2 == q (2
by the straight line x==-l.
Find the length of the arc OA of the curve
1

\037

cos

cp

\037

<p

cp

y ==a

wher.e 0 (0, 0);

- --

cpo

1.12.
7.11.13.

, a

7.11.14.
Compute the

In

In

:.

7.11.15.

off

-x '
2

of the curve

ta ined insi de the para bola y2 ==


Prove that the length of the
\037

cut

a2

a2

arc length

X)3

y2==\"3(x-l)3con-

ellipse

x==V2sint; y==cost
equal to the wavelength of the sinusoid y == sin x.
I
7.1
Prove tha t the arc of the parabola y == x2 correspon2p
ding to the interval 0 x a has the same length as the arc of
the spiral p==Pcpcorrespondingto the interval O\037p\037a.
Find the ratio of the area enclosedby the loop of the
x Vx to the area of a circlethe circumference
curve Y = +
)
(
of which is equal to the length of the contour of this curve.)))
is

1.16.

\037

7.11.17.

+-

<

\037

7.1t .18.Find

7.12.
Computing

Physical

the volume of the

segment cut

cal paraboloid + = x by the plane x ==


2p 2q
Z2

y2

7.11.19.
Compute the
2

367)

Quantities)

off from

a.

the ellipti-

volume of the solid bounded by the hyperand the planes z ==c and z == I > c.

== -1
2+ ---:F-T
x

boloid a

y2
tJ

Z2

c)

7.t t .20.Find the volume of the right ellipticalcone whose base


a and b, its altitude being equal to h.
ellipsewith semi-axes
7.t t .21.Find the volume of the solid generated by revolving
about the x-axisthe figure bounded by the straight lines y ==x+ 1;
y==2x+1 and x==2.
7.t t .22.Find the volume of the solid generated by revolving
x2
== 1,
about the x-axisthe figure bounded by the hyperbola a2 y2
b2
the straight line 2ay-bx==0 and the axis of abscissas.
is an

7.1t .23.Find

curve p ==a cos2

the volume of the


about the polar

cp

sol.idgeneratedby revolving the

axis.

7.11.24.

Find the areas of the surfacesgeneratedby revolving the


following curves:
about the x-axis;
(a)

y=tanx(O\037x\037 )

(b) y=x

\037

\037

(O\037x\037a)

about the

x-axis;

(c) +y2-2rx==Oabout the x-axisbetween 0 and


x2

\037

h.)

7.12.ComputingPressure,Work and Other Physical

Quantities
by

the

DefiniteIntegrals

I.

To compute the force of liquid pressure we use Pascal's


law, which states that the force of pressure of a liquid P on
an area S at a depth of immersion h is P = yhS, where y is the
specific weight of the liquid.
If a variable force X ==f (x) acts in the directionof the x-axis,
then the work of this force over an interval [Xl' x2] is expressed
by the integral)

II.

X2

\037

f (x) dx.

1)

III. The kinetic energy of a material point of mass m and velocity v is defined as)
n1V'l.

K==y-)))

Ch. VII.

368)

of the Definite

Applications

IV. Electricchargesrepulse each

other

Integral)

a force F ==
,
r
where e1 and e2 are the values of the charges,and r is the distance
wi th

el\0372

between them.
Note. When solving practicalproblems we assume that all the
in one and the same system of units and omit
data are expressed
the dimensionsof the correspondingquantities.
7.12.1.
Compu te the force of pressure experienced
by a vertical
triangle with base b and altitude h submerged base downwards in
water so that its vertex touches the surface of the water.
Solution.Introduce a systemof coordinatesas indicated iR Fig.107
and considera horizontal strip of thickness dx located at an arbitrary depth x.
Assuming this strip to be a rectangle,find the differential of area
dS== MN dx. From the similarity of the trianglesBMN and ABC
bx
bx
M N == x
dS == h
whence MN == hand
dx.
\\Ve h ave

f) Ii'

o)
70)
A)

A)

b)

c)

:c)
Fig.

Fig.

107)

108)

The force of pressureexperienced


by this strip is equal to dP ==xdS
of higher order (taking into consideration
that the specific weight of water is unity). Consequently,the entire
force of water pressure experienced
by the triangle is equal to

accurateto infinitesimals
h

\302\267

-'.
h

P ==.J xdS== h
{

r\037

\\

X2

dx == 3

bh 2

(I

of
7.12.2.
Find the force of pressure experienced
by a semicircle
radius R submerged vertically in a liquid so that its dianleter is
the liquid surface (the specificweight of the liquid is V).
flush with
A vertical dam has the form of a trapezoid whose upper
7.12.3.
base is 70 m long, the lower one 50 m, and the altitude 20 m.
Find the force of water pressure experienced
by the dam (Fig.108).
Solution.The differential (dS) of area of the hatched figure is
approximately equal to dS == MN dx. Taking into consideration
the)))

7.12.Conzputing

\037

Physical

369)

Quantities)

similarity of the triangles OML and OAE, we

find
\037OL

= 202-;;-x ;

ML==20-x,
MN==20-x+50==70-x.
Thus, dS==MNx
x dx == (70-x)dx and the differential of the force of water pressureis
equal to)
dP== xdS= x (70-x)dx.
Integrating\\vith respect to x from 0 to 20,we get

\\vhence

20
\037

P = (70x-x2)dx=11.3333\"'
1

,\\

(I)

7.12.4.
Calculatethe

performed

\\vork

pumping the water out

in

semisphericalboiler of rad ius R.


A rectangular vessel is filled with equal volumes of \\vater
7.12.5.
and oil;water is twice as t:eavy as oil.Show that the force of pressure of the mixture on the wall will
Z
0
reduce by one fifth if the water is
!J
rep laced by oi 1.
Solution.Let h be the depth of
of a

\037

the vessel and I the length of the

wall. Let us introduce a system of 2


coordinates as shown in Fig. 109. x
Sincethe oil is situated above the h
water and occupiesthe upper half of
J
the vessel, the force of the oil presI
m
sure experiencedby the upper half
of the wall is equal to

Fig. 109

2
lh2
P = 2S xl dx = 16
1

\302\267

The pressure at a depth


oil column of height

-2'
h

\037

x>

is made up of the pressureof the

and that of the water column of height

and therefore

dP2 =

[ x -}+ ( x
\037

-; ]
)

1dx =

\037

) 1dx.

Consequently,the force of pressure of the mixture on the lower half

of the

\\va

11 is)

P2=,h 1 ( x-T) dX=T'


\037

\\

2)))

It

lh2

Ch. VII.

370)

of the Definite

Applications

Integral)

The entire pressure of the mixture on the wall is equal to


2

lh
lh
== 5 lh2 .
P = P1+ p2 = '4
+ 16
T6

If the vessel were filled only


the same wall would be)

p ==

oil,the

with

force of pressureP on

r xl dx ==lh'l.

'..1

\302\267

o)

Hence,)

p-p- ==16 5P.


I

lh2

===

7.12.6.
The electricchargeE concentratedat the origin of coordinates repulses the chargee from the point (a, 0) to the point (b,O).
Find the work A of the repulsive force F.
Solution.The differential of the work of the force over displaceeE
ment dx IS dA =Fdx=2\"dx.
x
Hence)

---).

IJ

dx
=eE\\ 2=eE
x
(a
I

\037

a)

As

b --+00 the work

I \\

tends to eE .
a)

Calculate the
7.12.7.

work performed in launching a rocket of


from
the
ground vertically upwards to a height h.
weight
Solution.Let us denote the force of attraction of the rocket by
the mass of the rocket by mR' and the mass of
the Earth by
the Earth by mE' According to Newton's law)

F,

k mRmE
2
x

,)

where x is the distance between the rocket and the centre of the
Earth. Putting kmRmE=K, we get F(x)= , R\037x\037h+R,
R being the radius of the Earth. At x == R the force F (R) will be
whence K = P R2
F (R) = P =
the weight of the rocket P,

i.e.

PR 2

\0372

and F (x) = \037


x
Thus, the differential of the work is
PR
dA == F (x) dx ==

X2

dx.)))

'

\037

7.12.Computing

Integrating,we obtain
R+h

371)

Quantities)

Physical

R+h

F (x) dx = PR2 S =
S
R
R
=
The limit !irn A (h) = Iim
P
is
R
equal to the work performed
h_oo :\037hh
h_oo
by the rocket engineto achieve complete
escapeof the rocket from.
the Earth'sgravity field (the Earth'smotion is neglected).
A

\037:

\302\267

\037hh

7.12.8.
Calculatethe work that has to be done to stop an iron
sphere of radius R rotating about its diameter with an angular velocity w.

Solution.The amount of required work is equal to the kinetic


energy of the sphere. To calculatethis energy divide the sphere
into concentrichollow cylinders of thickness dx; the velocity of the
points of such a cylinder of radius x is wx.
The elementof volume of such a cylinder is dV ==4Jtx VR2 x2 dx,
the element of mass dM == dV, where y is the density of iron, and
2 dx.
the differential of kinetic energy dK = 2Jtyw 2x3 VR2

-x

'\\'

Hence,

-x

VR2 2 dx= 43tI'R3


3
S
o
Calculate the kinetic energy of a
7.12.9.
X3

K == 2JtYw 2

S-=

Mw 2R2

\302\267

disk of mass M and raan angular velocity w about an axis passing:


its centre perpendicular to its plane.
Find the amount of heat releasedby an alternatingsi-

dius R rotating
through

w 2R2

\302\267

with

7.12.10.

nusoidal current

I=

sin
t -<p
)
a cycle T in a conductor with resistanceR.
during
Solution. For direct current the amount of heat releasedduring
a unit time is determined by the Joule-Lenzlaw

C;

III

2R.
Q==O.24/
For alternating current the differential of amount
dQ = 0.24 (t) R dt, whence)

of heat

IS

t\037

/2

Q =0.24R 12 dt.
\037

In

t his

case)

Q = 0.24

R/\037

T
2

sin 2;t
S
(
o

(p

dt

1)

[t- ;1I:

=0.12R/\037

23t
.
SIn:'
t-cp
'>

\037

] 0 =0.12RT/\037.)))

II. Applications

Ch. V

372)

of the Definite

Integral)

7.12.11.

Find the pressure of a liquid of specificweight d on a


vertical ellipsewith axes2a and 2b whose centre is submergedin
the liquid to a level h (h b).
Find the pressure of a liquid of specificweight d on the
of a circular cylinder of base radius r and altitude h if the
\\vall
\037

7.12.12.

cylinder is

full

of

liq uid.

7.t 2.13.Calculate the

work performed to overcome the force of


water
the
out of a ronicalvesselwith the vertex
gravity
pumping
downwards;the radius of the cone base is R and its altitude is
Compute the work required to stretch a spring by 6 cm,
if a force of one kilogram is required to stretch it by 1 en1.)
in

H.

7.12.14.

* 7.13.ComputingStaticMomentsand Momentsof
Inertia.Determining
Coordinates
of the
Centreof Gravity
In all problems of this paragraph we will assume that the mass
is distributed uniformly in a body (linear,two- and three-dimensional)
and that its density is equal to unity.
1.For a plane curve L the static moments Mx and My about
the x- and y-axis are expressed
by the formulas
Mx

= ydl,
\037

My=

\037

xdl.

L)

The moment of inertia about the origin of coordinates


10

=--=
\037

(x2 + y2) dl.

L)

If the curve L is given by the explicit equation y== y(x) (a\037 x\037 b),
then dl has to be replacedby VI y'2 dx in the above formulas.
If the curve L is given by the parametric equations x ==x (t),
==
t
t 2), then dl should be replaced by V X'2
y'2 dt in
Y (t) (t 1
Y

\037

\037

these formulas.
2. For the plane figure bounded by the curves y ==Yl (x),Y == Y2 (x),
== a, x ==b (a
x<:b) the
Yl (x) <:Y2 (x) and the straight lines x
the
formulas)
static moments are expressed
by
\037

Mx

=+ S (Y\037-yDdx;

l'

3. l'he centre of
dinates: Xc =

My= S X(Y2-Yl)dx.
a)

plane curve has the following COOfwhere l is the length of the curve

gravity of a

Mv

Yc

T'
Mx

L.)))

7.13.Conzputing

\037

Static Moments

and Moments of Inertia

The centre of gravity of a plane figure has the


= , where S is the area of the figure.
Yc

373)

==
coordinates:
,
Xc
sY

\037

\037

7.t 3.I. Find

the stat ic moment of the upper portion of the


x2

+ 7fi =

a'\037

x-axis.

about the

ellipse

y'l

1)

Solution.For the ellipse

since y2

b2

-==

dl

dl ==y V 1

+ y'2 dx ==Vy2 + (yy')2dx;

-di
b2

b2

2
(i2 x and yy'

===

x, we

have

... b2 _!?:.-.
2
2 -e2x2 dx
2
==
'
a2 x + a4 x dx a V a
\037

\037

where e is the eccentricity of the


to a, we
Integrating from

-a

ellipse,e == V

a\"J-b2
a

find)

a' Va2-e2x2dx==S

M x ==\037 r
v

-Q

2b

Va2-\342\202\2542X2dx==

-8

2
2a 2
+ :2arcsin 8) = b (b + arc sin f).
(a Va
In the case of a circle,i. e. at a = b, we shall have Mx== 2a2,
. arcsine 1.
. e==0 an d I 1m
since

--=

e 0
Find the moment of inertia of a rectanglewith base band
altitude h about its base.
Solution. Let us consideran elementary strip of width dy cut
out from the rectangleand parallel to the base and situated at
The mass of the strip is equal to its area
a distance y from
dS==b dy, the distances from all its points to the base being equal
to y accurate to dYe Therefore, x == by2 dy and
e)

\037

7.13.2.

it.

dl

Ix == S by2 dy ==
o

7.13.3.
Find

3
bh:l

\302\267

the moment of inertia of an arc of the circle + y'!- == R2


quadrant about the y-axis.
lying
7.t 3.4.Calculate the moment of inert ia about the y-axis of the
4ax and the straight line x::figure bounded by the parabola y2
==
Solution.We have dlx
dS, where dS is the area of a verticaJ
strip situated at a distance X from the y-axis (Fig. 110):
dS=-:; 21yldx =-= 2 V' 4ax dx.)))
in

the

x\037

first

-=--:

x\"J

a.

Ch.

374)

Hence,)

II. Applications

Ix = 5

of the Definitp

4x2

Integral)

Vaxdx = 4 Va S xT dx =

a4
\037

In designingwooden girder bridges we often have


7.13.5.

logs flattened on two opposite

with

to deal

sides.Figure III shows the

cross-section
of such a log.Determinethe
moment of inertia of this cross-section
about the horizontal centre line.)

!I
v.\037\037

\037q
\037)

x=a)

:c)

a)

o)

x)

Fig. 110)

Fig. III)

Solution. Arrange the system of coordinates as

is shown

in

the

accompanyingdrawing. Then

dlx = y2 dS, where dS==MN dy == 2xdy ==2 V R2-y2 dye

Whence)
h

Ix=2-h y2VR2_y dy=4 0 y2VR2_.tldy.


\037

Substituting
\\V\037

('et
\"\"

1x = 4

5 y2

VR2

= R sin t;

arc sin (h/ R)

dy ==R
arc sin (h/

y2 dy

()

=4R4

\037

=4

cost dt;

R)

R2 sin2

5
0

t1

= 0;

t 2 ==arc sin (h/R),

t.R castR cast dt =

arc sin (hi R)

sin2tcos2tdt= 5
0

(1-cos4t)dt=

R2) V

\0374

= R4 arc sin Rh + Rh (2h2


When

h == R,

about one of its

we obta in the moment of

diameters:Ix =

.)))

:rr\0374

inert

R2-h2.

ia of the

circle

\037

7.13.Computing

7.13.6.
Find

Static

Moments and Moments

the moment of inertia about the

of

Inertia

x-axisof

375)

the figure

bounded by two parabolas with dimensionsindicated in Fig.112.


Solution.Arrange the system of coordinatesas shown in Fig. 112
and write the equations of the para-

bolas.

The equation of the left parabola is:


the equation of the
x+
y2 =

).
parabola. =

\037:

right

\037

y2

dlx ==y2 dS

MN = x2
I

===

y2

=2

Xl

\037

x) ,

:c

strip the moment

For the hatched


of inert ia is
where

\037:

=a-

:\037

Hence,)
W2

Ix = S y2 (a
-W2

\037

M N dy,

\037\037

y2

=
Fig. 112)

y2,

\037\037

\0372

y2

dy = 2

7.13.7.
7.13.8.

y2 a
S
(
0

\037\037

y2

) dy

=
a:\037

Find the static moments about the x- and y-axis of the


arc of the parabola y2 ==2x between x = and x == 2 (y > 0).
Find the static moments about the axesof coordinates
of the line segment + t = 1 whose end-points lie on the coordinate axes.
Find the static moment about the x-axisof the arc of
j'[
the curve y ==cosx between Xl =
and x2 = 11:
\302\260

\037

7.13.9.

7.13.10.
Find
bounded by the

7.13.11.
Find

-\"2

the static monlent about the

lines y = x 2;

2.

x-axisof

the

figure

==V\"X.

the moments of inertia about the x- and y-axis of


the triangle bounded by the lines x ==0, y == 0 and + t = 1 (a>0,

\037

> 0).

its

Find
7.13.12.

base AD

if

the moment of inertia of the trapezoid ABCDabout


AD ==a, BC== b and the altitude of the trapezoid

is equaI to h.
Find the centre of gravity of the sen1icircle
x2 + =-=a2
7.13.13.
situated above the x-axis.
Solution.Sincethe arc of the sen1icircle
is symmetrical about the
the
centre
the y-axis, i.e.Xc ==
of
of
the
arc
on
lies
y-axis,
gravity
V\"!.

O.)))

376)

Ch.

..

II. Applications

of the Definite

Integral)

- -.

To find the ordinate Yc' take advantage of the result of Problem


2aJ = 2a
2a
2. th eref ore
Th us, Xc ==0 , Yc ==
IVJ X -== 2
7 t 3 t .)lff
Yc
na
n
n

a,

Find
7.13.14.

-.

the coordinatesof the centre of gravity of the ca-

= + (eX + e- ) = coshx between A (0, 1) and B (a, cosha).


y
X

tenary

Solution.We have
dl == V-I

we

\\\\'hence

+ y'2 dx == V I + sinh 2 x dx ==coshxdx

find)
a

I=
\037

dl = coshx dx = sinh a.
\037

Then)

= X dl =
My
\037

Hence,)
Xc

Ana

0)

X coshX dx = X sinh X
\037

I\037

sinh

\037

dx =

==a sinh

--

a-I)_ a _ cosh. a- _ a
_ asinh a-(cosh
.

logously

sIn h

a)

t an h

\037

,)

Xdx=
Mx=Sydl=Scosh
2

sIn h

a-cosha + 1.)

\037

S(l+cosh2x)dx=
0)

=2
\037

a
\"2
Yc

7.t 3.15.Find
x ==a (t sin t),

sinh

===

sinh

4
a

2a

sinh
= sinh 2a .
+
2 ) l0 2 + 4
(
2X

a
2 sinh a

\037

')

+ cosha
2')

the centre of gravity of the first arc of the cycloid:


cast) (0 t 2n).
y ==a
Solution. The first arc of the cycloid is symmetrical about the
straight line x = na, therefore the centre of gravity of the arc of
the cycloid lies on this straight line and Xc ==na. Sincethe length
of the first arc of the cycloid l ==8a, we have)

- S --Yc-,
I

dl

(1-

\037

\037

2n
'
a C .3t
4
t
SIn
dt _
1 -cast) Sll1\"2dt
(
-3\"a.
2
-2J
S
2n

Sa

2a 2

222
0)

7.t\037.t6. Determine the coordinatesof


portion of the arc of the astroid x
quadrant.)))

\037

the centreof gravity of the

+y = a
\037

\037

situated in the

first

7.13.Computing

\037

Static

Moments and Moments

of

Inertia

377)

7.13.17.

Find the Cartesian coordinatesof the centreof gravity of


between cp==Oand cp==Jt.
the arc of the cardioid p==a(l+coscp)
Solution. Let us represent the equation of the cardioid in parametric form:)
x ==p cos ==a (1+ coscp) coscp;
==a (1+ coscp) sin
n
y ==psi
cp

cp.)

cp

As the parameter varies between0 and Jt the running point describes


the upper portion of the curve.Since the length of the entire cardioid equals 8a and
dl V (x' + (y' dcp == 2a cas dcp (see Problem 7.9.3),we have)
cp

===

\037)2

\037)2

\037)

JT

=T
Xc
1

\037

Y dl

= 4aI r a S
J

11

qJ

(l+ casqJ) 2a cas

dcr

\037

0)

.
2 SIn\"2d

1t

==2a () cos4 cp

(j)

<p

==

4
a cos5 \"2
5

\"'(

cp

0 ==\"5

a.

o)

Analogously,)
j{

YC=4a

C xdl =

j
L

4a 5

acosqJ(1+cosqJ)2acos
dqJ =
\037

0)

\037

= a cosqJ cos3
S

\037

dqJ = a

Putting

===

5 ( 2cos
5

\037

('OS3
\037

dqJ.

0)

we get (see Problem 6.6.9))

\037

= 2a
Yc

5 (2 cosot -cost) dt
3

= 4Ct

::\037

-2a

= a.
\037

\037

o)

And

4a
so,xc==Yc==5'

It is interesting to note that the centre of gravity of the above.


considered half of the arc of the cardioid lies on the bisector of
the first coordinate angle, though the arc itself is not symrrletrical

about this

bisector.

7.t 3.18.
Find the centre of gravity of the
ellipse4x2 + 9y2 ==36 and the circlex2 + y2
first quadrant (Fig. 113).)))

===

figure

bounded by the

9 and situated

in

the

Ck. V II. Applications

378)

Solution.Let

us

calculatethe static

first

'.

of the Definite

Integral)

moments:)

,.

My=j X(Y2-Yl)dx=
j x lV9-X2- V9-x] dx=
2

\037

0)

=+JX V9-x dx=3;


2

Mx = +

-;(9-

dx =

(Y\037-yD

\037

j\037

X2

[(9-X2)

.\\'
0)

o)

)]

dx =

= ;J

3)
2)

40<

,,\037

2'
_
_
S -4-2-4')
- - _-S-_
c-S-n')
x)

9n

Fig. 113)
Thus,)

\037

The area of a quarter of a circleof


91t
radius 3 is equal to 4 , and the area of a
a ==3
quarter of an ellipsewith semi-axes
3n
therefore the area
and b == 2 equals
of the figure under consideration is

.?

\037\037Jo)

(5- x )dx=5.

My

3n

3n

\302\267

Yc

20

Mx

3n

\302\267)

7.1 t 9. Find the centre of gravity of the figure bounded by


parabola x2 + Y = a2\" and the axesof coordinates.
\037.

the

2\"

7.t 3.20.Find

the

Cartesian coordinatesof the centre of gravity


3

figure enclosedby
Solution.Since p 0

of the

>

the curve p ==a cos (a 0).


all cases, the given curve is traced
to
By virtue of evennessof the funcp

in

\037

.
changes from
ction cos it is symmetricalabout the polar axisand passesthrough
when

cp

\037

\037

;.

cp

the origin of coordinatesat == +


Compute the area S of the figure obtained:)
cp

:rr

1[
2

S == 2 X 2\" S p2 dcp = a 2 cos6


S
o
1

0)))

cp

dcp == a

2I

x3x5

2x 4 X 6 X

2==32Jta .
n

\037

7.13.Computing

Static Moments

379)

and Moments of Inertia

Now arrange the axes of coordinatesas shown


the parametric equations of the curve are)

in

Then
Fig. 114.

x==:p cas ==a cos4 <p;


3
p sin ==a sin cos
y
<p

===

<p

<p

<p.)

i.e.

== 0
The centre of gravity of the figure lies on the x-axis,
Yc
virtue
about
the
determine
x-axis.
of
by
Finally,
symmetry
xc:)

Xc===

2\037XYdX;
8a3
o
1
S

=8

\037

S cos

\302\260<psin

Ba3

<pd<p=s
S

(cos1\302\260<p-COS

12

<p)d<p=

0)

8a3

x3x5x7x9 lX3X5X7X9Xli n = 21a.)


2 40

lOX 12)
-(5/32)na2 ( 2x4x6x8xl0) 2x4x6x8x

!I
1
0

0
Fig.

1[;

114

1[,

Fig. 115)

7.13.21.
Find

the coordinatesof the centre of gravity of the


bounded by the straight line y = x and the sinusoid
figure
==
y sin x (x 0) (Fig. 115).
Solution.The straight line y = Y:...
x and the sine line y == sin x intern
sect at the points (0,0) and
The area of the figure
bounded by these lines is)
\037

\037

(-i-,1).

Jt

.
2
n
S= S ( slnx-nx) dx= 44
2

o)))

\302\267

380)

Ch. V

II. A pplications

x2

of the Definite

Integral)

Hence,)
J[
2
\037

Xe =

50 ( sin2 x
4

n2
\037

J[

dx

- 5(
2

===

0)

sin2 x -\037 x2
1[2 )

dx =
J[)

4- [2 x2

===

\037

4)

3n 2

n
2'

Yc==

5o x ( sin

x-

\037

4-n

dx

:.2

sin 2x

x3

==

1[

o)

(4-n) ,

.
4-1[5 XSlnxdx\037

===

0)

- (4-1[

3t

)5

rc

.J
x.-dx

===

4-

3'(2

3 (4-n)

===

12

'
12-3n

Prove the following theorems (Guldin'stheorems).


7.13.22.
Theorem 1.The area of a surface obtained by revolving an

1[2

arc

of a plane curve about some axis lying in the plane of the curve and
not intersectingit is equal to the product of the length of the curve
by the circumference of the circledescribedby the centre of gravity
of the arc of the curve.
Theorem 2. The volume of a solid obtained by revolving a plane
figure about some axis lying in the plane of the figure and not
intersectingit is equal to the product of the areaof this figure by
the circumference of the circledescribedby the centre of gravity of
the

figure.

Proof. (1) Compare the formula for the area of the surface of
revolution of the curve L about the x-axis(see 7.10)
\037

P = 2n
\037

y dl

with

that for the

ordinate of the centre of gravity of this curve


Yc

Hencewe concludethat

=
\037x

=+ 5 ydl.
L

P = 2Jt lYe = l.2'JtYe'


arc, and

where 1 is the length of the revolving


of a

circleof radius Ye'

the centre of gravity

2JtYe is the length


length of the circledescribedby
revolving about the x-axis.)))

i.e.the

when

\037

7.13.Computing

Static

381)

Moments and Moments of Inertia

(2) Compare the formula for the volume of a solid generated


by revolving a plane figure about the x-axis(see 7.6)
\037

:rt
\037

dx

(y\037-yD

a)

with

that for the

ordinate of the centre of gravity of this

figure)

Yc

Hence

\\\\\"e

= 2'S (y\037-yD dx.


5a

\037;t

conclude that
V

==

n.2SYc S.2nyc
===

where S is the area of the revolving figure, and 2nyc is the length
of the circumference described by the centre of gravity when revolving about the x-axis.
Using the first Guldin theorem, find the centre of gra-

7.13.23.

vity

of a

of radius a.
semicircle

Solution.Arrange

virtue of symmetry

coordinateaxesas shown

the

Xc

== O. Now it

circlerevolves about the x-axis,then


the surface P of the solid of revo-

remains to

find

in

By
Fig. 116.

Yc'

If

the

semi-

lution is equal to 4J[a\037, and the


arc length l ==na. Therefore, according to the first Guldin theorem,

4na2 ==na.2Jtyc;

Yc

7.13.24.
Using the

-.

=2 a
3t)

second Gul-

-a

o)

a)

x)

din theorem, find the coordinates


Fig. 116)
of the centre of gravity of the
bounded by the x-axis and one arc of the cycloid:x =
figure
==a(t
cast).
t); y ==a
Solution.By virtue of the symmetry of the figure about the
straight line x = na its centre of gravity lies on this straight line;

(1-

-sin

hence,Xc ==na.

The volume V obtained by revolving


is equal to 5n2a 3 (see ProbleIT1
being equal to 3na2 (see Problem
theorem, we

this

about the
figure
S of the
the
area
7.6.14),

7.4.3).Using the

get)
Yc

x-axis
figure

secondGuldin

2a3
5a
V
= 23t.5n3na
= 23tS
2 = 6.

7.13.25.
An equilateral triangle

with

side a revolves about an

axisparallel to the base and situated a t a distanceb > a from the base.
Find the volume of the

solid of

revolution.)))

Ch. VII.

382)

Applications

of the Definite

Integral)

Solution.There are

two possible ways of arranging the triangle


respect to the axis of revolution which are shown in Fig.117,
a and b.
a
The altitude of the equilateral triangle is h = \0373 , the area
with

S=

a2

\0373

The centre of gravity

0' is situated at

intersecti-on of the medians and at a distance of


a
and b + \0373
axis of revolut ion in the first

cas:,

b-a
in

the point of
\0373

from the

the second.

!I)

If)
b)

b)

x)
A)

x)

o)

c)

c)

fa))

(IJ))

Fig.
By the

second Guldin theorem


V3
= 21ta:

a2
=
n
(b- ) ( 2b{3_ ) ,
a
V2 =
21ta:V3
(b+ a\0373) = n ( b{3+
VI

Find
7.13.26.
R

117

a\0373

\0373

\0373

subtending a central angle 2a.

Find
7.13.27.

the centre of gravity of the


the arc of the cosineline y = casx betweenx =
the straight line

).

the centre of gravity of the arc of a circleof radius

figure

bounded by

and x =

\037

and
\037

y=-}.

7.13.28.
Find the coordinatesof the centre of gravity of the
3
4
figure enclosedby line y2 = ax -x .
7.13.29.
Find the Cartesian coordinatesof the centre of gravity
from cp] = to
of the arc of the logarithmic spiral p =
;: ==n.
A regular hexagonwith
side a revolves about one of
7.13.30.
its sides.Find the volume of the solid of revolution thus generated.
7.13.31.
Using Guldin's theorem,find the centre of gravity of
ae\037

of radius
semicircle

R.)))

CP2

\037

\037

7.14.Additional

Problems)

383)

7.14.AdditionalProblems

7.14.1.

Find the area of the portion of the figure bounded by


the curves ym = xn and yn = xm (m and n positive integers)situated
in the first quadrant. Consider the area of the entire figure depending on whether the numbers m and n are even or
(a) Prove that the area of the curvilinear trapezoid
bounded by the x-axis,straight lines x = a, x = b and parabola
= Ax 3 BX2 Cx D can be computedusing Chebyshev'sformula
y
b a
b
S b3a
y
y
2
2
[y
)
)

odd.

7.14.2.
+

+ +

-;2

+ (a;b + (a;b+ ;2 a)].

(a;b

(b) Prove that an analogousarea for a parabola of the


y=f (x) = Ax 5 + Bx4 +Cx3 + Dx2 + Ex+F
can be computed using the Gauss formula

_v

S=b 9a[5f(atb

\037b

/ )+8f(a;b)+

b
+5f(at +

order

fifth

}/;

a)]

.)

7.14.3.
Show that the area of a figure bounded by any two radius vectors of the logarithmic spiral p = aem 1' and its arc is proportional to the difference of the squares of these radii.
7.14.4.
Prove that if two solids contained between parallel planes P and Q possess the property that on being cut by any plane
R parallel to these planes equivalent figures are obtained in
their section,then the volumes of these solids are equal (Cava..
lieri'sprinciple).

7.14.5.

Prove that if the function S(x) (O\037x\037h) expressing


the area of the sectionof a solid by a plane perpendicular to the
x-axisis a polynomial of a degreenot higher than three, then the
S
volume of this solid is equal to V
(0) 4S

\037

[S +

( ) + (h)]
\037

Using this formula, deduce formulas for computing the volume of


a sphere,spherical segments of two and one bases,cone,frustrum
and paraboloid of revolution.
of a cone,ellipsoid,
Prove that the volume of a solid generatedby revolving
7.14.6.
about the y-axis the figure a x b, 0 y y (x), where y (x) is
a single-valuedcontinuous function, is equal to
\037

\037

\037

xy

(x)dx.

V = 2n
\037
a)))

\037

Ch.

384)

II. Applications

of the Definite Integral)

7.14.7.
Prove that the volume of the solid formed by revolving,
about the polar aXIS, a figure O\037(X\037cp\037p\037Jt, O\037p\037p(cp),
is equal to)
B

= S p3 (cp) sin

\0371t

dcp.

cp

a)

7.14.8.
Prove

that the arc length of the curve given by the pa-

rametricequat ions
x =={\"
y ==
is

(t) cost + f' (t) sin t,)

f\"

(t 1

sin t + {'(t) cost

(t)

\037

\037

t2

))

equal to [f(t)+f\"(t)]\037\037.
Find the arc length of the curve represented parametri.14.9.

ca11 y)

- -

x-- S cos
I

dz,

z
== sin
t.,\\

dz

t)

1)

between the origin and the nearest point from the vert ical tangent

line.

Deduce the formula for the arc length in polar coor7.14.10.


dinates proceedingfrom the definition without passing over from
Cartesian coordinates to polar ones.

7.14.11.
Prove

== coshx

that

the arc length l (x) of the catenary

measured from the point (0, 1) is expressed


by the formula 1 (x) ==sinh x and find parametric equations of this line,
using the arc length as the parameter.
A flexible thread is suspended at the points A and B
7.14.12.
located at one and the same height. The distance between the
points is AB ==2b,the deflection of the thread is f. Assuming the suspended thread to be a parabola, show that the length of the thread
2

2t
+
3\"
)
(

l ==2b 1

b\037

at a sufficiently small

\037

Find
7.14.13.

the ratio of the area enclosedby the loop of the


curve y
V:X to the area of the circle,whose circum(
ference is equal in length to the contour of the curve.
Compute the length of the arc formed by the intersection
of the paraboliccylinder)

=+

i-x)

7.14.14.

(y+ Z)2 = 4ax)))

\037

and the

ellipticcone)

7.14.Additional
4
3 +
'}
x'\"

,)
Y\"

385)

Problems)

0)

Z'\"

==0 ,

between the orIgIn and the point M (x, y, z).


Prove that the area of the ellipse

7.14.15.
AX2+

is equal

2Bxy+Cy2 + 2Dx+2Ey + F ==0

to)

s=

JT\037

where

')

(AC

ABD
.
DEF)

-= B C E

\037

(AC\037B\037);Jj2

7.14.16.

-8 > 0)

Find: (a) the area S of the figure bounded by the hythe positive part of the x-axisand the radius
perbola X\"!._y2 =
vector connectingthe origin of coordinatesand the point M (x, y)
lying on t his hyperbola.
(b) The area of the circularsectorQ bounded by the x-axisand
the radius drawn fronl the centre to the point N (x, y) lying on
the circlex2 + y'J ==
Prove that the coordinatesof the points M
and N are expressed
respectively through the areas Sand Q by
the formulas

1,

1.

XM

2S,

==cosh

YA1

==sinh

2S, xN ==cos2Q,y N ==sin 2Q.

7.14.17.
Using Guldin's theorem,

prove that the centre of graangle is one third of t he a 1 t itude distant from its base.
Let be the abscissa of the centre of gravity of a curvilinear trapezoid bounded by the continuous curve y ==f (x), the
x-axisand the straight I ines x a and x b. Prove t he val idit y
of the following equality:)
vi t y of a tri

7.14.18.

\302\243

\037

=--=

\037

(ax + b) l(x)dx = (as + b)

\037

(x)dx

a)

(Vereshchagin'srule).

7.14.19.

Let a curvil inear sectorbe bounded by two radi us vectors and a continuous curve p ==f ((p). Prove that the coordinates
of the centreof gravity of this sectorare expressed
by the following

fornlu

las:)

(P2

<J)2

p3

CDS

p3 sin !p dip

!p dip

\037

x ==
C

\037

(f)1

Yc

<J)2

p\037

== 2

3\"

\037

<[) I

<f2

dip

p\037dcp

\037

\037

<PI)

(f)1)))

Ch. VII.

386)

Applications

of the Definite

Integral)

7.14.20.

Prove that the Cartesian coordinatesof the centreof


by the folgravity of an arc of the curve p ==f (<p) are expressed
lowing formulas:)
fP2

fP2

r cos'P V p2
\037

x ==
C)

+p'2

p sin<p

drp

Yc

<[J2

V p2
\037

CPt)))

Vp2+p'2d(jl

\037

fPt

==)

<PI)
<P2

V P2

p'2d(jl)
\037

([>1)

+p'2

d(jl

Chapter8)
INTEGRALS)

IMPROPER

8.1.ImproperIntegralswith InfiniteLimits

\037

Let the

interval

any

ion f (x)

funet

be defined for all x

Then !im

[a, A].

by

the symbol

\037

f (x)dx is

interval

f (x)dx. We

called the improper


[a, + 00] and is de-

similarly define the integrals

a)

+00

f (x) dx a nd

f (x) dx.

-00

-00
\037

A_+oo a
(x) in the
\037

integral of the function+00 f


noted

a and integrab Ie on

\037

\037

Thus,)
+00

f (x) dx =

\037

f (x)dx= !im

-00

f (X) dx = !im

A--oo A

-00

A--.-ooA

+00
\037

f (x) dx;

\037

B
\037

!im

A-+oo a

\037

\037

(X) dx;

!illl\\ f (X) dx.


(x)dx+ B-+(7;
C)

If the above limits exist and are finite, the appropriate integrals are calledconvergent; otherwise, they are calleddivergent.
Comparison test. Let f (x) and g (x) be defined for all x a and

integrable on each interval

all

x a,
\037

then from

[a, A],

convergenceof

\037

a. If

\037

\037

f (x)
('

\037

00

the integral

\037

g (x)

g (x)dx

it

for

fol-

a
00

00

lows

that

the integral

\037

f(x)dx

is

also convergent,and

\037
a)))

f (x) dx

\037

Ch.

388)

III.Improper

Integrals)
00

\037

\037
\037

g (X) dx; from divergenceof the integral

\037

f (x) dx

follows that

it

00

the integral

a
\037

g (x) dx

is

also divergent.

as x---+00the funct ion f (x) 0 is an


infinitesimal of order A > 0 as compared with \037,
then the integral
x

Specialcomparison test.

If

\037

+\037

dx
for
S f (x) converges

> 1 and

A.

diverges for

A.\037

Absolute and conditional convergence. Let the

1.
ion f (x) be

funct

00

defined for all

\037

a. If

the integral

\037

f (x)! dx converges.then the

00

integral
t his

In

\037

f (x) dx

also converges and is calledabsolutely convergent.

case)
00

\037

\037

f (x) dx

\037

\037

f (x) I dx.

00

If the integral

\037

00

f (x) dx converges,and

\037

f (x)

dx diverges. then

if:;

the integral

\037

f (x) dx is

calledconditionally

convergent.

The changeof the variable in an improper integral is based on


the following theorem.
Theorem.Let the function f (x) be defined and continuous for
x
If the function x == (t), defined on the interval a < t <
and (0), is monotoand
may also be improper numbers
(a
nic, has a continuous derivative cp'(t) =1= 0 and lirn
(t) == a,

a.

\037

-00

cp

\037

\037

cp

lirn
t ....{i

(t) ==

cp

t-..ex+O)

+ 00,then

0)

00

Ii

f (x) dx =

\037

\037

f (q>

qJ' (t)

(t\302\273

dt.

ex

Integrationby parts involves no difficulties.

8.t.t.

Evaluate the following improper integrals with infinite


divergencetaking advantage of their defini-

limits or prove their

tion.)

Jj

(a)

2)

00

00

Xl\037\037X

dx
2x

(b) 5 x 2 +
-00)

+5 ' (c) 5 x sin x dx.


o)))

\037

Solution.(a)
\037

t.

e
\037

dx
xln 3 x

5 xln 3. x
e
dx

I 1m
A\037+oo

==

I In1

A\037+\037

with

Integrals

By definition,)

00

8.1.Improper

Infinite

21n2 x e

=
1

2 ) =8')
( g-21n
A

=A\037\037\037

(b) By

389)

Limits)

definition,)
0

00

= I' x2+ dx2x+5+ r


x 2 +2x+5
j
5
-00
B
dx

B:\037oo

dx

5 x2+2x+5

A\037\037oo

0)

(instead of the point x ==0 any other finite point of the x-axismay
be taken as an intermediate limit of integration).
Compute each of the limits standing in the right side of the
above equality:
o

.
hm

dx

B\037-oo

\0371\037

00

r
J
o

x2

x+ 0
Jt
2arctan\037B =2arctan\"2+T'

dx
=
+2x+5 A

B\037-oo

:1\037

.
A

== 11m

5B x2+2x+5

00

\"2

I
+I A=
arctanx\037
4-2\"arc tan\"2'
11

Hence,)
00

dx

x2-\0372x+5

j'[

='2')

-\037)

(c) By

definit

ion,)
A

00

x sin x dx =
\037

\037

A\037+ooo)

u==x, dv==sinxdxand integrating by parts, we

Putting
A

HIn

x sin xdx.

lim

xsinxdx== Hrn
A\037+\037

A\037+\037\037

-xcosx0 + r COSXdX =
A

\037

lim
A\037+oo)

But

the last limit

00

\037
o)))

get)

x sin xdx diverges.

(-AcosA+sinA).

does not exist. Consequently, the integral

Ch.

390)

III.Improper

Integrals)

8.1.2.Evaluate the following improper integrals


limits on the basis of their definition:)
00

00

xdx

(a) 5 y(x2

3)3

(b)

(c)

5 X\037\\3

2)

x dx

dx

(e) 5 x2-::+
10 ;
-00

1)

00

00

(d) 5 x t (l +x) ;

Y!4x2 + 1)3

o)

I)

00

infinite

00
\037

with

5 e- sin xdx.
X

(f)

0)

Solution.(a) By definit ion


A

\037

xdx

52

V(X\037-3)3

Hm
A

-+

00

xdx
Y(x 2

(x2_3)-1/2A =
= Hin
-3)3 A + [ 2 -1/2 2 ]
\037

-+

00

8.1.3.Prove that

=-Iim
-I =I.
_
[ ylA2-3 J

2)

the integrals of the


+00

00

form)

e-PX dx and ePX dx


-00)
constant p > 0 and diverge for p < O.
\037

\037

converge for any


Test the integral)

8.1.4.

00

dx

S I +2x2 +3x4
o)

convergence.
Solution.The integrand

for

f (x)=

+2x2+3x4
is positive and is an infinitesimal of order A = 4 as comparedwith
\037 00.Since 4 > 1,the
integral convergesaccording to the
x as x
I comparison test.
specia
I

\037

8.1.5.Test

the integral)

00

dx

5 x+ sin 2 x
1)

convergence.
Solution. The inte g rand
tive for x\037l.
for

As x --+ 00

f (x ) == x+sin 2 x is cont inuous and POSt-

the function f (x) is an


as compared with
accordingto the

x;

integral diverges.)))

nitesimal of order A = 1
specialcomparisontest the
infi

8./.Improper

\037

Test the
8.1.6.
In

(a)

I)
(x:+ dx; (b)

\037:n

\037

391)

Limits)

;x dx;
r

(e) J

J2 l+xVx

Vx

Test
8.1.7.

Infinite

3+arcsin +
r
dx;
(d)

r
x
(c) 2+cosdx;
1

with

following integrals for convergence:

\037

Integrals

'

the integral

arc t an x
x)

x.

<JJ

(x+V\037+])dx
x2

\\

to-

+2 V x4 +

1)

for

convergence.
Solution. The integrand

1.

is continuous and positive for x


Determine its order of smallnessA with respect to J-. as x ---+00;
since)

+ Vx+
x 2 + 2 V x4 +
x

\037

=-x
I

the order of smallness A ==

X)

1.According to

the

00

test the integral r X+


t

x4 + I
J x +2\037X+f

I'

1+-..V J..+\037
xt
.
1+2V 6\"\"\"+10
x
x

specialcomparison

dx diverges.

8.1.8.
Test the integral
00

S YX(X_d\037(X_2)
3)

for

convergence.
Solution.Sincethe function
f (x) =

VX (

1- ) (1\037

=-x
1

))

2)

V(I- )(I\037

))

is an infinitesimal of order 'A = with respect to


as x -> + 00.
accordingto the specialcomparisontest the integral converges.
\037

Test the
8.1.9.

\037

integral)
00

)
2)

for convergence.)))

+2xt

3
V x3

\037/

-1dx

Ch. V

392)

III.Improper

Integrals)

Solution. The integrand is continuous and positive for


Determine its order of smallness with
/

2x.a

as x ---++ 00:

x2
I

x 35

11

x3)

{/2as x ---.+ 00,we

the limit

= 35< 1.Consequently,the

Test
8.1.10.

2.

\302\267

\037

Sincethe second multiplier has


have

x)

\037

V +2=-x
3--1
1--

;/3+
f1

respect to

given integral diverges.

the integral)
00

5 ( I-cas ) dx
1)

or convergence.
Solution.The integrand

f(x)= I-cos\037=2sin2\037
x
x

=x ,
is positive and continuous for x 1.Since2 sin2 x \"'-12 ..!..
( x ) \037
the given integral converges(by the specialcomparisontest).
2

\037

\037

Test
8.1.11.

the integral
00

J In

eX+(n-I)

dx.

n)

>0

convergence.
Solution.Transform the integrand:

for

f{x)=ln 3-+\037n-l)=In[I+
1

Since the function


1

f (x)\"\",

the

eX

\"'\"

eX

-.
I

nx

-1 is

other words,

Test
8.1.12.

t he

the integral)
00

S
1)

for convergence.)))

Hm
X-+OO

specialcomparisontest

].

'
I

(X)

/x

=n

\037.

x-+oo,)then)
According to

given integral diverges.)

4 sin

X3

an infinitesimal as)

n)

In

3--l

2x

+ V -;

dx

8.1.Improper

\037

the change

x
+ P=-x

changesits sign together

sign of the numerator. Test the

in

11-4 -2x

00

f
.]

x3

+v

dX

+V

00

intrgral)

00

sin
,

11-4sin 2x- <


x3
x

convergence. Since

for

x3

393)

Limits)

Infinite

1-4

Solution.The function f (x)=


with

with

Integrals

4 sin

\037
X3

, and

the

inte gral

dx convergesas well (acx converges,the mtegral S x 3 +


SI -a
x
cordingto the comparison test). Thus, the given integral converges
5dx

2x

v-

absolutely.

8.t.13.Prove

that the

Dirichlet

integral)

00

sin

I=

tJ\\

dx

o)

convergesconditionally.
Solution.Let us represent the given integral as the sum of two
integra Is:)

J()

00

1=

r Sinx
J x
o

00

dx= S
0

dx.
dx+ S Sinx
x

sinx
x

n
2

= 1 . Applying the
lim
proper integral since x0 x
)
(
method of integration by parts to the second integral,we have
The

\302\253>

\037

t.
11

sinx

is a

first

- .

SIn x
X

dx = 1 1m

A .
n SIn x

dx =
A-oo :rtj x

'\"2

\"2)

==

!im

A_oo

x2
-_ j COSXdx
A

COSX

Jt

oo

\037

:rt

2\"'

l.J

==_

x2

cO\037X

dx.

1t

'2)

00

COS

the improper integral \\


J

But

x2

dx

converges absolutely, since

2)

I cosx

x2

00
I

\037
\037

\037
X2

r
, and the inte gral J
:rt
2)))

dX

x2

converges.

Ch.

394)

Ill.Improper
00

sin-x

5o -Xdx converges.

Therefore, the Integral


00

Reasoning in a similar way

r cosx

IS easy to prove

it

dx a Iso converges.N ow 1et us prove

Integrals)

the integral

that

t he

t h at

in t egraI

:rr

2
oc

xI

Si\037

dx di verges. Indeed,

1t
2)

sin x

\037
\037

cos2x

2x)

the integral)

but

5
oc

x-- . - 5 ---5
n:rr
A

cos2xd

sin 2 x _
x

2x

1m
-+

00

00

dx
x

x--

cos2x d
x

2\"\"2)
=-2

I.

--In---5
00

hm In A

A-+oo

cos2x
x

dx

J1
\037)

00

diverges,since

lim In A

= 00,and the

000
A-+<X:'

8.1.14.
Prove

that the following

(a) sin (x2) dx;


\037

\037

dx

converges.

00

cas(x2) dx; (b)

Solution.(a) Putting

x ==

2x cas(x4) dx.

\037

Vi, we

find)
00

00

sin (x

)dx=-}J
0

Let us represent the integral on the


integra Is:)
J1.
2

rr:

dt
\037\037

dt.

So

cos2x

5
1[

integrals converge)

00

OC

integral

J0

\037n/

side as the

right

sunl

of two

00

dt
\037\037

+S

dt.
\037\037

:rr)

2)

=.

sin t
The rst summan d is a proper In t egra 1 ,since 1 1m -==0 Let
fi

t-++O

t)))

\037

8.1.Improper

with

Integrals

395)

Limits)

Infinite

apply to the second summand the method of integration by


parts, put t ing)
u == l/Vt, sin t dt ==dv,)
-\037 costdt =_\037 cost dt.
si\037 dt==us

00

00

00

00

CO\037

tf

1(/2

13t/

t 3/ 2

1(/2

t 3/2

'It/2

t
2

'

and)

can prove analogously that

the

The last integral converges absolutely, since

\037\037;

\037
t3\0372

00

the integral

I ::/2 converges.We

1t
2)

00

cos(x2) dx is convergent.The integra Is considered areca1Fresnel'sintegrals.They are used in explainingthe phenonle-

integra 1
led

\037

diffraction.
the
substitution x 2 == t this integral is reduced to the
(b) By

non of

light
00

integral

\037

cas(t2) dt. The

latter integral convergesas has

just

been

proved.
Note. Fresnel'sintegrals show that an improper integral can converge even when the integrand does not vanish as x \037 00.The last
convergent integral consideredin item (b) shows that an improper
integral can converge even if the integrand is not bounded.Indeed,
the integrand attains the values
at x == V nn (n == 0, 1,2,
+ V nJt , i.e.it is unbounded.
8.1.t 5. Evaluate the improper integral)

...)

00

dx

n natural

S
(l +x2)n'
o

number.

Solution.Make the substit ut ion x == tan t, where 0

x= 0

at

= 0, x \037 + 00 as t --+ 3t2

-0and

\037

xi = cos2 t

quently, by the theorem on changing a variable


integra
1t

in

< . Then

O. Conse-

an improper

1)

1t)

00

dX

I
2n

(l +x2)n S sec
S
o
0

sec2 t dt = 5 COS2n -2 t dt.


o

On changing the variable we obtain the proper integral)


in Problem 6.6.9.)

was t'onlputed

which)))

Ch.

396)

III.Improper

Therefore,)

==

:rt

== f

5o (I +x\037)n

12,n.. 1,
.3.5.
(2n 3)
2.4.6..
.(2n-2)2\"' n > 1.)

00

dx

Integrals)

\\

j'[

\302\267

00

8.t.t 6. Compute the

integral

1= 5 +x x4 dx.)
o
1

-(1It

Solution.Apply the substitution


x=

lit;

dx ==

t 2 == 0;)

t 1 ==00,)

) dt;)

00

00

() dt

('\302\243

4) dt
1=5 l+x4 dx= (I/t
4 = f4+I '
1+1//
J
o
0
'\"'

X2

\037

0)

If another integral

added to the

is

get)

00

and

right

00

=f

5 Z2\0372
-00

-lit,
-++ 00,

o
ion z == t

Make the substitut


Z ---+ 00 and as

-++ 0,

=f

Bl\037\037\",,\037

t\037+

(1+ Ilt2) dt

z \037

Z2d\0372

o)

Z2d\0372

==

. arc an
..r-+ 2 1rr 2 -.+00
1

I 1m

Then, as

2;2( ;+

-=

\037

)=2;2.
\037

the following improper integrals:)


00

00

50

}\037\037'\"

B
1
I 1m arc t an
1/\"2
2r
B-.-oo
r 2

Evaluate
8.1.17.

==dz.

+ 00.Hence,

+
=

(a ) C

\\\\\037e

+t 24 dt = 1//2+ 2 dt .
21== 5 l+t
5 1!t
I

sides then

left

(b) 5 e-x'x2m +1dx.

In x
1

+x2 dx;

o)

8.1.18.
Compute the

integral)
oc:
\"

V x3

I ==j

xC>

+_

-x2 2-t-l dx
x

+]

1)

accurateto two decimal places.


Solution.Represent the given integral
two integra Is

11

\037

+
+ +

3
r Vx
J
xC>

1)

x\037

x\037

in

the form of a sum

00

dx,

I2 ==j
N)))

+
+x2 + dx.

V X 3 _X2
xl)

of)

\037

8.2.Improper

397)

of Unbounded Functions)

Integrals

Compute the former with the required accuracy, using Simpson's


1 we have
formula, and estimateth e latter. Since for x

..rx3 -x2 +

\037

X3/2

0< x +x2+ <


\"

then)

00

0<12 = 5 X- /2 dx=
7

At

we get the estimate/2


Computation of the integral
N == 7

===X- 7 / 2 ,)

< 52 X

N-&/2.

49

r
7

< 0.0031.

/1==.
\\

YX3_X2+1 dx
x5

for a

Simpson'sformula

by
.

and for a step

h
\"2

+x2 +

step

===

gives

81== 0.2155,)

== 0.5)

80.5 0.2079.
===

Sincethe difference between the values is 0.0076,


the integral 11
0.2079with an errorof the order
gives a more accuratevalue SO.5==
O.

\"'-'

\03776

0.0005.

Consequently,the sought-for integral is approximately equal to

I 0.208
with
0.004,or 1==0.21
exceeding
\037

with

mal
\037

an

errornot

places.)

all true

deci-

8.2.ImproperIntegralsof [JnboundedFunctions

If the function f (x) is defined for a x < b, integrable on any


0<8<
and unbounded to the left of the
interval [0,b
point b, then, by definition, we put

b-a

-8],

\037

\037

b-\302\243

f (x)dx =!im
c\037+o

f (x) dx.

\037

If this limit is existent and finite, then (he improper integral is


said to be convergent. Otherwise it is calleddivergent.
Analogously, if the function f (x) is unbounded to the right from
the point

a,

then)
b

\037

f (x)dx =!im

B--t0 a +

f (x)dx.

\037
B)))

Ch. V

398)

III. Improper

Integrals)

if the function is unbounded in the neighbourhood of an


interior point c of the interval [a, b], then, by definition,)

Finally,

h
\037

f (x)dx =

f (x)dx

\037

\037

f (x)dx.

r)

Let the funct ion f (x) be cont inuous on the interva I [a, b] except
at a finite number of points. If there existsa funct ion F (x) continuous on [a, b] for which
(x) f (x) exceptat a finite number
of points, then the Newton-Leibnizformula

F' ::::::

\037

f (x)dx=F

(b)-F(a)

a)

holds good.

Sometimesthe function F (x) is calleda generalized


antiderivative
for the function f (x) on the interval [a, b].
For the functions defined and positive on the interval a x b
convergencetests (comparison tests) analogous to the comparison
tests for improper integrals with infinite limits are valid.
Comparisontest. Let the functions f (x) and g (x) be defined on
the interval a \037x < b and integrable on each interval [a,
If O<:f(x)
<:g(x),then from the convergence of the

<

\037

b-B],

O<B<b-a.
b

integral

\037

g (x)dx follows the convergenceof

the integral

\037

f (x)dx,

/}

and

\037

f (x) dx

\037
\037

a)

g (x) dx;

from

the divergence of the

integral)

a)

S f (x) dx follows the divergenceof the integral a g (x)dx.


Specialcomparisontest. If the function f (x) 0 is defined and
continuous on the interval a <:x < b and is an infinitely
large
as compared with b\037X as x-+-b-O,then
quantity of the order')..
\037

\037

the integral 5 f (x)dx convergesfor ')..< I and diverges for ')..


a
In particular,the integral)

\037

1.

dx

5 (b-x)'\\
a)

1.

'A < 1 and


diverges for 'A
Absolute and conditional convergence. Let the function f (x) be
defined on the interval a x < b and integrable on each interval)))

convergesfor

\037

\037

\037

8.2.Improper

of Unbounded Functions)

Integrals

399)
I)

[a,b-8];

the convergenceof the integral

then from

If (x)I dx)

\037

a)
b

follows the convergence of the integral

\037

(x)dx.

a)
b

casethe

In this

integral

f (x)dx

a
\037

is calledabsolutely convergent,

But

the integra I

if

\037

f (x) dx converges,and the integra I

\037

f (x) dx
I

Q
b

diverges, then the integral

\037

f(x)dxis calledconditionally convergent.


b

Analogous tests are also valid for improper integrals


where f (x) is unbounded to the

from the

right

8.2.1.

point

a.

\037

f (x)dx,)

a)

Proceedingfrom the definition, evaluate the following


improper integrals (or prove their divergence):
(a)

1t

e
r

2
dx

dx

(c) .

V 4x x 2

\\

l
X3

dx
r
(d) J V , I

x2 I

+ VX-2dx;

l-x3'
dx

(f)

Vxa

dx
CDS

\037

--

10

(e) .f

(b)

x inX
,) V
3

50

Solution, (a) The integrand f (x)=

1.

vlnx

is unbounded

in

the

neighbourhood of the point x == It is integrable on any interval


+ E, e],since it is a continuous function.
Therefore)

[1
e

SI V
x

dx
3 In

--

dx

== I 1m

e +0
\037

S
I +e

tim
e\037+O

Vln
[
_

\037

.
= 1m

[ V

3
e +0 2
1

\037

\037

In

te

= '
Vln2(1+8)
J

\037is unbounded
(b) The integrand f (>c)== cos
x

of the point x =

.e

2x

the neighbourhood
and integrable on any interval
[0, 8J
in

\037

as)))

Ch.

400)

Ill.Improper

Integrals)

a continuous function. Therefore)

1[
--E;:
2

:rr

So

\037==
lim
x

\037=

\\

e-++O J cosx

COS

0)

Iim In tan

+ )

\037

== Iim In tan

-\342\202\254

\037

P,-++O)

323
\342\202\254-++O

\037

-; - 00.
)

Hence,the given integral diverges.


(c) The integrand is unboundedin the neighbourhoodof the points
x == 1 and x == 3. Therefore, by definition,)

dx

dx

St V4x-x:!-3St Y4x-x2
(instead of the
the interva I
2
dx

\\
t.I

\302\243-++0

=
.S Y4x-x2-3
2
dx

dx

== I 1m

VI-(x-2)2

e)

= ;:;
[O-arcsin (B-1)]

\342\202\254-++o)

'

3-f

I 1In

.
hm
=
(x-2)\037

dx

\\

\302\243-++O\037,

. (x-2) 2 ==
l+f

arc Sin

\302\243-++0

1-

\342\202\254-++O

2)

lirn
\342\202\254-++o

.Hence,)

2)

== lim

point x == 2 we can take any other interior point of


us now compute each summand separately:

[1,3)). Let
. 2'
== 1m

.S Y4x-x -3
1
2

dx

-3+ S Y4x-x -3

3
dx

j'[

. (x-2)3-1:
=
.
[arcsin( 1-8) 0]
arc SIn

- =2-)
2)

n.

j'[

S Y4X\037-X2_3 =2'+2=
t)

is unbounded in the neigh(d) The integrand f (x)= V\037-2


-x
II
'bourhood of the point x == 1, which is an interior point of the
1

int\037rval

of

integration.Therefore,

definition,)

by

-S JI/I-x +S YIIS Yll- 12

_dx

dx

dx

x2 1

x2

I)))

\302\267

8.2.Improper

\037

Evaluate each summand separately.


1
1-1;;
dx

5o Y

!I-x
2

dx

r
J

Y I_x

dx

YI-x2

< 1, then

0 \037X

If

0)

=
If

Ii In
2 =8-++0

401)

of Unbounded Functions)

Integrals

arcsinx 0l-\"=lirn

liIn

8-++0

8-++0)

.
[arcsil1(l-e)-O]=
\037

< x 2, then
\037

dx

5I V II

x2 I

dx
Yx 2

tf
1

.
= In1
I

+0

-+

E'

-=

dx

5+e)

V x2

In(x+Vx2-1)
8-++0

== lim

I+\037)

== In
(I+ c + V(I + E)2_1)]
[In (2-t-- JI/\"3)-ln

lim
8

...+0)

Hence,)

5 11 \\

11

x2

\\

(e) Represent the given integral as


ing

+ V3).

= 2 + In (2+ V3).

dx

\0372

each term of the numera tor

by

a sum of three
V x 3,
1

items, divid-

dx
I xa+.V\037-2dx=rX12/5dx+1
4/15_21 3/5 .
3
dx

The

first

L\037ibniz

J0

J0

Vx

0)

sumn,and is a proper integral evaluated

by

the Newton-

formula:)

X
S
o

5 17/o
12/ 5 dX=-X
17

1
1

=-5

17

\302\267

The second and third summands are unbounded to the


point x = O. Therefore,)
1

dx

'j\\ X 4/15
o

ana logously,)

-.

I 1m

_+0

'. dx
1

xAI 1 5

8)

1
r>

jo

--I'8-+0 X-II'

- 8

dx

X3/5

==

dx

I lIn

e -+ + 0 5
8)))

x 3/

I)

1m

\037

II

11/151_\037.
1

. _5
2

I 1m
8 -+ + 0

right

X2 I 5

8)

=-25 .

of the

Ch.

402)

Hence,)
1

r
J

x3

+ Vx-

V?

o)

(f)

III.Improper

Integrals)

- dx=T7+n-'2=2

15

Represent the integrand f (x)=

of part ia I fract ions:

000_

625
187\"

11
x3

in

the form

of a sum

I
x+2
f(x)= 1-x3 = (I-X)(I+X+x
2 =3\"
[ x+ l+x+x2]

Then

= S
\037

I\037X

++S

. 1=
x 8-+0S
S
o
1

\302\267)

1 1111

0)

dx. Since

1\037\037\037x2

.
- =-8-+0
dx

dx

l\037xX3

l-

I 1m

-x) - =00,

.
nt 1

the given integral diverges.There is no need to compute the second


representing a proper integra 1.
Note. Evaluation of the improper integrals from Problem
(a to f) can be considerablysimplified by using a generalizedantiderivative and applying the Newton-Leibnizformula. For instance,
t (a) the function F (x) =
in Problem
V ln 2 x is continuous on
the interval
e] and differentiable at each point of the interval
1 < x e, and F' (x) ==f (x) on this interval. Therefore)

summand

8.2.1

8.2.
[1,

\037

\037

_ 3 V nx e _ 23 ,
-T
J xV lnx
e
r

dx

Il

I)

8.2.2.Proceedingfrom

the definition, compute the


their divergence):)
(or
prove
improper integrals
da

(a)

J0

2/1t

2xdx
;
2 _a2
)2/3
(x

(b)

(c) J cos

-2

.
sIn

-.-'
dx
x2

\302\267

S
0

dx

dx

l-x'(I-x)2; (d) S V (411:

dx

(e) J' x V x2
-1

X)2

2
2

(f)

dx

5
1)))

x In P

\302\267

follo\\\\Ting

32_
8.2.I mproper

\037

8.2.3.
Evaluate
'\"

(a )

X2

t V9
-3
\\

dx

the following improper integrals:

../ + dx.
.r
2 x
2
x
JI
J

(b )

;
x2

0)

Solution.(a)
S

403)

Functions

of Unbounded

Integrals

Find the indefinite integral

(9 arc sin

\037

\0372\037Xxt

\037

x V9-X2) + C.

a generalizedantiThefllnctionF(x)=+(9arcsin
-xV9-x2)iS
2
x
\037

[-3,3],

since it is
derivative for f (x) =
.2 on the interval
V
continuous on this interval and F'(x) ==f (x) at each point of the
interval
3). Therefore, applying the Newton-Leibnizformula,
we get)

9-x

(-3,
3

-- =X2

dx

2 (9

x2

-3t\\ V 9

. x Vg -x2 3 =,Jt.
9
arCSln--x
3
) -3 2
1

(b) Transform the integ rand


/-

I 2+x
- = 2+x- = -

f (x) == J

V4

x\037

2
V 4 x2

-.

+ V 4x

x2

The indefinite inte gral is equal to

../2-x
2+\037

4-x

2
dx==2arcsin...:.-V
+C.
2

-x

2 is a
The function F (x) = 2 arc sin
V4
generalizedant iderivative for f (x) on the interva 1 [0,2], since it is continuous on
this interval and F'(x) ==f (x) on the interval [0,2).
Therefore, applying the Ne\\vton-Leibniz formula, we get
\037

2
\\

t.,

V ;+;dx=(2arcsin-V4-x )1:=n+2.

8.2.4.Test th e integral)

dx

Sx
-1)

fi\037x

for cOFlvergence.

Solution.At the point x == the integrand goes to infinity. Both


I
o
and
since = > 1.Consequentintegrals S x
x Vi diverge,
j'
Vi
-1
\302\260

'A.

\037

0)))

Ch.

404)

III.Improper

Integrals)

ly, the given integral diverges.If this were ignored,and the Newton-Leibniz formula formally applied to this integral, we would
obtain the wrong result:

=
-S x Vi ( V x )

=
\037

-6.

I)

this is because the integrand is positive.


the following improper integrals for convergence:

And

8.2.5.Test
1

y-

eX

(a) r
J

cosx

b) r
J

dx;

VI

Solution.(a) The integrand

.I-cosx=

is

V 2 sin

si?x+cosx dx.
x:i

large as x \037

infinitely

x
2

1\"'0..'

+ O. Since

Y2 x as x ---+
+ 0,

the integrand has the order A = I as compared with


According
to the specialcomparison test the given integral diverges.
(b) Rewrite the integrand in the following way:
\037

t (x) == ,:;in x+cos\0372


x
x

V +

\037.

Vtl\"

This function is -infinitely large as x \037 1, its order is equal


to A= as compared with l\037X ' since the first multiplier tends
to 1 as x\037 O. Therefore, by the specialcomparisontest, the given
\037

integral converges.
Test the following improper integrals for convergtnce:

8.2.6.
2
In

(a)

.o

esin

\\

.o
\\

x2 +
dx;
dx; (b) \\ Y16x4
1

:1

\037

1
r-

(c)

(I + VX\"3)

C\037

d\037

SIn x

I'

SoLution. (a) The integrand f (x) =


interval (0, 2) and is not defined at

In

f (x) ==

liIn
\037....

00.Indeed,since

is

(:st
==
x

\037\037)

positive in the

O. Let us show

0)

sin x
esinx_l,-....\".

1\"'0..'

x,

In

(I+ V- X3)

I\"'o..'V

xJ

as

x\037O,)))

that

8.2.Improper

\037

Functions)

of Unbounded

Integrals

405)

we have)
Jim

S1I1

\037

At

(I + VX3) = Jim
x

In

x 0

\037

V.iI= Jim

V.= 00.

x_ 0

that

f (x) \"'-'

the same time we have shown

X\"\"

\037-

V
order A =

as x \037 0,

i.e.

x\037

that t (x) is an

infinitely

large quantity

of

\037

<I

as com-

pared with \037.


x Consequently, by the specialcomparison test, the
given integra I converges.
(b) Determine the order of the

.;

x2
16

-x

to 2

in

large function t (x)==


the neighbourhoodof the point x = 2 with respect
infinitely

ssion for t (x):


x' To this en d 2tran sform the expre
2
Vx +
y x -tt(x)= ,) 16-x.4- = 4+ x2 3 r
2-x
V
V
V 2+x V-I

\302\267

Hence it

is obvious that the function f (x) is an infinitely


large
I as x
quantity of order A
According to the special
comparisontest the given integral converges.
x. is unbounded 111 the neigh(c) The integrand t (x)
x SIll X

-..2.

=+ <

bourhood of the point


cosx
==
f (x)

Since

V\037os

x ==O.

casx

t1x-sinx-V x (I-

---

Vx

V )

x-++,
0)

as x ---++ 0 the function f (x) is an infinitely large quantity of order


A=
< I as comparedwith and, by the specialcomparis0ntest,

\037

the integra I converges.


8.2.7.Investigate the following improper integrals for conver-

gence:

--

eX dx

(a).o YI-x'!

(b)

\\

(c)

I-\\

1-x dx;
X

\"

dx

\\
t.o)

dx

..o V (I x2)0 ;
. dx
1

(e)

x\037

\\

x-sinx '

(d)

Jo I _ x3 + x6

(f),.rln(V;+I)
e
-1 d
tan x

o)))

X.

Ch.

406)

8.2.8.Prove

Ill.Improper

Integrals)

the integral

that

.
Slnx

dx

Vi)

-converges.

Solution.For 0 < x

.
X
yi

1)

\037

sIn

\037

I
\037

\302\267)

Yx

the integral

But

.f
o

:x converges,therefore,

by the

comparison

test,

sin

the integral r,
u
o

(Ijx) dx

also converges,and consequently

Vx

the given integra I convergesabsolutely.


of the integral
Prove the convergence

8.2.9.

JT

I = Insinxdx
\037

o)

.and eva]ua te it.


Solution.Integra te
;r
2
\037

r
,

- 2

ns\037nxdx=xlnsinx
l

lim
K_

+ 0 t an x

x- --0
2

Now make the


x=O at t 1

T'f

\037

\037

= In (sinx), dx
JT

dx=_

x C?SX
sIn x

= 0, the

S
0

t an

==

dv:)

x dx.

last integral

IS

proper

initial integral converges.


in integral
Then dx ==2dt;
On substituting we get:
at t 2 =

/.

substItution x = 2t

=0; x=

t an x

JT

f
0

1, Iim

one.Consequently,the

n/2

-'

1(

Since

parts, putting

by

\037

1[/4

1(/4

Insinxdx=2 Insin2tdt=2 (ln2+lnsint+lncost)dt=


\037

\037

n/4

1[/4

Incostdt=
=2tln21\037/4+2 o Insintdt+2
\037

\037

0)

n/4

JT/4

In
\037

2+ 2

S
o)

In

sin t dt + 2

S
o)

In

cost

dt.)))

8.2.Improper Intel!rals

\037

In the last integra

make the substitution

407)

Funct ions)

of Unbounded

t ==nj2

-z. Then

dt==-dz;t=O at zl='Jtj2;t-=n/4 at z2=='Jtj4. Hence,


n/4
/2
25 Incostdt2c=-2
5 Incos( -z)dz=25 Insinzdz.

JT/4

\0372

.'T\"f

\0374

Thus,)
n/2

n/2

.1t/4

1=5lnsitlxdx= In2+2 5 Insintdt+25 lnsinzdz=


o

n/4

.1t/2

In2 + 2

Whence)
JT/2

I= 5
o

8.2.10.
Compute the

sin xdx =

In

In

sin t dt =

o)

-;

In

2 + 2/

2.

In

integral

r xndx (n a natural number )


J V I _ x2

Solution.The integrand

').\"

large quantity of order


x\037l-O.
Therefore,the integral conx as

respect to

with
\037

is an

)1

verges.

Make the substitution x ==sin t

x ==0

at

t ==

infinitely

in

the integral. Then dx == cost dt,

0, x == 1 at t ==nj2.On substituting we get


1
dx
t cos t dt
= sinn t d t .
= sinn cos
t
_
x
.o ..r
r
.\\
j0
\0372

\0372

\037

\037

Xn

\\

\302\267

\037

-.
-_

The last integral is evaluated


JT

\0372

sinn t

jo

0)

Problem 6.6.9:

in

n-l n-3
' even,
_2. .
dt ==
n-l n-3 _2 ' odd.
{
n-2 3
I

\302\267

\302\267

\302\267

\302\267

2\" \"2

\302\267

Consequently,the given integral is also computed by the same

formu

la.

Eva luate
8.2.11.

divergence)

the following improper integrals (or prove their

:)

(a)

j xlnx;
o)

2
dx

dx

\037

(b)

j
1)

V lnx

(c)

J
o)))

+ dx.

3x2 2

x\037

Ch.

408)

8.2.12.
Compute the

11/.Improper

improper

Integrals)

integral)

m>-I).

(n natural,

/n=\037xmlnnxdx
o)

Solution.At

n == 0

the integral is evaluated directly:)


1

10 =

+1

I
xm dx=xm
m+10=m+l '
1

o)

For

>0

1n

integrate

by

= Inn

parts,

putt ing

-'

dv ==xm dx;

xm

n-l X dx ..=:

x;

+
du=nlnn-lx dxx' v= xm+1)
.
\\V

\037

get)

= Inm +
X

fl

+1

-+.

- + I -1

nn X

In

\\

\302\267

o)

This gives a formula


any natural

In ==

And

means of which one can reduce In to 10 for

by

- + 1 -1 + n(n-l)
+ 1n:

==

finally,)

1)2

(m

(_1)nn!
1n = (m+
l)n+l

8.2.13.
Compute the

2 ==

\302\267

\302\267

\302\267

== (-l)nn!
(m -t- 1 )n

(J

\302\267

integral)

- S V2+x-x
12.0

e-Xdx

0.3)

accurateto 0.03.
Solution.The integral has a singularity at the point x = 2, since
2 + X-X2 == (2
(1+ x). Let us represent it as the sum of two in-

-x)

tegrals:)

2-\342\202\254

11

e-Xdx

===

0.3)

2
2,

t!2+x-x

1'1.==\\
,_

2-\302\243)

x dx

V 2+x

X2

\302\267

Now compute the first integral to the required accuracy,and estimate


the second one. For E
we have)

o < /2

\037

0.1

3:\037

1.
4
dx
= O.115X 3
= 0.153
-=
84
< e-2.9
2-x
SV
V 2-e)))
9

4\"

\302\267

8.3.Geometric and Physical Applic's of Improp. Integr's


Evaluation
E=O.l,we get the estimate 12 < 0.028.

\037

Putting
integral)

e-Xdx

0.3)

Simpson'sformula

by

and

And

error
is

a step h/2

with

with

0.4,)

so,integral 11gives

not

step

x2

===

80.8 = 0.519,

===

0.8gives

80.4 ==0.513.
the more

accuratevalue,

with
0.513,

that
0.001.
Taking into consideration
exceeding

positive, we round

the obtained value to

off

of the

- SV + 1.9

J1

409)

an

integral

12

I 0.52
\037

an error not exceeding0.03.


we get the estimate12 < 0.005,
but the
Note. By putting E=O.OI,
computation of the integral) 1.99
e-Xdx
with

I1

- S V2
0.3

+X-X2

cumbersomecalculations.
8.2.14.
Investigate the following integrals for convergence:

would involve much more


1

(a) r

.J
o
1

(c)

;
SIn x
:\037

cas2 x dx

S (I

X)2

.'

(b)

S ex\037\037osx

0)

1
(d )

tan

SVI

x dx
x2

.'

o)

o)

6
5

(e)

dx

sin

S1 YII -x2 1.
\037

2)

of Improper
* 8.3.Geometricand PhysicalApplications

Integrals

bounded by the curve y = I


(the witch of Agnesi) and its asymptote.
Solution. The fund ion y = I x2 is cont inuous throughout the en1ire number scale,and Iim y = O.Consequently,the x-axisis the asympx-oo
It is required
tote of the given curve which is shown in Fig.118.

8.3.1.
Find the

to

find

the area

area of the

S of the

figure

figure

that

\037X2

extendswithout

bound along the)))

Ch.

410)

x-axis.In
ra S =

III. Improper

Integrals)

other words, it is required to evaluate the improper integ-)

00

dx

S)

I +x\037

. By

virtue of the symmetry of the

-00
y-axis we have)

00

00

S= r

dx

-00

about the)

figure

. arc tan x A J(
=2
0 =2'\"2=n.
I+X2
1:rr;,
dx

I+X2 =2.J

0)

8.3.2.Find the

surface area generatedby revolving about the


arc of the curve y ==e-X between x ==0 and x == + 00.
Solution.The area of the sury
face is equal to the improper in1)
tegra I

x-axisthe

+00

-1)

S=2n5

.:c

=. 1,x == 00 at
)

S = 2n

\"

tJ

dt===-e-x dx,

0; hence

t ==

[t

\037

Vl+e-

2X

we get

x\037O at)

VI + t 2 + In (t + VI + t 2)] =
\037

o)

[V-2+In (1+ V 2)].

==J1

8.3.3.
Compute the area enclosedby the

cartes

x3

Solution.The

loop of the folium of Des-

+ y3-3axy==O.
Descartesis

folium of

dx.

substitution e-X == t,

Making the

+ t 2 dt = 2n.

Fig. 118)
t

shown

in

Fig.86.Let

us

re-

present the curve in polar coordinates:


x==pcoscp;y==psincp.
3
3
3a p2 cos sin ==0,whence,cancelling p2,.
Then p3 cos + sin
cp

we

cp

p:\037

get)

cp

3acos

cp

sin

P ==cos3cp+sin 3 cp')
cp

cp

Since the loop of the curve correspondsto the variation


o and

the sought-for area is equal


3t)

3'1)

s ==_21 S
2

to)

p2

d ==
cp

2
9a2

TS
0)))

sin2
3

(sin

cp

cos2
+cosa

cp

cp
cp)2

dcp.

of

cp

between

8.3.Geometric and

\037

Integr's

Applic's of Improp.

Physical

411)

To evaluate the obtained proper integral make the substitution


n
tan<p==t;cas'\".) ===dt; cp==Oat t==O, CP===2 at t===oo.Thus we get
d\037

cp

00

9a2 r [2dt

9a2

S==TJo (I+[3)2==T

l\037noo

=-T .
3a2

t 2 dt

S (l+t3r\037
0)

[ 1+/3
]0

1\037\037

3?

=='ia ....

the volume of the solid generatedby revolving the cis8.3.4.Find


x3
==
==
2a-x about its asymptote x 2a.
Solution.The cissoid is shown in Fig. 119.
Transfer the origin of
coordinatesto the point 0' (2a, 0) without changing the directionof
the axes. In the new system of coordinates
X ==x-2a,Y == the equation of the cissoid !I

soid

y2

has the following form:


Y 2 ==

(X

+ 2a)3

-X

\302\267

The volume of the solid of revolut ion about


the axis X == 0, e. about the asymptote, is
by the integral)
,expressed

i.

00

=:n;

:c)

o)

00

X 2 dY

= 2:n;

X2

dY.

-00
Let us pass over to t he variable X. For this
purposewe find dY == Y' dX. Differentiating the
equation of the cissoid in the new coordinates
.as an identity with respect to X, we get
\037

\037

0)

' _ 3 (X +2a)2

2YY ==

whence for

Y
Y

> 0 we

(X

X2

+ 2a)3 _ __ 2 (X

' _ _ (X +2a)22 (X -a)_ _


X Y

Hence,)

2a)2 (X

-a)

+ 2a) (X -a)
(X + 2a)jX

(X
X2 V

\302\267)

(X+2a)(X-a) dX.
(X+2a)/X

,f V

2a)

Makethe substitution (X + 2a)/X ==


t

--1-

X2)

have

V=-2n

Fig.

t 2;

X ==

-2aat

t ==

= 00.Then:
2at 2
4at
X ==- 2a
dX=
dt;
X+2a-==
1+12)
(l+t2)2

X-a==- l+t .
3a+at2
2

')))

l+t\037

0,X ==0 at
;

Ch.

412)

III.I mproper

Integrals)

whence)
00

== 2n

r
J

2at 2 (3a+ at 2) 4al dt


t 2)2
(l +(2) (l t 2) (1-1-

o)

'

00

== 481ta3

00

, (1
\\

Putting

z ==

t == tan

z,

n/2.Hence,

dt == sec2 z dz,

14

. (I
\\

II)

t == 0

get

Wl'

r-

16Jta3

at z ==

+t 2)4 dt.

0, t == 00 at

N2

\0372

/2
dt
t 2)4

= 48na3 sin2 z cos4 z dz + 16na3 cos2 z sin4 z dz =


\037

\037

rr!2

n/2

= 48na3 cos4 z dz 48na3 cos6 z dz +


\037

\037

'!II 2

+ 16na
3

sin4 z
\037

31/

dz-16na

\037

sin6z dz.

31/2

Using the

formulas for

known

the

integrals

sin\"
\037

xdx,

o)
1\302\243/2

\037

cos\"xdx (see Problem

o)

nIx3
\302\267

8.3.5.

I X 3 x5
3
3
= 2a
64nQ\"2
2 X 4 64J1a
2 X 4 X 6 2n a
Prove that the area of the region bounded by the curve
1
axis of abscissas,the axis of ordinates and the
2

we get
6.6.9),

T'

y l-x ,the

asymptote x== 1 is

8.3.6.Prove

finite

and equals

\302\267

;.

that the are,a of the region bounded by the curve


the
axis
of abscissas and the straight lines x == + 1 is
y
3 x'\".) ,
V
finite and equals 6, and the area of the region contained between
the curve y =
the axisof abscissasand the straight lines x= + I
is infinite.
Find the volumes of the solids
enclosedby the surfaces
==
==
the
X
lines
(;
x,
0, y ==0 (0 x < 00):
b.y
generated revolvihg
y
(a) about the x-axis,
(b) about the y-axis.)))
l

===

\0372

'

8.3.7.

\037

-\037

\037

8.3.Geometric

Comp ute
8.3.8.

Applic's

and Physical

the area containedbetween the

and its asymptote.


Compute the area bounded by the curve
and the axesof coordinates.

8.3.9.

413)

Integr's

of Improp.

--

cissoidY ?\"'-2ax

-:=::

Find
8.3.10.

e-2x

(at

x > 0)

the volume of the solid generatedby revolving, about


for x\037
the x-axis,the infinite branch of the curve y = 2

8.3.11.

\037

\037:J

1.

Let a mass m be located at the origin 0 and attract a


material point M found on the x-axisat a distance x from 0 and
with
a force F ==
(accordingto Newton's
having a mass of
law). Find the work performed by the force F as the point M moves
along the x-axisfrom x-:=::r to infinity.
Solution.The work will be negative, since the direction of the
force is opposite to the direction of motion, hence)

1,

.-

00

A ==

f
r

\037

dx =

lim
N

-+

00

- =x

\037

dx

.E:.
r .

,)

During the reverse displacementof the point M from infinity to


the point x==r the force of Newtonian attraction will perform posiThis quantity is called the potential of the force
tive work r
under considerationat the point x == r and serves as the measureof
potential energy accumulated at a point.
In studying a decaying current resulting from a discharge
instruments are sometimes used whose readings are pro\"ballistic\"

!!!.-.

8.3.12.

00

portional to the

\"integral

current

intensity\"

g = I dt

or the

\"inte-

\037

00

S == /2 dt and not to the instano


taneous value of the current intensity / or to its square /2. Heret
is time measuredfrom the beginning of the discharge;I is alterna-

gral square of current intensit y\"

\037

ting-current intensity depending on tirne. Theoretically, the process


continues indefinitely, though, practically, the current intensity becomesimperceptiblealready after a finite time interval. To simplify
the formulas we usually assume the tinle interval to be infinite in
all calculations involved.
Compute g ktand S for the following processes:
(a) I == loe- (a simple aperiodic
proces3);k is a constant coeffi.

cient,which is greaterthan

zero.)))

Ch.

414)

Ill.Improper

Integrals)

coefficients k and
(b) I == Ioe-kt sin wt (simpleoscillatingprocess);
are constant.

U)'

Solution.)

\037

kl

(a) g ==S J oe- dt

!im r oe-kl dt

A\037oo.J

= 10 !im
A\037oo

_\037_kt

= lo/k;

0)

00

/2

\037

S== 12e-2ktd
t ==\037.'
o
2k
J

00

(b)

g= o J lIe-kl sin

\037

dt = !im \\ Ioe-kl sin ffit

ffit

L\\

sin2 wt dt = !im r
A\037ooJ

:\037

}\037\037

I Inl

A
1\037e-2kl

/
'
w cas(I) t + k sIn wt) e k t J A _
0
+k2 A\037oo [(
W\037+k2
.

I0

w2

00

S=

dt

A\037oo(\037

[1-

W2\037k2

o(f)

n e-2kl 1-2 2 wi dt =
CDS

(k

cos2ffit + ffik sin


2ffit)]

e-2kt /:=)
/2W

4k

8.3.13.
Let an

.
\302\267

(k'l.

+ 2)

\302\267)

extended(in both directions)beam lying

infinitely

concentratedforce P. If thethe initial positionof the axis


of the beam (before the latter is bent) and the y-axis is drawn\\
the point 0 (at which the force is applied) and directed
through
downwards,then, on bending, the beam axiswill have the fo11o-

elasticfoundation be bent by
with
x-axisis brought to coincidence
on an

\\ving

equation)
y

e-Cl.Ixl (CDSax+sin a xl>.)


I

\037;

a and k are certainconstants.Compute


elasticdeformation by the formula)

where
of

the

potential energy

00

= Ee

\037

dx)

(y\2")

(E, e const).)

())

Solution.Find
y\"

y\":

- (-

= P;\"e-ax[(CDSax+sin ax)

sin ax+cosax)+

;
J

p
+ (-sill ax-cas
ax)]== e-ax (sin ax-cosax).)))

8.4.Additional

\037

415)

Problems)

Hence,)
00

==

5o e-

2ax

p2a,6\302\243e

k2

(1-2 sin axcosa.x)dx


=

==

p2a,6\302\243e

k2

[
I

2a,

4a,2

2a,
1 p2a 5
+4a2 J == 4k2

\302\243e
\302\267

8.3.14. work has to be performed to move a body of massm


Earth'ssurface to infinity?
Determine the work which has to be done to bring an
8.3.15.
==
\\Vhat

from the

electric charge

chargeel

e2

1 from

to a

infinity

unit

distance

from

.)

* 8.4.AdditionalProblems
8.4.1.Prove

that the integral)


00

dx

5 xP

1n

qx

1)

convergesfor p > 1 and

8.4.2.Prove

< 1.

that the integral)


00

.x;P
\037

sin xq dx,

q =1= 0

o)

--1
1.

< (p + 1)/q< 0 and converges conditionally for 0 (p + 1)/q <


Prove that the Euler integra 1 of the first kind (beta func-

convergesabsolutely for
\037

8.4.3.

tion))

- (1-t)q-l dx

B (p, q) = xp 1
\037

o)

p > 0 and q > O.


8.4.4.Prove that)

converges for

T-..oo+-j
lim

if

a,

*
I

'Sinax'sin\037xdx=o.

o)

\037

I.

8.4.5.Prove

that)
00

1=5 e-x'.x2n +1dx =


o)))

\037t

(n

natural).

Ch. V

416)

Ill.Improper

Integrals)

00

8.4.6.Prove
tive a and

that

tends to

f (x)

if

the integra I

if

5a ;X) dx convergesfor any posi-

as x -?-0, then the

integral)

00

'1

f(ax)-f(\037x)

dx

\\
t..'

(ex,

> 0, > 0)
\037

convergesand equaIs

8.4.7.Prove
00

A In

e- '1.X_ ex

'

\037

\037x

5o

(\037/a).

that)

dx == cosax x casBx dx == In 1.a .


J
II)
.rT

8.4.8.At

va lues of m

what

does the

integral)

'.
/2)

C OS

xm

dx

con-)

o)

verge?)
31

8.4.9.Prove

-5 (SIn x)k converges if

that the integral

\037x

< 1, and

o)

diverges if

1.)

\037

00

8.4.10.
Prove

that

the integral

sin

5o

o < s < 4, and convergesabsolutely if


Suppose the integral)

x (l

xs

8.4.11.

cos x) dx

converges

if)

< s < 4.

+00
\037

f(x)dx)

(1))

a)

convergesand the function


Does the integral)

(x)

<p

is bounded.

+00
\037

f (x) (j)

(x)dx

(2)

a)

necessarilyconverge?
What can be said about the convergence
of integral (2), if integral (1) convergesabsolutely?

8.4.12.
Prove

the validity of the relation

f (x) == 2f (n/4

where f (x)=

\037

o)))

+ xj2)-2f (n/4-x/2)-xIn 2,)

In cosy dy.

\037

Compute

with

8.4. Additional

417)

Problems)

the aid of the relation obtained)

-J
JT

( )
\037

In

casy dy.

o)

8.4.13.
Deduce the

reduction formula for the

integral)

JT

In= Incosx.cos2nxdx(n
\037

())

and evaluate this

integral.)))

natural))

ANSWERS

AND

HINTS)

Chapter

I)

1.1.5.
(b) Hint.

Prove by

the rule of contraries,

putting

p2
,
2=--:;-

\\\\There

q\"

integers without common multipliers.


Hint. You may take

are positive

p and

k=s2-2
.
1.1.8.
2s
1.1.9.
(c) -4\037x\0372.
(b) x\0374,
I. Hint. The equality is valid for those values
1.1.11.
(a) x < -I or
true for those
3. Hint. The equality
2.;;;;
:;;,.
x';;;;
of x for
0;
(b)
:
:+
values of x for
x2-5x+6\0370.
or
x
1.1.13.
<
(a)
; x > 8; (b) x < 0 or 0 < x < 5. Hint. The
x\037O;

x\037

which

hold\037

which

a
a

inequality

> a I-I b

<

holds

good

when

b I.

and b

are opposite

in sign

or

when

I.2.3. ., [a (a2a++3a2+3)]., (a3+a) (a3 ).


Y2+ .' 2 VIO-5.)))
?
2., (a+ h)3
1.2.6.
4 ..r1.2.4.
r 2+I,.
b-+ab+a
8
2
x.
1.2.11.
(x)=10+5X2
6
2
5x + .
r _ 45x + .
1.2.13.(3x
) 2-3x' (x3) _ 2--x
3 '
15x2+3
125x6 +75x4 + 15x2 + .
31(x)= 2-x ; [I (x)]3== 8-12x-t-6x2 -x3
; 1(3)=8.
1.2.14./(2)=5;/(0)=4;/(0.5)=4;/(-0.5)=
it
follows
that
Hint.
Frornx
1.2.15.
Yn+l=aXn+l=aXn+d=axnad.
n +l=xn +d
2 -5x+6.
1.2.16.
x=:f:2;:f:3.1.2.17./(x)=x
1.2.18./(x)=--=23;cp(x)=527
1or x:;;\" 2. 1.2.20.
1.2.19.
x; S=b 1- x.
x.;;;;-I
P=2b+2
(
(
)
)
1t
1.2.21.
(b) (2,3); (c) (-00,-I) and (2,00); (d) x==2\"+2k1t(k==O, :f:1,
Hin!.Since sin x J, the function is defined only when sin x I;
:I:2,
+2kn<x<
(g) (-00,2) and (3, 00); (h) [I,4); (i) (-2,0) and (0, 1); (j)

-1.
-

\0373

\037

...).

\037

< 2+2k1t (k=O, :1:1,


:f:2, ...).)
1t

\037

-;

==

Answers and Hints

Ch.

to

419)

I)

1.2.22.
(d) The function is defined over the entire number scale, exceptthe
x :I:
2.
1.2.24.
3];
(a) (-00,00); (b) (3-2n,3-n) and (3, 4); (c) [-1,
(b) 5 x 6.
(d) (-I,0) and (0, 00). 1.2.25.
...);(b) [ ,
1.2.26.
(2k+1)n (k=O. :I:1,:1:2.
(a) 2kno;;;;;;xo;;;;;;

poin ts

==

\037

- -1].

\037

\037

1.3.3.
(b) Hint.
1.3.4.
(b) It

and

increasesfor

decreaseson

x2

Consider the difference

-5;

X2

+kn < x <

xt
xi

.
+

(k = 0,

+ kn

\037

1.3.7.
The function

decreaseson

the interval

<x

+ 00 to 2 and

from

\037
\037

increaseson

...)

:I:I, :I:2,

the other intervals.

the interval

\037

1.3.9.
(c) The function

x<

from

\037

to

\037

is

+ 00.
even.

neither even,
ode!,(d)
1.3.10.
(a) Even; (b) odd; (c) odd; (d) neither even, nor odd; (e) even.
1.3.12.
'
(a) A 1=5, ro=4, cp=O, T== ; (b) A 1=4,ro=3,
T=2;; (c) IA =5.ro={.ljJ=arctan ' T=4:rt.Hint. 3 sin +4cos =
nor

= 5 sin

cos1jJ=

( The ). value
1.3.16.
greatest
where the
where

+1jJ

\037

value at the point

value.

, sin

IjJ

: :. 1.3.13.

\037

\037

\037

(b) T = 2n; (c)T =

(1)=2. Hint.
\037

<p==

1.

reachesthe greatest
The
trinomial 2x2-4x+3
reachesthe least

quadratic

function

1.3.17.
(b) even;
(a) Even;
(c) odd; (d) even.
1.3.18.
(a) T==n; (b) T==6n.
Then
1.3.19.
Hint. (a) Assume the contrary.

whence

since

cos
the

x+T+sin (x+T)=x+ sin

=- 2 sIn.T T ' which


(x+ )
side is

left

cos V x+T=cosY x,

not

T,

constant

:I: :I: ...),


_ +arcsiny .' (b) x=3sin y;
1.4.6.
(a) x-

Then

contrary.

Yx+T+vX=2nk, or y---':
x+T+ Y_x

for any

which is impossible, since the left-hand mem bers


(k = 0, I, 2,
these equalities are functions of a continuous argument x.

= 2nk
of

is impossible

(b) suppose the

constant;

whence either

x,)

IOg2 Y

log2

y-I

log y

(0 < y

1.6.9.
Hint.

The

(y

> 0),.

(d) x =

...; -2'4' -8\"' ' ...;


-

Y3
JI3 V3
1.6.3.
(a) -y-' -y-'O,--y-,
10
113
'
(c) 2; 2.25;2 ; 2
256

\302\243)

< 2 or 2 < y < 00).

10gL
2

27

g
(c) x=y 10]

...
\302\267

inequalit y

(b)

3
I \037n:

21

< e is

16

satisfied

lor

>N

=)))

Answers and flints)

420)

- ).

e = 0.1the inequality is fulfilled beginning with n = 10, at


8=0.01
beginning with n= 100,at
beginning \\vith n= 1000.
Hint. Verify
that the sequence{X2n-l} tends to I as n\037 00,
1.6.10.
and the sequence{x2n } tends to 0 as n \037 00.
1.6.12.
(a) It has; (b) it doesnot have; (c) it has; (d) it does not have.

=E

At

\037

\302\243=0.001

1.6.14.
Hint.
Hint.
1.6.19.

(b)

(a)lxnl\037\037;
n

IXnl\037\037.

> 1 put Va= l-t-an (an>0) and, with the aid of the
a= (1+an)n > nan, prove
that an is an infinitesimal. For a < I put
inequality
I
n/(;= I .+an (an> 0) and make use of the inequality \037=(l
+ all)n > nan'
V
a
For a

2'

5
1.7.1.
(b)

1.7.4.(b)
and then

1.7.2.
(b)

(c) 0; (e)
(f) O. Hint. Multiply

4\"

I;

divide

by n

(e) I;

3; (g) + ;

I. Hint.

(h)

l.

(f)

and divide by imperfect of

16;

sum, square

each

Represent

summand

of Xn in t he form of the difference


I

IX2==1-2 ,
which

1.7.5.
(a) +;

2x3

bring Xn to the

will

2-3;

form Xn

(c) 0,

(b) I;

1-n+

'

-+.

cosn 3 is a bounded quantity;


1.8.6.
(b) Hint. The sequenceis bounded
X3x '\" Xn 2n-l and therefore
\037

+ + + ( ) + ...+ ( +)

-1= 3 _

\037

1.8.7.
(b) O. Hint.
1.8.9.
Hint. For
< a < n are

( +)

-1<

is an

2n

in-

n! = 1 X 2X)

3.

1.

of the fact that Xn+l


<
xn
n+3
a
certain
with
the
value,
beginning
inequalities
therefore
n =)
< a < n, and lirn

Take advantage
all

The quantity
4
(e) 0; (f) 3\"

due to the fact that

\037

Xn

Hint.)

(d)

and

tinitesirnal,

n-n+I'

n(n+l)

n,

fulfilled;

1/\\1

\037

= lim nr- = l.

f/

il

n)

1.8.10.
flint.
\302\245

Yn (Yn

The

> I).

The sequence Yn }

boundedness

of the

t
d ecreases,since Yn-tl=a211+1

sequence from

lim Yn by b and from the relat ion Yn + 1

1.8.11.
Hint. Ascertain
the

fl\037OO

ness from

that

the

inequalities

find

follows from
b

= l.

sequence increases.Establish

=---

-:)
n- < (n-1) n-1
1

below

= YYn

=a2n X2

> 1.Denote
the bounded-

(n\0372);

- )=2Xn<I+(I-+)+(+-+)+...+
C\037I
\037

\037

.)))

A nSWl'rs and flints

1.8.12.
Hint.

xn

Transform

tage of the inequalities)

Y +I

=I

and

ascertain

that

t
lirn

1.9.3.
(e) Hint.

2 xn

arc tan x <

= + 00,)

arc tan

; (e)

; (f)
\037

trinomial

by i\037perfect

X-3
Y-

x-3[loga x+6-3]
hrn

(j) 1 2'

and

1.10.3.
(b) 32.

> 0).

(x

\"6)

11

Hint.

. Hint.

\037

a I.
the

Multiply

numerator

(h)

\037\037

x=ZID;

Put

to the denomini1tor

irrationality

(f)

x=2-z:z--+Oas x--).2\";(g) -3.Hint.


n

00. Hint.

Put

sinx=y.

Put

t)

3
1.10.5.
(b) e ; (c) e- ; (d) elnk;
1.10.7.
(b) 1; (c)
(b) 4' 1.10.8.

i.

1.10.11.
(a)
1.IO.12 (a) 20;)

(b))

\302\267

(b)

3
4

-2;

loga

6.

(c) 2\";
(c)

(d)

\037;
I

cot

e
2

(d) \"5;

11

2;)

(g)

(d)

I
;)

\037

a;

z;

(h) 2.)

a.
(e) 0;)

(f)

(e) -24.)))

divide

;-x=

(f) 4;

de-

11m

\037

and

. (X-3)(Y
. 2
X+6+3)
[x-3 x-3 ] ==loga 6; (1)_3 ;

On removing the
denominator by x.
(c)

a!

(VlO-x+ 2): (g)

square

= loga

sin

1.10.2.
(e) {-.Hint.

the numerator

X =\037

.
2=srn. x-sin

. x-

\037

Hlnt.

\037

(2

product nnd apply the inequality

nominator

=0.)

of the inpquality

tan

sIn

1.10.1.
(d)

lirn

Xn

the difference

(f) Hint. Transform

into

have

1
,

Take advantnge

.::-

values of the function

appropriate

xn

comparing

...)

n)

sequencesof

the

limits:)

different

sequenceby

the

,
Xn=-(n=l, 2,

and

n)

advan-)

and take

n\037

1.8.13.
Hint. See Problem 1.8.7
(a).
1.8.14.
Hint. Establish the boundedness of
with the sum of some geometric progression.
1.9.2.
(b) Hint. Choosethe sequences)
X'l

2n

..r
J' n 2 + 1+ n
<1.
+

2n

<

421)

1)

xn =

into the form

2n
2n +

Ch.

to

1.)

Answers and Hints)

422)

-a-B.

I.IO.13.(a) e4;

(h) 1;

(b)
(j) I; (k)

(i) 9;

11m
x-o

(e) e

e('1-\037)X_I

fj

order

.
11m

a_
are of
is an

tan
()

--

smallness. Hint.

of

a
a-sin
3
a

same order;

the

I;)

X)

Replacearc cos(I-x)by arc sin

third

It

(f) e J; (g)

2)

a-\037.)

el\037X

1.10.14.
(a) Y2.Hint.
is of the
1.11.5.
(b)

1.11.6.
(b) They
1.11.8.
(a) 100x
an
sirna

Hint.

e'1.X-e\037X'

ltm

x-o

1
(d ) e3;

(c) 2 1n a;

Y2x-x2; (b) I; (c)a.

.)

(c) they are

equivalent.
sarne order as x;
(b) x2 is an
order
I of
infinite
x; (c) 6 sin x is an infinitesimal of the
of an order higher than x;
sarne order as x;
(d) sin 3 x is an infinitesimal
l
of
of
an
order
tesimal
tan
IS
an
infini
x
smallne\037s
lower than x.
(e)
(a) It is of the fourt-h order of smallness;
(b) of the first order of
smallness;
(c) of the third order of smallness; (d) of the third order of
smallness;
(e) of the first order of smallness; (f) of the order of smalJness
of the

infinitesimat
higher than

V
1.11.9.

2;of the(g)

(i)

of

the

-t;/
1.11.

cosx

order of smallness; (h) of


of smallness. Hint. Multiply

first

order

\037econd

the first order of smallness;


and divide the difference

cosx by

im perfect trinomial square;


(j) of the first order of smallness.
t O. The diagonal d is of the first order of sma llness; the area
is of
the second order of smallness; the volume V is of the third order of smallness.

1.12.3.
(b) 4;

(f)

1.12.7.
(a) I;
(f)

3;

(g)

(b)

(g) -2;
4;
1.13.1.(b)

\"3'

(h)
f (1

-l.

\037;

(i)

1.12.6.
(a) I;

2.

1.12.8.
(a) '5;

4
(b) 5\";

(b)

2.

(c)-i;(d) 2\"; (e)g;

1.12.9.
Hint. 1042=103 X (I +0.042).
10.14.
-2, (1+0) = 2; (f) 1(2 0) = 00;)
( +0) ==0; (b) 1 (-0) 0,) f(+O)==+oo;)

0) ==

- -

(2+0)=+ co.
.
1.13.3.
(a) (-0)= 2t
1 ( +0) = l.
(c) 1 (-0)= -1,
of the first kind at the point
1.14.2.
(b) The function has a discontinuity
x = 3. The jump is equal to 27.
1.14.3.
has
(e) the function
(c) The function is continuous everywhere;
a discontinuity of the first kind at the point x=O; the jump equals 1t. Hint.
1t
1t
arc tan (-00) = 2\"'arc tan ( + 00) = +2 .
1.14.6.
(b) At the point xo=5 there is a discontinuity of the first kind:
= 0, discontinuity of the
=
, (5+0) = ; (c) at the point
(5 0)
f

first

- -;
kind:

continuity

f(+O)==O; (d) at
1(-0)=1,
of the

1.14.7.
(a) At

the discontinuity

==

second kind:)

Xo

the point

xo =

;'

1( -0)=+00'1( +0)=-00.

an

infinite

the point x=O there is a removable discontinuity.


it is sufficient to redefine the function, putting f (0) ==

dis-

To remove
1;

(b)

at)))

Ch.

to

Answers and flints

423)

I)

To remove the discontinuity


the point x=--=O there is a removable discontinuity.
it is sufficient to extend the function putting f (0) I;
(c) at the point x=O
there is a discontinuity of the second kind: f
1
+ 00; (d) at

(-0)=0, (+0)=

x=(2kr-1) ;: (k=O,

the points

::f:

I, ::f:2,

.
. .)
1 (x)= 11m (slnx)-n =

(e) at

if

{I

removable discontinuities,
sin x
.
sin x

..
'-I
...

n\037oc

1f

(x) =

Sn x
i.
SIn x

i f sin

x=n==O,::f: I, ::f:2,

(f) at the points

(x) =

1.14.8.

-Io

x=

sin

if

since

< I,

= I .,

x=kn(k=--O.:f:I, :f:2, .), discontinuities

the points

si nce)

.),

\"

of the 1st kind,

x > 0,
x < 0;

removable discontinuities,

since

x n,
x 1=, n.

\037f

1f

I there is an infinite
of the second
(a) At the point
discontinuity
a discontinuity of the first kind (the jump
kind;
(b) at the point
of the second
being equal to 2); (c) at the point x=O, an infinite discontinuity
I, a discontinuity of the first kind (the jurnp being equal
kind, at the point
to
I, an infinite discontinuity of the second kind.
(d) at the point

x=-2,

x=

-4);

x=

3
1.14.9.
(a) f(O)=I; (b) 1(0)=-2\";(c) 1(0)=\"2;
(d) 1(0)=2.
1.15.2.
is
function
on
the
The
continuous
interval
(b)
(0, + 00).
1.15.3.
(b) The function is continuous everywhere. At the only possible point
of discontinuity x = 0 we have
lim
y==-limu'!.=I;
y= limu 2 =1; ylx=o=ylu==-t=L
u-+l
.t'-++O
there are removab Ie d is1, 2,
(c) at the points x = ;:+ nn (n = 0, :I:
I

Urn

\037\037-o

U\037-l)

...)

::f:

continuities,

since

liIn

'1=-1.
y= U lim:!::
31

X--+2

00

No. For instance,


1.16.2.
Yes. 1.16.12.
1.53. 1.16.13.
l-I,
I].
1.17.1.
(a) Hint. 1\\\\ultiply the obvious inequalities:
n
r I.n< +
2 ;

on the interval

.r-

the function

y=x'J

............
rr

\"1

1rr

(b) Hint.

Let)))

-2
I

B
Then A

< B since

X
---=

..)

\037\037)

-I-I

(n-I).2< \037;

1rn.1< n+
\037.
I

r
5

6\"

2
3

2n-1<
2n

2(n-l)< n 4-

-4 X
X

2n

2n+ 1

...

2n

...

\302\267

\037n

\"\"7

and

A'2

2n

< AB =--

2n
-j-

I
I

2n+

')

424)

and

Answers and Hints)

1.17.2.
(a) Hint.

Extract

IOlst

the

reduce both sides by 1012

(b)

the obvious

Multiply

from both

root

sidesof

the inequality

inequalities:

99 X 101< I 00 ,
\037

98 X 102<

I X

100:'>',)

2 X 198< 1002,
100X 199X 200 < 1004

5
1.17.3.
(a) -3< < -lor < x < 3; (b) x < -3\"or x > \"3 ; (c) the
sol u tions, since it is equi va]en t to the con tr ad ictory system
ineq ua it has no
Yes. 1.17.5.
x-2
> 0, x(4x2 -x+4)< O. 1.17.4.
(a) No; (b) Yes.
Hint.
the method of mathematical
1.17.7.
induction.
At n = the
I

relation

\"

... -

Apply
Supposing

is obvious.
(1

+Xl) (1+x2)

that

the inequality

(1+ xn d

\037

+ Xl + x2 + ... +xn -}

...,

both its sides by +xn and take into consideration the


0, xi,xn > 0 (i= I, 2,
n-l). (n=O, 1,2,
+x
1.17.8.
(a) [I, +00);
(b) (2nj[):'>'\037x\037(2n+l)\037j(\037
:1:2, (d) (-00,0)for f(x); g(x) is nowhere defined;
(c) x=O,:1:1,

holds true, multiply


conditions 1
n >

(e)

...;

:1:2,
...).
[-4,-2]or [2, 4J; (f) x=(2n+I)T(n=O, :1:1,

1.17.9.
(a) No:

...);

j[

I, and t (0) is not defined;


(b) No: t (x) is defined
cp (0)
for all x
0, and cp (x) only for x > 0; (c) No: t (x) is defined for all x, and
(e) No: t (x) is defined only for x > 2, and cp (x)
(J) (x) on I y for x \037 0; (d) Yes;
for x > 2 and for x <
V=8Jt
3 < x < 6.
00).
(a) (0, (0); (b)
(a) x 5. Hint. The domai n of defin it ion is specified by the inequ a\\vhich are fulfilled
lities x-+- 2 \037 0,
\037 0,
0,
only at the point
that the number
satisfies the given inequality. (b) Hint.
Verify
The domain of definition is specified by the contradictory inequalities
> 0;
i=-

I.

1.17.10. =
1.17.12.

[1,

x-5;.?:5-xx=5

x=5.

1.17.11. (x-3)(6-x),

2-x> O.

2
1.17.17.
(a) (x)=
+x2 + + x 2 ;
blem

(b) aX =

1.17.16).
An even extension
1.17.18.
<p(x)=

x-3

aX

(seePro+ aX -a->':
2

-1-aX

defines the function)


2
t (x) ==x

+x for

\037

x \037

3,

{ f(-x)=x2-xfor-3\037x<0.

defines the function

An odd extension

f(X)=X2+Xfor

0\037x\0373,

2
+x for -3\037x<0.
{ -f(-x)=-x
If
.21.
functi
on
has
a period T1, and the function
1.17 flint. the
t (x)
(x)
has a period T2' and T] = nld, T2 n 2d (nl, n'!, positive integers), then the period of the sum and the product of these functions will be T = nd, where n is

(X)=

'\\(J

cp

==

the

least common

1.17.22.
Hint.

nntltiple

For

But there

is

no

least

any

ot the numbers
rational number

(x) ==

(x + r) ==

number

in the

n} and n2'

for rational x,
irrational x.

{ 0 for

set

of positive

rational

numbers.)))

.23. Hint.
1.17
f (0)= f
\037

whence sin

425)

I)

denote the perioc1 of the function


we get
sin
sin + cosaT I
T) cos

(-T)T

(T)

Ch.

to

Answers and Hints


If

\\\\lC

(-

= =

T = 0, cosaT= I,

(x)

+ (-aT),

2;

of two increasing
tonic function. For example, the functions
x \037 0, but their difference f (x)
(x)

-g

and

[0,+]
1.17.26.
Example:)

==

+y
1.17.27.
(a) x=2ln
I-y

(b)
for

(y _
Yy

x= {

co < Y

\037

00).

\037

<

I);)

!J)

< I,

qCY

\037

\037

for

Hin!.
1.17.28.
+
(x \037-I)andY=- I+YX\"
+2x+1
inverse, but the
0) are mutually
\037

e.x2+2x+=x

1
has
equation y==x, i.
no real roots (seeProblem
If E is the dornain
t7
(c) H
of the function t (x),
of definition
then the function y== f [f (x)] is defined only for those xEE for \\vhich
f (x) E E. How the points of the desired
graph are plotted is sho\\\\rn in Fig. 120.
Hint. The quantity
and f
symmetry f (a-t-x)==t

1.4.4).

I. .30. in!.

1.17.32.

\037/

16,
Y
16 < Y < co.
Thefunctions y=x2

for

( log2 Y

(x

x'\037

x is rational,
if x is irrational.

X if

(-1<

x-x

decreaseson
y

then from

hence T = kn, aT= 2nn, a = is rational.


functions is not necessari
ly a monoincreasefor
f (x) = x and g (x) =
= 2 is not monotonic for x 0: it

and

The difference
1.17.25.

increaseson

T,

by

/)

/
/
/

o)

/)

x)

x)

f(x))

Fig.

T=2

120)

(b-a) period:
(a-x) (b+x)=f
(b-x) it follows that
==
[x+2(b.-a)] [b+ (b+x-2a)] (2a-x)= [a+(a-x)]= (x).
1.17.33.
(a) It diverges; (b) it may either converge or diverge. Examples:
'. = [ + (-I)n]: lirn
0,
n
==

=1

Xfl

Yn

Yn ==

n 2;

lirn
n

Example: xn==n;

(xnYn)

==

.....

(xnYn)

==

00.

00)

..Yn==-n+l;

(b) No.

an = n (n-2)
1.17.35.
Hint. Take
(n=3, 4, .). 1.17.36.
n
The
converseis
x
a
that
incorrect. Example:
xn-aI.
II n I-I II
Hint.
1.17.38.

0, I,

...,

number

\037

The sequencean

of

n.....oo)

n)

1.17.34.
(a) No.

from the conditions

==

is

rnto
Xn

==

account

(-I)n+1.

attain
only the following values:
9. If this sequenceturned out to be monotonic, then the irrational
would be represented by a periodic decimal fraction.

Ifin!.
1.17.39.

at'

bi

-'

If

the

< ban +1
n+l)))

sequence
\037n

may

increases,then

i.e b 1a,.<a I)i(i=1,2, ...,n),


ll +

n+1

Answers and Hints)

426)

+...

whence it follows that


bn +1(a1
and hence
Ul
U

+a2+ ...+an + I
+b2-t- ...+ bn +

+a2 +an ) < an+l (bl +b2


al +a2 + .+an
b 1+b2 +...+b
n

1.17.40.
(a)

nx-

\037

Hint.
1.17.42.

nx

follows

n)

n +l )

(bI

x--iE

From the

Hint.

x-

<

(nx)

<

n)

\037

> O.

inequalities

x.

From the inequalities)


n

(kx-1)

\037

\037

k=l
it follows

n ),)

+...+b+bn)-bn+1(al
+a +...
+a
+b +...
+...
+b
1.17.41.

an + 1 (b1 +b 2
(bl +b 2
0;
(c) O.
(b)
.It
that

2\037

< E (nx)

.. -

1)

+...+b

\037

k=1

E (kx)

n+

2n

kx,

\037

k=l)

x---\0372L

that)

\037

n+
E (kx)\037x2n
I

\302\267

k=1)

1)

Hint.
1.17.43.

Take advantage

of the fact that

an

lim
n

-jo

V;=

lim

00

fl

-\037

;x;)

(see

1)

lim a
1.6.19),
n

Problem

oc

-jo

1)

qualities

the

fI-a

lim
-jo

00)

= I,

-1< ah-l< an -1

take

expression by

1.17.47.
(a))

place.

for

:j:=
\037

Hint.
1.17.48.

-jo

for

In

2)))

(n=O, :I:1,

\037

the me-

-jo

of the identity
x2
x2n

)=

To find

the coefficient

..

:i:2, .).

1-x .
2n

one can't. For example,

(1

m
by x

x=2\"+nn

In

x-x=0.
x

+nn,

speaking,

+x)+
(I-x)=
2

replaceIn (I +x)
0

<

\037

(1-x) (I +x) (1+ ). ..(I +

lim

Take advantage

1.17.49.
Generally

lim

f(x)=

result:

pass over to the limit.


x 1,
I for 0
(x) = x for x > 1.)

(b))

we

and

\037

if

> I,

I)

Hint. Divide the numerator and denominator


1.17.45.
1.17.46.
(a) a= I; b=-I; (b) a= I; b=i-.Hint.

divide

and

and for

by

and In

In

lim

-jo

(1-x)

by

2)
(l-x
=_
x2

I ')

we will

get the wrong

1.17.50.
-}.Hint.
then

sideration,
R

(1-cosa) -

su btended

angle

by the

-- Ra,

to 2R sin

equal

427)

I)

and

arc

under

con-

the

sagitta

to

\037

R\"2')

The difference
is equal to)

perimeters of a circumscribed and

of the

regular n-gons
2R n

where

is

the cr.ord
a2

1.17.51.
2. flint.

inscribed

a is a central

If

Ch.

to

Answers and Hints

j[
and
a==-,

. j( = j[
2
an--Sln(
)
J1

the

side of

tan

an inscribed

1.17.52.
On the

sin

equivalence of

-j[ a,
R 2

is

2Ra.

f\"OoJ

and
(l+a)3-1
x

In(l+x)
1.17.53.
No, 10g(l+x)= InlO

1.17.54.
=

sin

n-gon

!!:...
==2R si
n)

a)

n)

2R

3a as a-+O.

- lnlO asx\037O.

is continuous at
(a) Yes. Hint. If the function cp(x)==f(x)+g(x)
the point x xo, then the function g (x) ==cp (x) f (x) is also continuous at this
both
(see Problem
point; (b) No. Example:
functions are discontinuous
at the point
0, and their sum is identically
equa 1 to zero, and is, hence, continuous.

1.5.11

f(x)==-g(x)=signx

x=

(p\302\273;

1.17.55.
(a) No. Example: f (x)=xis continuous everywhere, and g(x)=sin\037
x
for x:j::
0, g (0)=0being discontinuous at the point x=O.The product of these
functions

is

pie: f(x)=-g(x) == J
point

x=0,

\\

their

for

x\037O

'

since lim x sin


X-'O

both

-1
=

or x < o;
(x) g (x) ==

functions

(x)

2A

where

of the first kind;

(1 0) ==1; (c)

cp

(x) is

(a)

lim

x...n-O

I,

lim
x-..n+O y

...)

I, :1:2,
(n=:I:

are

li m

The function

-I

(x)

is even;

== 0; (b) No. Exam-

discontinuous

(x) is the Dirichlet function


1
of the
a
kind,
discontinuity
of the first kind:
I is a discontinuity
(b)
discontinuous at all points exceptx ==O.
of the first
are discontinuities
1,
O.The function has a period of (b) x
y Ix=n

y==2n-l;

li\037

at the

wnte
. 1.14.4
Problem
(see
x= is discontisecond
(b\302\273.

=0,
{(I-O)

I;

kind:

= :I:yli

of the first kind:)

points of discontinuity

x_Vn-o
discontinuity
value I to

x=O is a

(a)

Example:

are

Jt

being continuous everywhere.


I if x is rational,
We may
. . .
.
I 1f X 1S 1rra t lona 1

{= (x)-I,
1.17.57.
x=
+
1.17.58.x=n=O, :I: :l:2,..,
=
=
=

nuity
f

product

1.17.56.
No.

at x==0

continuous

function

Y=Y'x=Vn=2n.

x-.Vn+o)

(c)

...)

x=:l:Yn (n=:I:
I, :1:2, are the points of
passesover from the

of the first kind; at these points the function


and returns to
TMe function is even;)))

1.

428)

Answers and Hints)

-},i.e.- +:nn< <


= -},i.e.x=:!: +nn.

(d))

(x

if

sin x

<

\037

if
\037

\037

sin

. sIn. x

If

\037

+nn,

\037

5n
> \"2' I.e.\"'6 + nn < x < (3+nn.
.

J[

x=:I: +nn are discontinuities of the first kind.


The function
1.17.59.
[g (x) has d iscon tinui ties of the first kind at the
0; + I. The function g [I (x)] is continuous everywhere. Hint.
points x = -1;
The function 1 (u) is discontinuous at u = 0, and the function g (x) changes sign
1.The function g [I (x) ==0, since (x) attains only the
at the points x = 0, :I:
I.
val u es 0, :i:
Hint. W rite the function in the form
1.17.61.
\037.

x+ I

(x) =

and from

the

\037

for

20

{ (x + 1)2
Make sure that

-2

for

x=O:f (-0)=I, f (+0)=0.


e
1.17.62.
Hint.
consider that)

Suppose

e\037 min

Choosethe

points Xl and

> is given

I (X 2)

(x)-x.
Hint. Apply
1.17.64.
xn ],

...,
1.17.65.

the in terva

[I (Xl)'

min

=2

[Xl'

is

r-2,0)

value theorem

discontinuous

at the point

is chosen.We

that

= f (xo) +e,

g (x)

on the intrrval

(b)-I(xo)].

[f (xo)-f(a), f
x2, Xl < Xo < x2 so

f (Xl) = f (Xo) -e,

6 = min

and the point '\\oEla, b]

\302\260

(XO-Xl' 2-xo).
Hint.
1.17.63.
Apply the intermediate
=

and put

to

the intermediate

The function
[-2,-I]and [0,21.

\037

2.

\037

-I

on the interval

to

to the intervals

may

0<x

tor

increasesfrom
[0, 2].Apply

function

< 0,
x=O,

the intermediate
noting that

(xn )] \037 \037


n [f (Xl)

Hint.

Hint.
1.17.66.
\037

theorem

value theorem

f (X 2)

+ '\" +f (xn )]

\037

value theorem

Apply the intermediate

on the interval

value

to

the

to the function
max

function
f

(x) on

..., )].=

If (Xl),
to the function

(X n

(x)

[-{-,I].

At sufficiently
large values of the independent variable the
va ues of the polynom ial of an even degreehave the same sign as the coefficient
at the superior po\\ver of x; therefore the polynomial changes sign at least twice.
Hint. The inverse function
I

1.17.67.

-\302\245-y-l

x:...=

{
is

continuous

y=O.)))

in the intervals

\302\245

y-

y<-I,
=

for
for y
for y

0,

>

I)

(-00,-1)and (1, 00) and

has

one isolated

point

Answers and Hints

C}Iapte r I

to

Ch. II)

429)

I)

20
2.1.2.(b) y'=IOx-2. 2.1.5. =25 m/sec.
2.1.1.(b) -21'
The function is non-differentiable at
2.1.6.
(a) y' 3x2; (b) y' == x23 ' 2.1.7.
v av

==

t he in d i cated poi n t

2.2.1.
(b) y' ==

s.

-5

4
ax 3 + bx

3\"

--3.
7

2.2.2.
(c) y' =

= 2x arc tan x + 1. 2.2.3.


(a) y' = 6x2 +3; (b) y' =
(b) -9000.2.2.4.
2 x
2 +2x+2
3
-3x
vT7X .,
Vi-+8Vx+2
.
9. ()
c Y'= 2
(d) Y'=_
+x,
(x -x+l)2'
2x Vx
6(x-2Vx)
cosq' sin
x cosx;
x
,
;
y'=2e
y'=2e
(h) y'=
(g)
(f)
(e) y'== (1x
cos
\037_

- - - -

x (cosx

+-;
1

cp

<pr\"

sin x

sin x)

eX

x 2eX
1
3
.
(f) 30In 4 (tan x) SIn
6x ;

2.2.5.

-3(3-

2.2.6.) (b) y' ==


(d)

y
1

1r+

_ sec2 r

- 2Yx+

x;

YI

(f)

X)2

cos

(c)

x;)

'
b
2.2.8.()
tj

--

.
SInh 2

.
-_ v 2 -

2.

sec2 x+ cosh 2 1cot x cosecx,


(t a n x)
(
)

.13.(a) f' (x) =}(COSh

= V cosh x+ 1;

(cash bx +sin h

2.2.14.
(a) y'
y

(c) Y

, ==

==

\037

+sinh

\037

--

,.

3X)4

\302\267

3(x-l)2lx-t- 2)3 (x +3)'2)

e (acosh ax+b).

- l-x-

);= 4(b) l'

x-

(cosx)sin x (COSx In cos


cos3x

--

+)))
\037

tan

'9+x2'

\037

(d)

3x X

bx

( 3x

\037
; (e) f' (x)
(0) f' (x)==cos
hx
a+
b)x
==
bx) (a b) e(

v sin 2 5x3x2(1+x sin24


;j

esi nh ax

+ x+ ) .

2.2

(b)

sin
\037

\037
(

!I'= 3cos3x

sin x
+ 2cos
3X;

a
a
(h) y'=
tj'==(2x-5)cos(x2-5x+l)-2sec2-;
x
x

2
2
3
X (x si nh 2x + coshx sinh 2x ); (e) y'
1
x
.
, 3 X
sIn 3xcos2 x
(c) Y
1 +X2
(x + t )
1
1
,
1n tan x
(d) y == (t an x)
sin 2x
2\"

. x
3 SIn
V sin 2 x)

(e)

.,

\302\267

2(I-x)
2 cosx

y')

5 ln 4 x

ln 2 x

2.2.9.

3
2

x
+arc-Ian x + 1+x2; (1)11= 21narctan\"3'
arc tan

I-x)
y-=--

+.x'

sin

2ex 2X In 2
3 V/ (2eX 2x 1 )2

(g) sin

sinh

2x
3cot
1+X2

x-2

(x) = tanh x; (c)

4x; (f)

x sin x);

t anx

),

I'(x) =

I' (x) = (a+b)e

aX X

=.

Answers and Hints)

430)

'
2.2.17.
(a) y
y' = _

(b)

In

-1

V81x

Varc sin 3 2x
;
Varc sin 2x
SIn I n 3 x.I n 2 x

tan

. 3-

+ V cos2 1n3 x)V(arc tan V cosIn 3 X)2


2n -1sin
sin
(2x+n ) ; (f) + (x+n ) +

5x V eos4 1n3 x (1

2.3.1.(b)

knekx ;

(e)

;
...]

\037

+ 3; (3x+n ) + 5; (5x+n ).
n
I)
ax sin
2
aP
2.3.4.
(b) eX (x + 48x+551); (c) e
+
an-2\0372+
[
{
n
.
+
IJ \037;-2)an-3\0373+...]
}
[
2
2x2 +3x
x2
2.3.6.
3x..
; (b) (1+2x)arcsinx
(a)
2)2 ; (c) 2e- x
2. + (l-x
sin

sin

\037

CDS

nan-l\037

\037x

\037x

(2x

(sin
1

(l-x2)

2)

x-60x eosx-1140
x

x+8640eosx;
(b) 2e(n-l)4]; (d) (-I)n [(4n2+2n+

(a) x3 sin
x); (c)

x2) eos

(7\037

(\037\037

(1+X2) VI +X2

x - ).
2.3.8.
x+eos
x+ 2

\037

eX

[3x2 +6nx+3n

x].

4nx sin

sin

2.3.9.
(a) 100!
[(x-2)101 (X-l)l0lJ ;
1

(b)

IX3X5X...XI97X(399-x)
'
201

(I-x)

2100

2)

y=2 (I-x)-T-(I _x)2.


4 cosx
'
2.4.1.
(b) Xyy=(6+sin x)3

Hint.

\"

..

2.4.3.(b)

y\037==

2 4 4 (b ) y xx ==
3
\302\267

2.4.5.(b)

\"

y'\"
xxx

cot

k-l

\037t;

(d) y\037=-

4t

- --

\"

; (C) Yxx
I)3
+
= 3 sin t sec2 t.
(t 2

y
2.4.6.
(b) y'x =-+e x;
X

==

a t cos3 t

_2-x
y-

(c) y'x ==
x +Y)
Y)
(1+eY)3

-e (l-e

2e-2ct .

5;

.
(d) y'x ==-

V -.
'

,,4e-+

-; ...

== 4 (x+y)
1)3 (x+y+ 1)3

'
2a-2x-y; (b) x+y (c) _eXsiny+e-Ysinx.' ()
2.4.9.
d -\037 .
(a) x 2
xcosy+e- cosx
e
+ y- 2-a
2.
c
sin t
+
.
t.
!!J.
2 4 0. (a)
b
'
yo' (b) 256 2 4 II (a) a (b +cost) , ()\"2
2.4.7.(b)

..
+3

(c) 4/
(f)

Yxx=

Yxx= (eX-Y
Y

\302\267

2-t

(a CDS /

et2

2/(2/2+2/+1);(e)

(d)

.../1-4/
2 ;
V

(c)

eX

2y2

/2

(eX

\"

(g)

-YI-t .
2

2.5.1.
(b) 6x+2y-9=O;
2x-6y+37=0.)))

-b

sin /) cos3
\037

4'

SIn

2\

Answers and Hints

Ch. II)

to

431)

2
10
2
2.5.2.
'
5+ 3 10 ' M 2 ( yf ' 5-3 Y3
(c) M 1
y3\" )
,
)
( Y3
2.5.8.(b) x +y 2 = 0; y = x. \\
2.5.3.(b) = arc tan 2 y2 .
2
39
n
5
2.5.15.
(a) 4; (b) y=:.:I, x+2y-2=0; (c) Y+T6==-3\" x+ 4
);
(

cp

(d)

4' 2.5.16.11.

\302\267

t2
;
g2

s=at
2.5.17.
26,450. 2.5.19.

= '
v=a-gt;) Smax = S t=.!:..
;g
j

M
28
:::;: =
2.5.20.
v = r; =
2n;8 sin (I + cosM). 2.6.3. dy 0.05.
fJ.y

0.9942.
2.6.5.
log 10.21 1.009; (d) cot
= (I +tan 2 x)
2.6.7.(b)
(d)
(c) = cosx
3x
4Inx-4-ln
2.6.9.
dx'1;
(b) d2y== 2
(a) d2y=4-x22In4(2x2In4-I)dx2;
x
45\302\26010'

\037

\037y

...

\037x;

-4

x=

at

x4)
tan t, d2y=

Y(ln:.'.X-4)3

(b ) d2Y == _

x,2.

two

:
\037

nM3 is the volume contained

dV = 41tr 2

between

\037r;

\037r

\037r.

++

1/;

time
a velocity

\037t

v=gt
2.7.1.
(a) It

2.7.2.

the entire

during

interval

doesnot exist; (b)

Hint.

90\302\260.

(0) =

-I,

Since)

(0)=

2.7.3.f'- (a) = -f\037

1.

(a);
For x 0
::j::.

f\037

derivative

\037

(a) = cp (a).

the derivative

f' (x) =

x=O the

y= e x, X <
0,)
{

cp

2.7.4.Hint.

of time.

exists and equals zero.

it

eX,

At

x,

is the vol urne of a thin layer


spheres of radi i rand r
a base area equal to the sphere'ssurface area 4nr 2 and a height
fJ.s = gt M
g M2 is the distance covered by a body within the
=
=
ds gt
v dt is the distance covered by a body which would move at

2.6.
12.

f\037

4 (I 3x 4) 2 _ 4x
dX
d 2.
I _ x4
(I _ x4):.'.

dt 2

cos 2t

2.6.11.
fJ.v=4nr2M+4nrM2+
with

\037x'

\037y

sin 2x dx:i.
(c) d 3y =
4
3x4
2 6 I0 (a) d 2y = _ (I_ 2)
1
in particular

\037

cos

( ) +2x

sin

\037

( )

\037

equals zero:)
fJ.x

f'

(0) ==

2 sin
fJ.\037

HIn

Lx
f' (x) exists
x = O.
-+

==O.

\037x
0
for all x,

but has a discontinuity


of the
Thus, the derivative
point
b
Hint. The formula for the sum of a geometric progression represents an identity with respect to
Equating the derivatives of both sides of the identity, we get
nx n +1 (n
I) xn I
1 2x 3x:.'.
-1-nx n 1==

second kind at the


2.7.5.a = 2xo,

x\037.

2.7.7.

+ + + ...

- _+

(1 x)

x.

2)))

432)

and Hints)

An.,>wers

both

multiplying

sidesof

+ + ...+

1.2 22x

this

-1=

.
n 2x n

+-

by x and differentiating
2 ll (2n'.! 217 1)x ll
-f- I ) x

equality
x (n

+.
(l--X)'s

-+

again,
nx +

get

\\\\'e

ll

\037

2.7.8.sin x+3sin 3x+...+ (217 -1)sin (217-1)


x

\037

(217+1) sin

Hint. To prove

the identity

2.7.9.(a) sin

[I'(sin 2 x)
(x)
(c)
'I'(x)

x- (217-1)sin (217+I) x
(217-1)
2
4 sin

side by 2 sin

its left

x and

the

apply
To deduce the desired formula
2sinasinp=cos(a-p)-cos(a+B).
and
both sides of the
the derivatives.

formula
differentiate

2x

multiply

(b) ef

f' (cos2 x)];


I

'1\"

2.7.10.
(a)

equate

identity

<p'

\302\267

In

cp

(x)

cp

(x)
(x)

In
\302\267

ln\037

[ext' (eX)

(X)

'I'(x) .

+f'(x)

(eX)];

(x)

<p

No;
(c) Y es; (d ) No.
Hint. Differentiate the identity
or f
(x).
This fact is easily illustrated geometrically if \\ve take into consideration that
the graph of the even function is symmetrical about the y-axis, and the graph
of the odd function about the origin.
Hint. Differentiate the identity
f (x).
f (X-1-T)
x I.
The composite function
(x)==6x2
f [cp (x)] may be non-differentiable
only at points where <p' (x) does not exist
of
and where
(x) u at which f' (u) doesnot exist.
(x) attains such values
X 2 has a derivative
But the function y u 2
y' 0 at the point x==0,
x has no derivative.
though at this point the function u

2.7.11.

(b)

No;

2.7.12.
F'
2.7.13.

I(-x)=f(x) (-x)==-f
=
2.7.15.

. 2.7.14.
y'=21

= =

<p

<p

2.7.16.
(a) y\"=6Ixl;

xx'\"

at
(b) Y\"=2sin\037-\037cos\037--!,sin\037
x x
x
discontinuous at x == O.
that t lk ) =C\037 (k=O,
,.\" n) and lake

y\" (0) doesnot exist, since y' (x) is

2.7.17,
Hint.

(a)

Verify

I,

\037!

tage of the
show that
induction.

property
u\037

= (n-1)

2.7.18.
Hint.

-1- -

of the
Un

binonlial coefficients.
U n 2 and
use the

Apply the Leibniz

formula

x)

duct of the functions

==e

and v

= x2.

advan-

(b) Designate: t (x) ==

method

for the nth

at

x;tO,

of

un;
mathematical

derivative

of the pro-

= 2k

2.7.19. (0)= [IX3X...X(2k-I)]2


at n 2k+1
_
y(n)

Hint.

2.7.21.
Hint.
and

the

Differentiate

Leibniz

y(n)

the

identity

...).
n-2 times(k-I,2,
and, putting x==O, obtain

(0)=(n-2)2y(n-2) (0)

\037

2).

advantage of the definition


x2 H n +1(x)
X2
+ 1)
)(Tl

e-

= (e-

formula

for the

nth

= (-2xe-)(n)

\037=

of the product

\037

\302\245

4=:f:VI-Y
4xj

X2

derivative

2.7.22.y 3
.
(y2
I)
2.7.23.Xl .2=:f:VI+ l-y (-00< y\037
=
xj

(n

Take

v= -2x,

x3

17

I-

(/-xn

(O\037y\037

I),

I),

(i = I, 2, 3, 4) for

Xj

;i: 0,

:!:
I.)))

==

e-x2 and

AnstQ'ers and Hints

to

Ch. III)

-2x2 +y=0
x=:i:I.

flint. Solve the biquCldratic equation x 4


of dcfini t ion of the obtained funct ions xi (y).
x2 1; (b)
(a) Xl
Hint. Note that the function

=-3;

2.7.25.
2.7.26.

433)

and

find

the domains

X=2t- ={3::

t
has no derix 0,
f
X
therefore we can express y = t 2 t It 1=
vative at t = O. But t
x < 0,
_ 2t 2, t\037O, th roug h x. y _ 2X 2, x\037O, Th IS f unc t Ion IS d I ff erent la bi e eve.
{ 0, t < 0
{I0, x < O.
I
2.7.28.
2.7.27.
Hint. The curves intersect at the
rywhere.
a=c=4\";
=
Sinceat these points cosax = 0,
points \\vhere sin ax
l

\037\037\037.

x,

\037

=) :3'

\\

'

. . '

b=2'

I.

Y;=f' (x) sin ax+f (x)acosax=f'


(x)=y;,

the curves are tangent.


i.e.
2.7.30.
Hint. For t
reduced to the form:

the equations

. :j::. :rrn

of the tangent

and the

are

normal

t
y=-tan2l (x-at),
y=cot\"2(x-at)+2a;
t = n (2k I) (k = I, 2,
the tangent line (y = 2a)

...)
= ...)

touches
respect ively. For
the circleat the highest point, and the normal (x==at) passesthrough the highthe tangent line
est and lowest points; for t=2kn (k=O, 1,
passes through both points, and the normal (y 0) touches the circle at the lowest
2
/
2d q>
The relDtIve error
2.7.34.dd t Y2 +y.
The most
.
pOInt.
/
sIn 2cp
the result with the least relative error, corresponds to the
re1iable result,
value

2.7.35.

i.e.

(x=at)

6==\037

\037

(p=45\302\260.)

Chapter III)
3.1.2.
(b) Yes; (c) No, since the
3.1.7.

3.1.5.
\037=e-l.
the
functions)

(x) = arc si 11

g(x) =arcsin
7
3.1.15.
(a) \037=2;

(b)

derivative

since g

No,

is

2x

non-existent

(-3)=g (3). 3.1.9.


(d)

+x2 +2 arc tan x

for

2x
+X2

for

2
\037==

ln3

2 arc tan x
;

(c)

10

x
x

> 1,

<

I.

:i:Y52'. (d)
24

\037=

at the point O.
Hint. Consider

it

is

not

appli-

cable,since the function has no derivative at the point x=O.


3.1.16.
1.26< In (l +e) < 1.37.Hint. Write the Lagrange formula for the
function
f (x) == In x on the interval
[e, e+ 1] and estimate the right-hand side
in the obtained

relation:

In

(I + e) = 1

+i- (e <

\037

< e+ I).

3.1.17.
flint. Apply the
interval
[1, +x],x > 0,

Lagrange formula to the function f (x) == In x 011 the


Clnd estimate the right-hand
side in the obtained re-

lation

< < 1 +x).

In

(I +x)=

(I
\037

3.2.3.
(b) O. flint.

Represent

cot

3.2.1.(c) 2;

x-tCln
X--=--=---'
(c) _
2
1

x
x t8n x

')))

(d)

0;

(f)

-; .

3.2.5.
(b) e1=e.

434)

Answers and Hints)

3.2.6.
(a)
(f) 0;

4
I. 3.2.9.
(a) \"7 ;

1; (b)

(g) 1 ,

(n) 2\"; (0)

3.3.6.
V 83

(i)

-1

a2

2'

\037

a;

In

(h)

(p)

30

\037;

(c) 2;)
(k)

(q) I ;

(r)

11

(d)

-I;

\0373

(I)

e;

3.3.5.(b)

-2')
1

; (e)

;
\037

(m)

3
2

;)

0.34201.)

1)

2
V 83 = V 81+2 = 3 ( +Sf
)
.
terms.
1

the

Apply

and retain four


Hints. (b) Write the Maclaurin formula for the function
the remainder R4 (x);
(c) write the MClclaurin formula for the

3.3.7.

with

, (j)

a-I;
j()

nomial formula

In

_ m 2!!:....
2

,)

3.018350.Hint.

(b)

bi-

f(x)=tanx

function

+x) 2 with remainders R 2 (x) and R3 (x).


x2 x:
3.4.2.
(a) f(x)=2x2_3\"X3_5\"X5+0(X5);
(b) f(x)=x-2+6\"-12+
x5
+ 24
+0(x5).
3.4.3.
(b) -\"2; (c)
12; (d) 3\"; (e) 1.
2
5
x
x6
3.4.4.(a) 1+2x+x2-3\"x3-6\"x4-T5x5;
45 ; (c) 1-\"2+
(b)-\"2--T2+
4
x2
f

(x) ==(I

Xci

x\037

x-l

+ 12-720'

3.5.1.
(d) The function

(0. (0); (e) the

function

decreaseson
(
scaIe.)
and

\037

decreaseson
increaseson

. 3);

3.5.2.
(b) The function

(f) the

the interval
the intervals

the

intervals

(0.

\037

and

decreaseson

(
3.5.8.
(a) The function
\037

5:
).
increases

and

increaseson

(-00.+) and

increasesover

function

increaseson

(-00,0)

the entire

and

(3.+ (0)
number)

( 5:. 2:rt)

throughout the number scale; (b) the funcinterval


0) and decreaseson (0, 1); (c) the function
the number scale; (d) the function increaseson both
where it is defined;
intervals
0) and
(e) the function decreases
on the intervals (0, 1) and
e) and increaseson (e, (0); (f) the function
decreases on the intervals
1) and
(0), increases on
is f (I)=--=f
the
b\037
a\037O.
(b) The minimum

increaseson the
decreasesthroughout
tion

(-00,

(-I,

(0,00)
(I,

+
'(I,
(-00,
(-I, 1).
3.5.11.
3.6.1.
l.
3.5.10.
(3)=3,
=-;4 ' 3.6.2.
maximum
(b) The
(2)= 4; (d) the
( )
minimum

minima

\037

are f (:f:1)=Y3; the maximum f (0)= 2.


= 160;the minimum f (0)= 2.
(b) The maximum is f
3.6.7.
(b) The minimum is f(O)=O.
is f
the maxi(b) On the interval [0. 2:rt): the minimum
( )
3:rt
mum f
(a) The minimum is f (0)= 0, the maxi mum f (2) = 4e
2)
(
the max i mum f (2)= 1 ; (c) t he ma x imum
2) =
( b) t he ill in irn u m is f

3.6.3.
3.6.8.

(-2)

=4.3.6.10.

(- -1,

; =-4;

-\037;

i\037)))

Answers and Hints

I (0) = 0,

the minimum

r3.6.11.

7;
(O):=:
V 44.

the minimum
f (2)=

ximum

is

is

the maximum

(-3)==3 V3, the

(a) There is no extremum; (b) there is no extremum;


(c) the ma(0)==0; (d) the minimum is f (0)= O.
(c) The greatest value is f (1)=\037,
e the least value f (0)==0; (d) the

value

is

- :8'

(:I:; )

(:I:I) = O.

the least value t

The greatest val ue is y (0)= , the least value y


the
greatest value is y (4) = 6, the least value y (0)== O.
(c)
16
3.7.6.(a) The greatest value IS.
the least value
=
= 0; (c)
value
the
least
the
is
value
f (0) 2,
1 (:1:2)
greatest
(b)
=
=
In 3, the least value t (V3)
value is

t( ;3)

.
( )
is f (1)= 1, the least value
the least value is f (0)= 1.

value

value
value,

is

3.8.3.
H = R Y2, where
=

37

1(3)=-4;
the

greatest

the

3:rt
least value t
2)
(
f (2)= 2 (1
2); (f)

=-2;(e) the

-In

is

there

no

greatgreat-

H is the height of the cylinder, R is the radius of


0.5 arc tan 2.
sin ex, y a cosex, where ex
reduced to finding the greatest val ue of the function
2
S=4xy+4x(y-x)==4a2 (sin
ex)

in the

interval

=2 V

;:.

0<

P
3.8.8.

< \"4 .

==
at
4W.I
\302\2432

max

The radius
3.8.10.

is the radi us of the

. .X
IS\"2

2ex-sin

ex

x a
sphere. 3.8.7.
Hint. The problem is

the

;3

=3

\037

-0.25In 3;(d) the

+0.25

\037

(:I:

f(-2)=3'

est
est

minimum

3.7.2.(b)

greatest

-1,

==
(:1:2)

(d) the maximum is f

9\"

:1

3.7.1.
greatest

=-925 V T;
)

435)

fi

\"3

(e)

Ch. III)

to

expensesare expressedby

appropriate

3.8.14.
cp=

At

Hint.

t1;}e

+cos

trough
equal to a2 (1
walls to the bottom.

3.8.15.. Hint.

cp)

the tank
locnted,
the expression)

cpo

where

desired

straight

hours

to

\037

the formula
width

where

cp

T=

cover one

a+b.v3
v

knot.

The

\037+bV2.

a the cross-sectionalarea of
is the angle of inclination of
v

the
the

from
point of fall of the jet is at a distance of
is the height at which the orifice should be
therefore the length of the jet is determined
f10\\V;
by

The
base, where H
v is the rate of
\037

board
sin

'

\037

of the

a\037p.

\037

is

h ==2R=
3.8.9.

==Wi'

base is r =

of the cylinder

3.8.11.
The equation

conebase.

y
+\"4='
3.8.12.
x=a-pfor a>p and x=O for
3.8.13.
v=
V ;b . Hint. It will take

1Ille

=-==

'

\037

h-x

V- igx

J/I

(h-x)
g)))

Vx

(h-x).

436)

Answers and Hints)

3.8.16.
After

hours the least distance

2v

3.9.1.
(b) The intervals
xity

; , I);

ale (

(0, V3); the

);(e)

\037

(0, Xl)
points

Y3, 0)

and

3.9.5.
(a) The point
concavefor x > 3; (b)
is concavein

and

- -

( V3,

00,

(f) the intervals

3-JlS

xI=e

3+Y5
Y2 =
2

(0, 0),

),

\03703

(-

(V3:

3+V5

x2 =e

< 3 and

Y5-3
2

,)

arc sin

x=O;

(c) y=O; (d)

are

is (3, 3); the curve is convex for x


abscissaof the point of inflection x=arc sin

(Xl' X2), where


(X 2, Y2)' where)

- ;.

)3.10.1.

Y3)

of concavity

3+Y5\"

.)

of inflection

in-

the

the

conve-

of

\037

00,

(Y3,00), of convexity
are

km.

2\"

(I, 00),
)
(+, 12 ) , (I, 13); (c) the
(

is concaveeverywhere;

3-Y5

to

equal

\037\037

and (x2, 00), of convexity


of inflection are (Xl' YI),

the curve

are

of inflection

points

the curve

YI =

\037

the points of inflection

tervals of concavity
and

are

of concavity

be

will

\037-I;

r11

(i)

\037-I

) ,and

convex

in

(arc sin

x--++00JtX and

y=2x as

-1

5
\037

Y=

-2xas

x-+-oo. 3.10.3.
(a)x==3,y=x-3; (b)Y=:l:T-1;(c)y=x;
.
(d) = :I:2; (e) = 2x
X

3.11.2.

Jt

\"2

(a) The function is defined everywhere, it is even. The graph is symis Y (0) --= I,
metrical about the y-axis and has no asymptotes. The minimum
maxima

(-I)= . The points

(:I:
>
+
-1 -(- -I) (-I,y=x-3.
.
(-4)
(-6,(+
):

function
asymptote
maximum

.'

(2,

\037\037

has

3
are
,
(b) the
in
and
has
The
a
vertical
00,
00).
graph
The minimum is y (0)=0,
x= and an inclined asymptote
3 6
=
The points of inflection are
and
y
12;5 )
00, 0) and (0, 00). The graph
(c) the function is defined in

(I) = y
is defined

asymptote

V2
(1,

inflection

_I

is

is defined

the function

and
00); it
vertical aSYlnptotes

\037\037

( )

=3.The point

\037

2'

.s ( '1rY3
r 3) = -r3

x=O.The minimum

(----y'0); is(d)odd. The

(-1,1)
t\\VO

\037

22\0376

vertical

lion IS

of inflection

\037

is (0, 0);

is

in the intervals

of inflec.

(-00. -1),

about the

has

origin,
symmetrical
all inclined (1:-;ymptote y=x. The mil1illlllJn
x=:i: and graph
1

th e

111 Xl 111tI n1

(e) the

functio\037l

3V 3
r
( 1 3) =
2
is defined every\\vhere,

_,1

'

Tl1 e po
it

11t

is evcn.

Tile)))

graph is

\037yrnrnetrica

minimum

is

Answers and Hints

to

y-axis and

has

abou t the

Ch. III)
a

horizontal

y\"2) = 2
(0) = V.\"4, the maxima y
2, V4); (f) the function is defined in

(:I:

437)

asymptote y O.The
The points of inflec-

V2.

(-2, + 00).The vertical


(:I:
asymptote is x= -2.The minimum is y (0)=0,the maximum y (-0.73)\0370.12.
The point of inflection is (-0.37;
0.075);(g) the function is defined every\\vhere.
=
The horizontal asymptote is y= 0 as x -++ 00.The maximum is
( ) ( :e
are

tion

4' (

The points of inflection

3-y3 3-4

y= 1.The
,

eV3-3 ,

4'

\302\267

3+V3 3+Y3 3 e-3-V3.,


4

is defined and continuous everywhere. The horizontal asymptote


minimum
is y (0) ==0, the point (0, 0) being a corner point on the

(h) the function

is

are (0, 0),

Y\"3

11, +2'j[
-2)
-

==
y- (0) ==
y+ (0)
==
3.12.6.
4.4934. 3.12.8.
0.6705.
2.330;x2 = 0.202;x3 = 2.128.
Xl
3.12.12.
1.325.
(a) 0.27;2.25; (b) 0.21. 3.12.13.
(a) 1.17;
(b) 3.07. 3.12.14.
0.5896and 2.2805.Hint. To approximate the smaller root more precisely
3.12.15.

graph:

3.12.11.

1, to find a more accurate value of the


8X
write the equation in the form x=eO.
larger root represent it in the form
(I In x).
No. Hint.
Show that at the point x== I the derivative is non-exis,
,
tent:
(1)== I; 1+(1)
a b
Hint. Check the equality t (b) t (a) (b
f'

3.13.1.

x= 1.25 +

= 1.
1= -a)
3.13.2.
).
(
3.13.3.
Hint. Apply the Rolle theorem to the function I (x) =aoXn
on the interval [0, xo].
...+an-l(x)
3.13.4.
Hint. Make sure that the derivative I' (x) ==4 (x3 -1)has only one
real root, x= I, and apply the Rolle theorem.
3.13.5.
Hint. The derivative I' (x) nx n -1+ p has only one real root at an
even n and not more than two real roots at an odd n.
is a polynomial of the third degreeand has
3.13.6.
flint. The derivative

+...

==

roots. Take advantage of the fact that between


lies the root of its derivative.
3.13.7.
Hint. From the correct equality lim cos

three

the

roots

of the polyno-

mial

determined

the mean value theorem, it

frorn

-i-=o(0 <

< x), where

\037

doesnot

follow that

lim

enceof values E

for which

is

\037

\037

-=0,

cos x1

x_ 0
since it cannot be asserted that the variable attains all intermediate
in the neighbourhood
of zero as x
O. Moreover, takes on onl y such

-+

\037

\037

values

a sequ-

(1;EE).

!im

cost=o
3.13.8.
The mist ake is that in the Lagrange form ula one and the saine
H in!.
and
is
for
taken
I (x)
(x).
point
3.13.9.
Hint. Apply the Lagrange formula to the function In x on the interval
[b, a]; (b) apply the Lagrange fornlula to the function zP on the interval [y, x].
3.13.10.
Hint. With
the aid of the Leibniz formula ascertain that tile
cp

\037

coefficients
powers of
x < O.

of

the Chebyshev-Laguerrc
polynomial
negative coefficients. \\Vhcnce

having

alternate

deduce

in sign, the odd


Lll (x) > 0 for

that

3.13.11.
liint. Us.ing the Rolle theorem, show
inside the interval
o, x !
least rcots of the
derivative, n-l roots of the second derion.
that

there are at
vative, and

3.13.12.
Hint.

tive\037

I.x

ll

first

\037;o

of both the

The L'I-Iospitalrule
llutnCf(}tor

81)(.1

is 110t C1pplicablc here, since the derivavall ish at all points \\vhcre the)))

dCl10ntil1<ltor

Answers and Hints)

438)

factor sin

(which we

tives) vanishes.

Hint.
3.13.13.

the Taylor

Write

the limit

computing
formula
h2

the remainder R 2
h3
f\"' (a -r.-81h

with
f\"

- -

the expansion

with

it

(a)
+
(a+81h)
31'\"
h
Hint. Prove by using
3.13.14.
f\"

are

where p and q
get for n > p

natural

).

given in the problem,


and pass over to the limit
rule of

the

numbers,

get

the

as

h ----+

equality

O.

e=!!...,

contraries. Suppose that

q
using the Taylor formula,

1,and,

>q >

of deriva-

ot the ratio

(a+h)==f(a)+hf'(a) +21 (a)+3f

Comparing
f\" (a
8h)

cancelledin

P
q=I+IJ+2f+'.'+ + (n+l)! ( q) (0<8<1).
p

n!

sides of

both

Multiply

this

equality

and

nt,

by

are positive integers


(I+\037+...+) n!result.
obtain a contradictory

and
n\0371

\037!

Hint.
3.13.15.
f

(x) = J
'\\

x'

sin

Ascertain

< x';;;;
' is continuous
2\"

in!.
3.13.17.
13..

3
2

x ==

points

origin.

2nn

')

' then

i.e.the

Ascertain

the function

is

\037

derivative

that

retains

sin

cos x

\037

(n

(2n \037 I):n;

< I,

].

changes

all

its sign
constant.

sign

function

...),

in any

and to

vicinity

oppo-

at
\037

of

the)

IPl'p(x)==f(x)-cp(x)

points of the dornain of definition of the


if ad-be==O,
1= O. But

ad-be
3.13.21.
p=-6,q= 14.

if

has

(x i= 0)

= 0, :!:
I, :!:
2,

the auxiliary

Make sure that at

A minimum
3.13.22.
\037

=-2 + 2x

at the poin ts x

the derivative

function
\037

\\V

increases.
2 increasesand

xeX

I).

f' (x)

3.13.19.
flint.
Hint.
3.13.20.

creases.

\302\267

and

t'

and take advantage of the fact that the function


Show that the function f (x) ==
H
site signs at the end-points of the interval (0,
3. 18 Hint. Sh 0 t hat t he der iva t i ve)

the

\037

that the derivative f' (x) < 0 is inside the interval.


Hint. Show that
(x) \037 o. Ascertain that
0 for a < I,
a >
f (0) ==I
< 0 for a > 1,

equal to

l0,

on the interval

x==O

3.13.16.

is

r < nll

j(

\302\260

1,

\\

\037

that the function

Verify

E n!

that

noting,

i.e.

f (xo) == 0 if cp (xo) > 0 and n is even; a maximum


and n is even; the point Xo is not an extremum if n
is odo. Hint. At an even n, in a certain neighbourhood of the point Xo the function retains its sign and is either rigorously greater than zero or rigorously less
tIian zero, depending on the sign of cp (xo
At an odd n the function changes
sign in a certain neighbourhood of the point

(xo) == 0

if

cp

(xo)

<0

)'

xo.)))

Answers and Hints

3.13.23.
Hint.

to

Ch.

I.

439)

IV)

For x:l= 0 (x) > 0, hence


>
(0) is minImum.
. S
. ' at
.
.
.
'
t
x=IS
e
t Ive
e denvat Ive (x)=2 -sln-+-cosx x
x
2
The case x < 0 is investigated anaand negative at the points x=
'
I)
f

tll

3.13.24.
(a) and

11

(2n\037
I

0; (b)

For x

th

pOSI

pOln

-2.

:rtn

and
the greatest value equals
(b)
(a) The least value is non-existent,
the function has neither the greatest, nor the lcast value.
Yes. Hint. SinceI\" (x) changes sign \\\\Then passing through the point Xo,
the latter is a point of extremum for the function f' (x).
The graph passesthrough the point M
2) and has a tangent
line
1); M is a point of inflection, the curve being concavedo\\vnward to the left of the point M, and upward to the right of it. Hint. The funcx
tion
increases and changes sign when
t\" (x)
through
passing
logously.

3.J3.25.

3.13.30.
3.13.31.
y-2= (x+

I;

(-1,

3.13.32.
h=
.
(J y_2
3.13.33.
Hint. According
I

-I.

to the Rolle theorem, between the roots of the first


derivative
there is at least one root of the second derivative. When passing
through one of these roots the second derivative must change sign.
2
Hint. The polynomial has the form
n tx
n.
Polynomials of this form with positive coefficients have no real roots.
Hint. Take advantage of the fact that a polynomial of an odd degree
(and, hence, also its second derivative) has at least one real root and changes
sign at least once.
2x4 x:J I
lim
Hint. Find
x _ 00 x:J _x --)I

3.13.35.
3.13.36.

+a - +a
aox2n+alx2n-2+...

3.13.37.

C hap t e r

-+

C)

+ .
)

IV

1=x3 +x2 +O.51n12x-11


4.1.2.
+C.

..!.

2..!.

2
4.1.7.1=3
(x-I-I)2 +3x 2 +C.Hint.

..

Eliminate

the irrationality

denominator.

from the)

+ .
+ +C.
4.1.15.
2
4.1.18.
1=lnlx+3+Yx
+6x+I/+C.
V-TOx-yr
4.1.20.
1=
n
+C.
2 Y 70
VTOx+yf
- C., (c)) 3 tan x +)
3
4.I.21.(a) 2\" arc tan \037+c, (b) 4
(x 4) V x -!2
tan x-I-C.
+2 cot x+C; (d) --I-arc
x
4.1.
22. (a)
sin x--cosx+C;)))
(x -t- V +x 2) +arc sin x +C;
(b))
2
x+
x +C; (d) -0.2
52cos
sin
,5x-x 5a+C.
(c) -\\ii5
5 In 2
37
5
1-12 Jr(2x 5):1+\"2 Y2x 5 4 J1r2x-5)+C.
4.2.3.
4 I 14 1=

2x

arc t an
C
5\"
2
2x I
I== ..r_arctan
y_3
r 3
TO

I.In

x-3.

--

<

\037

440)

Answers and Hints)

4.2.8.I =-2V cosx+C. 4.2.10.


1=2(x3 +3x+1)3 +C.
4.2.13.
(a) 0.75 V (I + In
+C; (b) In Iin x 1+C; (c) 2\" arc sin
\037

;3+C;

X)4

-+C; (e) -2cos ..rrx-+C;

_ x+

xn
a

(d) na arc tan

4.2.14.
(a)

\037

(In

In2

In

( In x

+C.

+C;

x-5)V + Inx+C;

2. 2.
1

1/
r sin 3 x C;

sIn
sIn x
)
( 3\"-7 x+rr

(c)

(f) 2

3
14x2
140(35-40x+ ) (I-x)

32

(b)

-T5(8+4x + 3x 4) V I-x+C.
4.3.2.
x arc sin x+ Y l-x2 +C.
4.3.14. X In t an x+ In ta n (4-) +C.
4.3.17.x In (x+YI + x2)-YI +x:>'+C.
4.3.18.x Vx [(In X)2- n x+ n +C.
4.3.19.
2 Y +x arc sin x+4 Y l-x+C.
x
4.3.20.-0.5SIn
x \\
.
( 2 x+cot ) +C.
43213X(sinx+cosxln3)
C
I+(ln3)\037
4.3.22.
x+ I:) e3X +C.
(fX3_X2+
4
2
4.3.23.(x -IOx+21)
cosx+C.
sin x+x(4x2 -20)
I8x
-11
2x + 2 .
+
4 3 24
cos3x + 9 sIn 3x +C
27
4.3.25. _ x2+3x In x _ + -3x+C.
(
)
1

(d)

CDS

\037

...
...I

+.

\037

9X2

\302\267

\037I

\0373

\0372

9-

x
x
4.3.26.x 4 1 arc tan x-T2+4\"+C.
x3
2+x2 ...r
2
4.3.27.\"3arccosx
r
+C.

4.3.28.(a)

18x2

+-:fx-x+C; (b)
'27
+ 320(2x+1) 3 +C.
1

l-x

6;-13sin(6x+2)-72t

6x l

+
CDS(6x+2)+-}x
3

4'(x -7x+l)(2x+I)340(2X-7)(2x+l)3+
3

\037

\037

4.4.2.(d)

expressI n

Hint. Apply the generalized


formula
from the relation thus obtained

I n --

n -1
a-:)sin
e'1.X
w

(a sin

by parts

for integration

I n _.)X-fl cosX ) +n (n-I)


- --;-In.
a
n

\037

a-)))

.}
w

and

Answers and

flints

- -(n-I)sin- +n-l
('osX

4.4.3. In =

Ck. \

to

Jn 2

(n

\037

441)

2);

_x2 +C.
=- 2cos. .) -/-=- 2cos. .) +-2
2 /1
4.4.4.(a) I n- tann-lx-ln_2; 11=-lnlcosxl+C;10=x+C;
1 =ln sinxl+C; 10=x+C; (c) In=
(b) I
n- cotn-tx-ln_2;
n-l
/ 1= ..r
=_x - ..r
r +a+C;lo=ln x+ ..r
r x +a +C.
r x\037+a--aln-2;
x

13

==-

SIn\"'

Sln--

tan

In

x2

n)

Chapter
x2
5.1.2.
2-2x -1-6 In

- +3 x+ +C.
5.1.5. x-I -ln -(x_l)2+C.
+ --arc (x+2)+C.
5.1.8.V- +
5.1. +
V)

16

(x+ 1)3

..--

21

x
21n
2x I
1
2
arctan
3 7
V 7 3
to. 5x 1n x 2 (x -+. 2) x 2 3
1

In

-1

. 4 (x +2) (x -1-3) + 8
5111

tan

C.
(X+I)(X+2)16

+3)11 1+c
5.1.12.x-2-arctan (x-2)+C.
9x\037+50x+68

\037I n

\302\267

(x

\037

13.)
5.1.

1n

6 (I +x)+\"6

(l
I-x +22xarc tan x-

+ X)2

-3 +C.
.rr

+x\037

3 V--3

arc tan

--

x+2
arc tan x + In VX+1 +C.
5.1.14.
2 (x2 + I) +2
V x2 +
x-II+C.
5.2.2.
4 VX+6 V/ x -1-24 l\037x+24In
V (t +
arc tan 2t -I- +In
5.2.4.)))
+
..r
3
3
r {2 +t+l
VyVt-I .X-l
+C, where t =
I

3.\037

5.2.7.-./x+1+C,
V

l_x

V X-.

3 -

5.2.
8. 2\"

2)\"

I+X
V l_x+ C,

5.2.9.
rcSinx+C.
(I--}X)V1 -X2-%a
5.3.3.-2arc tan ( VI +X;X2 + + I)+C.
5.3.5.:2 V Xl -j-2x -r- --x 1- . 2 3
I

In

<1

\037

(Vx

+2x+4-x-l)

-:T

In

1/
r

.)'

X\"

-1-2x-t-4

-x-1

+C

\302\267)

442)

5.3.6.+ V x

and Hints)

An\037we's

x'\"

+2arctan

+
V -+C.
1-x
x

(x+ VI
5.3.7. x-I
+C.
+C. 5.3.8.
15
V 2x
5.4.2.5 JI2 +2x +- 5-1n(x+I + V +2x+5)+C.
5.4.5.3X
V3x 2 2x+I+C.
5.4.6.2xtl y'x2 +x+1+ InI2x+I+2YX2-t-x+I/+C.

+X\037)15

x\037

x\037

+t-

x\037

}+4x+3-

661n x+2+Yx\" -t-4x+31+c.


5.4.8.
f(x -14X+Ill)Yx 2
5.4.9.
Y\037xl+5x+7+
InI4x+5+
\037(32x2_20x-373)
128 2
64
2
+ 2 Y34x + lOx
. + 14 +CC..
3x+5 ../.) 2
5.4.10.
r x-+
x-1farcsln(x+I)+
\037(x+l)2
x2 4x + 3 2 arc .
V
sIn \037+C.
5.4.11. x_ 15 -.V x +2 8x2 + 12x+7 C
5.4.12.
Ip
x+ 4 (x+
2
x2
+1
+3x
In
5.4.13.
+C.Hint. First make the substitution
2

32\037_

- -

\037

x2

x-\037

+.

+1+\302\245x
x)

=
5.5.2.3 arc tan V x+C. 5.5.4.3\"2 ( 2+x ) +

t.)

\037

11

!)

-5( 2+x ) + +C.


12

\037

3
') -;- C.
? :;- 3 ( I +x2) + 3 (l +x5.5.5.22
) +
( I -r-X-)
5.5.7.I: V(I+V--Xf-3
l/(I+V xt+ c.
3
C,
5.5.8.3 In Vx -+
x
1+v x l+V3/-+

-8

TO

;j

\037)

5.5.9.(I +X2)
5 .0.
D..
J::

VI

(3X2

15

-2)+C.
I) C.
+

(2x'\"

x\037

3x3

21
3/-) + C.
5.5.11.
32 V ( +
4
1

x-l

5.5.12.(I++)5---

ii -1-C.

(I +
tan x +
c. 5.6.6.
,I
3
5 SIn0 x +

\037

\037

\037

x -I-C.
5.6.2.
,I
3 SIn3 x
5.6.10.
(a) -cotx+fCOt3x-+cotDx-x+C;
tan 2 x -I-In cas x +C.)))
(b) tan 2 x- In (I + tan 2 x) +C
1

\037C,

\037

\037

tC1n:J

\037

Answers and Hints

Ch.

to

443)

V I)

x
5.6.12.sin x -\"3Sin 3 X+2 n + sin
C.
sin x +
1+2tan)
5.6.14.2 arc tan (
) +c.
I

1I

\037

Vl5

VI5

-8+
x

,
5.6.22.
(a)

sinh 4x

32

5.7.3.-{In(x+Yx

+C;

;+ )+C.

2 tanh

(b)

Y3

arc t an

Y3

-1)+{X(2x -1) -1+C.


2

Yx 2

+ 1)-V xx2+ +C.


+C.
x-I
5.7.8.I
+c.
4 V x2-2x+5
5.8.2.1=4V I-x+ 21n (2-x-2V I-x)-2(I + V
a
nt
5.8.5.I
+ c, where t = arc tan
co\037;:t
I

5.7.4.In (x+
5.7.7.I =arcsin

Vx\037

x\037

==

I-

x+C.

x) In

x.)

\037\037e.t

C hap t e r VI
19= 44 the
6.1.9.
1=4.3+
as
2
bases 4X

and

I ==

6.1.12.
Sn== 164
1

6.2.2.
(a)
(d)
(J)

and

3
I; (b)
j[
; (e)
4\"

(c)

\037;

3;

tan

(k)

= t (2) =

In

2n

+ 4n2 '

(g) arc tan

(f) I;

. 6.3.1.
(c) 3 < I

175 125

7
j(
(a) 72 ;
(3' 6.2.10.

2;

sin 2x
. 6.3.11.
(a) x ; (b)

e-4 ;
1t

1t.

2 2;
I

(b)

1n

n;

14

16; (1) 15;

(h)

= f (0)

(c)

5
2''

=
Y I +x4. 6.3.14.
(b) .63.15.
(b) =
< 5.

3.

5-1==4

is

whose height

trapezoid

4+

4-arc V-3-Y2
j(

area of a
4 X 5 1 = 19.
1
175 125
16
;
4n2 Sn=
2n

Htnt.

==

rn

,
' t
t
=-e-Ysinx.6.3.23.
(a)
(a) x; (b) x' 6.3.24.
lnt ;(b)Yx=T'
at x == 1; (b) the
6.3.25.
(a) The maximum is at x ==I, the
are at x==-2; 0; 2, the maxima at x==:i:l.
\037

In

minima

\037\0374

:.1

In

6.4.15.
2; (b)
(a) 2-21n

-t- Y

0.2

In

2)

(b)

Y-3 Y2 bstitution
(su
--;

- I);

../0
. j[
SIn

(b) 2 (r 3

112; (c)) .
SIn

24

11

\"8

. J[

sIn

12)

tan

Y\\==

minimum

6.4.3.
(substitution x = a sin t);
(a)
2
..r- .r6.4.6.
(a) r
+ 2+V 3- ;
3

:\037

\0372

(c) 8

(d) Y

x=tan I).

n.
+3 Y3
'2

3-0.5

In (2+)))

Answers and Hints)

444)

+ Jf 3);

(e) 0.25In 3

(substitution

x = a cost);

6.4.16.
(a)

x-cosx=t);

sin

(substitution

(f) a3

;+{

x = 2a sin 2 t); (h)


(g) n;2 (substitution
; (c) {-In (su bstitution x4 = t); (d)

\037

\037

; (b)
(substix2 = a2 cos2 t b2 sin 2 t).
will not do, since this function is disconti6.4.17.
The substitution x =
=
nuous at t O.
will not do, since this function is disconThe substitution t = tan
=
tinuous at x n.
Hint. The inverse function
yi5 is double-valued. To obtain
the correct result it is necessary to divide the initial interval of integration into
\037

tution

\037\037

\037

6.4.18.

6.4.19.

x=:I:

two parts:)

-2
\037

x2 dx =

x=-

-2
\037

and apply the substitutions


< x < 2.
6.4.20.I t is im possjb Ie,

+ V x2 dx

V x2 dx

\037

V15 in
since sec t

-2< x < 0

and the interva

\037

I].
It is possible;see Problem 6.4.12.
6.4.21.
a
0

[0,

6.4.22.Hint. On writing

-a

f (x) dx

x=-t

stitution

in the first

-1

f (n

\037

o
Represent

Hint.

\037

6.5.3.
(l)
3t
-n

as the

-1

(2n

\037

+ arcsin t) dt.

sum of three integrals

and

substitute

3t

(x) cosnx

dx=2

J[

(x) cosnx dx. and

\037

-n

(x) sin x dx ==0.

\037

\037

If

for the in-

variable: x =

the

an even function,

(x)

\037

(2)

is

. 3;), ( 3;,

\037

of in tegr at ion

(x) dx, make the sub-

2n)sin t
), (
= arc sin (0,
t, x=n-arcsin t, x=2n+arc
respectively.
then
If
is
\037

0<

\037

arc sin t) dt +

the given integral

in

Vt5

tervals:

x=+

(x) dx

\037

integral.

6.4.23.(arcsin t) dt +

-u

\037

and

(x) is an odd function, then

\037

(x) cosnx dx

=\037

0, and

\037

(x) sin nx dx=

\037

-rr

-.11)

J1

=2

t (x) sin n

dx.

\037

6-2e.6.6.5.V-2-4.6.6.6.
6.5.4.O. 6.6.3.
Jf-2. 6.6.13.
(a)
Jf

(b )
(g)

2j[

---nlr-+ -In-'
2\037'

j[
(c) 4

(h)

V-3
9

3-2
16J1

3.)))

--I;
2
---\"
In2--'
4 2' ( )
2' ( ) ln-'
8'
\037

(d )

JI

<:

flint.
6.6.14.
Integrate
by parts twice,
(arccos -1the second time.
Hint.
6.6.15.
ntegrate by parts, putting
6.7.4.(a) 0.601.
Hint. Estimate flv (x)

ti Ine and u

==

0.96
2n = 6; (b) 0.7462.6.7.5.

6.8.1.)

x2
2

F (x) =

(x-2)3

\037

1-

take advantage
x

-1

x\037

the derivative

(x) is continuous

both

In\037lde

reo
the

dx

= f' (b) 1'(0).

(x) dx+

\037

> 0 and
< n + I,

of the integral
2

\302\243

\302\243

\037

(x) dx+

... +

The antideri
6.8.6.

[0, I].

on the interval

for definiteness x
E (x) n \037 x

the wrong

rational,
irrational

r (x)

\037

\037

n
\302\243

n-l

(x) dx+

\302\243

(x) dx.

\037

\037

vative F 1 (x) wi 11 lead to the correctresu It and F 2 (x) to


in the interval [0, :rr].
function is discontinuous
x

one, since this

6.8.7.F (x) = Yo +

the

form

(t) dt. Hint.

Any

antiderivative

F (x) can be represen-

\037

xo

in

if

of the additivity

ted

x is
x is

I if

Putting

(x) dx

2,

\037

Consider the function

{
6.8.4. V3. Hint.

6.8.5.
Hint.

<x

\037

lim
f(x)=f(1).
f{x)=f(O) and x-+l-0
x-..+O

(x) ==

cp

1,

+2)for 2 < 3.
The assertion
x= 1,x=2. concerning

Continuity is checked directly.


quires checking only at the points
Hint. Make sure that the function
and at the end-points (lim
interval

6.8.2.
(0,1)
No. Hint.
6.8.3.

O\037x\037

first

[ . ] and put

on the interval

for

2\"

u=(arccosx)nthe

x.

==

for

\\

445)

V I)

putting

X)n

\302\243

Ch.

to

Answers and Hints

Puttingx=x
o. find C=Yo'
F(x)=\037f(t)dt+C.

e2b-e:!.a
6.8.8.
6 = 2\" In 2b-2a .
The function is defined
6.8.9.
the
xo

[-I,

on the interval
1], it is nod,
interval
0] and concave on the interval
is a point of inflection.
the point
O. Hint. The fllnct ion
at 0 < x \037 1
XX

creasing; convex

on

[0,0]
6.8.1

is

continuous

x:=::-and
I

e)))

(x) -=

on the interval,
the greatest

[-1,

vC1lue

it
IVl

at

=--.:

Clnd

In-

[0, 1];

0)

reaches the least


-= 1 at x == 0 and

valuc
at x

fll
=-=

==

1.

e e

\037

0.692

at

446)

..

Answers and Hints)

.
. H lnt.

'

6.8.12.
Hint.

Integrate

th e Inequa l 1t y

t
te
negra

6 8 11

oro

and write Schwarz-Bunyakovsky

5 Jlxsinxdx\037
th

[
_

\037

x2

j[

O\037x\0376

inequality

1//-5\037:
5 sinxdx=y

\037

Apply

sin x
\037
x

\037

/ -6==X... I-Tat

...

1t

the inequ ality


x4
x2
V
V

\302\245XSJnx>

6.8.14.
Hint.

xdx

Schwar z-Bunyakovsky
b
2

'y (x).

;x)

dx

Hint.
6.8.15.

Make the substitution

6.8.16.
Hint.

If f

\037

\0372

'

2Y2
n_,__

in the form

inequality
b

(x) dx

f ;X)

dx.

\037

\037

arc tan x=--}.)


x

(x) is an even

F (x) =

funct ion, then

(t) dt

IS an

odd

\037

o)

since)

function,

-x

F(-x)=

f(t)dt=-\037 f

\037

()

(-z)dz=-F
(x) (t=-z).

I')

And

if f

(x) is an odd function,

then F (x)

f (t) dt is

an

even function,

\037

o)

since)

-x

F (-x)=

(t) dt

\037

the remaining

Iso even

\037

o)

all

(-z)dz = F (x) (t = -z);)

1\\)

the

have

antiderivatives

ions.
6.8.17.
Hint. The derivative

dl
da =f(a+T)-f(a)=O.)

F (x)

fOftn

funct

of the integral

with

+C and,

are

therefore,

respectto a equals zero:

Chapter VII)
7.1.4.
(a)

In

1.2 2. (a) 2\"'.


1

\302\267

2;
(b )

5
7.2.13.
(a) =3\";

(b)
I

; (2 Y2-1);
2

2+ 2
1

e -i-I

= In 2;

0 283.
\037.

(c)

3
4\"')

(d) 1;

7.2.5._j[4 ' 7.2.10. 2i

.
n.

V d:?+h2
2/0
7.2.16.

It

8
(c) == In 3

2'

(e)

+2. 7.2.15.
3\"'
2 5
35
8
. 3
7.3.11.
7.3.16.
7.3.13.9.
7.3.6.
'
1.3,4.\"'6' 3+2\"arcsln
S
15
m+
l-!

(b)

l-!

2/z

l-!

_
1

')

.)))

Answers and Hints

to

Ch. V II)

447)

7.3.21.
211(2 J/ 3) In (2+ Y3).
7.3.20'\"3'
4
8

64
7.3.19.

3'
128

7.3.22.
0.7511.

91
7.3.23.
. 7.3.24'\"3
. 7.3.25.3\" . 7.3.26.15' 7.3.27.12' 7.3.28.
IS
30
Hint. The curve is symmetrical about the coordinate
7.4.6.8 7.4.8.0.7511ab.
1

.)

5'

axes and

x= :i:a,y= :i:b.

them at the points

intersects

' Hint.

7.4.9.(a)

The curve is symmetrical about the x-axis,intersecting


The loop is situated in the second and th ird
it twice at the origin at t
'
ot the curve are found
Hint.
The
(b)
quadrants;
points of self-intersection
in the following way: y=tx(t), therefore y(tl)=t 1X(t 1)=t 2X(t2) at t 1 1= t 2 and
8
t1
t2
(c)
X(tl)=X(t2), only if X(tl)=X(t2
\0373
7.4.10.
Hint. The curve is symmetrical \\\\lith respect to both axesof
0.2511ab.
coordinates and passestwice through the origin forming two loops.Therefore, it
is sufficient to compute a quarter of the desiredarea corresponding to the variation
the obtained result by 4.
and multiply
of t from 0 to

=:i:l.

\0375

\0375

)=0,i.e. =0; =2;

3c4
7.4.11.
8ab

JT

The curve resembles an

Hint.

direct ion.

7.5.2.(a)
the

through

. Hint.

3;

:rr:

pole and

symmetrical

; (b)

.
87.5.10. .
2
7.5.6.2a
(

511

.)

The

curve

extended

astroid

is a circle of

about the polar axis,


11a2
11a2

7.5.8.(a)

8'. 8'
(b)

in the vertical)))

radius

-;

passing
\037

\037<p\037

7.5.9.a2

3 .
12-..r)
\037

711

Hint. The curve passesthrough the pole forming t\\VO loops located
about the y-axis in the first and fourth quadrants. It is sufficient
symmetrically
to calculate the area cnclosedby one loop corresponding to variation of cp from 0
\037\037\037

to

:; and

dou b Ie the resu lt th

7.5.11. 2. Hint.

LIS

obta

nen.

The curve passes through

:rra

\037\037

the

IS symmetrica
quadrants. It is sufficient

about the polar axis and situated in the first and fourth
to calculate the area of the upper portion of the figure
variation

of

cp

from

and double the result thus

to

pole. it

which corrcsponds to
obtained.

\037

7.5.12. (1+
7.5.13.
. Hint.
:rr;2
a2

\037

intersects

them only

(a four-leaved

_
\0373

The curve

at the origin, forming


it is sufficient

rose).Therefore,

coordinate axes and


four loops-onein each quadrant
to find the 2irea of one loop corres-

is symmetrical

about the

and multiply
the result by 4.
ponding to the variation of cp from 0 to
Y2 na 2 Hint. The curvc is sYl1lInetrical about the axes of coordinates and the bisectorsof the coordinate angles; it cuts ofT equal intercepts on
the axes. The origin is an isolated point. It is sufficient to compute the area of)

7.5.14.

Answers and Hints)

448)
of the figure

one-eighth

7.6.2.9 2 31. Hint.

A plane

centre

the

of the
2n 2a2b.

7.6.10.
7.6.16.(a) 2nab

from

cp

of

triang

le situated

circle is equal to h Ya\037-x2.


7.6.11. (see Problem

31

to

+ );

(b)

6
13

a; (c)

at

ann mul.

the point

the

distance

7.6.17.

abk 2n.

will

cross-sectio-

7.3.9). 7.6.14.

\037

x-axis at

2, therefore
r= Jl16-x

radius

2
).
S(x)=n(16-x
2h. Hint. The area
7.6.5.
0.531a

arca

to the

perpendicular

a circle of

cut the sphere along


nal

of

8.

the result by

tiply

to variation

corresponding

from

5Jt2a3

a.

a3 tan

4
16. 64. d 31,
2. (e) 64.
7.6.18. (a) 231,. ()
b
31, (f) 3 3.
15n, (c) 5 , ()
3 :C_ e
7.6.19. . 7.6.20.7;. 7.6.21.
) +na2c= n;3 sinh +
-{-na (e
3\0372

\037

\037

31

31a

3\"

\037C

\037

\037\0373

7.6.22. (631+5Y\"3).

na 2c.

Hint. The abscissasof the points of intersec+


6
Jt
J(
127
16.T1c
19
7.6.23.
x2
n
31
7.6.25.
tion are:
48
=3\"
2
Hint. Represent the evolute of the ellipseparametrically
as follows: x=\037 cos:Jt;
a
4 3
C2 . 3
:rr-a3
h
7 6 26
c V a 2 b2
7 6 27
sIn t were
y
3\"
4
Hint.
Pass over to polar coordinates.
V2 In (I V2)
l
4 3
e
e
7.6.28.
7.7.2.27 7.7.4.In
In (Y2+y'3);
(a)
a 7.7.8.
\037:rra.
n
2
J(
_ . (c) Y3 7 7 9 a (a-t-2)
(b) 2ln (2 r 3). Hl nt. Xl =
X2
\037

. 7.6.24'7.

xI=-3;

--b

\"

nt

- r\"I

-.

112
'
\037

'

.. ..
3' ...2'
. Y2'.

. na.

]. ea-e--'
b

-2; - 3'

105ab\037

-)

Y6+

7.7.10.
10
7.8.2.8a. 7.8.5.I:. Hint. The curve intersects the
G;+ V5). ,.-16a. 7.8.9.
axes at t 1=0 and t 2 =1/8. 7.8.7.4 V3. 7.8.8.
831a. Hint.
3
3
3

- . 7.8.11.3' 7.8.12. t=3


e;- ),= 3;]. 7.9.5.cp- 7.9.9. . 7.9.10.
.J

SC'e Fig. 79.


M a

Hint.

4 (a
7.8.10.
ab

b )

31

5
2+

1.5:n:a.

The curve p

2 V a cos
\037

\037

the point

2 V 2 :rta.

In

\037

2Jt

At

\037

is a circle.

-3'
-

14:.1
62n
7.9.11. p [Y'i+In (I + y'2)]. 7.10.3.
3 ; (b)
7.10.5.
2JT
.
7.10.8.; 7.IO. 4. (3.1yT7 2) .
(I +3 \0373 )
56 2
2 Jl2
7.10.7. --;::-7.10.15.[V2+In(i + V 2)]. 7.10.16.
na .
(er. 2).
3
16a2 where a
7.10.18.
29.6Jt. 7.10.19.
4;t 2a2. 7.10.20. na2. 7.11.7.
(a))))

\037

2:n:

IT

t))

IS)

1\0378

Answers and Hints


the

radius
7

-4arc
I

(b) 50

(c)

the

of

tan

+6 J
{(511

211
7.11.17.

449)

VII)

8
7.11.8.1.5n. 7.11.10.
(a) 15;
2
2

base.

cylinders

Ch.

to

na
7.11.11.
(a) T;

2\"'

(b)
\037

(3+4V2);

7.11.13.
(2 In 3 -I). 7.11.14. (5 V5-2V 2).
-I+ ) .
7.11.19.
11a2 V pq .
. 7.11.18.
6
4
2a. 7.11.23.
7.11.21.
12:n:. 7.11.22.
:111a3.
)
(

3).

\0372

\037

11ab

\\\0371-

\037

C\0372

7.11.20.
:rra;h
7.11.24.
(b)
(a)
V\037-I
(2IVT3+
f(V5-V-2)+(V2+1)
M
2
.3+ VT3 .
+ 2ln 2 ) ' (c) 2JTrh. 7.12.2.
31'R. 7.12.4. 7.12.9.4 .
7.12.13.11R2H. 7.13.3.0.2511R3.
l1rdh 2.
7.12.12.
7.12.11.
11abhd.
7.13.7.
Mx =
Mx= (5 V5-1);
My=; V5+ 116In (2+V5). 7.13.8.
= Va 2+b2; My= Va 2 +b 2. 7.13.9.
V2+ln(l+V2). 7.13.10.0.15.
3
3
(a + 3b) . 7.13.
16.Xc = c = O.4a.
11.Ix = ab12; Iy = a b . 7.13.12.
7.13.
12
5a
sin a
a
7.13.19.
xc=R----a-;Yc=O. 7.13.28.
xc=S;Yc=O.
xc=Yc=S' 7.13.26.
l1b

V\037--

];

11

4'

R\037(U\037

1\037

\037

\037

Ii

1z3

0.2(2e -e)
7.13.29.
x c =1[

-e

e
7.13.31.
xc=O; Yc=

0.2a(e

21t

TI

2TI

Yc=

\037

1t

\037\037

even;

if

21

:\037\037

evenness. Hint.
and

42:\0372

:rr.Ri

(1, 1) in

quadrant

both

Tt

the first

\037

;-x:'

(x

if

4\\

:\037\037

if
:\037\037
Tn

m and n

are of

different

have two common points (0, 0)


of the figure situated
in the first

. Depending

) dx

bothmand narc

=x
I

and yn

The area

quadrant.
1

is equal to

n
ym = x

are odd;

m and n

7.13.30.4.5:rr.a:.

TI

1[

-e

' 7.14.1.1
:.;\037!
1;

The curves

_ 2e ) '

on evenness and

oddness

either about the coordinate


of m and n this figure is mapped symmetrically
axes (m, n even) or about the origin (m, n odd). If m and n are of different
evenness, then the curves encloseonly the area lying in the first quadrant.
Hint. Take advantage of the formula for computing the area in
polar coord inates.
Hint. Sincethe figures are of equal area, the function
appear-

7.14.3.
7.14.4.

S(x)

ing in the formula

for the volume V

= S (x) dx

is

the same and, consequently,

\037

a
are also equal.

the values of the integrals


Hint. The formula

7.14.5.

5 J (x)
o)))

follows oircctly

dx=
\037

from

Simpson's formula

II(O)-HI( ) +1(h) J
\037

\302\267

450)

Answers and Hints)

a sphere

for

S (x) = n (r 2

S (x) = 2npx

-x );
2

nr 2x2
h2

a cone S (x) ==

for

for

of

paraboloid

and so on.
Hint. Divide the curvilinear trapezoid into strips \037x wide and write
an expression for the element of volume \037v
2n xy \037x.
Hint. Use the formula for calculating the length of a curve represented parametrically.
H
The point (t I) nearest to the origin with a vertical)
In
revolution

7.14.6.
7.14.8.

;. in!.t==-,
V3
7.14.13.15'
-0.5

7.14.9.

tangent

correspondsto)

2)

(b)

7.14.16.) (a)) 0.5In (x+y);)

7.14.14.) Y2.z.)

2n

arc sinx.

\037

C hapt e r

VI I I

8.1.2.
(f) 2'
(b) 21n2; (c) I; (d) 1-ln2;(e) n;
In (x2 + I)
8.1.6.
> x for x > Y e-I; (b) converges;
(a) It diverges. Hint.
x
. t 2+ cos > ' (d) con verges; (e) d .verges.
.
(c) d verges. H tn.
x
I

xl.

001
I

8.1.17.
(a)
in

S 1+x2

O.

00

dx==

Yx

Hint.

Represent

\037

the

in
in
dx
dx. Make the
S 1+x2 + S 1+x2

00

r
J

th at

sh ow

con d surnrnan d an d

8.2.2.
(a)

9a

as

integral

oc

3;

..
I

the

substItutIon

2d

m!

(b) 2

diverges;
(d) 6
(c) diverges;
(f) converges for p < 1 and diverges for p \037
(c) converges;
(a) It converges;
(b) diverges;
(b)

it

I.

8.2.7.

x==t in

x
x= 5 I In+x
2 dx;
I +x
0
In

of two items:

sum

se-

the

'

V 2;)

(e)

\"3')))

1r- 7'
(d) converges;

51
2; (c)
diverges;
(e) diverges; (f) converges.
(a) It converges;
(b) diverges;
(c) diverges; (d) converges;
2.
3:rra
8.3.7.
211.
(e) converges.
(a) ; (a)
mgR. Hint. The law of attraction of a body by the Earth is determ{? R2
where In is the rnass of the body, r is the
mined by the formula f == 2
r
distance between the body and the centre of the Earth, R is the radius of

8.2.11.
(a) It

8.2.14.

8.3.14.

(b) 2 r

In

8.3.9.
+. 8.3.10..

8.3.8.

\03711

'

tl\037e

Earth.

8.3.15.
el' Hint.
are

the

magnitudes
8.4.1.
Hint. Represent
+00

51

where e1 and
r ,
the distance between them.
in the form of the sum
+00

Electric charges interact


of the charges and r is

xP

\037\037q

the integral

-5
a

xP

\037\037qX

+5
a)

with

xP

a force

\037\037q

el\0372

(a > I)

e2

A nswers and Hints

Ch. V III)

to

451)

that in the
and apply special tests for convergence, taking into consideration
as
I, and in the second integral
integral
function increasesslower for q < 0 than any power function.
the logarithmic
Hint. Making the substitution
reduce the given integral to the

lnx=ln[l+(x-l)]-x-Ix-+

first

8.4.2.

:!:+

form

}'\"

5
o

dt

xq=t,

5I

ges absolutely
1

==0 the

for

a=I

<a <2

and

is

integral

integral

and at
Si7 dt.

-+
P

5\"'

\037

I
I

and show that the inte g ra

to

the

0<a

for

conditionally

conditionally

the sum

conver-

\037

I. Note

converging

that

at

integral)

+\037

to the diverging

t
integral

5
0

dt.

s\037\037

1/2
the given integral

Represent

as the

-x)q-1dx+
xP-l(1

sum
\037

as

sin t dt

\037:

q)

reduced

P; = -

+\037

8.4.3.
Hint.

' where

dt

t\037

q)

5o

the

Represent

0)

sin t

dt.

sin t

II

o
+00

s\037\037

p+

t :\037

Xp-l (I -X)q-ldx
1/2

and apply the

\037

8.4.4.Hint.

If

aI

:f-

specialcomparison test.

T
I

\037

I.

then

sin

ax.sin

\037x

dx

is

bounded.

is

reduced

\037

8.4.5.

Hint. By
gam ma -funct ion.)

t=x2

substituting
00

8.4.6.)

Hint.)

the

integral

00

(\037x)

(ax)\037

00

dx= f
5
aa

\037)

dx-5

a(3

dx= f
5
aa)

f
\037X)

a(3

dx=
\037X)

(3a

A
= A In a + f (x)dx. Applying
x
5
aa

the generalized

the last integral

8.4.7.Hint.

8.4.8.

mean value theorem, show

-+

tends to zero as a
O.
Take the function f (x)=e-x for the
of the
f (x) = cosx for the second and take advantage
It converges for m < 3 and diverges for m
2
x
of the equ iva lence of 1 cosx and
as x --+
that

to the Euler

integral, the function


results of Problem
Take advantage
\037 3. Hint.
first

8.4.6.

O.)

31

31

8.4.9.
Hint.

dx
Represent

5o (sin x)k

as the

SUlll of two integrals

50 (sin.dxx)k +

31

+5

dx

j[ (sin x)k

2
tution

; reduce the second integral

x=n-tand

take advantage

to the first

one by

of the equivalence of sin

making

and

the substi-

x as x --+

O.)))

452)

Answers and Hints)

J[
2

cosx)
sin x (1'
sin x (1cosx) dx
8.4.10.
Ht nt.
dx ==
+
S
S
S
xS
o
00

00

sin x
-I-

0)

(I. cosx) dx. The

of the first summand

integrand

X S)

sideis

on the right

-y)

s-3 x-+s-3 i.e.

of order
as
O. By the specialcomparrson
large quantity
infinitely
s < 4, and diverges
test the first integral converges absolutely for
< I,
for
4. The second integral in the right side convergesabsolutely for 5 > I,
since the function sin x
x) is bounded. But if 0 < s\037 I, the second inas the difference of two conditionally converging)
tegra) converges conditionally
an

s;;:::
00

sin

Integra Is

S
iT

(I-cos
00

dx and
xS

S
J[

sin x.cosx dx
X

(see Problem

8.1.13).

8.4.11.
Hint.

(2) can diverge.

Integral

(x) =

cp

I,

{ -I,)

2nn

(2n

\037

For

\037

example,

let)

(2n -t-- 1)1t,

+ I) n < x < (2n+2)

n.)
00

00

The

Integral

sin x

dx
S --X

(see Problem

converges

sin x
But
8.1.13).
S ----x-

cp

0)

00

00

S
o

(x) dx =

dx diverges

Si\037

(see the same problem).

But

if

the integral

S f (x)

dx

con-

a)

00

then the integral

verges absolutely,

S f (x)

cp

(x) dx also converges absolutely:

if)

a)

cP

(x)

< C, then

t (x) cP (x)

< elf(x) I,

and it retnains

theorem.)

to

use

the comparison

J[

8.4.12
Hint.

Transform

the

mtegral

(x) into

(x) =

2-.k
In sin

z dz

by the

\037

y=

substitution
the

above to

j[

;-

z. Taking

--

that sin

T'I)

-S .
2

1 ity)

I,z

sin 2nx

2n

()

sin '2nx

= sin

(2n

sin

cos x dx,

\302\267

2) x. cos

2,,\\

dx=dv,

Integrate

by

.
# O. SIr.ce

+ sin 2x.cos(2n-2)

cos

,reduce
\037

\037

the sum ot three integrals.


In cos x, cos 2nx
Hint. Putting u

8.4.13.

the equa

into account

z= 2 sin

x,)))

part\037

and

get)

Answers and Hints

r
In

I
==2n

453)

-5 . (2n-2)x-dx+
2

sin

SIn

cosx

0)

.\"1

+
equal

Ch. V //1)

.1t

Check

to

by direct

5o sin
n

for

2) x.sin 2x dx

+ 2 5 sin 2x,cos(2n

2) x dx

for n

that

calculation

zero. Therefore,

(2n

:n:

T
\037

the

\037

second and

the third summands

\037 2)

;TT

n-I '
.
1=--5 sIn(2n-2)x-dx=--1
- 4;'3
I, 2 S
2

11

sin x

2n

cosx

n _1

o)

,n

Since

sin 2x

\037-

sin

J(

cosux dX===4

dave

\\ve

\037

o)

and

by

InductIon,

j(
In=(-l)n-,
4n

')))

I'},=

Jt

\302\267

\037

=-=

' '2
'J

\302\267

2\"

j[
. 4j[ 3.4
===

Você também pode gostar